You are on page 1of 342

线性 代数学 习指南

居余马   林翠琴   编著

清 华 大 学 出 版 社
北 京
内 容 简 介

本书是与居余马等编著的《线性代数( 第 2 版)》配套的辅导教材 , 也 可为
学习其他教材的读者提供有益的 指导。全书 以章为 单位进 行指导。在每 章
中 , 首先 , 明确基本要求 , 指明了学习的目标和努力的方向 , 再给出内容提要 ,
提纲挈领地概括了本章的基本内容。然后 , 逐节进行指导 , 通过对基本概念、
定理和方法的深入分析 , 通过对一些基本、典型题目的讲解和演练 , 引导 读者
深入地学习和领会 每节 的基 本内容。最 后 , 对部 分难 题 和补 充题 给 出了 题
解 , 以帮助有余力的读者进一步提高分析问题和解决问题的能力。书后 还附
有历年硕士研究生入学试卷中线性代数题目的解答 , 以利于读者及时地检查
自己的掌握程度。

图书在版编目 ( CIP)数据

线性代数学习指南/ 居余马 , 林翠琴编著 .—北京 : 清华大学出版社 , 2003


ISBN 7-302-06507-1

Ⅰ. 线…  Ⅱ . ①居… ②林…   Ⅲ . 线性代数 - 高等学校 - 教学参考资料


Ⅳ . O151 .2

中国版本图书馆 CIP 数据核字( 2003) 第 023988 号

出 版 者 : 清华大学出版社 ( 北京清华大学学研大厦 , 邮编 100084 )


h tt p :/ / w ww .t up .com .cn
责任编辑 : 刘颖
印 刷 者:
发 行 者 : 新华书店总店北京发行所
开     本 : 850×1168 1/ 32   印张 : 10 .75   字数 : 259 千字
版     次 : 2003 年 6 月第 1 版   2003 年 6 月第 1 次印刷
书     号 : ISBN 7-302-06507-1/ O・292
印     数 : 0001~   000
定    价:       元
  序  言

本书是为居余马等编著 的《线性 代数 ( 第 2 版 )》教材 ( 清华 大


学出版社出版 ) 配套的辅导教材 .它也可为学习其他教材的读者提
供有益的指导 .本书还对 历年 硕士研 究生 入学 考试中 线性 代数 试
题给了题解 , 这将有助于 准备 考研的 读者 在学 习阶段 更好 和更 灵
活地掌握线性代数的基本内容 .
本书的书名《线性代数学习指南》表明了本书主要着眼于指导
学生如何学好线性代数课程 , 为此本书的内容作了以下安排 .
首先明确指出每章的“基本要求”, 并给出“ 内容提要”.课程的
“基本要求”是读者学习 的目标 和努 力的方 向 .每学 习一个 概念 及
有关的理论和计算 , 都 要按“基 本要 求”来掌 握它们 ; 每 学完 一章 ,
读者应 该以“ 基 本要 求”为 镜 子 , 对照 和 检查 是否 掌握 了“ 基本 要
求”, 对“基本要求”要能说出个一、二、三 , 绝不能含糊不清 .每章的
“内容提要”提 纲 挈 领 地 概 括了 该 章 的 基 本 内 容 , 它 是 每 一 章 的
“纲”.读者每学完一 章 , 都 应该 把“ 内容 提要”所 涉及 的基本 概念、
基本理论、基本计算以及分析和解决问题的基本方法 , 深深地印在
脑海之中 , 闭着眼睛都能熟练地陈述“ 内容提要”所述的方方面面 ,
这样你在思考各种问题和解题时 , 就有可能“ 纲举目张”, 顺利地抵
达彼岸 .
然后 , 逐节指导如何学 习每 章每节 , 这 是本 书的 重 点所 在 .这
里一般是从两个方面来引导读者深入地学习和领会每节的基本内
容 , 掌握分析解决问题 的方法 , 提 高解 题的能 力 .第一 个方 面是 对
每节涉及的基本概念及有 关的 理论和 计算 的方法 , 进 行深 入的 分
Ⅱ 序 言

析 , 力求准确地理解概 念 , 掌 握有 关定 理的条 件和 结论 , 掌 握计 算
的基本方法 .另一个方面是通过列举一些基本的、典型的和有一定
灵活性的计算题、概念题和证明题 , 帮助读者在理论的指导下提高
分析和解决各类问题的能 力 .对于各 种类 型的 计算题 要熟 练掌 握
它的基本计算方法 .有些题可以一题多解 ; 对于概念题要能准确地
判别各种说法的真伪 , 澄清一些似是而非的模糊观念 ; 对于证明题
要善于应用基本概念和基本的定理加以证明 , 要思路清晰 , 对各种
类型的证明题要 概 括出 一 些有 效 的 证明 方 法 ( 如 直 接 证法 , 反 证
法 , 数学归纳法等 ) .
在每章的最后 , 对部分疑难习题与补充题给出了题解 .这些题
多数是证明题和比较综合、比较难的计算题 , 题解一般都提出了解
题的思路 , 以及要用到 哪些基 本概 念和定 理 .读者 对于 这些 题 , 应
该在认真思考以后仍不会进行证明或计算时 , 再看题解 , 这样对比
自己的思考过程 , 才能深刻领会解题的关键所在 , 从而切实提高证
明和计算的能力 .
本书最后 , 对历年硕士研究生入学考试中线性代数试题 ( 按本
书章的顺序汇编 ) 给了题解 .这不仅可供考研的学生作为备考的参
考 , 而且更有意义的是 , 读者学完每一章都检查一下自己能否解这
些题 , 从中可以发现自己还有哪些基本内容掌握得不够 , 需要进一
步深入和提高 , 这有助于读者更好地学好线性代数课程内容 .
由于编著者水平和经验所限 , 不妥之处在所难免 , 恳请读者们
批评指正 .

编者
2003 年 2 月于清华园
  目  录

第 1 章   行列式 …………………………………………………… 1
1 .1   基本要求与内容提要 ………………………………… 1
1 .2   行列式的计算 ( 展开 ) ………………………………… 4
1 .3   克拉默法则 …………………………………………… 19
1 .4   部分疑难习题和补充题的题解 ……………………… 21

第 2 章   矩阵 …………………………………………………… 37
2 .1   基本要求与内容提要 ………………………………… 37
2 .2   高斯消元法 …………………………………………… 42
2 .3   矩阵的基本运算———加法、数量乘法和乘法 ……… 49
2 .4   矩阵的转置 …………………………………………… 62
2 .5   可逆矩阵及其逆矩阵 ………………………………… 65
2 .6   矩阵的初等变换和初等矩阵 ………………………… 73
2 .7   分块矩阵 ……………………………………………… 78
2 .8   部分疑难习题和补充题的题解 ……………………… 82

第 3 章   线性方程组 …………………………………………… 97
3 .1   基本要求与内容提要 ………………………………… 97
3 .2   n 维向量及其线性相关性 ………………………… 102
3 .3   向量组的秩及其极大线性无关组 ………………… 122

3 .4   矩阵的秩   矩阵的相抵标准形 ………………… 125
3 .5   齐次线性方程组有非零解的条件及解的结构 …… 131
Ⅳ 目 录

3 .6   非齐次线性方程组有解的条件及解的结构 ……… 142


3 .7   部分疑难习题和补充题的题解 …………………… 151

第 4 章   向量空间与线性变换 ………………………………… 167


4 .1   基本要求与内容提要 ……………………………… 167
n
4 .2   R 的基与向量关于基的坐标 ……………………… 171
4 .3   R n 中向量的内积   标准正交基和正交矩阵 ……… 177

  4 .4   部分疑难习题和补充题的题解 …………………… 192

第 5 章   特征值和特征向量   矩阵的对角化 ………………… 218


5 .1   基本要求与内容提要 ……………………………… 218
5 .2   矩阵的特征值和特征向量   相似矩阵 …………… 221
5 .3   矩阵可对角化的条件 ……………………………… 229
5 .4   实对称矩阵的对角化 ……………………………… 237
5 .5   部分疑难习题和补充题的题解 …………………… 248

第 6 章   二次型 ………………………………………………… 255


6 .1   基本要求与内容提要 ……………………………… 255
6 .2   二次型的定义和矩阵表示   合同矩阵 …………… 258
6 .3   化二次型为标准形 ………………………………… 262

  6 .4   惯性定理和二次型的规范形 ……………………… 274
6 .5   正定二次型和正定矩阵 …………………………… 276

  6 .6   其他有定二次型 …………………………………… 285
6 .7   部分疑难习题和补充题的题解 …………………… 287

历年硕士研究生入学考试中线性代数试题的题解 …………… 296


  第 1 章

行 列 式

1. 1 基本要求与内容提要
1  基本要求

(1 ) 理解行列式 的定 义 , 熟 悉每 一个 元素 的余 子 式和 代数 余
子式的含义 .
(2 ) 理解行列式 的性 质 , 并 能熟 练利 用性 质展 开 数字 行列 式
和文字行列式 .
(3 ) 熟悉一些特 殊行 列式 ( 如对 角行 列 式 , 副对 角行 列 式 , 上
( 下 ) 三角行列式 , 范德蒙 ( V andermonde ) 行列式等 ) 的展开结果 .
(4 ) 理 解 克 拉 默 ( Cra mer ) 法 则 , 会 利 用 它 求 解 一 类 线 性
方程组 .

2  内容提要
n
(1 ) n 阶行列式 D = | ai j | 1 的定义为
D = a11 A11 + a12 A12 + … + a1 n A 1 n ,
1+ j
其中 , A1 j = ( - 1) M1 j 是元素 a1 j 的代数余子式 , M1 j 是元素 a1 j 的
余子式 ( 它是 D 中去掉第 1 行与第 j 列全部元素构成的 n - 1 阶行
列式 ).
D 的展开 式是 n2 个 元素 ai j ( i, j = 1 , … , n) 的 n 次 齐次 多 项
2 第 1 章 行列式

式 , 共有 n ! 项 , 每项都是不同行不同列的 n 个元素的乘积 .
二阶、三阶行列式可按沙路法展开 .
(2 ) 行列式的性质 ( 对行与列皆成立 ).
① 行列式的行与列 ( 按原顺序 ) 互换 , 其值不变 .
② 行列式对任一行 ( 或列 ) 展 开 , 其值 相 等 ( 定 义是 对第 1 行
展开 ).
③ 线性性质 : 其一是行列式某行 ( 或列 ) 元素都乘 k, 则等于行
列式的 值也 乘 k; 其 二是 如果行 列式 某行 ( 或 列 ) 元 素皆为 两数 之
和 ( 如第 i 行为 a i1 + bi1 , ai2 + bi2 , … , ain + bin ) , 则其行列 式等于 两
个行列式之和 ( 其第 i 行分别为 a i1 , ai2 , … , ain 与 bi1 , bi2 , … , bin ).
④ 行列 式中 如有两 行 ( 或 列 ) 完全 相同 , 则 其值等 于零 ; 进 而
有两行 ( 或列 ) 成比例 , 其值也等于零 .
⑤ 把行列式某行 ( 或列 ) 元素都乘非零常数 k 加到另一 行 ( 或
列 ) 对应元素之上 , 行列式的值不变 .
⑥ 反对称性质 : 行列式两行 ( 或列 ) 对换 , 其值反号 .
⑦ 行列式某行 ( 或列 ) 元素 乘 另一 行 ( 或 列 ) 对 应 元素 的代 数
余子式之和等于零 , 即
ai1 A j1 + ai2 A j2 + … + ai n A jn = 0   ( i≠ j) ,
或 a1 i A1 j + a2 i A2 j + … + ani A nj = 0   ( i≠ j ).
(3 ) 行列式的计算 ( 或称展开 ).
展开行列式的基本方法有三个 : 其一是直接按定义展开 ; 其二
是利用性质 , 将行列式化为上 ( 下 ) 三角行列式 ; 其三是利用性质将
某行 ( 或列 ) 元素化为只剩一个非零元 , 然后对该行 ( 或列 ) 展开 , 将
n 阶行列式展开化为 n - 1 阶行列式的展开 , 此为降阶展开法 .
(4 ) 一些特殊行列式的展开结果 .
① 上 ( 下 ) 三角行列 式与 对 角行 列式 的值 都等 于 其主 对角 元
的乘积 a11 a22 … ann .
1. 1 基本要求与内容提要 3

0 … 0 a1 0 … 0 a1
0 … a2 0 0 … a2 *
② =
… Y … … … Y … …
an … 0 0 an … * *
* … * a1
* … a2 0
=
… Y … …
an … 0 0
n( n - 1)
= ( - 1) 2 a1 a2 … an .
③ 范德蒙行列式
1 1 … 1
x1 x2 … xn


2 2 2
Vn = x1 x2 … xn = ( xi - x j ).
1 ≤ j < i≤ n
… … …
n- 1 n - 1 n- 1
x
1 x 2 … xn
A 0 A *
④ = = | A| | B| ,
* B 0 B
其中 : | A| 与 | B| 分别 为 m 阶 和 n 阶 行列 式 ;0 所在 位 置的 元素 全
为零 ; * 所在位置元素为任意元素 .
(5 ) 克拉默法则
n

若线性方程组 ∑ aij x j = bi ( i = 1 , 2 , … , n) 的系数行列式 D =


j= 1

n
| aij | 1 ≠ 0 , 则方程组有惟一解 . 即
Dj
xj =       ( j = 1 , 2 , … , n) ,
D
其中 Dj 是用常数项 b1 , b2 , … , bn 替换 D 中第 j 列的 n 个 元素 所
成的行列式 .
4 第 1 章 行列式

1. 2 行列式的计算 ( 展开 )

3 2 3
例 1   计算 2 - 3 4 .
4 - 5 2
解   对于这个三阶数字 行列式 , 如 果利 用性质 将 其化 为上 三
角行列式 , 或将某行 ( 或列 ) 元素化为只剩一个非零元再展开 , 都有
较大的工作量 , 还不如直接用沙路法或行列式的定义 ( 对第 1 行展
开 ) 来计算 .
3 2 3
① 2 -3 4 = - 18 - 30 + 32 + 36 + 60 - 8
4 -5 2
= 128 - 56 = 72.
3 2 3
- 3 4 2 4 2 - 3
② 2 -3 4 =3 - 2 +3
- 5 2 4 2 4 - 5
4 -5 2
= 3 ( - 6 + 20 ) - 2 (4 - 16 ) + 3 ( - 10 + 12)
= 42 + 24 + 6 = 72.
对于三阶数字行列式一般都用这两种方法展开 .
λ+ 1 2 2
例 2   已知 -2 λ+ 4 -5 = 0 , 求 λ.
2 -2 λ+ 1
解   这里三阶行列式的展 开式是 λ的 三次 多项 式 , 所 以本 题
是三次方程的求根问题 . 如果 用沙路 法展 开 , 自然 易得 λ的三 次
多项式 , 但一般来讲三次 多项 式的因 式分 解是 比较麻 烦的 . 如 果
利用行列式 的 性 质 展 开 这 种 行 列 式 , 有 时 会 出 现 它 的 一 种 因 式
分解.
1. 2 行列式的计算 ( 展开 ) 5

λ+ 1 2 2 λ+ 3 0 λ+ 3
① +③
-2 λ+ 4 -5 -2 λ+ 4 - 5
2 - 2 λ+ 1 2 -2 λ+ 1
λ+ 3 0 0
[3 ] + [ 1 ] × ( - 1)
- 2 λ+ 4 -3 ( 对第 1 行展开 )
2 -2 λ- 1
λ+ 4 -3
= (λ+ 3 )
- 2 λ- 1
= (λ+ 3 ) [ (λ+ 4 ) (λ- 1 ) - 6 ]
2
= (λ+ 3 ) (λ + 3λ- 10 ) = (λ+ 3) (λ+ 5) (λ- 2 ) = 0 ,
所以 λ= - 3 , - 5 , 2 是这个 λ的三次方程的 3 个根 .
其中① + ③表示第 1 行加第 3 行 ; [ 3] + [ 1 ]× ( - 1 ) 表示第 1
列乘 ( - 1 ) 加到第 3 列上 .
2 5 4 9
-3 3 1 10
例 3   计算 D = .
3 4 5 15
4 3 14 19
解   对于三阶以上的数 字行列 式 , 一般 都是利 用 性质 将其 化
为上三角行列式求其 值 . 化为上 三角行列式的步 骤是规范化的 .
首先利用第 1 行第 1 列的非 零元将第 1 列 其他元素 全化为 零 , 然
后利用第 2 行 第 2 列的非零 元将第 2 列以 下元素全 化为零 , 如 此
等等 , 直到化为上三角行列式 . 如果化的过程中出现全零行 , 则行
列式的值等于零 .
这里第 1 行第 1 列 的元 素 为 2 , 如 果利 用 它 将第 1 列其 余 元
素全化为零 , 中间就会出现很多分数 , 继续化下去就比较麻烦 . 所
以这里先把第 1 行乘 - 1 加到第 3 行 , 再把第 1 行与第 3 行对换 ,
就使第 1 行第 1 列 元素为 1 , 这 样再将 第 1 列其 余元 素化 为零 就
比较简便 , 即
6 第 1 章 行列式

1 - 1 1 6
② + ①× 3
③ + ①× ( - 1) -3 3 1 10 ③ + ① ×( - 2 )
D -
①\ ③ 2 5 4 9 ④ + ① ×( - 4 )

4 3 14 19
1 - 1 1 6 1 -1 1 6
0 0 4 28 ④ + ③× ( - 1 ) 0 7 2 -3
-
0 7 2 -3 ②\ ③ 0 0 4 28
0 7 10 -5 0 0 8 -2
1 - 1 1 6
④ + ③× ( - 2) 0 7 2 -3
0 0 4 28
0 0 0 - 58
= 1×7 ×4× ( - 58 ) = - 1624.
其中 : ①\ ③表示第 1 行与第 3 行对换 , 此时行列式反号 .
-1 1 1 1
2 -1 3 -2
例 4   计算 D = .
4 1 -1 -2
-3 1 5 -1
解   此题按例 3 的方法 将其化 为上 三角行 列式 也 可求 其值 ,
但仔细观察会发 现各 行 元素 之 和均 为 2 , 此时 把 各列 都 加 到 第 1
列 , 第 1 列元素全为 2 , 而且第 2 列中也有 3 个元素为 1 .这样将其
化为上三角行列式就比较简便 , 即
2 1 1 1
[ 1] + [2 ] + [ 3] + [ 4 ] 2 -1 3 -2 ② + ①× ( - 1 )
D
2 1 - 1 -2 ③ + ①× ( - 1 )
④ + ①× ( - 1 )
2 1 5 -1
1. 2 行列式的计算 ( 展开 ) 7

2 1 1 1 2 1 1 1
0 -2 2 -3 ④ + ③× 2 0 -2 2 -3
0 0 -2 -3 0 0 -2 -3
0 0 4 -2 0 0 0 -8
= 2× ( - 2) 2 × ( - 8 ) = - 64.
1+ x 2 3
例 5   已知 2 1+ x 2 = 0 , 求 x.
3 3 1+ x
解   这里三阶行列式各列元素之和均为 x + 6 , 此 时把各行 加
到第 1 行 , 第 1 行元素全为 x + 6 , 然后把第 1 列乘 - 1 加 到第 2 , 3
列 , 就把行列式化为下三角行列式 , 即
1+ x 2 3 1 1 1
2 1+ x 2 = ( x + 6) 2 1+ x 2
3 3 1+ x 3 3 1+ x
1 0 0
= ( x + 6) 2 x- 1 0 = ( x + 6) ( x - 1 ) ( x - 2) = 0 ,
3 0 x-2
所以 x = 1 , 2 , - 6 就是题中的 x 的三次方程的 3 个根 .
这个行列式当然还可用其他方法展开 , 但这里的方法最简便 .
例 4 , 例 5 是行列 式中 值得 注 意的 一种 类 型 . 展 开 行列 式 时
首先要观察一下 各 行 ( 或 列 ) 元 素之 和 是否 相 等 , 如 果 相等 , 按 例
4 , 例 5 的方法展开比较简便 .
x1 a a a
a x2 a a
例 6   计算 D =   ( a≠ x i , i = 1 , 2 , 3 , 4).
a a x3 a
a a a x4
解   此题仍可将 D 化为 上三角 行列式 , 先将 第 1 行 乘 - 1 加
到其余各行 , 得
8 第 1 章 行列式

x1 a a a
a - x1 x2 - a 0 0
D= .
a - x1 0 x3 - a 0
a - x1 0 0 x4 - a
a - x1
再将第 j 列乘 - ( j = 2 , 3 , 4) 后加到第 1 列 , 得
xj - a
4
a - x1
x1 - a ∑ a a a
j=2 xj - a
D = 0 x2 - a 0 0
0 0 x3 - a 0
0 0 0 x4 - a
4
1
= x1 + a( x1 - a) ∑ ( x2 - a) ( x3 - a) ( x4 - a)
j=2 xj - a
4
x1 1
= a +∑ ( x1 - a) ( x2 - a) ( x3 - a)・
a( x1 - a) j=2 xj - a
  ( x4 - a)
4 4
1 + 1
= a
a ∑ j=1 xj - a ∏( x
j= 1
j - a) ,
4

其中连乘积 ∏ ( x j - a) = ( x1 - a) ( x2 - a) ( x3 - a) ( x4 - a).
j=1

a+ x a a a
a a+ x a a
例 7   计算 D= , 其中 ax y≠0.
a a a+ y a
a a a a+ y
解   法 1: 利 用 性 质将 其 化为 上 三角 行 列式 , 先将 第 1 行乘
- 1 加到其余各行 , 得
1. 2 行列式的计算 ( 展开 ) 9

a+ x a a a
- x x 0 0
D= ,
- x 0 y 0
- x 0 0 y
x
再将第 2 列加到第 1 列 , 第 3 , 4 列均乘 加到第 1 列 , 得
y
x
a + x + a+ 2a a a a
y
    D = 0 x 0 0
0 0 y 0
0 0 0 y
x 2 2 2 2 2
= 2a+ x + 2 a x y = 2 ax y + x y + 2 ax y.
y
法 2: 将 D 中 a 均表示为 a + 0 , 于是 D 中每个元 素都是两 数
之和 , 这样的行列式按线性性质可将其表示为 24 = 16 个行列式 之
和 , 但其中有 11 个 行列 式等 于 0 ( 它们 两 列或 三列 或四 列 相同 ) ,
另 5 个行列式也很好计算 . 即
a+ x a+0 a+ 0 a+ 0 a 0 0 0
a+ 0 a+ x a+0 a+ 0 a x 0 0
= +
a+ 0 a+ 0 a+ y a+0 a 0 y 0
a+ 0 a+ 0 a+ y a+ y a 0 0 y
x a 0 0 x 0 a 0
0 a 0 0 0 x a 0
            + +
0 a y 0 0 0 a 0
0 a 0 y 0 0 a y
x 0 0 a x 0 0 0
0 x 0 a 0 x 0 0
            +
0 0 y a 0 0 y 0
0 0 0 a 0 0 0 y
10 第 1 章 行列式

= ax y2 + ax y2 + ax2 y + ax2 y + x2 y2 = 2 ax y2 + 2 ax2 y + x2 y2 .


其中等号 右 端 第 2 , 3 个 行 列 式 分 别 对 第 1 , 4 列 展 开 即 得 ax y2
2
和 ax y.
0 a1 b1 0
a2 0 0 b2
例 8   计算 D = .
a3 0 0 b3
x a4 b4 y
解   此题可通过行、列对换化为
A *
= |A|| B|   ( 其中|A|, |B|均为二阶行列式 ).
0 B
先将第 2 列与第 1 列对换 , 再将第 3 列与第 2 列对换 , 得
a1 b1 0 0
0 0 a2 b2
D = ( - 1) ( - 1 )
0 0 a3 b3
a4 b4 x y
a1 b1 0 0
②\ ④ a4 b4 x y
-
0 0 a3 b3
0 0 a2 b2
a1 b1 a3 b3
= - = ( a1 b4 - a4 b1 ) ( a2 b3 - a3 b2 ).
a4 b4 a2 b2
1 x x2 x3 x4
2 3 4
1 x+ 1 ( x + 1) ( x + 1) ( x + 1)
2 3 4
例9   计算 D5 = 1 x+ 2 ( x + 2) ( x + 2) ( x + 2) .
2 3 4
1 x+ 3 ( x + 3) ( x + 3) ( x + 3)
2 3 4
1 x+ 4 ( x + 4) ( x + 4) ( x + 4)
解   由于行列式的行与列按顺序互换 , 其值不变 , 所以这也是
1. 2 行列式的计算 ( 展开 ) 11

主教材 1. 2 节中例 8 的范德蒙行列式 . 直接利用其结论 , 得


D5 = [ ( x + 1 - x ) ( x + 2 - x) ( x + 3 - x) ( x + 4 - x ) ]・
[ ( x + 2 - x - 1) ( x + 3 - x - 1) ( x + 4 - x - 1 ) ] ・
[ ( x + 3 - x - 2) ( x + 4 - x - 2) ] ( x + 4 - x - 3)
  = (4 !) (3 !) (2 !) ×1 = 288.
x- 1 1 1 0
1 x-1 1 0
例 10   求方程 = 0 的根 .
1 1 x-2 1
1 1 1 x- 2
解   由观 察 , 当 x = 2 时 , 行列 式第 1 , 2 行 相 同 , 行 列式 等 于
0; 当 x = 3 时 , 行 列式第 3 , 4 行 相同 , 行列 式 也等 于 0 ; 当 x = - 1
时 , 行列式每行元素 之和均为 0 , 把各 列加到第 1 列 则第 1 列元 素
均为 0 , 从而行列式也等于零 . 所以 x = 2 , x = 3 与 x = - 1 都是方
程的根 . 但这是 x 的四次方程 , 现在不能断言方程只有这 3 个根 ,
必须展开此行列式 . 将它记作 D, 将 第 3 行乘 2 - x 加到第 4 行 ,
然后对第 4 列展开 , 得
x-1 1 1 0
1 x-1 1 0
D = ( 对第 4 列展开 )
1 1 x-2 1
3- x 3- x 1 - ( x - 2) 2 0
x-1 1 1
= - 1 x-1 1
3- x 3- x (1 - x ) ( x - 3)
x-1 2- x 1
[ 2] + [1 ] × ( - 1 )
- 1 x- 2 1
3 - x 0 ( 1 - x) ( x - 3 )
12 第 1 章 行列式

x 0 2
( 1 ) + (2 )
- 1 x- 2 1
对第 2 列展 开
3- x 0 ( 1 - x ) ( x - 3)
x 2
= - ( x - 2)
3 - x (1 - x ) ( x - 3 )
= - ( x - 2 ) [ x (1 - x ) ( x - 3) - 2( 3 - x ) ]
= - ( x - 2 ) ( x - 3) [ x( 1 - x) + 2]
= ( x - 2 ) 2 ( x - 3) ( x + 1 ) = 0 ,
所以方程的根为 x = 2 , 3 , - 1( 2 是二重根 ).
例 11   证明恒等式
1 1 1 1 1 1
2 2 2
x y z = ( x y + y z + z x) x y z .
x3 y3 z3 x2 y2 z2
证   证明恒等式的一般方法为 : 一是从左推出右 , 或从右推出
左 ; 二是等号两端分别展开 , 得到同样的结果 . 这里采用后者较为
方便 .
等号右端的行列式是范德蒙行列式 , 直接利用公式可得
右端 = ( x y + y z + z x ) ( y - x) ( z - x) ( z - y). ( 1)
等号左端的行列式不是 范德蒙 行列 式 , 如果直 接 按定 义或 按
沙路法展开 , 其因式分解较麻烦 , 所以也用展开范德蒙行列式的方
法 , 将第 1 列化为只剩一个非零元 1 , 然后对第 1 列展开 , 即
1 1 1 1 1 1
③ + ② ×( - x)
x2 y2 z2 0 y2 - x2 z2 - x2
② + ①× ( - x ) 2
3 3 3
x y z 0 y3 - x y2 z3 - x z2
( y - x) ( y + x ) ( z - x) ( z + x)
=
( y - x ) y2 ( z - x) z2
y+ x z+ x
= ( y - x) ( z - x )
y2 z2
1. 2 行列式的计算 ( 展开 ) 13

= ( y - x) ( z - x ) [ ( y + x ) z2 - ( z + x ) y2 ]
= ( y - x) ( z - x ) [ z y ( z - y) + x ( z2 - y2 ) ]
= ( y - x) ( z - x ) ( z - y ) ( z y + x z + x y). ( 2)
这里 (2 ) 式 = ( 1) 式 , 所以恒等式成立 .
例 12   证明 : n 阶三对角行列式
α+β α
β
1 α+β α
β
1 α+β   w
Dn =
w w  w
w α+β αβ
1 α+β
( n + 1 )αn ,     当 α=β;
= βn + 1 - αn + 1
, 当 α≠β.
β- α
证   采用的 证 法 为 : 对 Dn 的 第 1 行 展 开 , 从 中 找 到 Dn 与
D n - 1 及 Dn - 2 之间的关系 ( 称为递推公式 ) , 然 后递推得 到要证的 结
果. 即

1 αβ
0 α+β αβ
1 α+β  w
Dn = (α+ β) Dn - 1 - αβ .
w   w  w
w   α+β αβ
1 α+ β n - 1 阶

将上式右端的 n - 1 阶行列式对第 1 列展开得 Dn - 2 , 所以

Dn = (α+β ) Dn - 1 - α
βDn - 2 . ( 1)
14 第 1 章 行列式

把这个递推公式 (1 ) 改写成
Dn - αDn - 1 = β( Dn - 1 - αDn - 2 ). ( 2)
记 An = Dn - αDn - 1 , ( 3)
则 Dn - 1 - αDn - 2 = An - 1 , 于是 ( 2) 式即为
An =βAn - 1 ( 4)
递推公式 (4 ) 中的 n 可为任何正 整数 , 也 就是 当 n 取为 n - 1 时 也
成立 , 即
An - 1 = βA n - 2 .
如此 , 将 ( 4) 式继续递推下去 , 即得
2 n- 2
An =βAn - 1 =β An - 2 = … =β A2 , ( 5)
由 (3 ) 式知 , 其中
A2 = D2 - αD1 .

α+ β αβ
D2 = = (α+β) 2 - αβ,
1 α+ β
D1 = |α+β| =α+β  ( 一阶行列式 ) ,
2 2
于是 A2 = (α+β) - αβ- α(α+β) =β . ( 6)
将 (6 ) 式代入 ( 5) 式 , 得
n
An = β ,
n
即 Dn - αDn - 1 =β. ( 7)
将 (7 ) 式中的 n 分别取为 n , n - 1 , n - 2 , … , 3 , 2 , 得到
n
Dn - αDn - 1 =β ,
n- 1
Dn - 1 - αDn - 2 =β ,
n- 2
Dn - 2 - αDn - 3 =β ,
( 8)
  …………
3
D3 - αD2 = β ,
D2 - αD1 = β2 .
1. 2 行列式的计算 ( 展开 ) 15

将 (8 ) 式中 n - 1 个 等式两 端依次用 1 ,α,α2 , … ,αn - 3 ,αn - 2 相 乘 , 然


后再将这些等式相加 , 即得
  Dn - αn - 1 D1 = βn + α
βn - 1 + α2 βn - 2 + … + αn - 3β3 + αn - 2 β2 . (9)
再把 D1 =α+β代入 ( 9) 式 , 并移项 , 得
n n- 1 2 n- 2 n- 2 2 n- 1 n
Dn = β +αβ +αβ + … +α β +α β+ α.
当 β=α时 , Dn = ( n + 1)αn ;
当 β≠α时 , 利用
n+1 n+ 1 n n- 1 2 n- 2 n - 1 n
β -α = (β- α) (β +αβ +αβ + … +α β+α ) ,
即得
n+1 n+ 1
-α β
Dn = .
β- α
此题的另一证法 : 把 Dn ( 不失一般性 , 取 n = 4 ) 表示为
α+β αβ+ 0 0 + 0 0+0
0+1 α+β αβ+ 0 0+0
D4 = . (10)
0+0 0+1 α+ β α
β+ 0
0+0 0+0 0 + 1 α+β
4
再利用行列式的线性性质② , 将 D4 表示为 2 = 16 个行列 式之 和
( 如例 7 那样 ) , 这 16 个行列式中的每一列都是行列式 (10) 中该列
的两个子列之一 . 容易看 出 , 每一列 的第 二子 列与下 一列 的第 一
4
子列成比例 , 因此 , 在 2 个行 列式 中只 有 4 + 1 = 5 个 行列 式不 为
0, 即
α αβ 0 0 α α
β 0 0
0 α αβ 0 0 α αβ 0
D4 = + +
0 0 α αβ 0 0 α 0
0 0 0 α 0 0 0 β
α αβ 0 0 α 0 0 0 β 0 0 0
0 α 0 0 0 β 0 0 1 β 0 0
+ + .
0 0 β 0 0 1 β 0 0 1 β 0
0 0 1 β 0 0 1 β 0 0 1 β
16 第 1 章 行列式

所以
5α4 ,     β= α;
4 3 2 2 3 4
D4 = α + αβ+ αβ + α
β + β = β5 - α5
,   β≠α.
β- α
例 13 ( 补充题 39 )   计算 n 阶行列式
cosθ 1
1 2cosθ 1
Dn = w w w .
1 2cosθ 1
1 2cosθ
解   注意 , 这不是三对角行列式 , 因为这里主对角线上的元素
不完全相同 .
展开这种 n 阶行列式 , 一般 都要 设法找 到一 个递 推公 式 . 这
里由于主对角线上第 1 个元 素为 cosθ, 而 其余 元素 均为 2cosθ, 因
此要找 Dn 与 D n - 1 及 Dn - 2 之 间的 关 系 , 不 能对 Dn 的第 1 行 ( 或
列 ) 展开 , 而要对第 n 行 ( 或列 ) 展开 . 现在将 Dn 按第 n 列展开 , 得
cosθ 1
 1 2cosθ 1
Dn = (2cosθ) Dn - 1 - 1・ w   w w .
 1  2cosθ  1
0 1 n - 1阶

再将上式中的 n - 1 阶行列式按第 n - 1 行展开 , 即得


Dn = ( 2cosθ) Dn - 1 - Dn - 2 . ( 1)
由递推公式 (1 ) 还难 以 递推 出 Dn 的结 果 . 这里 要 用数 学 归
纳法 , 首先易见
D1 = | cosθ| = cosθ,
cosθ 1 2 2 2
D2 = = 2cos θ- 1 = cos θ- sin θ= cos2θ,
1 2cosθ
1. 2 行列式的计算 ( 展开 ) 17

cosθ 1 0
3
D3 = 1 2cosθ 1 = 4cos θ- 3cosθ= cos3θ
. ( 2)
0 1 2cosθ
关于 (2 ) 式的结论 , 读 者 不 难由 cos3θ= cos ( 2θ+ θ) = cos2θcosθ-
3
sin2θsinθ推出 cos3θ= 4cos θ- 3cosθ

于是归纳出 ( 或猜想 ) Dn = cos nθ, 下面用数学归纳法证明它 是
正确的 .
当 n = 1 时 , 结论成立 , 假设结论对小于 n 阶的行列式都成立 ,
那么对 n 阶的情况 , 由递推公式 (1 ) 得

Dn = (2cosθ) cos ( n - 1 )θ- cos ( n - 2)θ ( 3)
1
利用 cosαcosβ= [ cos(α+ β) + cos (α- β) ] , 得
2
1
Dn = 2× ( cos nθ+ cos( n - 2 )θ) - cos( n - 2)θ= cos nθ

2
所以归纳的结论 Dn = cos nθ对任意正整数 n 都成立 .
读者应该从上面的例题 中 , 概 括出 展开 行列式 的 基本 方法 和
一些技巧 , 提高自己的解题能力 , 并在自己解题的实践中进一步总
结解题的规律和技巧 .
(1 ) 展开行列式最 基本 的方 法是 按定 义 展开 ( 即按 第 1 行 展
开 ). 如 上 ( 下 ) 三角行 列式 , 对 角行 列式 , 副对 角行 列式等 都是 用
定义展开的 . 一般的三阶数字行列式也常用定义或沙路法直接展
开 . 但是对于一般的 n 阶 ( n > 3 ) 行列 式按 定 义展 开 , 需要 计 算 n
个 n - 1 阶行列式 , 工作量较大 . 尤其是一些有规律的文字行列式
直接按定义展开 , 难以发现它们的规律 ( 例如难以因式分解 ).
(2 ) 利用性质 , 将行列 式化 为 上 ( 下 ) 三角 行列 式 是展 开行 列
式最常用的方法 . 如例 3 , 例 6 , 例 7 , 以及例 4 与例 5.
(3 ) 利用性质 , 将行列式某行 ( 或列 ) 化为只 剩一个非 零元 , 然
后对该行 ( 或列 ) 展 开 , 使 n 阶 行列 式 化为 n - 1 阶行 列 式来 展 开
18 第 1 章 行列式

( 称为降阶展开法 ) , 也是常用的展开法 . 如例 2 , 例 10 , 例 11.


(4 ) 如果行列式每 行 ( 或 列 ) 元 素 之和 均相 等 , 则 展开 的第 一
步是将各列加到第 1 列 ( 或 各行加 到第 1 行 ) , 然 后再 用前面 ( 2 ) ,
(3 ) 的方法展开 , 如例 4 , 例 5.
(5 ) 如果 n 阶行列式中每个元素均为两数 ( 一 般带有文 字 ) 之
n
和 , 则 可利 用 线性 性 质 , 将 其 化为 2 个 行列 式 之和 , 在 有些 情 况
下 , 这 2 n 个行列式很多都 等于 零 , 那 些不 等于 零的 行 列式 也很 容
易展开 , 如例 7 与例 12 的第 2 种解法 .
(6 ) 一些 n 阶文字行列式 , 常需要找到它的 递推公式 , 然后 利
用递推公式递推出它的结果 , 如果难以递推出结果 , 还需要善于归
纳出它的结果 , 再用数学归纳法证明之 . 如例 12 , 例 13.
(7 ) 要会识别范德蒙行列式 , 然后直接得展开结果 . 如例 9.
(8 ) 要会利用 : 当 | A| , | B| 分别为 m 阶和 n 阶行列式时 , 有
A 0 A *
= = |A||B|. ( 1)
* B 0 B
如例 8 , 再如对于
0 0 0 1 2
0 0 0 3 4
D5 = -1 0 0 1 2 ,
0 - 2 0 1 2
0 0 -3 1 2
如果令
- 1 0 0 1 2
1 2
| A| = ,   | B| = 0 -2 0 ,   C= 1 2 ,
3 4
0 0 - 3 1 2
其中 C 不是行列式 , 则
0 A
D5 = . ( 2)
B C
1. 3 克拉默法则 19

注意 : 这 不是 (1 ) 式的情 形 , 也不能如 二阶数 字行列式 那样展开 为


D5 = - | A| | B|.
这时 , 应先 将 (2 ) 式中 A 第 1 列所 在的 列与 B 所 在的 3 个 列
逐列对换 3 次 , 然后将 A 第 2 列所 在的 列与 对换后 的 B 所 在的 3
个列再逐列对换 3 次 , 得
0 A 2 ×3 A 0
D5 = = ( - 1) = | A| | B|
B C C B
= (4 - 6) ( - 1 ) 3 3 ! = 12.
具体问题具体分析是辩证法的灵魂 , 各种各样行列式的展开 ,
既有共性又有个性 , 除了以上 8 条 , 读者还会在学习中来丰富自己
的认识 . 在 学了矩 阵以后 , 还会 增长计算 n 阶矩阵的 行列式的 一
些知识 .

1. 3 克拉默法则

克拉默法则 ( 见内容 提要 ( 5) ) 是 求系 数行 列式不 等于 零的 线


性方程组的解的一种方法 . 克拉默法则清楚地揭示了这类 n 元线
性方程组的 n 个未知元 x1 , x2 , … , xn 的解与它们的系数及常数 项
b1 , b2 , … , bn 之 间 的 关系 . 但 是求 解 时要 计 算 n + 1 个 n 阶 行 列
式 , 其工作量较大 . 实际 上求 这类线 性方 程组 的解常 用的 方法 是
第 2 章中介绍的高斯消元 法 , 或通过 求系 数矩 阵的逆 矩阵 来求 其
解 ( 详见第 2 章 2. 2 节与 2. 5 节中 4). 所以克 拉默法则 主要是 其
理论意义 . 当然 , 对于三元线性方程组求解还是很方便的 , 因为三
阶行列式展开很容易 .
例 1   求解下列三元线性方程组
x1 - x2 + 2 x3 = 6 ,
2 x1 - x2 - x3 = 5 ,
x1 + x2 - 2 x3 = - 2.
20 第 1 章 行列式

解   系数行列式
1 -1 2
D= 2 -1 - 1 = 2 + 4 + 1 + 2 + 1 - 4 = 6≠0.
1 1 -2
用克拉默法则求方程组的解 . 因
6 -1 2 1 6 2
D1 = 5 -1 - 1 = 12 ,   D2 = 2 5 - 1 = - 12 ,
-2 1 -2 1 -2 - 2
1 -1 6
D3 = 2 -1 5 = 6,
1 1 -2
所以
D1 D2 D3
x1 = = 2 ,   x2 = = - 2 ,   x3 = = 1.
D D D
例 2   已知 某圆 过点 A( 2 , 1 ) , B( 3 , 4 ) , P( - 2 , - 1 ) , 试 求 其
方程及圆心和半径 .
解   此题用平面解析几何的方法容易求解 . 这里用待定系数
法来求圆的方程 . 设圆的一般方程为
2 2
x + y + a x + by + c = 0 ,
点 A , B , P 在圆上 , 它们的坐标满足方程 , 得
4 + 1 + 2 a+ b+ c= 0 , 2 a+ b+ c= - 5 ,
9 + 16 + 3 a + 4 b + c = 0 ,   即   3 a + 4b + c = - 25 ,
4 + 1 - 2 a - b + c = 0. 2 a + b - c = 5.
系数行列式
2 1 1
D= 3 4 1 = - 8 + 3 + 2 - 8 - 2 + 3 = - 10 ,
2 1 -1

1. 4 部分疑难习题和补充题的题解 21

-5 1 1 2 -5 1
D1 = - 25 4 1 = - 40 ,   D2 = 3 - 25 1 = 80 ,
5 1 -1 2 5 - 1
2 1 -5
D3 = 3 4 - 25 = 50 ,
2 1 5
所以
D1 D2 D3
a= = 4,   b= = - 8,   c= = - 5.
D D D
圆的一般方程和标准方程分别为
2 2
x + y + 4 x - 8 y - 5 = 0,
2 2
( x + 2) + ( y - 4) = 25.
该圆的圆心 O( - 2 , 4) , 圆的半径 r = 5.

1. 4 部分疑难习题和补充题的题解

1 ω ω2
1 3
1 ( 习题 7 )   计算 D3 = ω2 1 ω , 其中 ω= - +i .
2 2
ω ω2 1
解   法 1 : D3 中各行元素之和均为 1 + ω+ ω2 , 所以把第 2 , 3 列加到第 1
列 , 然后再把第 1 列后两个元素化为零 , 再对第 1 列展开 , 即
1 ω ω2
          D3 = ( 1 + ω+ ω2 ) 1 1 ω
1 ω2 1
1 ω ω2
② - ①
( 1 + ω+ ω2 ) 0 1 - ω ω- ω2
③ - ①
0 ω2 - ω 1 - ω2
1 -ω ω(1 - ω)
= ( 1 + ω+ ω2 )
- ω(1 - ω) 1 - ω2
22 第 1 章 行列式

1 ω
= ( 1 + ω+ ω2 ) (1 - ω) 2
- ω 1+ω
= ( 1 + ω+ ω2 )2 ( 1 - ω) 2 = ( 1 - ω3 ) 2 .

1 3
由于 ω= - +i 是方程 ω2 + ω+ 1 = 0 的根 , 所以 D3 = 0.
2 2
法 2:
1 ω ω2
② + ①× ( - ω ) 2

D3 0 1 - ω3 ω- ω4 = ( 1 - ω3 ) 2
③ + ① × ( - ω)
0 0 1 - ω3
= (1 - ω)2 (1 + ω+ ω2 ) 2 = 0.
此题法 2 比法 1 还简便 , 所以各行 元素和 相等时 , 并不 一定都 要把各 列
加到第 1 列后再展开 .
3 6 5 6 4
2 5 4 5 3
2 ( 习题 14)   计算 D5 = 3 6 3 4 2 .
2 5 4 6 5
1 1 1 -1 - 1
解   先将第 1 行与第 5 行对换 , 第 3 行与第 4 行对换( 反号两次 , 其 值不
变).
1 1 1 -1 - 1
2 5 4 5 3
    D5 = 2 5 4 6 5
3 6 3 4 2
3 6 5 6 4
1 1 1 -1 - 1
② + ①× ( - 2) 0 3 2 7 5
③ + ①× ( - 2)
0 3 2 8 7
④ + ①× ( - 3)
⑤ + ①× ( - 3) 0 3 0 7 5
0 3 2 9 7
1. 4 部分疑难习题和补充题的题解 23

1 1 1 -1 - 1
0 3 2 7 5
③ + ②× ( - 1)
0 0 0 1 2
④ + ②× ( - 1)
⑤ + ②× ( - 1) 0 0 - 2 0 0
0 0 0 2 2
1 1 1 -1 -1
0 3 2 7 5
⑤ + ③× ( - 2)
- 0 0 -2 0 0 = - 3×( - 2)2 = - 12.
④\ ③
0 0 0 1 2
0 0 0 0 -2
1 0 2 a
2 0 b 0
3 ( 习题 23)   计算 .
3 c 4 5
d 0 0 0
A *
解   提示 : 用行列对换将其化为 = |A||B|, 答案为 abc d.
0 B
a 1 0 0
- 1 b 1 0
4 ( 习题 24)   计算 .
0 -1 c 1
0 0 - 1 d
解   这里 a, b, c, d 不相 同 , 所以 不是三对角 行列式 . 此时 , 可 利用性 质
将第 1 列化为只剩一个非零元 , 然后 按第 1 列 展开 . 但要 注意 , 不 要将 第 1
1
行乘 加到第 2 行 , 这样无形中假定了 a≠0 , 对于 a = 0 的情况 , 还要另 加讨
a
论 . 所以这里应是第 2 行乘 a 加到第 1 行 , 即
0 1 + ab a 0
1 + ab a 0
- 1 b 1 0
原式 = = - ( - 1) - 1 c 1
0 - 1 c 1
0 - 1 d
0 0 -1 d
1 + ab a ad
[ 3 ] + [2 ] × d
-1 c 1 + c d (按第 3 行展开 )
0 - 1 0
24 第 1 章 行列式

1 + ab ad
= - ( - 1) = (1 + ab) ( 1 + cd) + ad.
- 1 1 + cd
此题用了两次“降阶展开法”, 把四阶行列式化成二阶行列式展开 . 如果
这里也追求化为上三角行列式展开 , 就比较麻烦 .
a2 ( a + 1 )2 ( a + 2 )2 ( a + 3 )2
b2 ( b+ 1) 2 ( b+ 2) 2 ( b + 3) 2
5 ( 习题 25)   计算 .
c2 ( c + 1) 2 ( c + 2) 2 ( c + 3) 2
d2 ( d + 1) 2 ( d + 2) 2 ( d + 3) 2
解   提示 : 利用 x2 - y2 = ( x - y ) ( x + y) , 将第 4 列加第 3 列乘 - 1 , 第 3
列加第 2 列乘 - 1 , 第 2 列加第 1 列乘 - 1 ;再将变换后的第 4 列减第 3 列 , 第
3 列减第 2 列 . 如此第 3 , 4 列元素全为 2 , 所以行列式等于零 .
6 ( 补充题 37)   证明
x - 1 0 … 0 0
0 x - 1 … 0 0
0 0 x … 0 0
            Dn =
… … … w … …
0 0 0 … x - 1
an an - 1 an - 2 … a2 x + a1
n

∑ax
n n- k
= x + k .
k= 1

证   从这个文字行列式中 a1 , a2 , … , an 分 别在 n 个 列可见 其为 n 阶 行
列式 . 证明这样的恒等式 , 一般是先找到 Dn 的一个递推 公式 ( 即 Dn 与 比它
阶数低的同类型行列式的关系) , 然 后递推 出结果 , 如 递推困 难 , 再 用数学 归
纳法证明 . 这里先把 Dn 按第 1 列展开 , 得
Dn = xDn - 1 + ( - 1) n+ 1 an ( - 1 ) n - 1 = x Dn - 1 + an. ( 1)
在( 1) 式中 n 可为任何正 整数 , 将 n 换为 n - 1 , 就 得 Dn - 1 = x Dn - 2 + an - 1 , 如
此由( 1) 式递推 , 可得
Dn = x Dn - 1 + an = x( x Dn - 2 + an - 1 ) + an
= x2 Dn - 2 + an - 1 x + an
= x2 ( x Dn - 3 + an - 2 ) + an - 1 x + an
= x3 Dn - 3 + an - 2 x2 + an - 1 x + an
1. 4 部分疑难习题和补充题的题解 25

=…
= x n - 2 D2 + a3 x n - 3 + … + an - 2 x2 + an - 1 x + an . ( 2)
x - 1
注意 , 其中 D2 = = x2 + a1 x + a2 , 代入 ( 2)式 , 得
a2 x + a1
Dn = x n + a1 x n - 1 + a2 xn - 2 + a3 xn - 3 + … + an - 2 x2 + an - 1 x + an ,

n

∑ax   ( 其中 x0 = 1) .
n n- k
Dn = x + k
k= 1

这里也可利用递推公式 (1) , 按数学归纳法证明之 .


当 n = 2 时 , D2 = x D1 + a2 = x ( x + a1 ) + a2 = x2 + a1 x + a2 , 结论成立 , 假
设 n - 1 阶也成立 , 即
n- 1

Dn - 1 = x n - 1 + ∑ax
n- 1 - k
k ,
k= 1

则对于 n 阶有
n- 1

∑ax
n- 1 n- 1 - k
Dn = x Dn - 1 + an = x x + k + an
k= 1

n- 1 n

= xn + ∑ ak x n - k + an = xn + ∑ax
n- k
k ,
k= 1 k =1

故结论对任意正整数 n 都成立 .
1 5 2 3
  -  
3 2 5 2
21
 3 - 12   15
5
7 ( 补充题 30)   计算 D4 = .
2 9 4 5
  -  
3 2 5 2
1 2 1 3
-   -
7 7 7 7
解   第 1 列依次乘 2 , - 1 , 3 分别加到第 2 , 3 , 4 列 , 可将第 4 行化为 只剩
一个非零元 - 1/ 7 , 然后可按第 4 行 展开 .但这 样要作 很多分 数运算 , 比较 麻
烦 .所以先把第 1 , 2 , 3 , 4 行分别提出公因子 1/ 30 , 3/ 5 , 1/ 30 , 1/ 7 , 使行列式中
元素皆为整数 , 然后再作前面所说的列变换 , 即
26 第 1 章 行列式

10 - 75 12 45

1 3 1 1 5 - 20 7 25
D4 = × × ×
30 5 30 7 20 - 135 24 75
- 1 2 -1 3
10 - 55 2 75

1 5 - 10 2 40
=
35×300 20 - 95 4 135
-1 0 0 0
- 55 2 75
- 1
= ( - 1) - 10 2 40
35×300
- 95 4 135
- 55 2 75
② - ① 1
45 0 - 35
③ + ① ×( - 2 ) 35×300
15 0 - 15
- 2 45 - 35 - 2 1
= = ×15( - 45 + 35) = .
35×300 15 - 15 35×300 35

1 1 … 1 - n
1 1 … - n 1
8 ( 补充题 41)   计算 Dn = … … Y … … .
1 - n … 1 1
- n 1 … 1 1
解   各行元素和皆为 - 1 , 把各列加到第 1 列 , 再将第 1 行乘 - 1 加到其
余各行 , 得
1 1 … 1 - n
0 0 … - n- 1 1+n
Dn = ( - 1) … … Y … …   (按第 1 列展开 )
0 - n- 1 … 0 1+n
0 0 … 0 1+n
1. 4 部分疑难习题和补充题的题解 27

0 … - n- 1 1+n
… Y … …
  = ( - 1)   (按第 n - 1 行展开)
- n- 1 … 0 1+n
0 … 0 1+n n - 1阶

( n - 2) ( n - 3 )
  = ( - 1) ( n + 1 ) ( - 1 ) 2 ( - n - 1) n - 2
1 + ( n - 2 ) ( n - 3 ) + ( n - 2)
      = ( - 1) 2 ( n+ 1 )n - 1
n( n - 3 ) n( n+ 1 )
  = ( - 1) 2 ( n + 1) n - 1 = ( - 1 ) 2 ( n+ 1 )n - 1 .
1 2 3 … n- 1 n
2 3 4 … n 1
3 4 5 … 1 2
9 (补充题 43)   计算 Dn = .
… … … … …
n- 1 n 1 … n- 3 n- 2
n 1 2 … n- 2 n- 1
n( n + 1)
解   各行元素之和均为 1 + 2 + 3 + … + ( n - 1) + n = , 把各 列元
2
n( n + 1 )
素加到第 1 列 , 提出公因子 , 并从最后一行起 , 依次减前一行 , 一直做
2
到第 2 行减第 1 行为止( 共做 n - 1次) , 即得
1 2 3 … n- 1 n
0 1 1 … 1 1 - n

n( n + 1 ) 0 1 1 … 1- n 1
Dn = .
2 … … … Y … …
0 1 1 - n … 1 1
0 1 - n 1 … 1 1
将上式按第 1 列展开 , 并 将 其“1”的 余 子式 的第 1 行乘 - 1 加 到 其余 各行 ,
即得
1 1 … 1 1 - n
0 0 … - n n
n( n + 1)
Dn = … … Y … … .
2
0 - n … 0 n
- n 0 … 0 n n - 1阶
28 第 1 章 行列式

再将上式的各列加到第 1 列 , 并对第 1 列展开 , 即得


- 1 1 … 1 1 - n
0 0 … - n n
n( n + 1 )
Dn = … … Y … …
2
0 - n … 0 n
0 0 … 0 n
0 … - n n
n( n + 1 ) … Y … …
= ( - 1)
2 - n … 0 n
0 … 0 n n - 2阶

将上面的 n - 2 阶行列式按最后一行展开 , 得
n( n + 1 ) ( n - 3 )( n - 4 ) n( n - 1 ) n + 1 n - 1
Dn = ( - 1) n( - 1 ) 2 ( - n) n - 3 = ( - 1 ) 2 n .
2 2
1 1 1 … 1
x1 x2 x3 … xn
x21 x22 x23 … x2n
10 (补充题 44)   证明 Dn =
… … … …
x1n - 2 x2n - 2 x3n - 2 … xnn - 2
x1n x2n xn3 … xnn
n

= ∑
i= 1
xi ∏
1≤ j< i≤ n
( xi - x j ) . ( 1)

证   法 1 : 用证明范德蒙行列式的方法 证明之 .先将第 n 行加第 n - 1 行


乘 - x21 , 然后依次从第 n - 1 行直到第 2 行 , 都将前一行乘 - x1 加到后一行 , 得
1 1 1 … 1
0 x2 - x1 x3 - x1 … x n - x1
0 x22 - x2 x1 x23 - x3 x1 … x2n - xn x 1
Dn =
… … … …
0 x2n - 2 - x2n - 3 x1 x3n - 2 - xn3 - 3 x1 … xnn - 2 - xnn - 3 x 1
0 x2n - x2n - 2 x21 x3n - x3n - 2 x21 … xnn - xnn - 2 x21
1. 4 部分疑难习题和补充题的题解 29

1 1  … 1
x2 x3  … xn
n

= ∏( x
i= 2
i - x1 ) … … … .
x 2n - 3 x 3n - 3  … x nn - 3
x 2n - 2 ( x2 + x1 )   x n3 - 2 ( x3 + x1 )   …   x nn - 2 ( xn + x1 ) n- 1 阶

将上式中最 后一 行改 写为 : xn2 - 1 + x1 x2n - 2 , x3n - 1 + x1 x3n - 2 , … , xnn - 1 + x1 x nn - 2 ,


则上面的行列式可以化为两个行列式之和 , 即
1 1 … 1 1 1 … 1
x2 x3 … xn x2 x3 … xn
n

Dn = ∏( x
i= 2
i - x1 ) … … … + x1 … … … .
n-3 n- 3 n -3 n -3 n- 3 n -3
x2 x3 … x n x 2 x3 … xn

x2n - 1 x3n - 1 … xnn - 1 x2n - 2 x3n - 2 … xnn - 2

上面第一个行列式是本题类型的 n - 1 阶行列式 , 可记作 Dn - 1 , 但 其 n - 1 个


元为 x2 , x3 , … , xn ;第二个行列式是由 x2 , x3 , … , xn 为元的 n - 1 阶范德蒙行
列式 , 于是
n

Dn = ∏( x
i= 2
i - x1 ) Dn - 1 + x1 ∏
2≤ j < i≤ n
( xi - xj ) . ( 2)

下面用数学归纳法证明 (1) 式成立 .当 n = 3 时 , 由 ( 2) 式得

1 1
D3 = ( x2 - x1 ) ( x3 - x1 ) + x1 ( x3 - x2 )
x22 x23
= ( x2 - x1 ) ( x3 - x1 ) ( x23 - x22 ) + x1 ( x3 - x2 )
= ( x2 - x1 ) ( x3 - x1 ) ( x3 - x2 ) x1 + x2 + x3
3

= ∑
i= 1
xi ∏
1≤ j< i≤3
( xi - x j ) .

所以( 1) 式对 n = 3 是成立的 .归纳假设 ( 1 ) 式对 n - 1 阶行列 式成立 , 下面 利


用递推公式 ( 2)证明 ( 1)式对 n 阶行列式也成立 .由 (2 )式和归纳假设 , 得
30 第 1 章 行列式

n n

Dn = ∏( x
i=2
i - x1 ) ∑xi =2
i ∏
2≤ j< i≤ n
( x i - x j ) + x1 ∏ (x
2 ≤ j≤ n
i - xj )

n n

= ∏( x
i=2
i - x1 ) ∑x
i=1
i ∏
2≤ j< i≤ n
( xi - xj )

= ∑x
i=1
i

1≤ j< i≤ n
( x i - x j ) = (1 ) 式右端 .

所以( 1) 式对任意的正整数 n 都成立 .


法 2 : 这是一个技巧性比较强的解法 .由 于该行 列式每 一列最 后两个 元
素为 xnj - 2 , x nj , 从而不是范德蒙行列式 .但是 , 可以将该行列式添一行 , 并加一
列 , 使之成为 n + 1 阶范德蒙行列式 , 即
1 1 1 … 1 1
x1 x2 x3 … xn y
x21 x22 x23 … x2n y2
    V n+ 1 = … … … … … ( 3)
x1n - 2 x2n - 2 x3n - 2 … xnn - 2 y n- 2
x1n - 1 x2n - 1 x3n - 1 … xnn - 1 y n- 1
x1n x2n x3n … x nn yn

= ( y - x1 ) ( y - x2 ) ( y - x3 )… ( y - xn ) ∏
1 ≤ j< i≤ n
( x i - x j ) . ( 4)

由( 3) 式可见 , 将 ( 3 ) 式 按最 后一 列 展开 , 其 y n - 1 的 系 数 就是 原 行 列式 的 值
乘 - 1 ;由 (4 )式又可见 , yn - 1 的系数为

- ( x1 + x2 + x3 + … + xn ) ∏
1 ≤ j< i≤ n
( xi - xj ) ,

所以原行列式 Dn 的值为

Dn = ( x1 + x2 + … + xn ) ∏
1 ≤ j < i≤ n
( xi - xj )

= ∑i= 1
xi ∏
1≤ j< i≤ n
( xi - xj ) .

11 ( 补充题 45 )   证明 (用数学归纳法 )导数关系式


1. 4 部分疑难习题和补充题的题解 31

d
a11 ( t) … a1 j ( t) … a1n ( t)
dt
a11 ( t) a12 ( t) … a1n ( t)
d
d a21 ( t) a22 ( t) … a2n ( t) n
a21 ( t) … a ( t) … a2n ( t)
dt 2j
dt … … …
= ∑ j= 1
.
… … …
an1 ( t) an2 ( t) … ann ( t)
d
an1 ( t) … anj ( t) … ann ( t)
dt
(1)
d n
证   下面将 aij ( t) 记作 aij ; a ij ( t) 记作 a′
ij ;行 列式 | aij ( t) | 1 记 作 D; 导
dt
d
数 D 记作 D′.
dt
对行列式的阶数 n 作数学归纳法证明 .当 n = 1 时 , (1 )式显然成立 , 假设
(1) 式对 n - 1 阶行列式成立 , 下面证明( 1) 式对 n 阶也成立 .由于
D = a11 A11 + a21 A21 + … + an1 A n1 ,
故 D′= a′ 2 1 A21 + … + a′
1 1 A11 + a′ n1 An1 +

a11 A′ 2 1 + … + an1 A′
11 + a21 A′ n1 , ( 2)
其中
a′
11 a12 … a1 n
a′
21 a22 … a2 n
a′ 2 1 A21 + … + a′
1 1 A11 + a′ n1 An1 = , ( 3)
… … …
a′
n1 an2 … ann
  a11 A′ 21 + … + an1 A′
11 + a21 A′ n1   ( 由归纳假设得 )

a22 … a′
2j … a2n a12 … a′
1j … a1n
n
a32 … a′
3j … a3n n
a32 … a′
3j … a3n
= a11 ∑ - a21 ∑ +
j =2 … … … j= 2 … … …
an2 … a′
nj … ann an2 … a′
nj … ann
a12 … a′
1j … a1 n
n
a22 … a′
2j … a2 n
( - 1) n+ 1 an1 ∑
j =2 … … …
an - 1, 2 … a′
n- 1, j … an - 1, n
32 第 1 章 行列式

a11 a12 … a′
1j … a1n
n
a21 a22 … a′
2j … a2n
= ∑
j=2 … … … …
. ( 4)

an1 an2 … a′
nj … ann
综上得 , ( 2) 式右端 = ( 3) 式 + ( 4) 式 = ( 1) 式右端 . 所以对任意的 n 阶行
列式求导数都等于 (1 )式中的 n 个行列式之和 .
12 ( 补充题 47 )   求使 3 个点( x1 , y1 ) , ( x2 , y2 ) , ( x3 , y3 ) 位 于一 直线 上
的充分必要条件 .
解   在平面直角坐标系中直线的一般方程为
a x + by + c = 0 . ( 1)
3 个点位于该直线上时 , 其点的坐标满足方程 , 即
a x1 + b y1 + c = 0 ,
a x2 + b y2 + c = 0 , ( 2)
a x3 + b y3 + c = 0 .
作为一条直线 , 方程( 1) 中的 a, b, c 不全为零 , 因此关 于 a , b, c 的齐 次线性 方
程组( 2) 有非零解 .所以 3 个点位于 同一直 线上 ( 即 3 个 点共线 ) 等 价于方 程
组( 2) 有非零解 .
由克拉默法则知 , 由 n 个方程构成的 n 元齐次线性方程组的系数行列式
不等于零时 , 齐次方程组只有全为零的 解 , 这 等价于 齐次方 程组有 非零时 其
系数行列式必须等于零 , 这里就是
x1 y1 1
x2 y2 1 =0 . ( 3)
x3 y3 1
以后在第 3 章中将证明式( 3) 也是齐次方 程组 ( 2 ) 有非零 解的充 分条件 . 所
以 3 个点 ( x1 , y1 ) , ( x2 , y2 ) , ( x3 , y3 ) 共线的充分必要条件就是 (3) 式 .
13 ( 补充题 49)   写出通过点 (1 , 1 , 1) , (1 , 1 , - 1 ) , ( 1 , - 1 , 1 ) , ( - 1 , 0 ,
0) 的球面方程 , 并求其半径和球心坐标 .
解   空间直角坐标系中球面的一般方程为
x2 + y2 + z2 + a x + by + cz + d = 0 .
球面上的点的坐标应满足该方程 , 于是由题设即得
1. 4 部分疑难习题和补充题的题解 33

1 + 1 + 1 + a + b+ c + d = 0 ,
1 + 1 + 1 + a+ b - c + d = 0,
1 + 1 + 1 + a - b+ c + d = 0 ,
    1 - a    + d= 0 .

a + b + c+ d = - 3 ,
a + b - c+ d = - 3 ,
( 1)
a - b + c+ d = - 3 ,
a    - d= 1 .
方程组( 1) 的系数行列式为
1 1 1 1 1 1 1 1
1 1 -1 1 0 0 - 2 0
D = =
1 - 1 1 1 0 - 2 0 0
1 0 0 - 1 0 - 1 - 1 -2
0 - 2 0
= - 2 0 0 = 8≠0 ,
- 1 - 1 -2
所以( 1) 有惟一解
D1 D2 D3 D4
a= ,   b= ,   c= ,   d= ,
D D D D
其中
-3 1 1 1
-3 1 - 1 1
D1 =
-3 -1 1 1
1 0 0 - 1
1 1 1 -3 1 1
按第 4 行
- 1 - 1 1 + ( - 1) -3 1 - 1 = - 8;
展开
- 1 1 1 -3 - 1 1
34 第 1 章 行列式

1 - 3 1 1
1 - 3 - 1 1
D2 = = 0   ( 第 1 , 3 行相同 ) ;
1 - 3 1 1
1 1 0 - 1

1 1 - 3 1
1 1 - 3 1
D3 = = 0   ( 第 1 , 2 行相同 ) ;
1 - 1 - 3 1
1 0 1 - 1

1 1 1 - 3 1 1 1 - 3
1 1 - 1 - 3 0 0 - 2 0
D4 = =
1 - 1 1 - 3 0 -2 0 0
1 0 0 1 0 -1 - 1 4

0 -2 0
= -2 0 0 = - 16 .
-1 -1 4

于是即得

- 8 - 16
a= = - 1;   b = c= 0;   d = = -2 .
8 8

所以该球面的一般方程为

x2 + y2 + z2 - x - 2 = 0 ,

化为标准方程 , 得

2 2
1 1 3
x- + y2 + z2 = 2 + = ,
2 4 2

所以球面半径 R = 3/ 2 , 球心坐标为 (1/ 2 , 0 , 0) .

14 ( 补充题 50 )   已知 a2 ≠ b2 , 证明方程组
1. 4 部分疑难习题和补充题的题解 35

a x1 + b x2 n = 1 ,
a x2 + bx2 n - 1 = 1,

a x n + b xn + 1 = 1,
b xn + a x n+ 1 = 1,

bx2 + a x2n - 1 = 1,
b x1 + a x2n = 1 .
有惟一解 , 并求此解 .
解   这里不仅 a2 ≠ b2 , 而且 a≠0 , b≠0 , 如果 b = 0 , a≠0 , 则方程组 显然
1
有惟一解 , 且 x i = ( i = 1 , 2 , … , 2 n) .
a
该方程组的系数行列式
a b
b
a b 第 2 n行加第 1 行乘 -
a
w Y
b
a b 第 2 n - 1 行加第 2 行乘 -
D= a
b a
………………………………
Y w b
b a 第 n+ 1 行加第 n行乘 -
a
b a
a b
a b
w Y
a b
a - b2
2
0
= a
Y w
a2 - b2
0
a
a2 - b2
0
a
a2 - b2 n
= an = a2 - b2 n
≠0 ,
a
所以方程组有惟一解 .
36 第 1 章 行列式

由第 1 和第 2 n 方程
a x1 + b x2n = 1 ,

bx1 + a x2n = 1 .

1 b
1 a a- b 1
x1 = = = ,
2
a - b 2 a2 - b2 a+ b

a 1
b 1 a- b 1
x2n = = = .
2
a - b 2 2
a - b 2 a+ b

1
同理 , 由第 2 和第 2 n - 1 方程可得 x2 = x2 n - 1 = ; … ;由第 n 和 第 n + 1 方
a+ b
1
程可得 xn = x n+ 1 = .所以该方程组的惟一解为
a+ b
1
xj = ,   j = 1 , 2 , … , n, n + 1 , … , 2 n .
a+ b
  第 2 章

矩  阵

2. 1 基本要求与内容提要

1  基本要求

(1 ) 会用高斯消元法解线性方程组 .
(2 ) 理解矩阵的概念 , 熟悉单位矩阵、数量矩阵、对角 矩阵、上
( 下 ) 三角矩阵和对称 ( 反对称 ) 矩阵的基本运算性质 .
(3 ) 熟练掌握矩阵的线性运算 ( 加法和数量 乘法 ) 、乘 法、转 置
及其运算规律 , 以及方阵乘积的行列式和方阵的幂 .
(4 ) 熟练掌握矩阵可逆的条件和求逆的 方法 ( 用定义 , 伴随 矩
阵和初等变换求逆 ) .
(5 ) 熟练掌握矩阵的初等变换和 3 种初等矩阵 .
(6 ) 掌握分块矩阵的运算及其应用 .

2  内容提要

(1 ) 高斯消元法是对线性方程组中的方 程做 3 种 初等变换 将
其化为 同 解 的 阶 梯 形 方 程 组 来 求 解 的 一 种 方 法 .具 体 操 作 见
2 .2节 .
(2 ) 矩阵的线性运算 ( 加法和数量乘法 )
设同型矩阵 A= ( aij ) 和 B= ( bij ) ∈ Fm× n , k∈ F( 数域 ) .规定
38 第 2 章 矩阵

A + B = ( aij + bij ) ∈ Fm× n ,


k A = ( kai j ) ∈ Fm× n ,
A - B = A + ( - B) = ( aij - bi j ) ∈ Fm× n .
矩阵的 线 性 运 算 满 足 以 下 运 算 律 ( A, B, C 为 同 型 矩 阵 ,
k, l∈ F) :
① A + B= B + A;         ② A + ( B+ C) = ( A+ B) + C;
③ A+ 0 = A(0 为零矩阵) ; ④ A+ ( - A) = 0(0 为零矩阵 );
⑤ 1 A = A; ⑥ ( kl) A = k( l A) ;
⑦ ( k + l) A= k A + l A; ⑧ k( A + B) = k A + k B .
( 3 ) 矩阵的乘法
m× n n× s
设 A= ( aij ) ∈ F , B = ( bi j ) ∈ F , 规定
m×s
AB= C= ( cij ) ∈ F ,
其中 cij 是 A 的第 i 行与 B 的第 j 列依次对应元素的乘积之和 , 即
cij = ai1 b1 j + ai2 b2 j + … + ai n bn j   ( i = 1, … , m, j = 1 , …, s) .
如果 A 的 列 数与 B 的行 数 不相 等 , 则 AB 没 有 意义 ( 即不 可
乘) .
矩阵的乘法满足以下运算律 :
① ( AB) C= A( BC) ;
② k( AB) = ( k A) B= A( k B) , 其中 k 是数 ;
③ A( B+ C) = AB + AC, ( B+ C ) A= BA+ CA .
矩阵乘法不满足以下运算律 :
① 不满足交换律 , 即一般来说 AB≠BA;
② 由 AB = 0( 零矩阵 )不能推出 A= 0 或 B= 0, 即 A≠0 且 B≠
0, 可能使 AB =0;
③ 不满足消去律 , 即由 AB = AC, 一 般不能推出 B = C .但如果
A 可逆 , 则必有 B= C .
方阵 A 与 B 的乘积的行列式 | AB| = | A| | B| .
( 4 ) 矩阵的幂 .规定方阵 A的 k 次幂为
2. 1 基本要求与内容提要 39

Ak = A A…A( k 个 A 连乘 ) .
由此可推出 : Ak Al = Ak + l ;   ( Ak ) l = Ak l   ( k, l 皆为正整数 ) , 但是 ,
一般来说
( AB) k ≠ Ak Bk .
如果 AB= BA( 即 AB 可 交 换, 此 时 A, B 必 为 同 阶 方 阵 ) , 则
k k k k k
( AB) = A B = B A .
(5) 矩阵的转置
m× n T T n× m
A= ( aij )∈ F 的转置矩阵 A = ( aj i ) ∈ F , 其中
aTji = aij     ( i = 1 , …, m, j = 1 , … , n) .
有的书将 AT 记作 A′.
T T
如果 A = A, 称 A 为对称 矩阵; 若 A = - A, 称 A 为 反对 称矩
阵, 反对称矩阵的主对角元全为 0 .
矩阵的转置运算满足以下运算律 :
T T T T T
① ( A ) = A;           ② ( A+ B) = A + B ;
③ ( kA) T = kAT ( k 是数 ) ; ④ ( AB) T = BT AT .
进而有
T T T T
( A1 A2 …Am ) = Am …A2 A1 .
(6) 可逆矩阵及其逆矩阵
① 若 AB= BA = I, 则称 A 为可逆矩阵 , 并称 B 为 A 的逆矩阵 ,
记作 A- 1 = B .
② 矩阵 A 可逆的充分必要条件为 | A| ≠0 .
③ 可逆矩阵的逆矩阵是惟一的 .
④ 求逆矩阵的方法
( i) 用定义 .如对角阵 A = diag( a1 , a2 , …, an ) 的逆矩阵为
1 1 1
A- 1 = diag , ,…, .
a1 a2 an
n× n
( ii) 用伴随矩阵 , A= ( aij ) ∈ F 的逆矩阵
- 1 1 *
A = A ,
| A|
40 第 2 章 矩阵

其中
A11 A21 … An1

A12 A22 … An2
A = ,
… … …
A1 n A2 n … Ann
式中 Aij 为 A 的元素 aij 的代数余子式 .
( iii) 用初等变换法
初等行变 换 - 1
( A, I) ( I, A ) ,
A 初等列变 换 I
.
I A- 1
⑤ 可逆矩阵满足以下运算律 :
- 1 - 1 - 1 1 -1
( i) ( A ) = A;       ( ii) ( k A) = A   ( k 是非零数 );
k
- 1 - 1 - 1 T - 1 - 1 T
( iii) ( AB) = B A ; ( iv) ( A ) = (A ) ;
- 1 1
(v) | A |= .
| A|
多个可逆矩阵乘积的逆矩阵为
( A1 A2 … Am ) - 1 = Am- 1 … A2- 1 A1- 1 .
(7) 矩阵的初等变换与初等矩阵
① 对矩阵的行和列的 3 种初等变换为: ( i) 对第 i 行 ( 或 列 ) 乘
非零常数 c; (ii) 将第 i 行 ( 或列 ) 乘 c 加到第 j 行 ( 或列 ) ; ( iii ) 将第 i
行 (或列 ) 与第 j 行 ( 或列) 对换 .
相应于 3 种初等变换的初等矩阵为:
( i) 倍乘初等矩阵 Ei ( c) 是将单位矩阵第 i 行 ( 或列 ) 乘 c;
( ii) 倍加初等矩阵 Eij ( c) 是将 单位矩 阵第 i 行 ( 或第 j 列 ) 乘 c
加到第 j 行 ( 或第 i 列 ) ;
( iii) 对换初等矩阵 Eij 是将单位矩阵的第 i 行和第 j 行 ( 或列 )
对换 .
2. 1 基本要求与内容提要 41

② 初等矩阵左乘矩阵 A, 即 Ei ( c) A, Eij ( c) A, Eij A, 它们分别表


示上述对 A 做 3 种初等行变换 .
初等矩阵右乘矩阵 B, 则是对矩阵 B 做 3 种列 变换 , 但 要注意
BEij ( c) 是表示将 B 的第 j 列乘 c 加到第 i 列 .
③ 初等矩阵都 是可逆矩阵 , 它们 的逆矩阵分别是 同类初等矩
阵, 即
1
Ei- 1 ( c) = Ei ,   Ei-j 1 ( c) = Ei j ( - c) ,   Ei-j 1 = Ei j .
c
④ 对可逆矩阵 A 只做若干次 初等行变换 ( 或列变 换 ) , 可将其
化为单位矩阵 .从而可 逆矩阵 A 可以 表示为若干初等 矩阵的乘积 ,
即存在初等矩阵 P1 , P2 , …, Ps , 使
- 1 - 1 - 1 - 1
Ps … P2 P1 A= I, 从而 A= ( Ps … P2 P1 ) = P1 P2 … Ps ,

- 1
A = Ps … P2 P1 = Ps … P2 P1 I .
- 1
这就是用初等行变换法求 A 的理论依据 .
(8) 分块矩阵的运算
① 加法和数量乘法 : 设分块矩阵 A = ( Ak l ) s× t , B= ( Bk l ) s× t , 且
A 与 B 的对应子块 Ak l 与 Bk l 是同型矩阵 , 则
A+ B = ( Ak l + Bk l ) s× t ,
λA = (λAk l ) s× t   (λ是数 ) .
② 乘法: 设分块矩阵 A= ( Ak l ) r× s , B= ( Bk l ) s× t , 且 A 的列的分
块法和 B 的分块法完全相同 , 则
AB= C= ( Ck l ) r× t ,
即 C是 r× t 分块矩阵 , 且 Ck l 是 A 的分块第 k 行与 B 的分块第 l 列
的对应子块乘积之和 , 即
Ck l = Ak1 B1 l + Ak2 B2 l + … + Aks Bsl   ( k = 1 ,…, r, l = 1, …, t) .
③ 转置 : 分块矩阵 A = ( Ak l ) s× t 的转置矩阵为
AT = ( Bl k ) t× s ,
42 第 2 章 矩阵

其中   Bl k = ATk l ,   k = 1 , … , s, l = 1 , … , t .

2 .2 高斯消元法

高斯消元法的基本思路 是 , 对 线性 方程 组中的 方 程做 3 种 初
等变换 ( 某方程乘非零常数 c; 一个方程乘常数 c 加到另一个方程 ;
两个方程对换位 置 ) , 将 其 化为 同 解 而又 易 于求 解 的阶 梯 形 方 程
组 .它是求线性方程组的解的一种基本方法 .
所谓消元就是把某个未知元的系数化为零 .因此, 为了使消元法
的解题过程书写简明, 我们就把线性方程组中的未知元 x1 , x2 , … , xn
去掉 , 将其对应为一张矩形数表 ( 称为方程组的增广矩阵 )
a11 a12 … a1 n b1
a21 a22 … a2 n b2
. ( 2 .1)
… … … …
am1 am2 … am n bm
消元过程就 变成对 线性方程 组的增广 矩阵做 3 种初 等行变 换 ( 某
行乘非零常数 c; 某行乘 c 加到另一行 ; 两行对换位置 ) .
高斯消元法是 一种 规 范化 的 消 元法 , 它 是 按一 种 固 定 的“ 程
序”来 消元的 .首先 做初等行 变换 , 把第 1 列的 元素化为 只剩一 个
非零元 , 并把该非零元置 于第 1 行第 1 列 , 然后依次对第 2 , 3 , … ,
n + 1 列都仿照第 1 列那样进行 , 把 (2 .1 ) 式化为阶 梯形矩 阵 .为 简
明起见 , 不妨设 m = n = 5 , 且化为
c11 c1 2 c13 c14 c1 5 d1
0 0 c23 c24 c2 5 d2
0 0 0 c34 c3 5 d3 . ( 2 .2)
0 0 0 0 0 d4
0 0 0 0 0 0
此时 , 方程组有解的充分必要条件 为 d4 = 0 .有 解时 , 求解步 骤为 ,
2 .2 高斯消元法 43

把每一行第 1 个非零元 c1 1 , c2 3 , c34 对应的未知元 x1 , x3 , x4 取为基


本未知量 , 把其 余的 x2 , x4 取为自 由未知量 , 并 取 x2 = k1 , x5 = k2
( k1 , k2 为任意常 数 ) , 将 其代 入 ( 2 .2 ) 式 所 对应 的 方程 组 , 然 后 先
求得 x4 , 代入第 2 个方程再求得 x3 , 将 x3 , x4 代入第 1 个方程 , 最
后求得 x1 .这样求解 , 必 须一 个一 个 回代 , 为了 省 去回 代 步骤 , 我
们继续在 (2 .2 ) 式上做初等行变换 , 将每行第 1 个非零元上方的元
素 c2 4 , c1 4 , c13 都化为 零 , 并把 每行 第 1 个非 零元均 化为 1 , 得到 一
个行简化阶梯形矩阵
1 c′
12 0 0 c′
15 d′
1

0 0 1 0 c′
25 d′
2

0 0 0 1 c′
35 d′
3 . ( 2 .3)
0 0 0 0 0 0
0 0 0 0 0 0
在行 简 化 阶 梯 形 增 广 矩 阵 对 应 的 方 程 组 上 , 求 解 极 为 方 便 , 令
x2 = k1 , x5 = k2 , 立即可得
1 - c′
x1 = d′ 1 2 k1 - c′
1 5 k2 ,

x2 = k1 ,
x3 = d′
2 - c′
2 5 k2 ,

x4 = d′
3 - c′
3 5 k2 ,

x5 = k2 .

x1 d′
1 - c′
12 - c′
15

x2 0 1 0
x3 = d′
2 + k1 0 + k2 - c′
25   ( k1 , k2 为任意常数) .
x4 d′
3 0 - c′
35

x5 0 0 0
这里解的表示形式 , 利用了后面的向量加法和数量乘法 .
44 第 2 章 矩阵

例 1   线性方程组
x1 + x2 + 2 x3 - x4 = 1 ,
x1 - x2 - 2 x3 - 7 x4 = 3 ,
x2 + ax3 + t x4 = t - 3 ,
x1 + x2 + 2 x3 + ( t - 2) x4 = t + 3
中的 a, t 取何值时 , 方程 组无 解 , 有 惟一 解 , 有 无穷 多组 解 ? 有 无
穷多组解时 , 求其解 .
解   记方程组的增广矩阵为 ( A, b) , 则
1 1 2 -1 1
1 -1 -2 -7 3
( A, b) = . ( 1)
0 1 a t t -3
1 1 2 t- 2 t+3
将其第 1 行乘 - 1 分别加到第 2 , 4 行 , 得
1 1 2 -1 1
0 -2 -4 -6 2
( A, b) ,
0 1 a t t -3
0 0 0 t- 1 t+2
1
把第 2 行乘 - , 再将其乘 - 1 加到第 3 行 , 得
2
1 1 2 -1 1
0 1 2 3 - 1
( 2)
0 0 a-2 t- 3 t- 2
0 0 0 t- 1 t+ 2
(1 ) 当 a≠2 , t≠ 1 时 , 方程 组 有 惟一 解 .此 时 求解 比 较繁 , 因
为将增广矩阵化为行简化 阶梯 阵也不 方便 .所 以先由 第 4 行对 应
t+ 2
的方程 ( t - 1 ) x4 = t + 2 , 得 x4 = ; 将其回 代到第 3 行对应的 方
t- 1
程,得
2 .2 高斯消元法 45

2 t- 4
x3 = ・ ;
2 - a t- 1
再将 x3 , x4 回代到第 2 行对应的方程 , 得
1 4( t - 4)
x2 = - 1 - + 3( t+ 2) ;
t- 1 2- a
最后把 x2 , x3 , x4 回代到第 1 行对应的方程 , 得
t+1
x1 = 6 .
t-1
(2 ) 当 t = 1 时 , 第 4 行 对 应 的 方 程 为 0 ・ x4 = 3 , 此 时 , 方
程无解 .
(3 ) 当 a = 2 时 , 令第 3 , 4 行最后两列元素成比例 , 即
t- 3 t- 2
= ,   得 t= 4 .
t- 1 t+2
此时 , 第 3 , 4 行对 应 的方 程 都是 x4 = 2 .从 而 方 程 组有 无 穷 多 组
解 .将 a = 2 , t = 4 代入阶梯形增广矩阵 , 并进而用初等 行变换将 其
化为行简化阶梯矩阵 , 即
1 1 2 -1 1 1 0 0 0 10
0 1 2 3 - 1 0 1 2 0 -7
.
0 0 0 1 2 0 0 0 1 2
0 0 0 3 6 0 0 0 0 0
它所对应的方程组为
x1 = 10 ,
x2 + 2 x3 = - 7 ,
x4 = 2 .
取 x3 = k, 得方程组的解为
( x1 , x2 , x3 , x4 ) = ( 10 , - 7 - 2 k, k, 2 ) , k 为任意常数 .
(4 ) 当 a = 2 , t≠4 时 , 矩阵 ( 2) 中 , 第 3 , 4 行 不成 比例 , 它们 所
对应的方程 是矛盾方 程 , 从而方程 组无解 .或者 对矩阵 ( 2 ) 做初 等
46 第 2 章 矩阵

t-1
行变换 , 将第 3 行乘 - 加到第 4 行 , 得
t-3
1 1 2 -1 1
0 1 2 3 -1
0 0 0 t- 3 t- 2 . ( 3)
t-4
0 0 0 0 2
t-3
从矩阵 (3 ) 对应的方程组可见 , t = 3≠4 , 或 t≠4 且 t≠3 时 , 方程组
均无解 .
例 2   非齐次线性方程组的增广矩阵为
4 - 2 13 0 3
2 4 p 10 1
( A, b) = .
3 - 1 8 1 2
5 3 -3 11 1
试问 : p 取何值时 , 方程组有解 ? 并求解 .
1 3 5
解   如果将第 1 行分别乘 - , - , - , 并依次 加到 第 2 ,
2 4 4
3 , 4 行 , 这样虽 然后 3 行第 1 列元素全 化成零 , 但其他元素有很 多
分数 , 如此再继续消 元时 计算 很 繁 .为使 消元 过 程简 便 , 先 把 第 3
行乘 - 1 加到第 1 行 , 将第 1 行化为
(1   - 1   5   - 1   1 ) ,
并把第 2 行换到第 4 行 ( 否则消元时第 2 行的 p 会引起麻烦 ) , 即
1 -1 5 -1 1
行 变换 3 -1 8 1 2
( A, b) .
5 3 -3 11 1
2 4 p 10 1
将第 1 行乘 - 3 , - 5 , - 2 , 并分别加到第 2 , 3 , 4 行 , 得
2 .2 高斯消元法 47

1 - 1 5 -1 1
行变 换 0 2 - 7 4 -1
.
0 8 - 28 16 -4
0 6 p - 10 12 -1
再将第 2 行乘 - 4 , - 3 , 依次加到第 3 , 4 行 , 并将第 3 , 4 行对换 , 得
1 - 1 5 -1 1
行变 换 0 2 - 7 4 -1
.
0 0 p + 11 0 2
0 0 0 0 0
当 p + 11 ≠0 即 p≠ - 11 时 , 方 程组 有 无穷 多 组解 , 取 x4 为
自由未知量 , 令 x4 = k( 任意常数 ) , 依次解得
2 3- p 5+ p
x3 = ,   x2 = - 2 k,   x1 = - k.
11 + p 2( 11 + p) 2( 11 + p)
例 3   齐次线性方程组的系数矩阵为
a 2 -1
A= 2 1 2 .
3 3 1
试问 : a 取何值时 , 方程组有非零解 ? 并求解 .
2 3
解   此时如果把第 1 行乘 - , - 分别加到第 2 , 3 行 , 把第
a a
1 列化为只剩 1 个非零 元 a, 这 样做 不 好 , 因为 首先 假 定了 a≠ 0 ,
解完后还需讨论 a = 0 时是否有非零解 , 而且 在第 2 , 3 行的 第 2 , 3
列元素出现含 a 的分式 , 继续消元也比较麻烦 .此题可用下面两种
方法求解 .
法 1: 把 A 的第 1 行与第 3 行对换 , 并把第 2 行乘 - 1 加到 第
1 行 , 使第 1 行第 1 列的元素化为 1 , 然后 , 继续对矩阵做初等行变
换 , 使之化为阶梯阵 , 再求其非零解 .
法 2: 把 A 的第 1 列与第 3 列对换 , 化为
48 第 2 章 矩阵

-1 2 a
A1 = 2 1 2 ,
1 3 3
再把 A1 的第 1 行与第 3 行对换 , 使之化为
1 3 3
A2 = 2 1 2 .
-1 2 a
这里以 A1 和 A2 为系数矩 阵的 齐次线 性方 程组是 一样 的 .但
要注意为使这个方程组与以 A 为系 数矩阵 的齐 次线 性方程 一样 ,
第 1 列对应的未知元应为 x3 , 而 第 3 列对应 的未 知元 应 为 x1 .读
者不难通过初等行变换 , 把 A2 化为下面的阶梯形矩阵
1 3 3
初等 行变 换
A2 0 - 5 -4 .
0 0 a-1
由此可见 , 齐次线性方程组有非零解的充分 必要条件 为 a - 1 = 0 ,
即 a = 1 .求 解时 把 第 3 列 对 应 的 未知 元 x1 ( 要 特 别 注 意 它 不 是
4
x3 ) 取为自由未知量 , 令 x1 = k 得 x2 = - k, 将 x1 , x2 代入第 1 个
5
方程 , 得
12 3
x3 = - 3 x2 - 3 x1 = k - 3k= - k.
5 5
所以 a = 1 时方程组的解为
4 3
( x1 , x2 , x3 ) = k, - k, - k , k 为任意常数 .
5 5
关于高斯消元法 , 再强调以下几点 :
(1 ) 高斯消元法是用初等行变换按“ 固定的程 序”一列一列 地
依次消元 , 将线性方程 组的增 广矩 阵化为 阶梯 形矩 阵 .求解 时 , 把
每行第 1 个非零元对应的 未知 量取为 基本 未知量 , 其 他的 取为 自
由未知量 , 并依次取任意常数 k1 , k2 , …将其代入方程组 , 求出基本
2. 3 矩阵的基本运算——加法、数量乘法和乘法 49

未知量 .为了使求基本未知量更为方便 , 尽可能如例 1 中 (3 ) 那样 ,


把增广矩阵化为行简化阶梯矩阵 .
(2 ) 关于矩阵的初 等变 换 , 以 后要 介绍 也 有 3 种 与行 变换 一
样的初等列变换 .但是必须注意 , 求解线性方程组将增广矩阵化为
阶梯形矩阵的过程中 , 不能用倍加列变换和倍乘列变换 , 而两列对
换可以使用 , 此时相应的未知元也要对换 , 如例 3 那样 .
(3 ) 为了使消元计 算 ( 尤 其是 手算 ) 时 比较 方便 , 要尽 可能 如
例 2 那样 , 先用初等 行变换将第 1 行第 1 列的 元素化为 1 , 然后 再
将第 1 列 其余元 素全 化为零 .以后对 第 2 , 3 , … 列元 素消 元 时 , 也
仿照第 1 列那样进行 .
(4 ) 当线性方程组 的增 广 矩阵 中含 有参 数 a, b, p , t 等时 , 如
果它们位于增广矩阵的左上方 , 消元时会比较麻烦 .通常要尽可能
用行对换或列 对换将 它们换到 右下方 ( 如 例 3 那样 , 列对换 时 , 未
知元也对换 ) .此外 , 还要避免将某行乘 k/ a 加到另一行的做法 , 因
为这样就假定了 a≠ 0 , 对 a = 0 的情形还要另行讨论 .

2 .3 矩阵的基本运算——加法、数量乘法和乘法

1  矩阵的线性 运算——加法 和数量乘法

矩阵的加法和数量乘法 的定义 及其 满足的 运算 律 , 在 内容 提


要中已经叙述 , 这里不再重复 .
定义了矩阵 B= ( bi j ) m× n 的负 矩阵 - B = ( - bi j ) m× n , 则 两个 矩
阵 A 与 B 相减就定义为
A - B= A + ( - B) .
两个矩阵 A 和 B 当且仅当它们是同型矩 阵时才 能相加 .例 如
3×3 矩阵和 3 ×4 矩阵不是同型的 , 它们不能相加 .
矩阵的加法和数量乘法 与数的 加法 和乘法 是类 似 的 , 掌握 它
们的运算没有什么困难 .但是 如果深 入地 追究 为什么 两个 同型 矩
50 第 2 章 矩阵

阵相加可以定义为对应元 素相 加 , 而 后面 讲的 两个矩 阵相 乘与 普
通的数的乘法相去甚远呢 ? 这就牵涉到矩阵的本质是什么 ? 在第
n
4 章揭示了一个 n× n 矩阵 A 实际 上是 表示 n 维 向量空 间 ( R ) 的
一个线性变换 .与此相关的是 : 一个线性方程组
a11 x1 + a12 x2 + … + a1 n x n = b1 ,
a21 x1 + a22 x2 + … + a2 n x n = b2 ,
( 2 .4)

an1 x1 + an2 x2 + … + an n x n = bn .
n

即 ∑ aij x j = bi   ( i = 1 , … , n) , 它的 n 阶系数矩阵
j =1

A = ( aij ) n× n
T
就是一个线 性变换 , 它 把 n 维 向量 x = ( x1 , x2 , … , xn ) 变换 为 n
维向量 b = ( b1 , b2 , … , bn ) T .如果还有另一个以 n 阶矩阵
B = ( bi j ) n × n
为系数矩阵的线性方程组
n

∑b
j =1
ij x j = ci   ( i = 1 , … , n) , ( 2 .5)

那么方程组 (2 .4 ) 与 ( 2 .5) 中对应方程相加所得的新方程组


n

∑(a
j= 1
ij + bij ) xj = bi + ci   ( i = 1 , … , n) ( 2 .6)

的系数矩阵就是我们前面定义的
A + B= ( aij + bi j ) n× n .
这个 A + B 就是两个线性变换 A 与 B 之和 , 从 方程 组 ( 2 .6 ) 可见 ,
T
它把 n 维向量 x = ( x1 , x2 , … , xn ) 变换为 n 维向 量 b + c= ( b1 +
c1 , b2 + c2 , … , bn + cn ) T .

2  矩阵的乘法

矩阵的乘法定义及其满 足的运 算律 在内容 提要 中 已叙 述 , 这


里不再复述 .
2. 3 矩阵的基本运算——加法、数量乘法和乘法 51

矩阵的乘法是很重要的 矩阵运 算 , 它与 数的乘 法 运算 很不 一


样 .因此 , 对于读者来讲 , 首先要 理解 为什么 矩阵 的乘 法要 做如 此
奇特的定义 ? 然后要搞清楚矩阵的乘法运算与数的乘法运算在运
算律上有哪些共同点和不同点 .
(1 ) 矩阵乘法定义的背景
数学中的定义 , 如微积分中的导数、微分、定积分 等概念的 定
义 , 都是研究某些实际问题的需要而抽象出来的 .
为什么两个矩 阵相 乘 不定 义 为 两个 同 型 矩 阵 对应 元 素 相 乘
呢 ? 因为这样的定义没有什么用处 .
两个矩阵 Am× n 与 Bn × s 相乘 , 其乘 积 AB = Cm× s 为 什 么要 做 前
面所述的定义呢 ? 寻根求源 , 它 是研 究向量 空间 中两 个线 性变 换
作乘法的客观需要 ( 读者可参看第 4 章中的定 理 4 .20) .与此 相应
的问题是下面的两个线性方程组
b11 x1 + b1 2 x2 + … + b1 n x n = y1 ,
b21 x1 + b2 2 x2 + … + b2 n x n = y2 ,
( 2. 7)

bn1 x1 + bn2 x2 + … + bn n x n = yn ;
a11 y1 + a12 y2 + … + a1 n y n = z1 ,
a21 y1 + a22 y2 + … + a2 n y n = z2 ,
( 2. 8)

an1 y1 + an2 y2 + … + an n y n = z n .
这里 的方 程 组 ( 2 .7 ) 可 理 解 为系 数 矩 阵 B = ( bi j ) n× n 把 向 量
x = ( x1 , x2 , … , xn ) T 变 换 为 向 量 y = ( y1 , y2 , … , yn ) T ; 方 程 组
T
(2 .8 ) 是系数矩阵 A = ( ai j ) n× n 把向 量 y = ( y1 , y2 , … , yn ) 变换 为
向量z= ( z1 , z2 , … , z n ) T .
把方程 组 ( 2 .7 ) 中 的 y1 , y2 , … , yn 代 入 方 程 组 ( 2 .8 ) , 就 会
得到
52 第 2 章 矩阵

c1 1 x1 + c12 x2 + … + c1 n x n = z1 ,
c2 1 x1 + c22 x2 + … + c2 n x n = z2 ,
( 2 .9)

cn1 x1 + cn2 x2 + … + cn n x n = z n .
其中 cij = ai1 b1 j + ai2 b2 j + … + ai n bn j   ( i, j = 1 , … , n) .
推导出这个结果比较麻烦 , 但是 , 如果令 n = 2 , 读者就很容 易
导出这个结果 .
这里的方程组 (2 .9 ) 是 系数 矩 阵 C = ( cij ) n× n 把 向 量 x = ( x1 ,
x2 , … , xn ) T 变换为向量 z= ( z1 , z2 , … , z n ) T .于 是 , 我们就 把矩 阵
C 定义为矩阵 A 与 B 之乘积 , 即
C= AB .
这就是矩阵乘法定义的实际背景 .
定义了矩阵的乘法 , 上述的 3 个方程组 ( 2 .7) , ( 2 .8) 就可简捷
地表示为
Bx = y, ( 2 .7)
Ay = z, ( 2 .8)
将方程组 (2 .7 ) 中的 y 代入方程组 ( 2 .8 ) , 即得
ABx = z,   即   Cx = z . ( 2 .9)
其中   A = ( aij ) n× n ,   B = ( bi j ) n× n ,   C = ( ci j ) n× n ,
x1 y1 z1
x2 y2 z2
x= ,       y= ,       z= .
… … …
xn yn zn
线性代数在它的发展史 上 , 最 早研 究的 问题就 是 一般 线性 方
程组的求解问题 .定义了矩阵的乘法 , 一般的线性方程组 , 如 ( 2 .1)
式表示的线性方程组就可表示为
Ax = b,
其中 : A = ( ai j ) m× n , x = ( x1 , x2 , … , xn ) T , b= ( b1 , b2 , … , bm ) T .
2. 3 矩阵的基本运算——加法、数量乘法和乘法 53

这样 , 一般线性方程的 矩阵表 示式 就与 初等代 数 中的 一元 一


次方程式 ax = b 相类似 , 这为研究问题提供了极大的方便 .也正因
为如此 , 线性代数从研究一般线性方程组的求解问题开始 , 进而研
究 n 维向量和矩阵的各种性质 .
(2 ) 矩阵乘法的运算律
矩阵乘法满足结合律和 数乘矩 阵的 数乘结 合律 , 以及 左分 配
律和右分配律 .这些与数的乘法是一样的 .
更重要的是区别于数的乘法它不满足以下运算律 :
① 矩阵乘法不满足 交 换 律 , 即 一般 AB≠ BA .这 可从 3 个 方
面来理解 :
一是 AB 可 乘 , BA 不 一 定 可 乘 .例 如 A 为 2 × 3 矩 阵 , B 为
3×4 矩阵时 , AB 可乘 , 而 BA 不可乘 .
二是 AB 与 BA 均可乘 , 但不一 定是同型 矩阵 .例如 A 为 2×3
矩阵 , B 为 3×2 矩 阵时 , AB 为 2 ×2 矩阵 , BA 为 3 ×3 矩阵 , 它 们
自然不能相等 .
三是即使 AB 与 BA 为同 型矩 阵 ( 此 时 A, B 必为 同阶 方阵 ) ,
也不一定相等 .例如
0 1 1 0
A= ,   B= ,
0 0 0 0

0 0 0 1
AB= = 0 ,   BA = = A≠0 .   (2 .1 0)
0 0 0 0
此时 AB≠ BA .
由于一般来讲 AB≠ BA, 因此 , 一般
( AB) 2 = ( AB) ( AB) ≠ ( AA) ( BB) = A2 B2 .
但是 , 如果 AB 可交换 , 即 AB = BA, 则
( AB) 2 = ( AB) ( AB) = ( AA) ( BB) = A2 B2 ,
2 2 2
( AB) = ( AB) ( AB) = ( BA) ( AB) = ( BB) ( AA) = B A .
54 第 2 章 矩阵

此时 , 对于任何正整数 k, 均有
k k k k k
( AB) = A B = B A .
k k k
那么 , 由此能否进一步得出结论 :“ ( AB) = A B 的充分必 要
条件是 AB = BA”呢 ? 这 个 结 论 是 不 对 的 , 只 能 讲“ AB = BA 是
( AB) k = Ak Bk 成立的充分条件 , 而不是必要条件”.例如 :
在 (2 .1 0) 式中 A 和 B, 满足 AB≠ BA, 但是 由于 AB = 0, A2 =
2 2 2 2
0, B = B, 所以仍有 ( AB) = 0 = A B .
② 由矩阵 A 与 B 的乘积 AB = 0( 零矩阵 ) , 不能推 出 A = 0 或
B= 0, 即 A≠0 且 B≠0 时 有可能 得 AB = 0 .这 就是 矩阵的 乘法 有
非零矩阵作为零因子 , 此时 称 B 为 A 的 右零 因子 , A 为 B 的 左 零
因子 .例如在 ( 2 .10 ) 式中 , 有
A≠0 ,   B≠0,   但   AB= 0 .
这又是与数的乘法截然不 同的 一种“奇 特现 象”.为 什么会 出现 这
种“奇特现象”呢 ? 其根源 仍是 矩阵乘 法的 定义 .它的 一个 具体 表
现如下 .
如果以 A= ( aij ) m× n 为系数 矩阵 的齐 次线性 方程 组 Ax = 0 有
非零解 ( 如有非零解必有无穷多个 )
x1 1 x12 x1 s
x2 1 x22 x2 s
, ,… , .
… … …
xn1 xn2 xn s
将这些非零解为列排成的矩阵记作 B, 即
x1 1 x1 2 … x1 s
x2 1 x2 2 … x2 s
B= ,
… … …
xn1 xn2 … xn s
则有 AB= 0 .
由此又可知 , 当 A = ( aij ) n× n 为 n 阶 不 可逆 矩 阵时 , 由于 齐 次
2. 3 矩阵的基本运算——加法、数量乘法和乘法 55

线性方程组 Ax= 0 有非 零解 , 所 以 必存 在矩 阵 B = ( bi j ) n× s ( 其 中
每一列都是 Ax= 0 的 非 零 解 ) , 使 得 AB = 0; 而 当 A 为 可 逆 矩 阵
时 , Ax = 0 只有零解 , 因此 , 如果 AB = 0, 则必有 B = 0( 因为 B 的每
一列都是 Ax= 0 的解 ) , 或者由
AB = 0,   可推出   B = A- 1 0 = 0 .
所以 , 不可逆矩阵 A 也称为奇异矩阵 , 因为它存在 B≠0 使 AB = 0
的奇异现象 ; 而可逆 矩阵 A 则 称 为非 奇 异 矩阵 , 因为 它 不会 出 现
B≠0使 AB= 0 的奇异现象 .
这个问题进一步的结论是 : 对 于 m× n 矩 阵 A = ( ai j ) m× n , 如
果秩 ( A) < n, 则 Ax= 0 有非零解 , 从而存在 B≠0, 使 AB = 0; 如 果
秩 ( A) = n, 则 Ax = 0 只有零解 , 因此 , 不存在 B≠0 使 AB = 0, 即由
AB= 0 可推出 B = 0 .
③ 矩阵的乘法不满足消去律 , 即当 A≠0 时 , 由 AB = AC 不能
消去 A, 得 B= C .
这一点是由上述第②点派生出来的 .因为
AB = AC   可推出   AB - AC = A( B - C) = 0,
而上式由 A≠0, 不能推出 B - C = 0, 即 B = C .
但是 , 如 果 A 是 可 逆 矩 阵 , 则 由 AB = AC 的 等 式 两 边 左 乘
- 1 - 1 - 1
A , 即得 A AB= A AC, 从而得 B = C( 此时可消去 A) .
(3 ) 一些特殊矩阵的乘法运算
① 对角矩阵、单位矩阵与数量矩阵
对角矩阵 Λ= diag( a1 , a2 , … , an ) 左乘 A = ( ai j ) n× s 是将主对角
元 ai ( i = 1 , … , n) 乘 A 中 第 i 行 每 个元 素 ; 对 角 矩阵 Λ 右 乘 B =
( bij ) m× n 是将主对角元 a i ( i = 1 , … , n) 乘 B 中第 i 列每个元素 , 即
56 第 2 章 矩阵

a1 a11 a12 … a1 s a1 a11 a1 a1 2 … a1 a1 s


a2 a21 a22 … a2 s a2 a21 a2 a2 2 … a2 a2 s
  = ,
w … … … … … …
an an1 an2 … an s an a n1 an an2 … an a n s
(2 .1 1)
b11 b1 2 … b1 n a1 a1 b11 a2 b12 … an b1 n
b21 b2 2 … b2 n a2 a1 b21 a2 b22 … an b2 n
  = .
… … … w … … …
bm1 bm2 … bm n an a1 bm1 a2 bm2 … an b m n
(2 .1 2)
单位矩阵 I 与数量矩阵 k I 都是特殊的对角矩阵 , 由 ( 2 .11 ) 式
与 (2 .1 2) 式立即可得
IA = AI = A,
( kI ) A = A( kI ) = k A   ( k 是数 ) .
这表明 , I 和 kI 与任何方阵相乘可交换 , 而且单位矩阵 I 在矩阵乘
法中的作用类似于数的乘法中的 1 .
关于对角矩阵与其他矩 阵相乘 是否 可交换 的问 题 , 有 以下 几
个结论 ( 以三阶矩阵为例 ) .
设 Λ= diag ( k1 , k2 , k3 ) , A = ( ai j ) 3 × 3 .则
( i) 与主对角元 k1 , k2 , k3 互异的对角矩阵可交换的矩阵 A 也
必是对角矩阵 .因为 , 由
ΛA = AΛ
即得
k1 a11 k1 a12 k1 a13 k1 a11 k2 a12 k3 a13
      k2 a21 k2 a22 k2 a23 = k1 a21 k2 a22 k3 a23 .   (2 .1 3)
k3 a31 k3 a32 k3 a33 k1 a31 k2 a23 k3 a33
于是就有
2. 3 矩阵的基本运算——加法、数量乘法和乘法 57

ki a ij = kj a i j     ( i, j = 1 , 2 , 3) .
由于 , 当 i≠ j 时 , ki ≠ kj , 所以当 i≠ j 时 , ai j = 0 .因 此 , A 为 对角 矩
阵 , 即 A = diag( a1 1 , a22 , a3 3 ) .
( ii ) 如果对角矩阵 Λ= diag ( k1 , k2 , k2 ) ( 其中 k1 ≠ k2 ) , 则与 该
对角矩阵可交换的矩阵 A= ( aij ) 3 × 3 必为
a11 0 0
A= 0 a22 a23 . (2 .1 4)
0 a32 a33
这从 (2 .1 3 ) 式中 k3 = k2 ≠ k1 , 立即 可得 A 中 元素 a12 = a13 =
a2 1 = a3 1 = 0 .
同理 , 如果对角矩阵 Λ= diag ( k1 , k1 , k2 , k3 , k3 ) ( 其中 k1 , k2 ,
k3 互不相等 ) , 则与其可交换的五阶矩阵 A 必为
a1 1   a1 2 0 0 0
a21 a22 0 0 0
A= 0   0 a33 0 0 . (2 .1 5)
0   0 0 a4 4   a4 5
0   0 0 a54 a55
以上 (2 .1 4) 与 (2 .1 5) 式的矩阵称为对角块矩阵 .
思考题   与对角矩阵 Λ= diag ( k1 , k2 , k1 ) ( k1 ≠ k2 ) 可交 换 的
矩阵 A= ( aij ) 3 × 3 是怎样的矩阵 ?
( iii ) 最简单的情形是 , 两个对角矩阵 Λ1 = diag( a1 , a2 , … , an )
与 Λ2 = diag ( b1 , b2 , … , bn ) 相乘可交换 , 且
Λ1 Λ2 = Λ2 Λ1 = diag ( a1 b1 , a2 b2 , … , an bn ) .
② 上 ( 下 ) 三角矩阵
两个同阶 上 三角 矩 阵 A = ( aij ) n × n 与 B = ( bi j ) n× n ( 其 中 i > j
时 , aij = bij = 0 ) 的乘积 AB = C = ( ci j ) n× n 仍 是上 三角 矩阵 , 且主 对
角元 cii = aii bii ( i = 1 , 2 , … , n) .
同样 , 两个同阶下三 角矩 阵 A 与 B 的 乘积 AB = C 仍是 下 三
58 第 2 章 矩阵

角矩阵 , 且 cii = aii bii ( i = 1 , 2 , … , n) .


但要注意 , 上述命题的逆命题不成立 , 即 , 由 AB = C 为上 ( 下 )
三角矩阵 , 不能推出 A, B 均为上 ( 下 ) 三角矩阵 .例如
1 0 1 1 1 1
= .
1 -1 1 2 0 -1
此外 , 三角矩阵在乘法可交换的问题上 , 与对角矩阵是不一样
的 .即任何 两 个 同 阶 上 ( 下 ) 三 角 矩 阵 相 乘 不 是 都 可 交 换 的 .例
如: 设
1 1 1 2 1 1
A= ,   B= ,   C= ,
0 1 0 1 0 2
则有
1 3
AB = BA = .
0 1
1 5 1 3
而 BC = ≠ CB= .
0 2 0 2
③ 对称矩阵与反对称矩阵的运算性质
在下面矩阵的转置中讨论 .
(4 ) 两个同阶方阵 A 与 B 的乘积的行列式
| AB| = | A| | B| .
例 1   设 n 阶矩阵 ( n 为奇数 ) A 满足 AT A = I, 且 | A| > 0, 求行
列式 | A - I| = ?
解   由于 A - I = A - AT A = ( I - AT ) A = - ( A - I ) T A, 所以
T n
| A - I| = | - ( A - I) | | A| = ( - 1) | A - I| | A| = - | A - I| | A| .
又因为 | AT A| = | AT | | A| = | A| 2 = | I | = 1 , | A| = ±1, 这里 | A| = 1 > 0,
代入上式, 得
| A - I| = - | A - I| ,   于是   | A - I| = 0 .
1
n
例 2   已知 A= αβ, 其中 α= 2 ,   β= 1   - 1   1 ,   求 A .
3
2. 3 矩阵的基本运算——加法、数量乘法和乘法 59

解  An = (αβ ) (αβ ) …(αβ ) (αβ )


= α(βα)… (βα)β,
其中
1
βα= (1   - 1   1) 2 = 1×1 + ( - 1)×2 + 1×3 = 2 .
3
1 -1 1
n n- 1 n- 1 n- 1 n- 1
所以 A =α(2 )β = 2 αβ = 2 A= 2 2 -2 2 .
3 -3 3
例3  设
a1 b1
a2 b2 n
A= ,   B= .   计算 ( AB ) .
a3 b3
a4 b4
解 
a1 b4 a1n b4n
n n
a2 b3 n
a2 b3
AB = ,   ( AB ) = .
a3 b2 a3n b2n
n n
a4 b1 a4 b1
n n n
试问 , 此时 ( AB) = A B 成立吗 ? ( 答案 : 不成立 .)
例4  设
2 3
2 a a a
2
0 2 a a n
A= ,  求 A .
0 0 2 a
0 0 0 2
解   由于
60 第 2 章 矩阵

2 3
0 a a a
2
0 0 a a
A = 2 I + B,   其中   B= ,
0 0 0 a
0 0 0 0
故 
n n n 1 n- 1
A = ( 2 I + B) = ( 2 I) + C n ( 2 I ) B+
2 n- 2 2 3 n - 3 3 n
  Cn ( 2 I ) B + C n (2 I) B +…+B
n 1 n - 1 2 n- 2 2 3 n- 3 3 n
= 2 I + Cn 2 B + Cn 2 B + Cn 2 B +…+B ,
其中
2 3
0 0 a 2a 0 0 0 a3
2
2 0 0 0 a 3 0 0 0 0 4 n
B = ,   B = , B = … = B = 0,
0 0 0 0 0 0 0 0
0 0 0 0 0 0 0 0
所以
2n C1n 2n - 1 a (C1n 2n - 1 + C2n 2n - 2 ) a2 (C1n 2n - 1 + C2n 2n - 2 + C3n 2n - 3 ) a3
n 1 n- 1 1 n- 1 2 n- 2 2
n
0 2 Cn 2 a (Cn 2 + Cn 2 )a
A = n 1 n- 1
,
0 0 2 Cn 2 a
0 0 0 2n
1 2 n( n - 1 ) 3 n( n - 1 ) ( n - 3)
其中组合数 C n = n,   C n = ,   Cn = .
2! 3!
例 5   设 A, B, I 为同阶矩阵 .下列命题哪些是正确的 ?
(1 ) ( A + B) 2 = A2 + 2 AB + B2 .
3 3 2 2 3
(2 ) ( A +λI ) = A + 3λA + 3λ A +λ I   (λ为数 ) .
(3 ) 若 A, B 可交换 , 则 ( A + B) 与 ( A - B) 相乘也可交换 .
2 2 2
(4 ) ( AB) = A B 当且仅当 AB= BA .
( 5 ) 若 BA = A, 则 B= I .
( 6 ) 若 A 与 AB 均为上三角矩阵 , 则 B 也是上三角矩阵 .
解   ( 1 ) 不正确 , 因为一般 AB≠ BA .
2. 3 矩阵的基本运算——加法、数量乘法和乘法 61

( 2 ) 正确 , 因为数量矩阵 λI 与 A 可交换 .
( 3 ) 正确 , 因为 ( A + B) ( A - B) 与 ( A - B) ( A + B) 都等 于 A2 -
2 2 2
AB+ AB - B = A - B .
2 2 2
( 4 ) 不正确 , 其 中“ 当 AB = BA 时 , 有 ( AB) = A B ”是 正 确
的 , 而“ 仅 当 AB = BA 时 , 才有 ( AB) 2 = A2 B2 ”是 不正 确 的 , 例 如 ,
2 2 2
( 2 .1 0) 式中 , AB≠ BA, 但 ( AB) = A B = 0 .
( 5 ) 不正确 , 例如
1 1
1 1 2 2
A= ,   B= .
1 1 1 1
2 2
(6 ) 不正确 , 例如
1 1 1 1 2 2
A= ,   B= ,   AB= .
0 0 1 1 0 0
例6  设
1 1 2 -1 x1 y1 z1
A= ,   B= ,   x= ,   y= ,   z=
1 -1 -4 2 x2 y2 z2
之间满足 Ax= y, By= z .
T
( 1 ) 若 x = ( 1, 2 ) , 求 z .
( 2 ) 当 z = ( 0 ,0 ) T 时 , 求 x .
解   由 Ax= y, By= z   得 ( BA) x = z .
2 -1 1 1 1
( 1 ) z= ( BA) x =
-4 2 1 - 1 2
1 3 1 7
= = .
-2 -6 2 - 14
( 2 ) ( BA) x = z   即为线性方程组
1 3 x1 z1 0
= = .
-2 -6 x2 z2 0
62 第 2 章 矩阵

用消元 法 , 得 x1 + 3 x2 = 0 ,   取 x2 = k ( 为 任 意 常 数 ) , 得 x1 =
- 3 k, 于是所求的全部 x 为
- 3k
x= ,   k 为任意常数 .
k

2. 4 矩阵的转置

将一个 m× n 矩阵 A = ( ai j ) m × n 的行与列互换 ( 即把 A 的 m 个
行依次换为 m 个列 , 或 把 A 的 n 个列 依次 换为 n 个 行 ) 所 得到 的
一个 n× m 矩阵 AT = ( aTj i ) n× m ( 其 中 aTji = aij ) 称为 A 的转置 矩阵 .
关于矩阵的转置 , 需要搞清以下问题 .

1  为什么要研 究矩阵的 转置

矩阵的转置是研究非常重要的对称矩阵和反对称矩阵的有力
工具 .
T
A= ( aij ) n× n 为对称矩阵的充分必要条件是 A = A, 即 aj i = aij
( i, j = 1 , … , n) .
T
A = ( aij ) n× n 为 反 对 称 矩 阵 的 充 分 必 要 条 件 是 A = - A, 即
aji = - aij ( i, j = 1 , … , n, 其中 aii = 0 ) .
线性代数的一 个重要 问题是研 究 n 元实二 次型 ( 即 n 元二 次
齐次多项式 )
2
f ( x1 , x2 , … , xn ) = ( a11 x1 + 2 a12 x1 x2 + … + 2 a1 n x1 xn )
+ ( a22 x22 + 2 a23 x2 x3 + … + 2 a2 n x2 xn )
+ … + ( an - 1 , n - 1 x2n - 1 + 2 an - 1 , n x n - 1 xn )
2
+ an n x n
( 其中 aij 均为实数 ) 的性质 .这个实二 次型对 应于一个 实对称矩 阵
A = ( aij ) n× n , 利用转置和 矩阵 的乘法 , 该 实二 次型 可表 示 为 ( 详 见
第 6 章)
2. 4 矩阵的转置 63

f ( x1 , x2 , … , xn ) = xT Ax,
其中   x = ( x1 , x2 , … , xn ) T .
以后 , 我们还将证明 , 对于任何实对称矩阵 A, 都存在可 逆 ( 或
满秩 ) 矩阵 C, 使得
CT AC = Λ= diag ( d1 , d2 , … , dn ) .
T
做坐标变换   x = Cy( 其中 y = ( y1 , y2 , … , yn ) ) , 则
T T T
f ( x1 , x2 , … , xn ) = x Ax = y C ACy
= d1 y21 + d2 y22 + … + dn y2n ,
即把一般的二次齐次多项 式化 为了纯 平方 项之和 .这 在空 间解 析
几何中研究一般的三元二 次方 程所表 示的 二次曲 面的 图形 , 以 及
在 n 元函数中研究极值点的充分条件都有重要的应用 .

2  矩阵转置的 运算律

见内容提要 .
这里要特别注意 , ( AB) T ≠ AT BT .   矩阵乘积的转置满足 :
( AB) T = BT AT ,
( A1 A2 … An ) T = ATn … A2T A1T .
这个运算律可形象地称为“ 脱 衣法 则”, 即 A1 表示 内衣 , … , An 表
示外衣 , 右端括号内是穿衣顺序 , 左端是脱衣顺序 , 先脱外衣 , 后脱
内衣 .
k
若 A 为对称矩阵 , 则对于任意正整数 k, A 仍然 是对称 矩阵 .
因为
k T T T T T T k k
( A ) = ( AA… A) = A … A A = ( A ) = A .
而对于反对称矩阵 A, 由于
( Ak ) T = ( AT ) k = ( - A) k = ( - 1) k Ak ,
k T k k
因此 , 当 k 为奇数时 , ( A ) = - A , 所以 A 仍是反 对称矩阵 ; 当 k
k
为偶数时 , 则 A 为对称矩阵 .
但要注意 , 当 A, B 都是同阶对称矩阵时 , 其乘积 AB 并不一定
64 第 2 章 矩阵

是对称矩阵 , 因为
T T T
( AB) = B A = BA   ( BA 与 AB 不一定相等 ) .
由此可见 , 若 A, B 均是对称矩阵 , 则 AB 仍为对称矩阵的充分必要
条件是 AB 可交换 ( 即 AB = BA) .
同理 , 若 A, B 都是反对称矩阵 , 而 AB 为对称矩阵的充要条件
是 AB = BA .
例 1   设 A, B 分别为 n 阶 对称 矩阵 和 反对 称矩 阵 .问 : 正 整
k m m k
数 k, m 取何值时 , A B - B A 必为对称矩阵或反对称矩阵 .
解   讨论矩阵的对称性要利用矩阵的转置 , 由
k m m k T k m T m k T
( A B - B A ) = ( A B ) - (B A )
= ( Bm ) T (Ak ) T - ( Ak ) T (Bm )T
T m T k T k T m
= ( B ) (A ) - ( A ) (B )
= ( - 1) m Bm Ak - ( - 1) m Ak Bm ,
可知 : 当 m 为偶数 , k 为任意正整数时 ,
( Ak Bm - Bm Ak ) T = - ( Ak Bm - Bm Ak ) ,
k m m k
所以   A B - B A 为 反对 称 矩阵 ; 同 理 , 当 m 为奇 数 , k 为 任 意
正整数时 , Ak Bm - Bm Ak 为对称矩阵 .
T T T
例 2   设 α = ( 1 , 0 , - 1 ) , β= ( 1 , - 1 , 2 ) , A = αβ , 且
| kI - A5 | = k3 + 1 , 试求 k .
5 T 5 T T T T T
解   A = (αβ ) = (αβ ) (αβ ) (αβ ) (αβ ) (αβ )
= α(βT α) 4 βT = ( - 1) 4 αβT = A,
1 1 - 1 2
A = αβ = T
0 (1 , - 1 , 2 ) = 0 0 0 ,
-1 -1 1 -2
k- 1 1 -2
5 k- 1 -2
| kI - A | = | kI - A| = 0 k 0 = k
1 k+2
1 - 1 k+2
3 2 3
= k + k = k + 1,
2. 5 可逆矩阵及其逆矩阵 65

于是 ,   k2 = 1 , 所以   k = ±1 .

2. 5 可逆矩阵及其逆矩阵

1  可逆矩阵的 基本概念

一个方阵是否可逆 , 这是线性代数所研究的一个重要问题 .例
如 , 以 n 阶矩阵 A 为系数矩阵的齐次和非齐次线性方程组
Ax = 0,   Ax = b
是否有非零解和无穷多组解 , 都与 A 是否可 逆有 关 .再如 , 在三 维
T
空间中做一一对应的坐标变换 , 把坐标 x = ( x1 , x2 , x3 ) 变换为坐
标 y= ( y1 , y2 , y3 ) T , 也必须通过一个三阶可逆 矩阵 Q = ( qij ) 3 × 3 来
实现 , 即
Qx = y .
所谓一个 n 阶矩阵 A 可逆 , 就是如果 A 与 B 可交换 , 且 AB =
I, 即
AB = BA = I .
并称 B 是 A 的逆矩阵 , 记作 A - 1 = B .此时 , 也可 称 B 可逆 , A 是 B
的逆矩阵 , B - 1 = A .
(1 ) 如果矩阵 A 可逆 , 其逆矩阵是惟一的 .
在数学中证明惟一性的一种常用方法为 : 假设有两个 , 证明这
两个相等 .这里可设 B, C 均为 A - 1 , 利用可 逆矩 阵的 定义 , 可证 明
B= C( 详见主教材 ) .
( 2 ) 矩阵 A = ( ai j ) n× n 可逆的充分必要条件是 | A| ≠0, 且
- 1 1 *
A = A   ( 详见内容提要 ) , (2 .1 6)
| A|
A* = | A| A - 1 . (2 .1 7)
这个定理 , 证明其必要性时 , 利用 AB = I 可推 出 | A | | B| = 1 ,
66 第 2 章 矩阵

从而 | A| ≠0; 证明其 充分性时 , 利用伴随矩阵 A* 的定义和行 列式


展开的性质 , 得
AA* = A* A = | A| I,
从而有
1 * 1 *
A A = A A= I .
| A| | A|
(3 ) 如果两个 n 阶矩阵 A 与 B 之乘积 AB = I, 则 A 与 B 可 交
换 , 即必有 BA = I .
证明这个命题时 , 由 AB = I 推 出 | A | ≠ 0 , 再 由 ( 2 ) 可 知 A 可
逆 , 于是在 AB = I 的两端分别左乘 A - 1 , 右乘 A, 即
- 1 - 1
A ( AB) A= A IA = I,
从而得   BA = I .
由这个命 题 可 知 , 只 要 由 AB = I   ( A, B 均 为 方 阵 ) 就 可 判
断 : ① A 与 B 均可逆 , 且互 为 逆矩 阵 ; ② BA = AB = I, 即 A, B 可
交换 .

2  求逆矩阵的 3 种方法
- 1 - 1
(1 ) 利用定义 , 或 AB = I A = B, B =A.
例 1   对角矩阵 A= diag( a1 , a2 , … , an ) 可 逆的 充要条 件为 主
对角元 ai ≠0( i = 1 , 2 , … , n) , 且其逆矩阵
1 1 1
A - 1 = diag , , …,   ( 这由定义立即可得 ) .
a1 a2 an
0 0 1
例 2   求 B= 0 2 0 的逆矩阵 .
3 0 0
解   因为 | B| = - 6≠0 , 所以 B 可逆 , 利 用这类矩 阵的乘法 性
质及可逆矩阵的定义 , 立即可得
2. 5 可逆矩阵及其逆矩阵 67

1
0 0
3
- 1
B = 1 .
0 0
2
1 0 0
容易验证 BB - 1 = I .
2
例 3   设方阵 A 满足 A = 2 A, 证明 A - I 与 A + 2 I 均可逆 , 并
求其逆 ; 再问 A - 2 I 是否可逆 ? 并列举几个这样的三阶矩阵 A .
2 2
解   由 A = 2 A 得   A - 2 A = 0, 于是
A2 - 2 A = ( A - I) 2 - I = 0,
- 1
即   ( A - I) ( A - I ) = I, 所以 A - I 可逆 , 且 ( A - I) = A- I.
2
A - 2 A = ( A + 2 I) ( A - 4 I ) + 8 I = 0 ,
1
即   ( A + 2I ) - ( A - 4 I) = I,   所 以 A + 2 I 可 逆 , 且
8
- 1 1
( A + 2 I) = - ( A - 4 I) .
8
2
再由 A - 2 A = A( A - 2 I) = 0,   得
| A| | A - 2 I | = 0 .
此时 , 无法判定 | A - 2 I | 是否不 等于零 , 因而 这里无 法断定 A - 2 I
是否可逆 .以 后学 了第 3 章 , 就 可知道 , 只要 A≠0 , A - 2 I 都是 不
可逆的 .
满足 A2 = 2 A 的 A 的例子 , 最简单的可考虑对角矩阵 .
2 2 2 2 2
设   A = diag( a, b, c) , 则 A = diag( a , b , c ) , 由 A = 2 A 得
a2 = 2 a,     b2 = 2b,     c2 = 2 c,
所以 , a = 2 或 0 ,   b = 2 或 0 ,   c = 2 或 0 .   例如 , 可取
A = diag( 2 , 2 , 0) ,   则满足   A2 = 2 A .
如果要列举非对角矩阵的例子 , 可取
A = P - 1 ΛP,
其中 : P 是任意三阶可逆矩阵 , Λ是上面的对角矩阵 , 此时
68 第 2 章 矩阵

A2 = ( P- 1 ΛP) ( P - 1 ΛP) = P- 1 Λ2 P = P - 1 ( 2Λ) P


= 2 P- 1 ΛP = 2 A .
T
也可取 A = α β,
其中   α= ( a1 , a2 , a3 ) ,   β= ( b1 , b2 , b3 ) ,   于是
A2 = (αT β) (αT β) = αT (βαT )β
T
= α ( a1 b1 + a2 b2 + a3 b3 )β
= ( a1 b1 + a2 b2 + a3 b3 ) A .
T 2
只要 a1 b1 + a2 b2 + a3 b3 = 2 , 所得的 A = α β就满足 A = 2 A .
- 1 1 *
(2 ) 利用伴随矩阵 , A = A .
| A|
例 4   下列矩阵是否可逆 ? 如可逆求其逆 .
1 1 2 1 -2 0
(i) A= 0 2 1 ,     ( ii) B = 3 -4 0 ,
0 0 -2 0 0 4
1 0 1
( iii ) C = 1 -1 1 .
1 1 a
解   ( i ) | A| = - 4≠0 , 则 A 可逆 . A = ( aij ) 3 × 3 的各元 素的 代
数余子式为
2 1 0 1 0 2
A11 = = - 4 ,   A1 2 = - = 0 ,   A13 = = 0,
0 -2 0 -2 0 0
1 2 1 2 1 1
A21 = - = 2,   A22 = = - 2,   A23 = - = 0,
0 -2 0 -2 0 0
1 2 1 2 1 1
A31 = = - 3 ,   A3 2 = - = - 1 ,   A3 3 = =2 .
2 1 0 1 0 2
-4 2 -3 1 - 1/ 2 3/ 4
1 * 1
所以   A- 1 = A = - 0 -2 -1 = 0 1/ 2 1/ 4 .
| A| 4
0 0 2 0 0 - 1/ 2
2. 5 可逆矩阵及其逆矩阵 69

一般结论 : 上三角 矩阵 的 逆矩 阵仍 是上 三角 矩 阵 , 且其 主 对
角元是原矩阵的主对角元的倒数 .
( ii ) | B| = 8 , 故 B 可逆 .将 B 表示为对角块矩阵
B1 0
B= ,
0 B2
其中
1 -2 - 1 1 -4 2 -2 1
B1 = ,   B1 = = ,
3 -4 2 -3 1 - 3/ 2 1/ 2
1
B2 = 4 ,   B2- 1 = .
4
根据后面讲的对角块矩阵的逆矩阵的结论 , 得
- 1 -2 1 0
- 1
B1 0
B = = - 3/ 2 1/ 2 0 .
0 B2- 1
0 0 1/ 4
( iii ) | C| = 1 - a, 因此 , 当 a = 1 时 , C 不可逆 , 当 a≠1 时 , C 可
逆 , 其逆矩阵为
-1- a 1 1
- 1 1 * 1
C = C = 1 - a a-1 0 .
| C| 1- a
2 -1 -1
(3 ) 用初等行变换或初等列变换求逆 矩阵 .( 见 2. 6 节初等 变
换与初等矩阵中的例 ) .

3  可逆矩阵满 足的运算 律

详见内容提要 .
这里要特别注意 : A 和 B 均可逆 , 则 AB 也可逆 , 且
- 1 - 1 - 1
( AB) =B A ; 
- 1 - 1 - 1 - 1
( A1 A2 … An ) = An … A2 A1   ( 脱衣法则 ) ;
T - 1 - 1 T
(A ) = (A ) ;
70 第 2 章 矩阵

- 1 1
|A | = .
| A|
但是 , A 和 B 均可逆 , 不能推出 A + B 也 可逆 ( 读者 不 难用 对角 矩
阵举出例子 ) .即使 A+ B 可逆 , 也可能
- 1 - 1 - 1
( A + B) ≠A +B   ( 也用对角矩阵举例 ) .
这在数的运算里也是如此 , 非零数 a 与 b 之和 a + b≠0 时 , 有
- 1 1 1 1 - 1 - 1
( a + b) = ≠ + =a +b .
a+ b a b
关于可逆矩阵满足的运算律 , 既要能熟练地加以证明 ( 详见教
材 ) , 又要会灵活地应用它们解决各种问题 .
例 5   关于对称矩阵与反对称矩阵的逆矩阵 .
- 1
(1 ) 可逆对称矩阵 A 的逆矩阵 A 仍是对称矩阵 .
因为
- 1 T T - 1 - 1
(A ) = (A ) =A .
(2 ) 可逆反对称矩阵 B 的逆矩阵也是反对称矩阵 .因为
- 1 T T - 1 - 1 - 1
(B ) = (B ) = ( - B) = - B .
( 3 ) 奇数阶反对称 矩阵 B 一 定不 可 逆 .只 要 证明 此 时 | B | =
0 .设 B 为 n( 奇数 ) 阶矩阵 , 由于 BT = - B, 所以
T n
| B | = | B| = | - B| = ( - 1 ) | B| = - | B| ,
因此 | B| = 0 , 故 B 不可逆 .
例 6   已知
1 0 0 2 3 4
B= 2 1 0 ,   C= 0 2 3 ,
3 2 1 0 0 2
- 1 T - 1 - 1
且三阶矩阵 A 满足   ( I - C B ) C A = I,   试求 A .
解   将上述矩阵等式两端 , 先左乘 I - C- 1 BT , 再左乘 C, 即可
- 1 T T
得   A = C( I - C B ) = C- B .
也可先将上述矩阵等式利用可逆矩阵的运算律加以化简 , 即
2. 5 可逆矩阵及其逆矩阵 71

( I - C- 1 BT ) - 1
C- 1 A = C( I - C - 1 BT ) - 1
A
T - 1
= (C- B ) A = I,
于是 , 立即可得
2 3 4 1 2 3 1 1 1
A = C - BT = 0 2 3 - 0 1 2 = 0 1 1 .
0 0 2 0 0 1 0 0 1
例 7   已 知同 阶 方阵 A, B, C 满 足 ( I - C B) C A = C - 1 , 且 T T T

T
CC = I, C - B 可逆 , 求 A .
解   此题要先利用矩阵 转置与 可逆 矩阵满 足的 运 算律 , 将 题
中所给矩阵方程加以化简 , 然后再求 A, 即
( I - CT B) T CT A = [ C( I - CT B) ] T A = ( C - CCT B) T A
T - 1
= ( C - B) A = C . ( 1)
由于 | C - B | = | ( C - B) T | ≠ 0 , 所以 ( C - B) T 也可 逆 , 于 是 由 ( 1)
式得
A = [ ( C - B) T ] - 1 C- 1 = [ C( C - B) T ] - 1
T T - 1 T T - 1
= [C( C - B ) ] = ( CC - CB )
= ( I - CBT ) - 1 . ( 2)
T - 1 T
如果利用 CC = I, 得 C = C , 并代入 ( 1) 式 , 则得
A = [ ( C - B) T ] - 1 CT = [ ( C - B) - 1 ] T CT
- 1 T
= [ C( C - B) ] . ( 3)
这里 (2 ) 式与 ( 3) 式的结论都是正确的 ( 读者不难证明 ( 2) 式 , ( 3) 式
右端矩阵是 相等的 ) .从形式 上看 , ( 2 ) 式 更简洁一 些 .其实两者 的
计算量是差不多的 , 都要作一次减法 , 一次乘法 , 一次转置 , 一次求
逆 , 只不过是这四种运算的顺序不一样 .
例 8   已知 A = diag( 1 , - 2 , 1 ) , 且 A* BA = 2 BA - 8 E( E 为 单
位矩阵 ) , 求 B .
解   先利 用 矩 阵的 运 算律 , 化简 题 中 矩 阵方 程 .由 A* BA =
2 BA - 8 E, 得
72 第 2 章 矩阵

A* BA - 2 BA = ( A* - 2 E) BA = - 8 E .
所以
        BA = - 8( A* - 2 E) - 1
B = - 8( A* - 2 E) - 1 A - 1 = - 8[ A( A* - 2 E) ] - 1
- 1
= - 8( | A| E - 2 A)   ( 其中 | A| = - 2 )
- 1 - 1
= - 8( - 2 E - 2 A) = - 8[ diag( - 4 , 2 , - 4 ) ]
1 1 1
= - 8diag - , , - = diag (2 , - 4 , 2 ) .
4 2 4

例 9 ( 习 题 73 )   设 A 为 三 阶 矩 阵 , | A | > 0, A = diag ( 1 ,
- 1 , - 4) , 且
- 1 - 1
ABA = BA + 3 I,
求 | A| 和矩阵 B .
解   由 AA* = | A| I, 得 | A| | A* | = | A| 3 ,   所以
| A| 2 = | A* | = 4 ,     | A| = ±2 ,   取 | A| = 2 > 0 .
- 1 - 1
再由 ABA = BA + 3 I, 得
- 1 - 1 - 1
ABA - BA = ( A - I) BA = 3 I,
所以
- 1 - 1 - 1 - 1 - 1
B = 3 ( A - I) A = 3[ A ( A - I) ] = 3( I - A )
- 1 - 1
1 * 1 *
=3 I- A =3 I- A
| A| 2
- 1
1 3 2 1
= 3 diag , ,3 = 3diag 2 , ,
2 2 3 3
= diag (6 , 2 , 1 ) .

4  以 n 阶 矩阵 A 为系数 矩阵的线 性方程组的 解

当系数矩阵 A 不可逆时 , 齐次线性方程组 Ax = 0 和非齐次 线


性方程组 Ax = b 均 用 高 斯消 元 法 求 解 , 前 者 必 有 无 穷 多 组 非 零
解 , 后者有解时 , 也有无穷多组解 .
当系数矩阵 A 可 逆时 , Ax = 0 只有 零 解 , 即 x = A - 1 0 = 0; 而
2. 6 矩阵的初等变换和初等矩阵 73

Ax = b 有且仅有一个解 , 此时可利用 A - 1 ( 一 个矩阵的 逆矩阵是 惟


一的 ) 求其解 , 即由
- 1 - 1 - 1
A Ax = A b   得   x = A b .
例 10   解下列线性方程组
x1 - x2 + 2 x3 = - 3 ,
2 x1 - x2 - x3 = 1 ,
x1 + x2 - 2 x3 = 5 .
解   由于系数矩阵 A 的行列式
1 -1 2
| A| = 2 -1 - 1 = 2 + 4 + 1 + 2 - 4 + 1 = 6≠ 0 ,
1 1 -2
所以 A 可逆 , 于是方程组的解
x1 -3 3 0 3 -3 1
- 1 1
x= x2 =A 1 = 3 -4 5 1 = 2 ,
6
x3 5 3 -2 1 5 -1
即方程组的惟一解为 x1 = 1 , x2 = 2 , x3 = - 1 .

2. 6 矩阵的初等变换和初等矩阵

对矩阵做初等变换是 线性代数中解决很 多问题的一种 重要方


法 .例如 , 对线性方程组的增广矩阵做初等行变换将其化为阶梯 形
矩阵 , 得到容易求解的同解方程组 ; 用初等行变换或初等列变换可
以判断矩阵是否 可 逆 , 并 求其 逆 矩阵 ; 用初 等 变 换 可以 求 矩 阵 的
秩 , 向量组的秩及其极大线性无关组 ; 对一般二次曲面方程对应的
三阶实对称矩阵 , 同时做同样类型的初等行变换和列变换 , 将其化
为对角阵 , 从而将一般二次曲面方程化为标准方程 , 并得到此时所
做的坐标变换式等 .因此 , 读者必须熟练地掌握矩阵的 3 种初等行
( 列 ) 变换及相应的 3 种初等矩 阵 , 并熟 悉 3 种初 等矩阵 左 ( 右 ) 乘
74 第 2 章 矩阵

矩阵所起的 3 种行 ( 列 ) 变换 的作 用 ( 详见 内容 提要 ) .这里 还要 搞
清楚以下几个问题 :
(1 )   为什么 3 种初等矩阵的逆矩阵都是同类型的初等矩阵 ?
即为什么
1
Ei- 1 ( c) = Ei ,   Eij- 1 ( c) = Eij ( - c) ,   Ei-j 1 = Eij   ?
c
这是因为初等矩阵是单 位矩阵 做一 次初等 变换 得 到的 矩阵 ,
所以对初等矩阵再做一次同样类型的初等变换就能使之回到单位
矩阵 .例如 , 对倍乘初等矩 阵 Eij ( c) , 再做一 次初 等 行变 换 , 将第 i
行乘 - c 加到第 j 行 , Ei j ( c) 就变为单位矩阵 I, 即
Eij ( - c) Eij ( c) = I,   所以   Eij- 1 ( c) = Ei j ( - c) .
(2 )   为什么初等变换能判定矩阵的可逆性
这是因为对方阵作初等 变换 , 不会 改变 方阵行 列 式的 零或 非
零性 .因此 , 如果对方阵 A 做初等行 变换将 其化 为阶 梯形矩 阵时 ,
当它出现全零行时 , 就说明 | A| = 0 , 从而 A 不可 逆 ; 当 它不出现 全
零行 , 即阶梯形矩阵 为上 三角 矩 阵 , 且其 主对 角 元均 不 等于 0, 就
说明 | A| ≠0, 从而 A 可逆 .
(3 )   为什么只用 初等行 变换 ( 或 只用 初等列 变换 ) 能 求可 逆
矩阵的逆矩阵
这是因为对可逆矩阵做 初等行 变换 , 可 将其化 为 主对 角均 非
零的上三角矩阵 , 再 对每 行 做倍 乘变 换 , 可 将主 对角 元 全化 为 1 ,
然后再做初等行变换 , 就可将其化为单位矩阵 .所以对可逆矩阵做
若干次初等行变换就可 将其 化为 单位 矩阵 , 即存 在初 等矩 阵 P1 ,
P2 , … , Ps , 使
Ps … P2 P1 A = I, ( 2 .18 )
从而
A - 1 = Ps … P2 P1 = Ps … P2 P1 I . (2 .1 9)
因此 , 由 ( 2 .18 ) , ( 2 .19 ) 式可见 , 对 A 与 I 做同样的初等行变换 , 当
2. 6 矩阵的初等变换和初等矩阵 75

A 变为 I 时 , I 就变为 A - 1 , 即
初等 行变 换 - 1
( A, I ) ( I, A ) .
同理
A 初等 列变 换 I
- 1
.
I A
这里要特别注意 , 用初等变换求逆矩阵时 , 如用初等行变换必
须始终做初等行变换 , 其中不能做任何初等列变换 .
(4 )   为什么可逆矩阵都可分解为若干个初等矩阵的乘积 ?
这是因为从 (2 .1 8) 式可得
- 1 - 1 - 1 - 1
A = ( Ps … P2 P1 ) = P1 P2 … Ps , (2 .2 0)
- 1
其中 Pi ( i = 1 , … , s) 是与 Pi 同类型的初等矩阵 .
例1  设
3 5 5 1 2
A= 2 4 3 ,   B= -2 1 ,
-2 -2 -3 1 3
且 AX = B, 试求 X .
- 1
解   法 1: 由 于 | A | = 2 ≠ 0 , A 可 逆 , 所 以 X = A B .求 出
A- 1 , 再算 A - 1 B, 即得 X .
法 2: 由于 A 可逆 , 所 以存 在若 干初 等 矩阵 P1 , P2 , … , Ps , 使
Ps … P2 P1 A = I, 于是
Ps … P2 P1 AX = Ps … P2 P1 B, ( 1)
X = Ps … P2 P1 B = A - 1 B . ( 2)
由 ( 1 ) , ( 2 ) 式可见 , 对 A, B 做同样的初等行变换 , 当 A 变成 I 时 , B
就变成了 X .即
3 5 5 1 2 1 1 2 3 1
① + ②×( - 1)
( A, B ) = 2 4 3 -2 1 2 4 3 -2 1
③+ ②
-2 -2 -3 1 3 0 2 0 -1 4
76 第 2 章 矩阵

1 1 2 3 1 1 1 2 3 1
② + ①×( - 2 ) ③ + ②×( - 1 )
0 2 -1 -8 -1 0 2 -1 -8 -1
0 2 0 -1 4 0 0 1 7 5

21
1 0 0 - - 11
1 1 0 - 11 - 9 ① + ②× -
1 2
②+ ③ 2
0 2 0 -1 4 1 .
① + ③×( - 2) 1 0 1 0 - 2
②× 2
0 0 1 7 5 2
0 0 1 7 5
所以
21
- - 11
2
X= 1 .
- 2
2
7 5
例2  设

1 -1 0 0 2 1 3 4
0 1 -1 0 0 2 1 3
B= ,   C= ,
0 0 1 -1 0 0 2 1
0 0 0 1 0 0 0 2
- 1 - 1 - 1
且 A( I - C B) C B= I, 求 A .
解   由 A( I - C - 1
B) - 1 C- 1 B = A[ C( I - C - 1 B) ] - 1 B
- 1
= A( C - B) B
= I,
- 1
得 A = B ( C - B) .
用初等列变换法求 B- 1 .
2. 6 矩阵的初等变换和初等矩阵 77

1 -1 0 0 1 0 0 0 1 0 0 0
0 1 -1 0 0 1 -1 0 0 1 0 0
0 0 1 -1 0 0 1 -1 0 0 1 0
B
0 0 0 1 [2] + [1] 0 0 0 1 [3] + [2] 0 0 0 1
= ,
1 0 0 0 1 1 0 0 [4] + [3] + [2] 1 1 1 1
I
0 1 0 0 0 1 0 0 0 1 1 1
0 0 1 0 0 0 1 0 0 0 1 1
0 0 0 1 0 0 0 1 0 0 0 1
所以
1 1 1 1 1 2 3 4 1 3 6 10
0 1 1 1 0 1 2 3 0 1 3 6
  A= B- 1 ( C - B) = = .
0 0 1 1 0 0 1 2 0 0 1 3
0 0 0 1 0 0 0 1 0 0 0 1
1 2
例 3   将可逆矩阵 A = 表示为若干初等矩阵的乘积 .
3 4
解   对矩 阵 A 做若 干次 初等 行 变换 将 其化 为 单位 阵, 按
( 2 .18 ) 式即得 A= ( Ps … P2 P1 ) - 1 = P1- 1 P2- 1 … Ps- 1 .由
1 0 1 2 1 2
= ,
-3 1 3 4 0 - 2
1 1 1 2 1 0
= ,
0 1 0 - 2 0 - 2
1 0
1 0 1 0
1 = = I,
0 - 0 -2 0 1
2

1 0
1 1 1 0 1 2
1 = I,
0 - 0 1 -3 1 3 4
2
78 第 2 章 矩阵

所以
- 1
1 0
1 2 1 1 1 0
A = = 1
3 4 0 - 0 1 -3 1
2
- 1
- 1 - 1 1 0
1 0 1 1
= 1
-3 1 0 1 0 -
2
1 0 1 -1 1 0
= ,
3 1 0 1 0 -2

A= E12 ( 3 ) E21 ( - 1 ) E2 ( - 2) .

2. 7 分块矩阵

把矩阵表示为 分块 矩 阵进 行 运 算是 矩 阵 运 算 的一 个 重 要 技
巧 , 它把大型矩阵化为 若干小 型矩 阵的运 算 , 使运 算更 为简 明 .而
且它有助于一 些命题 ( 定 理 ) 的证明 .例 如书中 例 4 证 明可逆上 三
角矩阵的逆矩阵也是上三 角矩 阵 , 用 数学 归纳 法和矩 阵分 块的 方
法加以证明就比较方便 , 如果不用分块的方法证明就比较麻烦 .
关于分块矩阵的加法、数乘、乘法、转置等运算 , 要特别注意乘
法和转置 .
两个可乘矩阵 A, B 相 乘用 分块 的 方法 计 算 AB, 不 仅 要求 A
的列的块数与 B 的行的块数相等 , 而且要 求 A 的 列的 分块法 与 B
的行的分块法完全一致 .以保证对应子块可乘 ( 见内容提要 ) .
A11 A12 A13
分块 矩 阵 A = 的 转 置不 仅 要行 ( 块 ) 与 列
A21 A22 A23
( 块 ) 互换 , 而且每一子块也要转置 , 即
2. 7 分块矩阵 79

T T
A11 A2 1
T T
A = A12 A2T2 .
A13T A2T3
有些可逆矩阵也常用分 块的方 法求 其逆矩 阵 .最 简单 的对 角
块矩阵 A( 也称准对角矩 阵 ) , 当 每一 个对 角块 Ai i 均 可逆 时 , 其 逆
矩阵
- 1
A11 A11- 1
- 1
- 1
A22 A22
A = = .
w w
- 1
Ann Ann
分块矩阵也可做初等行 ( 列 ) 变换 , 相应 地也 有 3 种分 块初 等
矩阵 , 它们左 ( 或右 ) 乘分块矩阵 A( 或 B) 的作用 , 与不 分块的初 等
矩阵的作用是类似的 ( 请详见教材 ) .
例 1   设分块矩阵 P = ( A, B) , 其中 A 是 n 阶 可逆矩 阵 , B 是
n× m阶矩阵 , 试求矩阵 Q, 使 PQ= In ( n 阶单位矩阵 ) .
解   P 是 n× ( n + m) 矩阵 , 因 此 Q 应 是 ( n + m) × n 矩 阵 .现
将 Q 也表成分块矩阵
C
Q= .
D
为使分块矩阵 PQ 可乘 , 其中子块 C 是 n 阶 矩阵 , 子块 D 为 m × n
矩阵 .于是由
C
PQ= ( A, B) = AC+ BD = In ,
D
A- 1
所以可取 C = A , D = 0m× n , 即 Q=
- 1
.
0
思考题   在什么情况下 , D 可取非零矩 阵 ? ( 答案 : 当齐次 线
性方程组 Bx = 0 有非 零解 , 把 它的 n 个 非 零解 按 列排 成 的 矩阵 ,
可取为 D .)
80 第 2 章 矩阵

0 0 A1
例 2   设 A= 0 A2 0 , 其 中 A1 , A2 , A3 分 别为 m 阶 , n
A3 0 0
- 1
阶 , k 阶可逆矩阵 , 证明 A 可逆 , 并求 A .
解   先证 | A| ≠ 0 ( A 可逆 ) .将 A1 所 在的 m 个列与 A2 , A3 所
在的 n + k 个列逐列对换 ( 共 对换 m ( n + k) 次 ) , 再将 A2 所在 的 n
个列与 A3 所在的 k 个列逐列对换 ( 共对换 nk 次 ) , 即得
A1 0 0
| A| = ( - 1) m n + m k + nk 0 A2 0
0 0 A3
= ( - 1) m n + m k + nk | A1 | | A2 | | A3 | ≠0 .
0 0 B1
取 B= 0 B2 0 , 其中 B1 , B2 , B3 分别为 k 阶 , n 阶 , m 阶
B3 0 0
矩阵 .此时分块矩阵 A 与 B 可乘 , 并得
0 0 A1 0 0 B1 A1 B3 0 0
AB= 0 A2 0 0 B2 0 = 0 A2 B2 0 .
A3 0 0 B3 0 0 0 0 A3 B1
- 1 - 1
当 A1 B3 = Im ( 即 B3 = A1 ) , A2 B2 = In ( 即 B2 = A2 ) , A3 B1 = Ik ( 即
B1 = A3- 1 ) 时 , AB= I, 所以 B= A - 1 .即
- 1
0 0 A1 0 0 A3- 1
- 1
0 A2 0 = 0 A2 0 .
A3 0 0 A1- 1 0 0
A C
例 3  设 P= , 其中 A, B 分别为 m 阶 , n 阶可 逆矩阵 .
0 B
- 1
 求P .
解   分块矩阵 P 显然是可逆的 , 因为 | P| = | A| | B| ≠0 .
2. 7 分块矩阵 81

由于对角块矩阵的逆矩阵容易求 , 所 以先 对分 块矩阵 P 做 初
- 1
等行变换 , 将其第 2 行左乘 - CB 加到第 1 行消去矩阵 C, 使 P 化
为对角块矩阵 , 即
Im - CB - 1 A C A 0
= .
0 In 0 B 0 B
于是
- 1 - 1
A C Im - CB A 0
= .
0 B 0 In 0 B
所以
- 1 - 1 - 1
- 1
A C A 0 Im - CB
P = =
0 B 0 B 0 In
- 1
A- 1 0 Im - CB A- 1 - A - 1 CB- 1
= = .
0 B- 1 0 In 0 B- 1
例 4   设 A, B, C, D 均为 n 阶矩阵 , | A| ≠ 0 , AB = BA .试计 算
A B
行列式 .
C D
解   这是 2 n 阶分块矩阵的行列式 , 读者不能像二阶数字行列
式那样展开为 | AD - BC| .
由于分块初等矩阵以及上三角块和下三角块矩阵的行列式容
易求 , 所 以 先 对 这 个 分 块 矩 阵 做 初 等 列 变 换 , 将 其 第 1 列 右 乘
- 1
- A B 加到第 2 列 , 消去 B, 使之成为下三角块矩阵 , 即
A B I - A- 1 B A 0
= - 1
,
C D 0 I C D - CA B
其中等式左端第二个倍加分块初等矩阵的行列式等于 1, 所以
A B A 0
    = - 1
= | A| | D - CA - 1 B|
C D C D - CA B
- 1 - 1
= | D - CA B| | A| = | ( D - CA B) A|
82 第 2 章 矩阵

= | DA - CA - 1 BA| = | DA - CA - 1 AB| = | DA - CB| .


上面第 3 个 等号处 利用了两 个矩阵的 行列式 相乘可交 换 , 第 4 个
等号处利用了两个矩阵乘 积的 行列式 等于 矩阵行 列式 的乘 积 , 第
6 个等号处利用了 AB= BA .
这个题也可对分块矩阵 做初等 行变 换 , 将其化 为 上三 角块 矩
阵 , 可得到同样的结果 .

2. 8 部分疑难习题和补充题的题解

1 ( 第 7 题 )   将军点 兵 , 三 三数之 剩二 , 五五 数之 剩三 , 七 七数之 剩二 ,


问兵几何 ( 求在 500 至 1 000 范围内的解 ) ?
解   设兵数为 s, 3 人 , 5 人 , 7 人 组的 组数 分别 为 x , y, z .按 题意 可列 一
个关于 s, x, y , z 的非齐次线性方程组
s - 3 x = 2,
s- 5y=3,
s- 7z=2,
其解应都为正整数 .此方程组的增广矩阵为
1 -3 0 0 2 1 0 0 - 7 2
初 等行变 换
1 0 - 5 0 3 0 3 0 - 7 0 .
1 0 0 - 7 2 0 0 5 - 7 - 1
取 z 为自由未知量 , 解得
s = 2 + 7 z,
7
x= z,
3
1
y= ( - 1 + 7 z) .
5
为保证 s, x, y , z 均为正整数 , 取 z = 3 + 15 k( k 为任意正整数 ) , 解得
s = 23 + 105 k,   x = 7 + 35 k,   y = 4 + 21 k .
为了求兵数 s 在 500 至 1 000 范围内的解 , 取 k = 5 , 6 , 7 , 8 , 9 , 得 s = 548 , 653 ,
758 , 863 , 968 .
2. 8 部分疑难习题和补充题的题解 83

2 ( 第 21 题)   已知 A = PΛQ,其中
2 3 1 0 2 - 3
P= ,   Λ= ,   Q= ,   QP = I2 .
1 2 0 - 1 - 1 2
计算 A8 , A9 , A2 n , A2 n+ 1 ( n 为正整数 ) .
解  Am = ( PΛQ) ( PΛQ) … ( PΛQ)
= PΛIΛ… IΛQ= PΛm Q .
当 m 为偶数时 , Λm = I, Am = PIQ= I2 ,所以 A8 = A2 n = I2 .
当 m 为奇数时 , Λm = Λ, 所以
2 3 1 0 2 - 3
A9 = A2 n+ 1 = PΛQ=
1 2 0 -1 - 1 2
2 - 3 2 - 3 7 - 12
= = .
1 - 2 -1 2 4 - 7
n
0 1
3 ( 第 23 题)   计算 .
-1 0
0 1
解   记 A= , 则 A2 = - I,   A3 = - A, A4 = I,于是
- 1 0
A5 = A, A6 = - I, A7 = - A, A8 = I,…
其一般结论为
A,     n 为奇数 ,
2 n- 1
A =
- A, n 为偶数 ;
- I,   n 为奇数 ,
A2 n =
I, n 为偶数 .
或利用
n
f ( n) = cos π,   f ( 1) = 0 , f (2 ) = - 1 , f (3 ) = 0 , f (4) = 1 ,
2
n
φ( n) = sin π,   φ(1 ) = 1 , φ(2 ) = 0 , φ(3 ) = - 1 , φ(4 ) = 0 .
2

f (1 ) φ( 1) f (2) φ(2 )
A= ,   A2 = ,
- φ(1 ) f (1) - φ(2) f ( 2)
84 第 2 章 矩阵

f (3) φ(3 ) f ( 4) φ(4 )


A3 = ,   A4 = ,
- φ(3 ) f ( 3) - φ( 4) f ( 4)
如此周期性地变化 , 一般地 , 有
nπ nπ
cos sin
f ( n) φ( n) 2 2
An = = .
- φ( n) f ( n) nπ nπ
- sin cos
2 2
4 ( 第 26 题)   求平方等于零矩阵的所有二阶矩阵 .
a b a2 + bc ab + bd
解   设 A= , 则由 A2 = 得
c d ac+ cd bc + d2
a2 + bc = bc+ d2 = 0 , ( 1)

ab+ bd = ac + c d = 0 . ( 2)
2 2
由方程( 1) 得 a = d , 所以 d = ± a, 再 由方程 (2 ) 可见 , 当 d = - a 时 , 等式 成
立 .因此取 d = - a, 欲使方程 (1 ) 成立 , 还要求 bc = - a2 .于是 即得 A2 = 0 的
所有 A 为
a b
A= ,   其中   bc= - a2 .
c - a
5 ( 第 30 题)   证明 : 两个 n 阶下三角矩阵的乘积仍是下三角矩阵 .
解   此题可按教材 2. 2.3 节中例 4 的方法类似地证明 .这里我们用 分块
矩阵的方法证明 .这要用对 n 作数学归纳法 .
当 n = 2 时 , 即两个下三角矩阵的乘积显然仍是下三角矩阵 ( 略去证明 ) ,
假设两个 n - 1 阶 下三 角 矩 阵的 乘 积 仍是 下 三 角矩 阵 .下 面 考 虑 n 阶 的 情
况 .设
a11 0 b11 0
A= ,   B= ,
α A1 β B1
其中: 两个 0 均为 1× ( n - 1) 零矩 阵 , α,β均 为 ( n - 1) ×1 矩阵 , A1 , B1 均 为
n - 1阶下三角矩阵 .于是
a11 b11 0
AB = ,
αb11 + A1 β A1 B1
其中 0 是 1× ( n - 1) 零 矩阵 , 根据归纳法 假设 A1 B1 为 n - 1 阶下三 角矩阵 ,
所以 , 两个 n 阶下三角矩阵 A 与 B 的乘积 AB 仍是下三角矩阵 .
2. 8 部分疑难习题和补充题的题解 85

附带说明一点 ,这里用数学归纳法 ,也 可直接 说结论 对 n = 1 成立 (因 为


一阶矩阵也可以说是下三角矩阵 , 上三 角矩阵 , 对 称矩阵 ) , 然后假 设结论 对
n - 1 阶成立 , 再证明 n 阶的情况 .
6 ( 第 38 题)   设 A 是实对称矩阵 ,且 A2 = 0,证明 A = 0 .
证   设 A = ( aij ) n× n , aji = aij   ( i, j = 1 , 2 , … , n) .
由 A2 = B = ( bij ) n× n = 0 .得
bii = ai1 a1 i + ai2 a2 i + … + ain an i
= a21i + a22 i + … + a2ni = 0   ( i = 1 , 2 , … , n) . ( 1)
因为 ai j ( i, j = 1 , … , n) 均为实数 , 所以由 (1) 式得
a1 i = a2 i = … = ani = 0   ( i = 1 , 2 , … , n) ,
即 A 的所有元素 a ij = 0 ( i, j = 1 , … , n) , 故 A = 0 .
7 ( 第 44 题 )   设 A, B 都是 n 阶矩阵 , 问 : 下列命题是 否成立 ? 若成立 ,
给出证明 ; 若不成立 , 举反例说明 .
( 1) 若 A, B 皆不可逆 ,则 A + B 也不可逆 ;
( 2 ) 若 AB 可逆 ,则 A, B 都可逆 ;
( 3 ) 若 AB 不可逆 ,则 A, B 都不可逆 ;
( 4 ) 若 A 可逆 ,则 kA 可逆 ( k 是数) .
1 0 0 0
解   ( 1) 不 成 立 .例 如 ( n = 2) : A = , B= 均不可逆, 但
0 0 0 1
A + B = I可逆 .
( 2 ) 成立 .因为 AB 可逆 | AB| = | A| | B| ≠0 ,所以 , | A| ≠0 且 | B | ≠0,
因此 , A, B 均可逆 .
( 3) 不成立 .因 为由 | AB| = | A | | B| = 0 不能 推 出 | A | = 0 且 | B | = 0 .
  例如 :
1 0 0 0 0 0
A= ,   B= ,   AB = ,
0 2 0 1 0 2
AB 不可逆 , 但 A 可逆 .
( 4) 成立否与 k 有关 .由 A 可逆 , 即 | A| ≠0 及 | kA | = kn | A| ≠0, 得 k≠0 ,
所以 k≠0 时 , kA 可逆 ,而 k = 0 时 , kA = 0 不可逆 .
8 ( 第 48 题)   试求上 (或下 )三角矩阵可逆的充要 条件 , 并 证明 :可 逆上
(或下 )三角矩阵的逆矩阵也是上( 或下) 三角矩阵 .
86 第 2 章 矩阵

解   由于上 (或下 ) 三角行列式等于主对角元的乘积 a11 a22 … ann , 所 以上


(或下 )三 角 矩 阵 可 逆 的 充 要 条 件 是 a11 a22 … ann ≠ 0 , 即 主 对 角 元 aii 均 不
等于零 .
证明可逆上三角矩阵的逆矩阵也是上三角矩阵 , 我们用对矩阵阶数 n 作
数学归纳法的方法 .当 n = 1 时结论 显然成 立 , 假 设结论 对 n - 1 阶 上三角 矩
阵成立 , 下面证明对 n 阶也成立 .
先将 n 阶上三角矩阵分块表示为
a11 α
A= , ( 1)
0 A1
其中 A1 为 n - 1 阶上三角矩阵 .设分块矩阵 B 为
b11 β
B= ,   ( 其中 B1 为 n - 1 阶矩阵 ) ( 2)
γ B1
为 A 的逆矩阵 ,则
a11 α b11 β
AB =
0 A1 γ B1
a11 b11 + αγ a11 β+ αB1 1 0
= = = In . ( 3)
A1 γ A1 B1 0 In - 1
由( 3) 式得
A1 γ= 0, 因为 A1 可逆, 所以 γ= A1- 1 0 = 0;
A1 B1 = In - 1 , 根据归纳假设 B1 = A1- 1 为上三角矩阵 .
b11 β
所以 A - 1 = B = 为上三角矩阵 .
0 B1
这里再进一步 ,可以证明 B = A- 1 的主对角元 b11 , b22 , … , bnn 分别 为 A 的
对应主对角元的倒数 ,即
b11 = a11- 1 ,   b22 = a22- 1 ,   … ,   bnn = ann- 1 .
同样用数学归纳法证明 , 由 (3 )式得
a11 b11 + αγ= a11 b11 + 0 = 1 ,   所以   b11 = a11- 1 .
由归纳假设 , 又知 B1 = A1- 1 的主 对角 元分 别为 A1 主 对角 元的 倒数 .故结 论
成立 .
同理 , 对可逆下三角矩阵也可证明相应的结论 .
2. 8 部分疑难习题和补充题的题解 87

9 ( 第 53 题)   用初等变换法求矩阵 A 的逆矩阵 .
0 a1 0 … 0
0 0 a2 … 0
A= … … … w … ,   其中   ai ≠0 ,   i = 1 , 2 , … , n .
0 0 0 … an - 1
an 0 0 … 0
解   法 1 : 将 A 的第 n 行与前面的 n - 1 个逐行对换 , 把第 n 行 换到第 1
行 , 而第 n - 1 , … , 2 , 1 行 依 次 换 到 第 n, … , 3 , 2 行 , 这 样 变 换 为 对 角 矩 阵
dia g( an , a1 , a2 , … , an - 1 ) ;继续对每行做倍乘变换 , 对 ai 所在行乘 a i- 1 , 就把 A
化为单位矩阵 , 从而得 A - 1 .即
an 0 1
a1 1 0
逐行对换
( A, I ) a2 1 0
w w w
an - 1 1 0
1 0 a -n 1
1 a1- 1 0
a i 所在 行
1 a2- 1 0 = ( I, A - 1 ) .
乘 a i- 1

w w w
1 an- 1- 1 0
( 1)
法 2 : 将矩阵 A 分块表示为 ( 副对角块矩阵 )
0 A1
A= ,   其中   A1 = diag( a1 , a2 , … , an - 1 ) .
an 0
根据 2. 7 节中例 2 可见 , 取 A - 1 为 n 阶分块矩阵
0 bn
B= ,   其中 B1 为 n - 1 阶矩阵 ,
B1 0
则 AB 为对角块矩阵 ,即
A1 B1 0 In - 1 0
A A - 1 = AB = = . ( 2)
0 an bn 0 1
88 第 2 章 矩阵

由( 2) 式得 :
A1 B1 = In - 1 ,   即   B1 = A1- 1 = diag( a1- 1 , a2- 1 , … , an- -1 1 ) ;
an bn = 1 ,   即   bn = an- 1 .

- 1
0 an- 1
从而得 B = A = - 1
, 这与( 1) 式中 A - 1 是一样的 .
A1 0
关于这个题 , 如果对矩阵乘法掌握得很熟练 , 直接由定义可得 A- 1 如 (1 )
式中所示的那个矩阵 .

10 (第 65 题 )   将 n 阶矩阵 A 分块为
An - 1 b
A= ,
c ann
其中 An - 1 是 n - 1 阶可逆矩阵 , 如果 A 可逆 , 且已 知 An- -1 1 , 试 求 A- 1 ( 这种 利
用 An- -11 求 A - 1 的方法 ,称为加边法 ) .
解   先将 A 的第 1 行左乘 - cAn- -1 1 加到 第 2 行 , 消去 c, 再做行 变换消 去
b,使 A 化为对角块矩阵 , 如此即可求得 A - 1 , 即
In - 1 0 An - 1 b An - 1 b
= . ( 1)
- cAn- -1 1 1 c ann 0 ann - cAn- -11 b
记上式中
ann - cAn- -11 b= k( k 是一个数) , ( 2)
则 k≠0 .因为由( 1) 式得
| A| = | An - 1 | ・ k≠0 .
b
于是再将 ( 1) 式右端第 2 行左乘 - 加到第 1 行消去 b, 即
k
b
In - 1 - An - 1 b An - 1 0
k = . ( 3)
0 k 0 k
0 1
综合( 1) , (3 )式 , 得
b
In - 1 - In - 1 0 An - 1 0
k A= .
- cAn- -1 1 1 0 k
0 1
从而有
2. 8 部分疑难习题和补充题的题解 89

b
An- -1 1 0 In - 1 - In - 1 0
k A = In .
0 k- 1 - cAn- -1 1 1
0 1
所以
b
An- -1 1 0 In - 1 - In - 1 0
A -1
= k
0 k- 1 - cAn- -11 1
0 1
1 -1
An- -1 1 - A b In - 1 0
= k n- 1
-1
- cA n- 1 1
0 k- 1
1 1 -1
An- -1 1 I+ bcAn- -1 1 - An - 1 b
k k
= . ( 4)
1 1
- cAn- -11
k k
这里要 注 意 : 其 中 k 是 ( 2 ) 式 所 示 的 一 个 数 , c 是 1 × ( n - 1 ) 矩 阵 , b 是
( n - 1 )×1矩阵 .
11 ( 第 69 题 )   设 A 为 四 阶 矩 阵 , 已 知 | A | = a ≠ 0 , 计 算 行 列 式
det ( | A* | A) .
解   利用 AA* = | A| I,得 | A| | A* | = | A| 4 , 从而 | A* | = | A| 3 = a3 , 因此
det ( | A* | A) = det ( a3 A) = ( a3 ) 4 | A| = a13 .
12 (第 72 题 )   设 α= ( x1 , x2 , … , xn ) T ,β= ( y1 , y2 , … , y n ) T , 已知 αT β=
3 , B = αβT , A = I - B .证明 :
( 1) Bk = 3 k - 1 B( k≥2 为正整数) ;
( 2) A + 2 I 或 A - I 不可逆 ;
( 3) A 及 A + I 均可逆 .
证   ( 1) Bk = (αβT ) (αβT )… (αβT ) = α(βT α) … (βT α)βT , 其中 有 k - 1个
βT α, 且 (βT α) = (αT β) T = (3 ) T = 3 , 所以
Bk = α( 3) k - 1βT = 3 k - 1 αβT = 3 k - 1 B .
( 2) 由 (1 )的 结果知 B2 = 3B, 再由 已知条件 B = I - A, 可推 出矩阵 A 满
足的一个方程 , 即
B2 = ( I - A) 2 = 3 B = 3 ( I - A) ,
I2 + A2 - 2 A = 3 I - 3 A,
90 第 2 章 矩阵

A2 + A - 2 I = 0 .
于是 ( A + 2 I) ( A - I) = 0 , 从而 | A + 2 I | | A - I | = 0 , 所以 A + 2 I 和 A - I 的 行
列式至少有一个等于零 ,因此 A + 2 I 或 A - I 不可逆 .
( 3 ) 再由 A2 + A - 2 I = 0 , 可 得 A( A + I) = 2 I, 从而 | A | | A + I | = 2 n ≠ 0 ,
所以 | A| ≠0 , | A + I | ≠0 , 即 A 与 A + I 均可逆, 且
1 1
A- 1 = ( A + I) ,     ( A + I) - 1 = A.
2 2
13 ( 第 75 题 )   设 A 为 奇 数 阶 可 逆 矩 阵 , 且 A - 1 = AT , | A | = 1,
求 | I - A| .
解   设 A 为 n( 奇数 )阶矩阵 .由 A - 1 = AT 得 AA T = I,于是
| I - A| = | AAT - A| = | A| | AT - I | = | AT - I | .
再由 | AT - I | = | ( AT - I) T | = | A - I | = | - ( I - A) | = ( - 1 ) n | I - A| =
- | I - A| , 得
| I - A| = - | I - A| ,   所以 | I - A| = 0 .
14 ( 第 80 题 )   设 B 是元素全为 1 的 n 阶矩阵 ( n≥2 ) , 证明 :
1
( 1 ) Bk = nk - 1 B( k≥2 为正整数 ) ; (2 ) ( I - B) - 1
= I- B.
n-1
证   ( 1)
1 1 … 1 1
1 1 … 1 1
B= = (1 , 1 , … , 1) = ααT ,
… … … …
1 1 … 1 1
其中 α= (1 , 1 , … , 1 ) ,α α= n .与 前 面 12 题 ( 1 ) 的 结 果 类似 地 可 得 Bk =
T T

nk - 1 B .
( 2) 由 (1 )的结论得 B2 = nB 即 B2 - nB = 0 , 欲证题中所 给的 ( I - B) - 1 .
应将 B2 - nB = 0 , 化为
( I - B) ( ?) = I .
于是由
B2 - n B = ( I - B) [ ( n - 1 ) I - B] - ( n - 1) I = 0,

( I - B) [ ( n - 1) I - B] = ( n - 1 ) I,
2. 8 部分疑难习题和补充题的题解 91

1 B
即 ( I - B) [ ( n - 1) I - B] = ( I - B) I- =I.
n-1 n -1
所以
1
( I - B) - 1 = I - B.
n- 1
1
此题的另一证法 : 欲证 ( I - B) - 1 = I - B,只要证
n-1
1
( I - B) I- B = I,
n- 1
1 B2
即 I- 1+ B+ = I,
n- 1 n-1
n B2
也即 - B+ =0 .
n- 1 n- 1
这只要证 B2 = nB .而由 (1) 的结论得 B2 = nB, 故结论成立 .
15 ( 第 82 题 )   已 知 P, A 均为 n 阶 矩阵 , 且 P- 1 AP = dia g( 1 , 1 , … , 1 ,
0 , … , 0) ( 其中 1 有 r 个 ) , 试计算 | A + 2 I| .
解   这里解题的关键是要把 2 I 表示为 P ( 2 I) P- 1 或 P - 1 ( 2 I) P .
法 1 : 由 P - 1 AP = diag ( 1, 1, … ,1, 0,… , 0 ) ,得
A = Pdiag ( 1 ,1 ,… ,1 ,0 ,… ,0 ) P - 1 .
于是
A + 2 I = Pdiag ( 1,1 ,… ,1 ,0 ,… ,0 ) P - 1 + P( 2 I) P- 1
= Pdiag ( 3,3 ,… ,3 ,2 ,… ,2 ) P - 1 ,
其中“3”有 r 个 “2”
, 有 n - r 个 , 所以
| A + 2 I| = | P| | diag ( 3, … ,3 ,2, … ,2 ) | | P- 1 |
= | P| 3 r 2 n - r | P| - 1
= 3 r ・2n - r .
法 2 : 由 P - 1 AP + 2 I = P- 1 AP+ P - 1 (2 I) P = P- 1 ( A + 2 I) P, 得
P- 1 ( A + 2 I) P = diag ( 1 ,… ,1 ,0 ,… ,0 ) + 2 I = diag ( 3 ,… , 3,2 ,… ,2 ) .
于是
| P - 1 ( A + 2 I) P| = | P- 1 | | A + 2 I| | P| = | A + 2 I |
= | diag ( 3 ,… ,3 ,2, … ,2 ) | = 3 r ・2n - r .
16 ( 第 83 题 )   设 A 为 n 阶 ( n≥2 )可逆矩阵 , 证明 :
( 1) ( A* ) - 1 = ( A - 1 ) * ;
92 第 2 章 矩阵

( 2) ( AT ) * = ( A* ) T ;

( 3 ) ( kA) = kn - 1 A* ( k 为非零常数 ) .
证   这里主要是利用可逆矩阵 B 的逆矩阵 B - 1 与 B 的伴随矩阵 B * 的关
系 ,即
1
B- 1 = B* ,     B* = | B| B - 1 . ( 1)
| B|
1
( 1) ( A* ) - 1 = ( | A| A - 1 ) - 1 = A = | A- 1 | A . ( 2)
| A|
再将 ( 1 ) 式中的 B 视为 A - 1 , 则有
( A- 1 ) * = | A- 1 | ( A- 1 ) - 1 = | A- 1 | A . ( 3)
由 ( 2 ) , ( 3 ) 式即得   ( A* ) - 1 = ( A - 1 ) * .
( 2) ( A* ) T = ( | A| A - 1 ) T = | A| ( A - 1 ) T = | A| ( AT ) - 1
= | AT | ( AT ) - 1 = ( AT ) * .
这里利用了 | A| = | AT | , 第一个等号和最后一个等号利用了 ( 1) 式 .
( 3) 利用 (1) 式 ,   得
1 -1
( k A) * = | k A | ( k A) -1
= kn | A| A = kn - 1 | A| A - 1
k
= kn - 1 A* .
这里需要指出 , ( 2 ) , ( 3 ) 的结论 ,当 A 不可逆时也 是成立 的 .这 要用伴 随
矩阵的定义来证明 .
设 A = ( aij ) n× n , 则 AT = ( aTji ) n× n , 其中 aTji = aij .由于 行列式 转置 其值 不
变 , 因此 , 矩阵 A 的 a ij 的余子 式与矩阵 AT 的 aTji 的余子 式相等 , 从而其代 数
余子式也相等 , 即
ATji = A ij ,     i, j = 1 , 2 , … , n .
于是( 以三阶为例)
T
A11 A21T T
A31 A11 A12 A13
( AT ) * = T
A12 A22T T
A32 = A21 A22 A23 = ( A* ) T .
T
A23T T
A13 A33 A31 A32 A33
对于 k A = ( kaij ) n× n 来说 , 它的元素 kai j 的代数 余子式 ( 其余子 式是 n - 1
阶行列式 )
( k A ) i j = kn - 1 Ai j ,
2. 8 部分疑难习题和补充题的题解 93


所以 ,   ( kA) = kn - 1 A* .
17 ( 第 85 题 )   证 明 : 与 任 意 的 n 阶 矩 阵 可 交 换 的 矩 阵 必 是 n 阶 数
量矩阵 .
证   在 2. 3 节的 (2 ) 矩阵 乘法 中已 证明 过与 主对 角 元互 异的 对 角矩 阵
Λ= diag( k1 , k2 , … , kn ) ( 其中 ki ≠ k j , i≠ j ) 可交换的矩阵必是对角矩阵 .下面
证明 , 这个对角矩阵还要与任意的 n 阶矩阵可交换 , 则必是数量矩阵 .取
0 a1 0 … 0
0 0 a2 … 0
A= … … … w …   ( 其中 ai ≠0 ,   i = 1 , … , n) ,
0 0 0 … an - 1
an 0 0 … 0

0 k1 a1 0 … 0 0 k2 a1 0 … 0
0 0 k2 a2 … 0 0 0 k3 a2 … 0
ΛA= … … … w … ,   AΛ= … … … w … .
0 0 0 … kn - 1 an- 1 0 0 0 … kn an - 1
kn an 0 0 … 0 k1 an 0 0 … 0
于是 , 由 ΛA = AΛ,得
ki a i = ki + 1 ai ,   从而 k i = ki + 1 ,   i = 1 , 2 , … , n - 1 ,
即 k1 = k2 = … = kn , 所以与任何 矩阵可交换 的矩阵必是数 量矩阵 kI ( k 为 任
意非零常数 ) .
前面已经讲过 , 数量矩阵 kI 与 任何矩 阵可交 换 .综 上所述 , 与任 意矩 阵
可交换的矩阵 A,当且仅当 A = kI ( 数量矩阵 ) .
18 ( 第 86 题 )   n 阶矩阵 A = ( ai j ) n× n 的主对角元之和称 为矩阵 A 的迹 ,
记作 t r( A) , 即
n

t r( A) = ∑a
i= 1
ii .

证明 : 若 A 是 m× n 矩阵 , B 是 n× m 矩阵 , 则
tr ( AB) = t r( BA) .
证   设 A = ( aij ) m× n ,   B = ( bij ) n× m ,   则
94 第 2 章 矩阵

n m

( AB) ii = ∑a
k= 1
ik b ki ,   ( BA) k k = ∑bi=1
ki aik .

于是
m m n

tr ( AB) = ∑ ( AB)
i=1
ii = ∑∑ a
i =1 k= 1
ik bk i
n m n

= ∑∑ b
k= 1 i = 1
ki aik = ∑ ( BA)
k= 1
kk = tr ( BA) .

19 ( 第 88 题 )   若 n 阶矩阵 A 存在正整数 k , 使得 Ak = 0, 就称 A 为幂零


矩阵 .
设幂零矩阵 A 满足 Ak = 0( k 为正 整数 ) , 试 证明 : I - A 可 逆, 并 求其 逆
矩阵 .
解   证明 I - A 可逆 , 如果 A 的 元素是 具体数 字, 可 用行列 式 | I - A | ≠
0, 或用初等行 ( 列 ) 变换将其化为单位 矩阵来证明 .这里 A 是 抽象的 , 只能 从
定义的角度考虑 , 即是否存在矩阵 B, 使 ( I - A) B = I .
在初等代数里 , 我们知道
(1 - x) (1 + x + x2 + … + xk - 1 ) = 1 - xk .
由于单位矩阵 I 与 任何 矩阵 相乘可 交换 , 所 以上 式中“1”换为“ I”,“ x”换 为
A,其等式成立 ,即
( I - A) ( I + A + A2 + … + Ak - 1 ) = I - Ak = I .
( 读者不难用矩阵乘法满足的分配律及 I 与 A 可交换来验证上式成立 ) ,所以
I - A 可逆 ,且
( I - A) -1
= I + A + A2 + … + Ak - 1 .
20 ( 第 89 题 )
a 1 0 0
0 a 1 0
设 A= , f ( x) = ( x - b) n , 试求 f ( A) ;当 f ( A) 可逆时 ,求其
0 0 a 1
0 0 0 a
逆矩阵 .
解   由 f ( x ) = ( x - b) n , 求 f ( A) 时 , 不仅要把 x 换为 A,而且要把 常数 b
换为数量矩阵 bI ,即
f ( A) = ( A - bI ) n
2. 8 部分疑难习题和补充题的题解 95

n
a- b 1 0 0
0 a- b 1 0
= .
0 0 a- b 1
0 0 0 a- b
0 1 0 0 a- b 0 0 0
0 0 1 0 0 a- b 0 0
记 = B,   = ( a - b) I,   a - b=λ, 则
0 0 0 1 0 0 a- b 0
0 0 0 0 0 0 0 a- b
n
f ( A) = (λI + B)
= λn I + C1nλn - 1 B + C2nλn - 2 B2 + C3nλn - 3 B3 + C4nλn - 4 B4 + … + Bn ,
其中
0 0 1 0 0 0 0 1
0 0 0 1 0 0 0 0
B2 = ,   B3 = ,   B4 = … = B5 = 0 .
0 0 0 0 0 0 0 0
0 0 0 0 0 0 0 0
所以
λn C1nλn - 1 C2nλn - 2 C3nλn - 3
0 λn C1nλn - 1 C2nλn - 2
f ( A) = .
0 0 λn C1nλn - 1
0 0 0 λn
当 λ= a - b≠0 时 , f ( A) 可逆 ,用其伴随矩阵求逆 ,得
-1 f * ( A)
f ( A) =
| f ( A) |
1 - C1nλ- 1 [ (C1n )2 - C2n ]λ- 2 [ - (C1n )3 + 2C2n C1n - C3n ]λ- 3

1 0 1 - C1nλ- 1 [(C1n )2 - C2n ]λ- 2


= n .
λ 0 0 1 - C1nλ- 1
0 0 0 1
21 ( 第 92 题 )   证明 : n 阶反 对称矩 阵可逆 的必要 条件是 n 为偶 数 , 举
例说明 n 为偶数不是 n 阶反对称矩阵可逆的充分条件 .
证   设 A 为 n 阶反对称矩阵 , 则 AT = - A, 于是
| A| = | AT | = | - A| = ( - 1 ) n | A| .
96 第 2 章 矩阵

当 n 为奇数时 , | A| = - | A| ,所以 | A| = 0, 因此不可逆 .故 n 阶反对称矩 阵可


逆的必要条件为 n 是偶数 .
但偶数阶反对称矩阵也不一定可逆 ( 即 n 为偶数不是可逆的充分条件 ) ,
例如 : 四阶反对称矩阵
0 - 1 0 0
1 0 1 0
A= ,   | A| = 0 ,   A 不可逆 .
0 - 1 0 0
0 0 0 0
22 ( 第 96 题 )   若 n 阶 矩 阵 A 与 B 可交 换 , 则 A 与 B 的 任 意 多 项 式
f ( A) 与 g( B) 也可交换 .
k

∑ aA
2 k i
证   设 f ( A) = a0 I + a1 A + a2 A + … + ak A = i ,
i=0
m

g( B) = b0 I + b1 B + b2 B + … + bm B = ∑ bB
2 m j
j ,
j= 0

其中   A0 = B0 = I .于是
i+ j= k+ m

            f ( A) g( B) = ∑
i+ j = 0
ai bj Ai B j ( 1)
j+ i= m+ k

= ∑
j+i = 0
bj a i B j A i = g( B) f ( A) ,

其中第二个等号处 A B = B A 是利用 AB = BA 而得到 的 .( 1 ) 式的 含义是 :


i j j i

i + j = 0 的 项 为 a0 b0 I; i + j = 1 的 项 为 a1 b0 A + a0 b1 B; i + j = 2 的 项 为
a2 b0 A2 + a1 b1 AB + a0 b2 B2 ; i + j = 3 的 项 为 a3 b0 A3 + a2 b1 A2 B + a1 b2 AB2 +
a0 b3 B3 ; 如此等等 .
  第 3 章

线性方程组

3 .1 基本要求与内容提要
1  基本要求

(1 ) 熟练掌握 n 维向量 的线 性 运算 ( 向量 的加 法 及数 与向 量
相乘的向量的数量乘法 ) , 熟悉它们所满足的 8 条运算规则 .
(2 ) 准确理解 n 维向量 的线 性 相关 性的 定义 ( 包 括线 性相 关
与线性无关 ) ; 熟悉一组向 量线 性相关 和无 关的充 分必 要条 件 ; 熟
练掌握判别一组向量线性相关或无关的方法 .
(3 ) 准确理解向 量组 的秩 及其 极 大线 性无 关组 的概 念 , 会 用
初等变换的方法求秩与极大线性无关组 .
(4 ) 准确理解矩 阵的 秩的 概念 ; 会用 初等 变换 的 方法 求矩 阵

的秩 ; 熟悉矩阵相加和相乘后的秩 与原矩阵 的秩的 关系 ; 了解 矩
阵的相抵标准形的概念 .
(5 ) 准确理解齐 次线 性方 程组 有 非零 解的 条件 , 及其 基础 解
系和一般解 ( 通解 ) 的结构 ; 熟练掌握求基础解系的方法 .
(6 ) 准确理解非 齐次 线性 方程 组 有解 的条 件 , 及 其一 般解 的
结构 ; 熟练掌握求一般解的方法 .

2  内容提要

(1 ) n 维向 量与 矩 阵 是 线 性代 数 中 两 个 基本 的 运 算 对 象 . n
98 第 3 章  线性方程组

维向量的线性运算 ( 加法与数量乘法 ) .
n
设   α= ( a1 , a2 , …, an ) ,   β= ( b1 , b2 , … , bn ) ∈ F , k∈ F, 规定
α+ β= ( a1 + b1 , a2 + b2 , … , an + bn ) ,
kα= ( ka1 , ka2 , … , kan ) ,   - α= ( - 1)α,
α- β= α+ ( - β) .
n 维向量的线性运算满足以下 8 条运算规则 :
① α+ β= β+ α;           ② (α+ β) + γ= α+ (β+ γ) ;
③ α+ 0 = α( 0 为零向量 ) ; ④ α+ ( - α) = 0(0 为零向量 ) ;
⑤ 1α= α; ⑥ k( lα) = ( kl)α( k, l 为数 ) ;
⑦ k(α+ β) = kα+ kβ; ⑧ ( k + l)α= k α+ l α .
(2 ) n 维向量的线性相关性
n
定义   向量 α1 , α2 , … ,αm ∈ F 线性 相关指的 是 : 存在不全 为
零的数 k1 , k2 , … , km ∈ F, 使
k1 α1 + k2 α2 + … + kmαm = 0( 零向量 ) .
α1 ,α2 , … ,αm 线 性 无关 指 的 是 : 仅 当 数 k1 = k2 = … = km = 0
时 , 才使 k1 α1 + k2 α2 + … + kmαm = 0( 零向量 ) , 也就是 说 , 若 k1 α1 +
k2 α2 + … + kmαm = 0, 则必有数 k1 = k2 = … = km = 0 .
单个向量 α 线 性 相 关 ( 无 关 ) , 当 且 仅 当 α为 零 向 量 ( 非 零
向量 ) .
① α1 ,α2 , … ,αm ( m≥2) 线性相 关的充 要条 件是 , 其中 有一 个
向量可由其余向量线性表示 ; 而线性无关的充要条件则是 , 其中任
一个向量均不可由其余向量线性表示 .
② 如果向量组 β1 , … ,βt 可 由 向量 组 α1 , … , αs 线 性 表示 , 且
t > s, 则 β1 , … ,βt 线性相关 .这个命 题的等 价命 题是 : 如果 向量 组
β1 , … ,βt 可由向 量组 α1 , … , αs 线性表 示 , 且 β1 , … ,βt 线性 无关 ,
则 t≤ s .
③ 如果 向 量 组 α1 , α2 , … , αr 线 性 无 关 , 而 向 量 组 β, α1 ,
α2 , … ,αr 线 性 相 关 , 则 β可 由 α1 , α2 , … , αr 线 性 表 示 , 且 表 示
3 .1 基本要求与内容提要 99

法惟一 .
④ 设 α1 = ( a11 , a21 , … , an1 ) T , α2 = ( a12 , a22 , … , an2 ) T , αr =
( a1 r , a2 r , … , an r ) T , 由于向量方程
x1 α1 + x2 α2 + … + xrαr = 0( 零向量 )
等价于齐次线性方程组
Ax = 0,
其中系数矩阵 A 是 以 α1 , α2 , … , αr 按 列 排 成 的 n × r 矩 阵 , x =
T
( x1 , x2 , … , x r ) .
所以 , 向量组 α1 ,α2 , … ,αr 线性相关 ( 无 关 ) 的充要条 件是 , 齐
次线性方程 组 Ax = 0 有 非 零 解 ( 只 有 零 解 , 即 x1 , x2 , … , x r 全
为零 ) .
(3 ) 向量组的秩及其极大线性无关组
定义   如果向量组 α1 ,α2 , … ,αs 中存在 r 个线 性无关 的向 量
αi1 ,αi2 , … ,αi r ( 1≤ i1 < i2 < … < ir ≤ s) , 且其中 任一 个向量 可由 这
r 个向量线性表示 , 则称向量组的秩为 r, 即秩 {α1 ,α2 , … ,αs } = r .
由前面的 (2 ) 段② , ③ , 可得这个定义的等价定义 :
秩{α1 ,α2 , … ,αs } = r 即为 α1 ,α2 , … ,αr 中存在 r 个线 性无 关
的向量 , 且任何 r + 1 个向量 都 线性 相关 .其 中 的 r 个 线性 无关 的
向量称为向量组的一个极 大线 性无关 组 .向量 组的极 大线 性无 关
组一般不是 惟一的 , 但不 同的极 大线性无 关组所 含向量“个 数”是
相同的 , 这个“ 个数”就是向量组的秩 .
求向量组 α1 ,α2 , … , αs 的 秩及 其极 大线 性 无关 组 的方 法 ( 不
妨设向量为四维向量 , 向量个数 s = 5) :
将 α1 ,α2 ,α3 ,α4 ,α5 按列 排成 4× 5 矩阵 A, 对 矩阵 A 做初 等
行变换将其化为阶梯形矩阵 U , 即
初等
A = α1 α2 α3 α4 α5
行变 换
100 第 3 章  线性方程组

c11 * * * *
0 0 c2 3 * *
= (ξ1 ξ2 ξ3 ξ4 ξ5 ) = U
0 0 0 c34 *
0 0 0 0 0
( 其中 c11 , c23 , c34 均不 等 于零 ) , 则阶 梯 形 矩 阵 U 的 非 零 行 行 数 3
为秩{α1 ,α2 , … ,α5 } , U 中 每个 非零 行的 第 一个 非零 元 c11 , c23 , c3 4
所在的第 1 , 3 , 4 列所对应的 α1 ,α3 ,α4 为 向量组 {α1 ,α2 , … ,α5 } 的
一个极大线性无关组 .其理论根据是 A 与 U 中对应的 列向量组 有
相同的线性相关性 ( 定理 3 .6 ) .
(4 ) 矩阵的秩
矩阵的行向量组的秩称 为矩阵 的行 秩 ; 矩阵的 列 向量 组的 秩
称为矩阵的列秩 ; 矩阵的行列式秩为矩阵非零子式的最高阶数 .
矩阵 A 的行秩与列秩及其行列式秩都相 等 , 并 称为矩 阵 A 的
秩 , 记作 r( A) .
初等变换不改变矩阵的 秩 , 所 以矩 阵的 秩是矩 阵 在初 等变 换
下的一个不变量 .对矩阵 A 做初等变换将其化为阶梯形矩阵 U, 则
U 的非零行行数就是矩阵 A 的秩 .
r ( A + B) ≤ r( A) + r( B) ,
r( AB) ≤ min( r( A) , r ( B) ) .
若 P, Q 可逆 , 则
r( A) = r ( PA) = r( AQ) = r ( PAQ) .

矩阵 A 的 相 抵标 准 形 : 对 秩 为 r 的 矩 阵 A 做初 等 行、列 变
换 , 可将 A 化为
Ir 0
0 0
( 其中 Ir 为 r 阶单位矩阵 ) , 并称 之为 A 的 相抵 标准 形 , 即 存在 可
逆矩阵 P 与 Q, 使
3 .1 基本要求与内容提要 101

Ir 0
PAQ = .
0 0
(5 ) 齐次线性方程组有非零解的条件及解的结构
① 以 m× n 矩阵 A 为系 数矩 阵的齐 次线 性方 程 组 Ax = 0 有
非零解的充分必要条件为 r ( A) < n, 或 A 的 n 个列向量线性相关 .
其等价命题是 : Ax = 0 只有零解的 充要条 件为 r( A) = n, 或 A 的 n
个列向量线性无关 .
当 A 为 n 阶方阵时 , | A| = 0( ≠ 0) 是 Ax = 0 有非零解 ( 只有零
解 ) 的充要条件 .
② 如果 齐 次 线 性 方 程 组 Ax = 0 有 p 个 线 性 无 关 的 解 x1 ,
x2 , … , xp , 且任一个解可由它们线性表示 , 则称这 p 个线性无关的
解为 Ax = 0 的基础解系 , 并称
x= k1 x1 + k2 x2 + … + kp x p
( 其中 k1 , k2 , … , kp 为任 意常 数 ) 为 Ax = 0 的一 般解 ( 或称 通解 ) .
当 A 为 m× n 矩阵 , 秩 ( A) = r 时 , 其中 p = n - r .
③ 求 Ax= 0 的基础解系的方法
对 m× n 矩阵 A 做初等行变换将其化为阶梯阵 U, 即
c11 … c1 i2 … c1 i r … c1 n
0 … c2 i2 … c2 i r … c2 n
… … … …
初等
Am × n
行变 换
0 … 0 … cr i r … cr n = U,

0 … 0 … 0 … 0
… … … …
0 … 0 … 0 … 0
其中 U 的 r 个非零行的第一个 非零 元 c11 , c2 i2 , … , cr i r 所在 列对 应
的未知元 x 1 , xi2 , … , xi r 称为基本未知量 ( 或称主元 ) , 其余 n - r 个
未知元称为自由未知量 .
102 第 3 章  线性方程组

将 n - r 个自由未知量依次取 1 其余均取 0 的 n - r 组自由未知量


(1 , 0 , … , 0) , ( 0 , 1 , … , 0 ) , … , ( 0 , … , 0 , 1 )
分别代入方程组 Ux = 0 求 基本 未 知 量 , 从 而得 到 Ax = 0 的 n - r
个线性无关的解 ( 即基础解系 ) x1 , x2 , … , xn - r .
(6 ) 非齐次线性方程组有解的条件及解的结构
① 非齐次线性方程组 Ax= b 有解的充要条件为
r ( A, b) = r( A) , 或 b 可由 A 的列向量组线性表示 .
Ax = b 有惟一解的充要条件为
r ( A, b) = r( A) = A 的列数 .
② 若 x1 与 x2 为 Ax= b 的两个解 , 则 x1 - x2 为对应齐次方程
组 Ax= 0 的解 .
③ 若 Ax= b 有解 , 则其一般解为
x = x0 + x,
其中 : x0 是 Ax = b 的一个特解 ( 某一个解 ) ,
x= k1 x1 + k2 x2 + … + kp x p
是对应齐次方程组 Ax= 0 的一般解 .
求 Ax= b 的一般解的方 法 : 对 增广 矩阵 ( A, b) 做 初等 行变 换
将其化为阶梯形矩阵 ( U, d) , 即
初等 行变换
( A, b) ( U, d) .
从 Ux = 0 求 Ax = 0 的 基础解系 与一般 解 x的 方法与 前面 (5 ) 段 ③
中所述一样 .从 Ax = b 的 同解 方程组 Ux = d 中 , 将 自 由未 知量 全
取为零 , 算出基本未知量 , 从而得到 Ax= b 的一个特解 x0 .

3 .2 n 维向量及其线性相关性

本章的中心问题是讨论线性方程组的解的基本理论 , 也就是 :
齐次线性方程组 Ax= 0 有非零解的条件 和解的 结构 , 以及 非齐 次
3 .2  n 维向量及其线性相关性 103

线性方程组 Ax= b 有解的条件和有无穷 多组解 时解 的结构 .在 前


一章中 , 我们虽然已经会 用高 斯消元 法求 解齐 次和非 齐次 线性 方
程组 , 但是其中还有很 多重要 的问 题没有 阐述 清楚 , 例 如 , 对非 齐
次线性方程组增广矩阵 ( A, b) 做 初等行 变换将 其化 为阶梯 形矩 阵
( C, d) 时 , 其 中 的 dr + 1 在什 么 情 况 下 ( 即 增 广 矩 阵 满 足 什 么 条 件
时 ) 必定等于零 ( 即 Ax= b 有解 ) ; 再如 , 对方 程组采 用不同 的消 元
步骤 ( 即对增广矩阵做 不同 的初等 行变 换 ) , 将 其化 为阶梯 形矩 阵
时 , 其非零行的行数是 否相同 ? 即求 解时自 由未 知量 的个 数是 否
相同 ; 又如 , 自由未知量一般 可以 有不 同的取 法 , 从而 解的 表示 形
式有所不同 , 那么它们 全部解 构成 的解 集合是 否相 同呢 ? 这些 深
层次问题不搞清楚 , 即使会用高斯消元法求出了线性方程组的解 ,
也不能说对线性方程组的解的问题有了透彻的认识 .
那么 , 应该从什么地方 , 通过什么途径来搞清楚上面提的那些
问题呢 ?
我们用高斯消元法求解线性方程组 Ax = b 的消 元过程 , 实 际
上是对其增广矩阵 ( A, b) 的 行向 量 做线 性运 算 ( 向量 的 加法 与 数
乘向量 ) , 通过这样的运算 ( 也就是矩阵的初等行变换 ) 把增广矩阵
( A, b) 化为阶梯形矩阵 ( C, d) 时会有几个 非零行 , 也就 是会出现 几
个全零行 , 这取决于 增广 矩阵 ( A, b) 的 行向 量 之 间在 线 性运 算 下
有怎样的关系 .例如 : 系数矩阵 A 为 5×5 矩阵时 , ( A, b) 的行向 量
有 5 个 , 记作 α1 ,α2 , α3 ,α4 , α5 , 如果 α4 ,α5 能用 α1 ,α2 ,α3 线性 表
示 , 而 α1 ,α2 ,α3 之间任一 个都 不能 用 另外 两个 线性 表示 , 那么 对
( A, b) 做初等行变 换 ( 也就 是对 α1 ,α2 , α3 ,α4 ,α5 作线 性运 算时 ) ,
就一定可以将 α4 ,α5 所在 的 行 化为 全 零行 , 于是 ( A, b) 化为 阶 梯
形矩阵 ( C, d) 时 就必 有 3 个 非零 行 .这 里所 涉 及 的就 是 一组 向 量
之间在线性 运算下 有怎样的 关系 , 这就是“ 向量的线 性相关 性”的
问题 .因此 , 必须从“向 量的线 性相 关性”入 手 , 才能搞 清楚 前面 所
提的求解线性方程组的深层次问题 .
104 第 3 章  线性方程组

1  向量的线性 表示

任何一个 n 维向量α= ( a1 , a2 , … , an ) 都可以由 n 个 n 维单位


向量 ε1 = ( 1 , 0 , … , 0 ) ,ε2 = ( 0 , 1 , … , 0 ) , … ,εn = ( 0 , 0 , … , 1 ) 做 线
性运算 ( 即数量乘法和加法 ) 来表示 , 即
α = ( a1 , a2 , … , an )
= ( a1 , 0 , … , 0 ) + ( 0 , a2 , … , 0 ) + … + ( 0 , 0 , … , an )
= a1 ε1 + a2 ε2 + … + anεn .
一般地 , 如果向量 α能用 向量 α1 , α2 , … , αr 做 线 性运 算来 表
示 , 即 α= x1 α1 + x2 α2 + … + x rαr ( 其中 x1 , x2 , … , x r 为数 ) , 就 称
α可由 α1 ,α2 , … ,αr 线性表示 ( 或称线性组合 ) .
例 1   设 α= ( 4 , 3 , 3 , 1 ) , α1 = ( 1 , 2 , 3 , 4 ) , α2 = ( 0 , 1 , 2 , 3 ) ,
α3 = ( 0 , 0 , 1 , 2) ,α4 = (0 , 0 , 0 , 1) .试 问 : (1 )α能 否由 α1 ,α2 ,α3 , α4
线性表示 ? (2 )α4 能否由 α1 ,α2 , α3 线性 表示 ? 如能表 示 , 写出 线
性表示式子 .
解   ( 1 ) α能由 α1 , α2 , α3 , α4 线 性表 示 , 就 是说 存 在 数 x1 ,
x2 , x3 , x4 , 使
x1 α1 + x2 α2 + x3 α3 + x4 α4 = α, ( 1)

  ( x1 , 2 x1 , 3 x1 , 4 x1 ) + ( 0 , x2 , 2 x2 , 3 x2 ) +
      (0 , 0 , x3 , 2 x3 ) + (0 , 0 , 0 , x4 )
= ( x1 , 2 x1 + x2 , 3 x1 + 2 x2 + x3 , 4 x1 + 3 x2 + 2 x3 + x4 )
= (4 , 3 , 2 , 1 ) . ( 2)
由 (2 ) 式得到 x1 , x2 , x3 , x4 的一个线性方程组
x1               = 4 ,
2 x1 + x2 = 3,
             
3 x1 + 2 x2 + x3 = 3,
4 x1 + 3 x2 + 2 x3 + x4 = 1 .                 (3 )
3 .2  n 维向量及其线性相关性 105

这个方程组容易 求解 , 将 x1 = 4 代 入第 2 个 方程 , 得 x2 = - 5 ; 再
将 x1 = 4 , x2 = - 5 代入第 3 个方 程 , 得 x3 = 1; 最后 将求 得的 x1 ,
x2 , x3 代入第 4 个方程 , 得 x4 = - 2 .所 以 α可由 α1 , α2 , α3 ,α4 线
性表示 , 其表示式为
α= 4α1 - 5α2 + α3 - 2α4 .
这里要注意两点 : ① ( 1) 式是一个向量方程 , 它通过 ( 2) 式 , 等
价于 (3 ) 式表示的四元线性方程组 Ax = b, 这个方程组的增广矩阵
1 0 0 0 4
2 1 0 0 3
( A, b) = = α1 α2 α3 α4 α
3 2 1 0 3
4 3 2 1 1
中的 5 个列 向量就 是题目所 给的 5 个向量 写为列向 量的形 式 .因
此 , 解题时 , 可直接将向量方程 ( 1 ) 等价 于线 性方 程组 ( 3 ) .一般 要
用高斯消元法求解这个方程组 .
② 方程组 (3 ) 有惟一解 , 是因为 α1 ,α2 , α3 ,α4 是 线性无 关的 ,
而 α,α1 ,α2 ,α3 ,α4 是线性 相关 的 ( 学 了线 性相 关性 就 会明 白这 一
点的 ) .
(2 ) 如果 α4 能由 α1 ,α2 ,α3 线性表示 , 则存在 x1 , x2 , x3 , 使
x1 α1 + x2 α2 + x3 α3 = α4 . ( 4)
(4 ) 式等价于下面的方程组
x1           = 0 ,
2 x1 + x2     = 0,
                 
3 x1 + 2 x2 + x3 = 0 ,
4 x1 + 3 x2 + 2 x3 = 1 .                 ( 5)
方程组 ( 5 ) 显 然是 无解的 , 因 为由 前 3 个方 程得 x1 = x2 = x3 = 0 ,
而这不满足第 4 个方程 .所以 α4 不能由 α1 ,α2 ,α3 线性表示 ( 这 也
是因为 α1 ,α2 ,α3 ,α4 是 线性 无 关的 , 以 后会 明 白这 4 个向 量中 的
任一个向量均不能由其余 3 个向量线性表示 ) .
106 第 3 章  线性方程组

2  向量的线性 相关性

向量的“线性相关性”的概念 ( 即“ 线性相关”和“ 线性无关”) 是


线性代数中最重要的基本概念 .
向量的“线性相关性”是描述向量 α1 , α2 , … ,αm ( m≥ 1) 在“ 线
性运算”下有怎 样的“关系”的一种概 念 .例如 , 在三维 几何向量 空
间中 , 任何两个共线 ( 或平行 ) 的 非零向量 α1 ,α2 , 都 可以互 相线 性
表示 , 即 α1 = kα2 或 α2 = l α1 ; 任何 3 个 乃 至 更 多 个共 面 的 向 量
α1 ,α2 , … ,αm ( m≥3 ) , 至 少存 在 一个 向量 可由 其余 向 量线 性表 示
( 详见主教材 p112~ 113) .这表明这两种情 况下的向 量之间 在“ 线
性运算”下是“有 着某种关 系”的 , 我们称 它们是线 性相关 的 .一 般
来说 :
“如果 α1 ,α2 , … ,αm 中有一个向量能由其余向量线性表示 , 就
称 α1 ,α2 , … ,αm 是线性相关的 .”但是我们并没有把这个命题作为
“向量线性相关”的定义 , 因为 一般 情况 下用这 个定 义判断 给定 的
α1 ,α2 , … ,αm 是否线性相关是 不很方便的 .因此 , 我们把这个命 题
的等价命题 :
“如果存在不全为 零的数 k1 , k2 , … , km , 使 k1 α1 + k2 α2 + … +
kmαm = 0( 零向 量 ) , 就 称 α1 , α2 , … ,αm 是 线 性 相 关的”, 作 为 α1 ,
α2 , … ,αm 线 性相 关的定 义 .后面 的例 题将表 明 , 用这 个定 义判 别
一组向量是否线性相关是 很方 便的 , 它把 这个 问题转 化为 齐次 线
性方程组是否有非零解的问题 .
这里讲的两个命题等价 指的是 它们 可以互 相推 出来 ( 请详 见
主教材中定理 3 .1 ( p113 ) 的 证 明 ) .于 是 我 们 把前 一 个 命 题 作 为
α1 ,α2 , … ,αm 线性相关的充分必要条件 .
“如果 α1 ,α2 , … ,αm 中任何一 个向 量都 不能由 其 余向 量线 性
表示 ( 即 它们在线 性运算下 无任何 关系 ) , 就称 α1 , α2 , … ,αm 是 线
性无关的 .”这等价于 :
3 .2  n 维向量及其线性相关性 107

“如果 k1 α1 + k2 α2 + … + kmαm = 0 ( 零 向量 ) , 则 k1 , k2 , … , km
必须全为零 , 就称 α1 ,α2 , … ,αm 线性无关”( 这里 , 如果 ki ≠0 , 那么
αi 就可由其余向 量 线性 表 示 ) .我 们 把后 者 作为 α1 , α2 , … , αm 线
性无关的定义 , 把前者作为 α1 ,α2 , … ,αm 线性无关的充要条件 .
(1 ) 如何判断向量的线性相关性
判断向量的线性相关性的问题 , 一般有两种类型 : 一是给定一
组具体的 n 维 向量 α1 ,α2 , … , αm , 如内 容提要 ( 2 ) ④中 所述 , 判 断
其线性相关性的问题就转化为齐次线性方程组
Ax= 0

( 其中系数矩阵 A 是以α1 ,α2 , … ,αm 为列向量的 n× m 矩阵 ) 有 无


非零解的问题 ; 另一类 是已知 一组 抽象 的向量 α1 ,α2 , … , αm 的 线
性相关性 , 讨论由它们做 线性 运算得 到的 另一 组向量 的线 性相 关
性 .例如 , 讨论 α1 + α2 , α2 + α3 , … , αm - 1 + αm , αm + α1 的 线 性 相
关性 .
下面通过例题来讨论这两类线性相关性的问题 .
例 2   设 α1 = ( 1 , 2 , 3 , 4 ) , α2 = ( 0 , 1 , 2 , 3 ) , α3 = ( 0 , 0 , 1 , 2 ) ,
α4 = ( 1 , 3 , 6 , 9) , 试判别 :
( i ) α1 ,α2 ,α3 ,α4 的线性相关性 ;
( ii ) 这 4 个向量中任何 3 个向量的线性相关性 .
解   ( i ) 此题容易观察到

α4 = α1 + α2 + α3 .

根据一组向量线性相关的 充要 条件 ( 有 一个向 量可 由其余 向量 线


性表示 ) , 即得 α1 ,α2 ,α3 , α4 是 线性 相 关的 .一 般是 不 容易 观察 到
一组向量是否有线性关系的 .一般的判别法为 : 设

x1 α1 + x2 α2 + x3 α3 + x4 α4 = 0, ( 1)

于是由 (1 ) 式即得齐次线性方程组
108 第 3 章  线性方程组

1 0 0 1 x1 0
2 1 0 3 x2 0
Ax = = . ( 2)
3 2 1 6 x3 0
4 3 2 9 x4 0
用高斯消元法解上面的齐次线性方程组
1 0 0 1 1 0 0 1
② + ① × ( - 2) 0 1 0 1 ③ + ② ×( - 2 ) 0 1 0 1
A ③ + ① × ( - 3) 0 2 1 3 ④ + ② ×( - 3 ) 0 0 1 1
④ + ① × ( - 4)
0 3 2 5 0 0 2 2
1 0 0 1
0 1 0 1
  =U. ( 3)
④ + ③ ×( - 2 ) 0 0 1 1
0 0 0 0
Ux = 0 与 Ax = 0 是同解 方程 组 , 解 Ux = 0 时 , 取 x4 = k( k 为任 意
常数 ) , 即得
x = ( x1 , x2 , x3 , x4 ) T = ( - k, - k, - k, k) T .
于是方程组 (2 ) 有非零解 , 从而有不 全为零 的 x1 , x2 , x3 , x4 , 使 ( 1)
式成立 , 故 α1 ,α2 ,α3 ,α4 是线性相关的 .
( ii) α1 ,α2 ,α3 ,α4 中任意 3 个向量都是线性无关的 .因为考 察
α1 ,α2 ,α3 的线性相关性时 , 设
x1 α1 + x2 α2 + x3 α3 = 0, ( 4)
于是得到 A1 x1 = 0, ( 5)
其中 A1 是 (2 ) 式中 A 的前 3 列 , x1 = ( x1 , x2 , x3 ) T .由于上面 ( 3) 式
中对 A 做行变换时 , 列之间互不干扰 , 所以
1 0 0
    0 1 0
A1 = α1 α2 α3 = U1 .
0 0 1
0 0 0
3 .2  n 维向量及其线性相关性 109

而齐次线性方程组 U1 x1 = 0 只有零解 , 从而 A1 x1 = 0 也只有零解 .


因此 , 要使 ( 4) 式成立 , 必须 x1 , x2 , x3 全为零 , 故 α1 ,α2 ,α3 是线 性
无关的 .
同理 ,α1 ,α2 ,α4 也是线性无关的 , 因为
1 0 1
    0 1 1
A2 = α1 α2 α4 = U2 ,
0 0 1
0 0 0
所以 A2 x2 = 0 也只有零解 ( x1 = x2 = x4 = 0) , 即要使
x1 α1 + x2 α2 + x4 α4 = 0
成立 , 必须 x1 , x2 , x4 全为零 , 故 α1 ,α2 ,α4 线性无关 .
读者不难证明 α1 ,α3 ,α4 以及 α2 ,α3 ,α4 也都是线性无关的 .
例 3   证明 : 任何 m 个 n 维向量αj = ( a1 j , a2 j , … , anj ) ( j = 1 ,
2 , … , m) , 当 m > n 时 , ,α1 ,α2 , … ,αm 必定线性相关 .
证   设 x1 α1 + x2 α2 + … + xjαj + … + x mαm = 0 . ( 1)
这等价于
a11 a12 … a1 j … a1 m x1 0
a21 a22 … a2 j … a2 m x2 0
Ax = = . ( 2)
… … … … … …
an1 an2 … anj … an m xm 0
当 m > n 时 , 求解这个齐次方程组 ( 2 ) 时 至少 有 m - n 个自 由未 知
量 , 从而必有非零解 , 即存在不全为零 的 x1 , x2 , … , xm 使 ( 1 ) 式 成
立 , 故 α1 ,α2 , … ,αm 线性相关 .
对于 n 阶矩阵 A = ( aij ) n× n , 如果 A 的行 列式 | A| ≠0 , 即 A 可
逆 , 则齐次线性方程组 Ax = 0 只有 零解 , 从而 A 的 n 个列 向量 线
T T
性无关 .此时 | A | = | A | ≠ 0 , A x = 0 也只 有零解 , 故 A 的 n 个 行
n
向量也线性无关 .所 以在 全 体 n 维实 向 量 构 成的 集 合 R ( 称 为 n
维实向量空间 ) 中 , 有各种各样 的 n 个 线性无 关的 向量 ( 以 后称 为
110 第 3 章  线性方程组

n
R 的基 ) , 其中最基本的是 n 个单 位向 量εi = ( 0 , … , 1 , … , 0 ) ( 第 i
n
个分量为 1 , 其余分量均为零 , i = 1 , 2 , … , n) .而R 中 任何 n + 1 个
n
向量都是线性相关的 , 所以由定理 3 .3( 教材 p116 ) 即得 , R 中任一
个向量都可由Rn 中任何 n 个线 性无 关的向 量线 性表示 , 且 表示 法
n
惟一 .这些是 n 维向量空间R 中最基本的结论 .
T T T
例 4   设 α1 = ( 1 , 2 , 3 ) , α2 = ( - 1 , 0 , - 1 ) ,α3 = (2 , 1 , 4) ,
T
α4 = ( 3 , 5 , 8 ) .试 判 别 α1 , α2 , α3 , α4 以及 α1 ,α2 , α3 的 线 性 相 关
性 , 并问 α4 可 否 由 α1 , α2 , α3 线 性 表 示 ? 如 可 表 示 , 写 出 它 的 表
示式 .
解   α1 ,α2 , α3 , α4 是 4 个三 维 向量 , 所 以 它 们必 是 线性 相 关
的 .判别 α1 ,α2 ,α3 的线性相关性时 , 设
x1 α1 + x2 α2 + x3 α3 = 0, ( 1)
即得
x1 1 -1 2 x1 0
Ax = α1 α2 α3 x2 = 2  0 1 x2 = 0 . ( 2)
x3 3 -1 4 x3 0
用高斯消元法解齐次线性方程组 (2 ) , 得
1 -1  2
初等
A 0  2 - 3 =U .
行 变换
0  0  1
Ux = 0 只有 零解 , 因 此 , 要使 ( 1) 式 成立 , 必 须 x1 = x2 = x3 = 0 , 所
以 α1 ,α2 ,α3 线性无关 .
判别 α1 ,α2 ,α3 的线性相关性更简便的方法是 : 由于 | A| = 2 ≠
0 , A 可逆 , Ax = 0 只有零解 , 所以 α1 ,α2 ,α3 线性无关 .
但要注意 , 用 | A| 来判断 α1 ,α2 ,α3 的线性相关性时 , A 必须 是
方阵 .如果 α1 , α2 ,α4 是 3 个 四维 向量 , 对应 的矩 阵 A 是 4 × 3 矩
阵 , 它无行列式可言 , 这时必须由 Ax= 0 是否有非零解来判 别 α1 ,
α2 ,α3 的线性相关性 .
3 .2  n 维向量及其线性相关性 111

3
因为在R 中任何向量均可由 3 个线性无关 的向量线 性表示 ,
所以 α4 可由 α1 ,α2 ,α3 线性表示 .因此存在 x1 , x2 , x3 , 使
x1 α1 + x2 α2 + x3 α3 = α4 . ( 3)
这个向量方程对应于 ( 或说等价于 ) 非齐次线性方程组
x1
α1 α2 α3 x2 = α4 .
x3
记作 Ax= α4 , 即
1 -1 2 x1 3
2  0 1 x2 = 5 . ( 4)
3 -1 4 x3 8
对方程组 ( 4 ) 的 增 广 矩 阵 做 初 等 行 变 换 , 将 其 化 为 行 简 化 阶 梯
阵,即
1 -1 2 3 1 -1  2  3
初等
( A,α4 ) = 2  0 1 5 0  2 - 3 -1
行 变换
3 -1 4 8 0  0  1  0
1 0 0   5/ 2
   
0 1 0 - 1/ 2 = ( C, d) . ( 5)
0 0 1 0
Ax = α4 与 Cx = d是同解方程组 , 由后者即得
5 1
x1 = ,   x2 = - ,   x3 = 0 .
2 2
所以 α4 可由 α1 ,α2 ,α3 线性表示 , 其表示式为
5 1 5 1
α4 = α1 - α2 + 0α3 = α1 - α2 .
2 2 2 2
此结果表明 ,α4 与 α1 ,α2 是共面的 , 它们线性相关 .
从 (5 ) 式也可见 α3 是不能由 α1 ,α2 ,α4 线性表示的 .因为 , 设
x1 α1 + x2 α2 + x4 α4 = α3     ( 记作 Bx = α3 ) , ( 6)
112 第 3 章  线性方程组


1 -1 3 2 1 0 5/ 2 0
初等
( B,α3 ) = 2  0 5 1 0 1 - 1/ 2 0 .
行变 换
3 -1 8 4 0 0 0 1
所以 Bx = α3 无解 , 也就是说不存在 x1 , x2 , x4 , 使 ( 6 ) 式成 立 .这 表
明 α3 不在 α1 ,α2 ,α4 所确定的平面上 .
例 5   设有两个向量组 α,β,γ与 α,β,γ, 其中
α= ( a1 , a2 , a3 , a4 ) ,β= ( b1 , b2 , b3 , b4 ) ,γ= ( c1 , c2 , c3 , c4 ) ;
在 α,β,γ中依次添加第 5 个分 量 a5 , b5 , c5 就是 α,β,γ.试 讨
论这两个向量组的线性相关性有何关系 .
解 设 x1 α+ x2 β+ x3 γ= 0, ( 1)
x1 α+ x2 β+ x3 γ= 0 . ( 2)
它们对应于齐次线性方程组为
Ax = 0   与   Bx = 0,
其中 : x= ( x1 , x2 , x3 ) T ,
a1 b1 c1
a2 b2 c2
A= (α β γ) = ,
a3 b3 c3
a4 b4 c4
a1 b1 c1
a2 b2 c2
B = (α β γ) = a3 b3 c3 .
a4 b4 c4
a5 b5 c5
( i ) 当 Ax= 0 只有零解 时 , 则 Bx = 0 中 的前 4 个 方程 只有 零
解 , 从而 Bx = 0 也只能 有零 解 .这表 明 : 当 α,β,γ线 性无 关 时 , 它
们添加分量后得到的 α,β,γ也必然线性无关 .但是 , 如果 α,β,γ线
3 .2  n 维向量及其线性相关性 113

性相关 , 即 Ax = 0 有非零解时 , 而 Bx = 0 则有可能 只有零解 ( 读 者


不难举出这样的例子 ) , 从而 α,β,γ线性无关 .
( ii ) 当 Bx = 0 有非 零解时 , 显然 Ax = 0 也有 非零 解 ( 因为 它
的 4 个方程就是 Bx = 0 中的前 4 个方程 ) .这表 明 : 当 α,β,γ线 性
相关时 , 它们去掉第 5 个分量后得到的 α,β,γ也仍然线性相关 .不
过仍要注意 , 如果 α,β,γ线性无关 , 它们去掉第 5 个分量后得到的
α,β,γ有可能是线性相关的 , 这与 ( i) 中后面的情况是一样的 .
例 6   试讨论一个向量组 α1 ,α2 , … , αp 的 整体 的线性 相关 性
与其子集 ( 即其中一部分向量 ) 的线性相关性的关系 .
解   ( i )“ 如 果 α1 , α2 , … , αp 中 有 一 部 分 向 量 ( 不 妨 设 α1 ,
α2 , … ,αm , m < p) 线性相关 , 则 α1 ,α2 , … ,αp 的整体 也线性相 关 .”
因 为 当 α1 , α2 , … , αm 线 性 相 关 时 , 就 存 在 不 全 为 零 的 数 x1 ,
x2 , … , xm , 使
x1 α1 + x2 α2 + … + xmαm = 0,
从而存在 不 全 为 零 的 数 x1 , x2 , … , xm , 0 , … , 0 ( 共 有 p - m 个
零) , 使
x1 α1 + x2 α2 + … + x mαm + 0αm + 1 + … + 0αp = 0,
所以 α1 ,α2 , … ,αp 的整体也线性相关 .
但是这个 命题 的逆 命题 不 成立 , 即 不 能说 :“ 如 果 α1 , α2 , … ,
αp 整体线性相关 , 则其中必有一部分 向量也 线性相关”.如前面 例
2 , 例 4 中的 α1 ,α2 ,α3 ,α4 线 性 相关 , 但 其中 任 何 3 个 向量 都线 性
无关 , 从而任何两个或一个向量也都线性无关 .更直观的例子为 :
3
在R 中 3 个共面的 几 何向 量 α1 , α2 , α3 是 线 性相 关 的 , 如 果
这 3 个 非零 几何 向量两 两不 共线 ( 见 图 3 -1 ) , 则其中 任何 两个 向
量都是线性无关的 .
( ii)“如果 α1 ,α2 , … ,αp 整体是线性 无关的 , 则其 中任何一 部
分向量也都是线性无关的”.因为如果 α1 ,α2 , … ,αp 中有一部分 向
量线性相关 , 则其整体也线性相关 , 这与命题的假设矛盾 .实际上 ,
114 第 3 章  线性方程组

图   3-1

( ii ) 的命题是 ( i ) 的命题的逆否命题 , 它们是等价命题 .


这里同样要 注意 , ( ii) 的命题 的逆命 题也是不 成立的 .也就 是
不能说 :“α1 , α2 , … , αp 中 任 何 一 部 分 向 量 都 线 性 无 关 , 则 α1 ,
α2 , … ,αp 的整体也线性无关”, 其反例与 ( i ) 中反例一样 .
例 7   设 α1 ,α2 , α3 线 性 无 关 ,β1 = α1 - α2 + 2α3 ,β2 = 2α1 +
α3 ,β3 = 4α1 + α2 - 2α3 , 试判别 β1 ,β2 ,β3 的线性相关性 .
解 设 x1 β1 + x2β2 + x3 β3 = 0, ( 1)

x1 (α1 - α2 + 2α3 ) + x2 ( 2α1 + α3 ) + x3 (4α1 + α2 - 2α3 ) = 0 .
按 α1 ,α2 ,α3 合并同类项 , 得
( x1 + 2 x2 + 4 x3 )α1 + ( - x1 + x3 )α2 + (2 x1 + x2 - 2 x3 )α3 = 0 .
( 2)
利用已知条件 α1 ,α2 ,α3 线性无关 , 由 ( 2) 式得
x1 + 2 x2 + 4 x3 = 0 ,
- x1 + x3 = 0 , ( 3)
2 x1 + x2 - 2 x3 = 0 .
由齐次线性方程组 (3 ) 的系数行列式
1 2 4
| A| = - 1 0 1 = - 5≠ 0     ( A 可逆 ) ,
2 1 - 2
得方程组 (3 ) Ax = 0 只有 零解 , 即 x = A - 1 0 = 0 .因 此 , 要 使 ( 1 ) 式
3 .2  n 维向量及其线性相关性 115

成立 , 必须 x1 = x2 = x3 = 0 , 所以 β1 ,β2 ,β3 线性无关 .


这里对此题再给出两种情况 :
( i ) 如 果 α1 , α2 , α3 与 β1 , β2 , β3 如 题 所 设 , 又 有 β4 = α1 +
α2 - α3 , 即 4 个向量 β1 ,β2 ,β3 ,β4 可 由 3 个向 量 α1 , α2 , α3 线 性 表
示 , 则根据定理 3 .4 ,β1 , β2 ,β3 ,β4 线 性 相 关 .此 时 , 也可 如 题 的 证
法,设
x1 β1 + x2 β2 + x3 β3 + x4 β4 = 0, ( 4)
得到
x1 + 2 x2 + 4 x3 + x4 = 0 ,
- x1 + x3 + x4 = 0 , ( 5)
2 x1 + x2 - 2 x3 - x4 = 0 .
此时齐次线性方程组 ( 5 ) 有非 零解 ( 求解 时 有一 个自 由未 知量 x4
可取任意常数 ) .因此 , 有不 全为零 的 x1 , x2 , x3 , x4 使 ( 4 ) 式 成立 ,
所以 β1 ,β2 ,β3 ,β4 线性相关 .
( ii ) 如果 β1 ,β2 ,β3 如题所 设 , 而 α1 ,α2 ,α3 线性 相关 .此时 讨
论 β1 ,β2 ,β3 的线性相 关性 , 不能 用 前面 的 方法 , 因为 当 α1 , α2 , α3
线性相关时 , 由 ( 2) 式不能得到方程 组 (3 ) .现 在 , 应该由 α1 ,α2 , α3
线性相关可 知, 其中至少 有一个向 量可由其 余向量线 性表示,
不妨设
α3 = aα1 + b α2 .
将此式代入 β1 ,β2 ,β3 的表示式中 , 即 得 β1 ,β2 ,β3 可 由 α1 ,α2 线 性
表示 , 再根据定理 3 .4 , 可知 β1 ,β2 ,β3 线性相关 .
例 8   设 α1 , α2 线性无 关 , 证明 α1 , α2 ,α3 线 性无关的 充要 条
件是 α3 不能由 α1 ,α2 线性表示 .
证   这个命题对R 3 中的几何向量在直观上是很明显的 α
. 1 ,α2 线
性无关, 即 α1 ,α2 不共线, 而 α1 ,α2 ,α3 线性无关, 则当且仅当 3 个向量
不共面, 即当且仅当 α3 不能由 α1 ,α2 线性表示( 如图 3-2) .
下面给出一般的证明 :
116 第 3 章  线性方程组

图   3-2
必要性 : 用 反证法 , 如果 α3 可由 α1 , α2 线性 表 示 , 则 α1 ,α2 ,
α3 线性相关 , 与题设矛盾 .
充分性 : 设
x1 α1 + x2 α2 + x3 α3 = 0, ( 1)
由已知 α3 不能由 α1 ,α2 线性表示 , 所以 ( 1) 式 中的 x3 = 0( 否则 α3
能由 α1 ,α2 线性表示 ) , 于是
x1 α1 + x2 α2 = 0 .
再由 α1 ,α2 线性 无关 , 必又 有 x1 = x2 = 0 .因此 , 当 α3 不 能由 α1 ,
α2 线性表示时 , 仅当 x1 = x2 = x3 = 0 时 , 才 能使 ( 1 ) 式成 立 .所 以
α1 ,α2 ,α3 线性无关 .
例 9( 补充题 62)   若向量 α可由向量组α1 ,α2 , … ,αr 线性 表
示 , 则表示法惟一的充要条件是 α1 ,α2 , … ,αr 线性无关 .
证   充 分 性 : 当 α1 , α2 , … , αr 线 性 无 关 , 而 α 又 可 由 α1 ,
α2 , … ,αr 线性表示时 , 其 表 示 法 惟 一 的 证 明 , 见 教 材 中 定 理 3 .3
( p116 ) .
必要性 : 用反证法 .已知
α= x1 α1 + x2 α2 + … + x rαr , ( 1)
假设 α1 ,α2 , … ,αr 线 性相 关 , 则 存 在不 全 为 0 的 数 k1 , k2 , … , kr ,
使得
k1 α1 + k2 α2 + … + krαr = 0 . ( 2)
于是
3 .2  n 维向量及其线性相关性 117

α= α+ 0 = ( x1 + k1 )α1 + ( x2 + k2 )α2 + … + ( x r + kr )αr . ( 3)


从而 α有两种 不同 的 表 示 法 ( 1 ) 式 和 ( 3 ) 式 , 与 表 示 法 惟 一 相 矛
盾 , 所以 α的表示法惟一时 ,α1 ,α2 , … ,αr 必须线性无关 .
注意证明必要性时 , 用反证法设 α1 , α2 , … , αr 线性相 关时 , 如
果用其中有一个向量可由 其余 向量线 性表 示的说 法 , 不容 易说 清
楚 α有两种不同的表示法 .因为 ( 1 ) 式中的 x1 , … , x r 可能有 0 , 如
果 xi = 0 , 而 αj ( j≠ i) 又都不能 用其 余向量 线性 表示时 , 就 无法 说
清楚 .
3
例 9 对 R 中 的 几 何 向量 在 直 观 上 也是 很 清 楚 的 .设 α1 , α2 ,
α3 ,α4 是 4 个共面的非零向量 , 而 且两两不 共线 .它 们中任 意 3 个
都线性相关 , 此时 α4 可由 α1 ,α2 ,α3 线性表示 , 其表示 法显然不 是
惟一的 , 例如 α4 可由 α1 ,α2 线性表示 , 也可由 α2 ,α3 线性表 示 ( 见
图 3 -3) , 也可由 α1 ,α2 ,α3 线性表示 .

图   3-3

例 10   设 α1 ,α2 , … ,αn ∈R n , 证明 : R n 中 任一 向量β可 由 α1 ,


α2 , … ,αn 线性表示的充分必要条件是 α1 ,α2 , … ,αn 线性无关 .
证   充分性 : 若 α1 ,α2 , … ,αn 线 性无关 , 由于 R n 中 任何 n + 1
n
个向量 β,α1 ,α2 , … ,αn 线性相关 , 根据定理 3 .3( 教材 p116) , R 中
任一向量β可由 α1 ,α2 , … ,αn 线性表示 .
必要性 : 用反证法 .若 α1 ,α2 , … ,αn 线性相关 , 不妨设 αn 可由
α1 ,α2 , … ,αn - 1 线 性 表 示 , 且 α1 , α2 , … , αn - 1 线 性 无 关 .于 是 存 在
118 第 3 章  线性方程组

n
η∈ R , 使 α1 ,α2 , … , αn - 1 , η线性 无 关 , 从 而 η不能 由 α1 , α2 , … ,
αn - 1 线性表示 , 也就不能由 α1 ,α2 , … ,αn - 1 ,αn 线性表示 .与题设矛
盾 , 所以 R n 中任 一 向 量 可 由 α1 , α2 , … , αn 线 性 表 示 , 必 须 要 α1 ,
α2 , … ,αn 线性无关 .
n
注意   这里 讲 , 存 在 η∈ R , 使 α1 ,α2 , … ,αn - 1 , η线性 无关 .
那么 , 如果给定了 n - 1 个 n 维线性 无关的 向量 αj = ( a1 j , a2 j , … ,
anj ) T ( j = 1 , 2 , … , n - 1 ) , 如 何 求 这 样 的 η 呢 ? 下 面 介 绍 两 种
方法 .
( i) 设 A1 是由 α1 ,α2 , … ,αn - 1 按列排成的 n× ( n - 1 ) 矩阵 , 后
面学了矩阵的秩 就知 道 , 秩 ( A1 ) = n - 1 , 从而 A1 中必 存 在 n - 1
阶非零子式 .如果 A1 中去掉第 i 行的 n - 1 阶子式不等于 零 , 那 么
取 η= ( 0 , … , 0 , 1 , 0 , … , 0) T ( 其中 1 是第 i 个分量 ) , 就有
    | A| = |α1 α2 … αn - 1 η|
a11 a12 … a1 , n - 1 0
… … … …
= ai1 ai2 … ai , n - 1 1
… … … …
an1 an2 … ai n 0
a1 1 a12 … a1 , n - 1
… … …
ai - 1 , 1 ai - 1 , 2 … ai - 1 , n - 1
= ( - 1 ) r+ ≠0,
n

ai + 1 , 1 ai + 1 , 2 … ai + 1 , n - 1
… … …
an1 an2 … an , n - 1
于是 A 可逆 , A 的 n 个列向量线性无关 .
( ii ) 学了第 4 章 4 .2 节的 R n 中向 量的内积 , 就 知道如果 η与
线性无关的向 量 α1 , α2 , … , αn - 1 都 正交 ( 即 (αi , η) = 0 , i = 1 , … ,
T
n - 1) , 则 α1 ,α2 , … ,αn - 1 ,η就线 性无关 .设 η= ( x1 , x2 , … , xn ) ,
3 .2  n 维向量及其线性相关性 119

于是
(α1 ,η) = a11 x1 + a21 x2 + … + an1 xn = 0 ,
(α2 ,η) = a12 x1 + a22 x2 + … + an2 xn = 0 ,
…………………………………………………
(αn - 1 ,η) = a1 , n - 1 x1 + a2 , n - 1 x2 + … + an, n - 1 xn = 0 .
这是由 n - 1 个 方 程构 成的 n 元齐 次 线性 方程 组 , 从 而必 有非 零
解 , 任取一 组 非 零 解 作 η= ( x1 , x2 , … , xn ) T ≠0, 则 α1 , α2 , … ,
αn - 1 ,η就线性无关 .
(2 ) 澄清一些模糊的观念
由于初学者没有准确地 理解向 量的 线性相 关性 的 概念 , 常 常
容易想当然地形成一些似是而非的模糊认识 .例如 :
①“若 一组向 量 α1 ,α2 , … , αm 线性 相关 , 则 存在 全不 为零 的
数 k1 , k2 , … , km , 使得 k1 α1 + k2 α2 + … + kmαm = 0”.这里的 错误 在
“全不为零”, 正 确的 应 该是“不 全 为零”.例 如 , 3 个 共 面的 几何 向
3
量 α1 ,α2 ,α3 是线性 相关 的 , 当 α1 , α2 共 线且 | α2 | = | α1 | 时 ( 如
2
图 3 -4) , 只存在 3 , 2 , 0 , 使
3α1 + 2α2 + 0α3 = 0 .

图   3-4

这里要注意的是“
: 不全为零”包含“ 全不为零”, 而“全不为零”
不包含“不全为零”.
②“向 量组 α1 ,α2 , … , αm 线 性相关 的充 要条件 是 , 其 中任 一
120 第 3 章  线性方程组

个向量都可由其余向量线性表示”.这里充分性是成立的 , 即“ 任一
个向量都可由其余向量 线性表 示时 , 则 α1 ,α2 , … ,αm 是线 性相 关
的”是正确的 .但必要性是不成立的 , 这里错在“ 任一个”, 正确的应
该是“有一个”或说“至少有一 个”.例如 图 3 -4 中的 α3 不能 由 α1 ,
α2 线性表示 .这里的 错 误与 ①中 的 错误 是 一脉 相 承的 , 如果 那 里
的 k1 , k2 , … , km 全不为 零 , 自然 就 推出“ 任一 个”向 量 都可 由其 余
向量线性表示 .
③“如果 k1 , k2 , … , km 全为零时 , k1 α1 + k2 α2 + … + kmαm = 0,
则称 α1 ,α2 , … ,αm 线性无关”.这个说 法显 然是 错误的 , 因 为 k1 =
k2 = … = km = 0 时 , 不论 α1 ,α2 , … ,αm 是线 性无 关还 是线性 相关 ,
k1 α1 + k2 α2 + … + kmαm = 0 总 是 成立 的 .对于 线 性无 关 的向 量 正
确的说法应该是 :
“仅当 k1 , k2 , … , km 全为零时, 才能使 k1 α1 + k2 α2 + … + k3 α3 =
0 成立 , 则称 α1 ,α2 , …,αm 线性无关”.或 者说:“若 k1α1 + k2 α2 + … +
kmαm = 0, 则 k1 , k2 , … , km 必 须 全 为 零 , 就 称 α1 , α2 , … , αm 线 性
无关”.
④“向 量组 α1 ,α2 , … , αm 线 性无关 的充 要条件 是 , 其 中任 意
两个向量都线 性无 关 ( 或说 两 两线 性 无 关 ) ”.这 里 必要 性 是 成 立
的 , 因为一组向量整体线性无关 , 则其中任一部分向量都是线性无
关的 .但充分性是 不 成立 的 , 例 如 图 3 -3 中 共 面的 4 个 非 零 向 量
α1 ,α2 ,α3 ,α4 , 它们两两不共线 , 即两两线性无关 , 但整 个向量组 是
线性相关的 .
对于线性无关的向量来 说 , 用 其中 任何 部分向 量 的线 性无 关
性 , 都不能推出整体 的线 性无 关性 .因为 对 于 α1 , α2 , … , αm , 即 使
其中任意 m - 1 个 向量 都线性 无关 , 也 不能 推出 α1 ,α2 , … ,αm 线
性无关 .如前 面例 2 , 例 4 中 的 4 个向 量线 性 相关 , 而其 中 任 何 3
个向量都线性无关 .
需要指出的是 , 在欧氏空间中 “
: 一组非 零向量 α1 ,α2 , … ,αm ,
3 .2  n 维向量及其线性相关性 121

如果它们两两正 交 ( 即垂直 ) , 则 α1 ,α2 , … ,αm 线 性无关”, 这是 正


确的 ( 见教材 p168 中定理 4 .5 ) .
⑤“若 α1 ,α2 线性相关 ,β1 ,β2 线性相关 , 则 α1 + β1 ,α2 +β2 也
线性相关”.这是 不正 确的 .例如 , 在 几何 向量 中 α1 与 α2 共 线 ,β1
与 β2 共线 , 并不意味着 α1 +β1 与 α2 +β2 也一定共线 .如 :
α1 = ( 1 , 0) ,   α2 = ( - 1 , 0 ) ;   β1 = (1 , 1 ) ,   β2 = (2 , 2 ) .
而 α1 +β1 = (2 , 1 ) ,α2 + β2 = ( 1 , 2) 不共线 ( 或说不成比 例 ) , 从而 是
线性无关的 .
⑥“若 α1 ,α2 , α3 , α4 , α5 线 性 无 关 , 则 α1 - α2 , α2 + α3 , α3 -
α4 ,α4 + α5 ,α5 - α1 也线性无关”.这也是不正确的 , 因为 , 由
(α1 - α2 ) + (α2 + α3 ) - (α3 - α4 ) - (α4 + α5 ) + (α5 - α1 ) = 0,
可知 α1 - α2 ,α2 + α3 ,α3 - α4 ,α4 + α5 ,α5 - α1 是线性相关的 .
⑦“若 α1 ,α2 , … ,αn 线性 相关 , 则 α1 + α2 ,α2 + α3 , … ,αn - 1 +
αn ,αn + α1 这 n 个 向量 不一 定 线性 相 关”.这 里不 是“ 不 一定”, 而
是“一定”线性相关 .因为 :
α1 ,α2 , … , αn 线 性相 关的 充要 条 件是 , 其 中 有一 个 向量 可 由
其余向量线性表示 , 不妨设 αn = a1 α1 + a2 α2 + … + an - 1 αn - 1 .于 是
α1 + α2 ,α2 + α3 , … , αn - 1 + αn , αn + α1 这 n 个向 量都 可 以由 n - 1
个向量 α1 ,α2 , … ,αn - 1 线 性表示 , 根 据定 理 3 .4 ( 教材 p120 ) , 它 们
是线性相关的 .

3  小结

( 1) 讨论一组向量 α1 ,α2 , … ,αm ( 不论是具体的还是抽象的向


量 ) 的线性相关性问题 , 一般都设
x1 α1 + x2 α2 + … + xmαm = 0, ( 1)
然后得到关于 x1 , x2 , … , xm 的一个齐次线性方程组
Ax = 0, ( 2)
T
其中 x = ( x1 , x2 , … , xm ) .如 果 Ax = 0 有 非 零解 ( 只 有 零解 ) , 则
122 第 3 章  线性方程组

α1 ,α2 , … ,αm 线性相关 ( 线性无关 ) .


当 然 , 有 的 时 候 是 可 以 直 接 看 出 有 没 有 不 全 为 零 的 x1 ,
x2 , … , xm 使 ( 1) 式成立的 , 如上面 ( 2) 段⑥的情况 ; 再如 , 对于两 个
非零向量 α= ( a1 , … , an ) ,β= ( b1 , … , bn ) , 如果 它们不 成比 例 , 就
一定线性无关 .
(2 ) 要熟悉定理 3 .1 , 定理 3 .3 , 定 理 3 .4 的结 论 及它 们的 推
论 , 不仅要会证明它们 , 而且要会灵活地应用它们分析和解决一些
向量的线性相关性的问题 .前面的例子中多处用到它们 .
(3 ) 要 善于 用反 证法 证明 问 题 .如前 面 的例 6 , 例 8 , 例 9 , 例
10 中都用反证法证明了有关的结论 .

3 .3 向量组的秩及其极大线性无关组

向量组的秩的概念在内 容提要 中已 经叙述 .向 量 组的 秩也 是


线性代数中重要 的 概念 .关于 向 量组 的 秩的 基 本 理 论 , 是 以 定 理
3 .4 为基础 .
定理 3 .4   设向量组 β1 ,β2 , … ,βt 可由向量组 α1 ,α2 , … ,αs 线
性表示 , 如果 t > s, 则 β1 ,β2 , … ,βt 线性相关 .
这个定理的等价命题为 : 在如 定 理的 所设 下 , 如 果 β1 ,β2 , … ,
βt 线性无关 , 则 t≤ s .
由此即得 : 如果向量 组 β1 ,β2 , … ,βt 可 由向 量组 α1 ,α2 , … ,αs
线性表示 , 则
p = 秩{β1 , … ,βt }≤秩{α1 , … ,αs } = r .
因为{β1 , … ,βt }中由 p 个向量组成的极大线性无关组 {βi1 ,βi2 , … ,
βip } , 可由{α1 , … ,αs } 中 由 r 个 向 量组 成 的 极大 线 性无 关 组 {αj1 ,
αj2 , … ,αj r }线性表示 .
进而就有 : 如果两个向量组 {β1 ,β2 , … ,βt }与 {α1 ,α2 , … ,αs } 是
等价的 ( 即它们可以互相线性表示 ) , 则它们的秩相等 , 即
3 .3 向量组的秩及其极大线性无关组 123

秩{β1 , … ,βt } = 秩{α1 , … ,αs } .


这正是 : 初 等变 换不 改变矩 阵的 秩 ( 即矩 阵的 秩是矩 阵做 初
等变换时的一个不变量 ) 的理论根据 .
如何求向量组的秩及其 极大线 性无 关组 , 在内 容 提要 中已 讲
过 .对于下面的阶梯形矩阵

1 - 1 3  1  2
0  2 1 - 3  6
U = (ξ1 ξ2 ξ3 ξ4 ξ5 ) =
0  0 0  5 -8
0  0 0  0  0

的列向量组 ξ1 ,ξ2 ,ξ3 ,ξ4 ,ξ5 ( 此 时都 去 掉第 4 个 分量 而 视为 三 维


向量 , 其线性相关性不 变 ) , 显 然存 在 3 个 列向 量线 性无关 ( 例如 :
{ξ1 ,ξ2 ,ξ4 } , {ξ1 ,ξ2 ,ξ5 } , {ξ1 ,ξ3 ,ξ4 }等 都线 性无关 , 因 为它 们去 掉
第 4 个分量后做成的三阶 行列 式都不 等于 零 , 相应的 三阶 矩阵 是
可逆的 ) , 而任 何 4 个列向量 都线性相 关 ( 因为 任何 4 个三维向 量
必线性相关 ) .所以 U 的列向 量组 的秩为 3 , 上 面提 到 的 3 个线 性
无关的列向量 ( 如 ξ1 ,ξ2 ,ξ4 等 ) 都是它的极大线性无关组 .
再利用“
: 对矩阵 A 做初等行变换将其化 为阶梯形 矩阵 U 时 ,
A 与 U 的对应的列向量组有相同的线性相关性”( 定理 3 .6) , 就 可
容易地求得向量组的秩及其极大线性无关组 .
例 1   求下列向量 组 的秩 及其 一个 极 大线 性 无关 组 , 并 将 其
余向量用这个极大线性无关组线性表示之 .
T T
α1 = ( 1 , 2 , 3 , 4 ) , α2 = ( - 1 , - 1 , - 2 , - 2 ) , α3 = ( 2 , 3 , 5 ,
T T T
6) ,α4 = ( - 2 , - 2 , - 1 , - 1) ,α5 = ( 1 , 1 , 3 , 3) .

解   将 α1 ,α2 ,α3 , α4 ,α5 按列 排成一 个 4 × 5 矩 阵 A, 对 A 做


初等行变换将其化为阶梯形矩阵 , 即
124 第 3 章  线性方程组

1 -1 2 -2 1
2 -1 3 -2 1
       A=
3 -2 5 -1 3
4 -2 6 -1 3
1 -1  2 -2  1
② + ①× ( - 2 ) 0  1 -1  2 -1
③ + ①× ( - 3 ) 0  1 -1  5  0
④ + ①× ( - 4 )
0  2 -2  7 -1
1 -1  2 -2  1
③ + ②× ( - 1 ) 0  1 -1  2 -1
④ + ②× ( - 2 ) 0  0  0  3  1
0  0  0  3  1
1 -1  2 -2  1
    0  1 -1  2 -1
=U . ( 1)
0  0  0  3  1
0  0  0  0  0
U 中第 1 , 2 , 4 三个列 向 量是 U 的列 向 量组 的 一 个极 大 线性 无 关
组 , 所以 A 的列向量组 α1 ,α2 ,α3 ,α4 ,α5 的一个极大线性无关组 为
α1 ,α2 ,α4 , 从而
秩{α1 ,α2 ,α3 ,α4 ,α5 } = 3 .
下面将 α3 和 α5 分别用{α1 ,α2 ,α4 }线性表示 : 设
x1 α1 + x2 α2 + x4 α4 = α3 ,
由此得到非齐次线性方程组
T
A1 x = α3     ( 其中 x = ( x1 , x2 , x4 ) ) . ( 2)
T
对其增广矩阵 ( A1 ,α3 ) 做初等行变换化为阶梯矩阵 , 即
1 -1 -2  2
利用 0  1  2 -1
( A1 ,α3 ) = (α1 α2 α4 α3 ) . (3)
( 1) 式得 0  0  3  0
0  0  0  0
3 .4 矩阵的秩  * 矩阵的相抵标准形 125

(3 ) 式右端的阶梯阵 , 就是由 (1 ) 式 U 中的第 1 , 2 , 4 , 3 四个 列依 次


排成的阶梯矩阵 ( 因为对矩阵做初等行变换时 , 列向量之间是互不
影响的 ) .由 (3 ) 式得到方程组 ( 2) 的同解方程组
x1 - x2 - 2 x4 = 2 ,
    x2 + 2 x4 = - 1 ,
        3 x4 = 0 .
由此即得 : x4 = 0 , x2 = - 1 , x1 = 1 , 所以
α3 = 1・α1 + ( - 1 )α2 = α1 - α2 .
再设
x1 α1 + x2 α2 + x4 α4 = α5 , ( 4)
同理 , ( 4) 式对应的方程组 A1 x = α5T 的增广 矩阵 ( A1 ,α5T ) 做初等 行
变换 , 可化为
1 - 1 -2 1
    0 1 2 - 1
( A1 ,α5T ) .
0 0 3 1
0 0 0 0
1 5
由此即得 : x4 = , x2 = - , x1 = 0 , 所以
3 3
-5 1 5 1
α5 = 0・α1 + α2 + α4 = - α2 + α4 .
3 3 3 3


3 .4 矩阵的秩  矩阵的相抵标准形

矩阵的秩也是线性代数 中一个 极为 重要的 概念 , 它在 线性 方


程组解的理论中具有举足轻重的地位 .
矩阵的列向量组的秩 ( 简称列秩 ) , 矩阵行向量组的秩 ( 简称行
秩 ) , 矩阵的行列式秩 ( 即矩 阵非零 子式 的最 高阶数 ) , 三者 是相 等
的 ( 定理 3 .8 , 定理 3 .10 ) , 我们把它们统称为矩阵的秩 .
126 第 3 章  线性方程组

对矩阵 A 做初等行变换 ( 即对 A 的行向 量组做线 性运算 ) , 将


其化为矩阵 B, 即存在初等矩阵 P1 , P2 , … , Ps , 使
Ps … P2 P1 A = B .
从而
- 1 - 1 - 1
P1 P2 … Ps B = A
- 1 - 1 - 1
( 其中 Ps , … , P2 , P1 也是初 等 矩阵 ) , 这 表明 对 B 做初 等行 变
换也可化为 A .所 以 B 的 行向 量 组 是 由 A 的 行 向 量 组 线 性 表 示
的 ; 反之 , A 的行向量组也是由 B 的行向 量组线性 表示 的 .因此 , A
与 B 的行向量组是等价向量组 , 它们 的行秩 相等 , 这 就是说 , 初 等
行变换不改变矩阵的行秩 .同 样初等 行变 换也 不改变 矩阵 的列 秩
( 定理 3 .6 ) .同 理 , 初等 列变 换 也不 改变 矩阵 的列 秩 和行 秩 .综 合
起来就是 , 初等变换不改变矩阵的秩 .
对矩阵做初等变换 , 矩阵的元素会呈现出各种各样的变化 , 但
是它的秩始终是不变的 .所以 , 我 们说 , 矩阵 的秩 是矩 阵在 初等 变
换下的一个不变量 .
我们用高斯消元法求解线性 方 程组 Ax = b, 其 消 元过 程就 是
对增广矩阵 ( A, b) 做 初等 行 变换 , 将 其化 为 阶梯 形矩 阵 ( C, d) .此
时 , Ax = b 的同解方程组
Cx = d
有解的充分必要条件是 : 阶梯形 的系数 矩阵 C 的非 零行行 数与 阶
梯形的增广矩阵 ( C, d) 的非 零行 行数 相 等 .而 阶 梯形 矩 阵的 非 零
行行数就是它的行秩 , 也就是矩阵的秩 ; 再由于初等变换不改变矩
阵的秩 , 所以 Ax = b 有解的充分必要条件是 , 秩 ( C) = 秩 ( C, d) , 即
秩 ( A) = 秩 ( A, b) .
如果 A 是 m× n 矩阵 , 秩 ( A) = r, 那么不论做怎样的初等行变
换将增广矩阵 ( A, b) 化为阶梯形矩阵 ( C, d) 时 , 其非零 行行数都 为
r, 求解时都有且仅有 n - r 个自由未知量 .以上就是矩 阵的秩在 线
性方程组解的理论中所起到的重要作用 .
3 .4 矩阵的秩  * 矩阵的相抵标准形 127

对于 n 阶矩阵 A, r( A) = n 的充要条件是 A 为可逆矩阵 ( 也 称


非奇异矩阵 ) , 即 | A| ≠0( 定理 3 .9) .
求矩阵 A 的 秩的方 法 : 对 A 做 初等 行变换 , 将其 化为 阶梯 形
矩阵 U , 则 r ( A) 就等于 U 的非零行行数 .
例1  设
1 -1  2 -1 0
2 -2  4 -2 0
A= ,
3  0  6 -7 1
0  3  0 0 1
求秩 ( A) , 并指出它的一个最高阶的非零子式 .
解   对 A 做初等行变换 , 将其化为阶梯形矩阵 U, 即

1 -1 2 1 0
② + ① × ( - 2) 0  0 0  0 0
A ③ + ① × ( - 3) 0  3 0 -4 1
0  3 0 0 1
1 -1 2 1 0
④ + ③ × ( - 1) 0  3 0 -4 1
= U,
再做 两次 0  0 0  4 0
行对换
0  0 0  0 0

所以 , 秩 ( A) = 3 , U 中一个最高阶的非零子式为
1 -1  1
0  3 - 4 = 12 ≠0
0  0  4
它对应于 A 中第 1 , 3 , 4 行与第 1 , 2 , 4 列交点排成的三阶行列式
1 -1 - 1
1 -1
3  0 - 7 = -3 = 12≠0 .
3 -7
0  3  0
128 第 3 章  线性方程组

例2  设
1 -1 2 -1 0
2 -2 4 -2 0
A= .
3  0 5 -7 1
a  3 b  c 2
问: a, b, c 为何值时 , A 的列向量组的极大线性无关组只有两个向量 .
解   A 的列向量组的极大 线性 无关组 所含 向量 个数就 是A的
列秩 , 也就是 A 的秩 .所以此题就是问 : a, b, c 取何值时 , r ( A) = 2 .
对 A 做初等行变换 , 将其化为矩阵 B .即
1 -1  2 -1 0
② + ①× ( - 2) 0  0  0  0 0
A = B.
③ + ①× ( - 3) 0  3 -1 -4 1
④ + ①× ( - a)
0 3+ a b - 2a c + a 2
由于 r ( B) = r ( A) , 欲使 r( B) = 2 , B 中第 3 行与第 4 行的行向量必
须成比例 , 所以得
3 + a = 2× 3 ,   a = 3 ;
b - 2 a = - 1×2 ,   b = - 2 + 2 a = 4;
c + a = - 4×2 ,   c = - 8 - a = - 11 .
即 a = 3 , b = 4 , c = - 11 时 , r ( A) = 2 .
A C
例 3   设分块矩阵 P =   ( 其中 C 为任意矩阵 ) , 证明
0 B
r( P) ≥ r ( A) + r( B) .
证   不妨设 A= (α1 , … ,αs ) 的列向 量组的一 个极大线 性无 关
组为{α1 ,α2 , … ,αm } ( 其中 m≤ s) , 从而 r ( A) = m; B = (β1 , … ,βt )
的列向量组的一个极大线性无关组 为{β1 ,β2 , … ,βn } ( 其 中 n≤ t) ,
从而 r ( B) = n .
( i ) 当 C= 0( 为零矩阵 ) 时 , P 中α1 , … ,αm 与β1 , … ,βn 所 在列
的 m + n 个列向 量是 P 的列 向量组 的一 个极大 线性 无关 组 , 所以
3 .4 矩阵的秩  * 矩阵的相抵标准形 129

r ( P) = m + n = r( A) + r( B) .
( ii ) 当 C≠0 时 , B 中 线 性相 关 的 列向 量 添 加 了 C 中 的 分 量
后 , 有可能是线性无关的 , 所以 , P 的 列向 量组 的极 大 线性 无关 组
所含向量个数可能等于 m + n, 也可能大于 m + n, 因此
r ( P) ≥ m + n = r( A) + r( B) .
例 4   设 A 是 m × n 矩 阵 , 且 m < n, 证 明 : 齐 次线 性 方 程 组
T
( A A) x = 0 必有非零解 .
T T T
证   A A 是 n × n 矩 阵 , 由于 r ( A) = r ( A ) ≤ m, r ( A A) ≤
T T
min( r( A) , r ( A ) ) ≤ m < n, 那 么 , 用 高 斯 消 元 法将 系 数 矩 阵 A A
通过初等行变换化为阶梯形矩阵时 , 其 非零行行 数小于 n, 从而 求
T
解时有 n - r( A A) 个自由未知量 , 所以必有非零解 .
或者根据齐次线性方程组解 的理论 ( 定理 3 .12 ) , 以 n 阶 矩阵
T T
A A 为系数矩阵的齐次 线 性方 程组 ( A A) x = 0 有非 零 解的 充 要
T
条件为 r ( A A) < n .
例 5 ( 习题 18 )   设 A 是 s× n 矩阵 , B 是由 A 的前 m 行构成的
m× n 矩阵 .证明 : 若 A 的行向量组的秩为 r, 则 r( B) ≥ r + m - s .
证   设 αi = ( ai1 , ai2 , … , ai n ) ,   i = 1 , 2 , … , s,
α1

α1
αm
A= ,     B= … .
αm + 1
αm

αs
设 r ( B) = p, 于是 B 的 行 向量 组 的 极 大线 性 无 关 组 {αi1 ,αi2 , … ,
αi p } 含 p 个向量 .因此 , A 的行向量组的 一个极 大线性无 关组是 向
量组{αi1 ,αi2 , … ,αip ,αm + 1 , … ,αs }的一个子集 , 也就是 它所含向 量
个数≤ p + ( s - m) , 即
r( A) = r≤ p + ( s - m) ,
130 第 3 章  线性方程组

从而
r( B) = p≥ r + m - s .

例 6 ( 习题 25 )   设 A 是 m × n 矩 阵 ( m < n) , r ( A) = m, 证
明 : 存在 n× m 矩阵 B, 使 AB= Im .
证   利用矩阵的相抵标准形和分块矩阵的乘法来证明 .
由于 r( A) = m, 所以存在 m 阶可逆阵 P 和 n 阶可逆阵 Q, 使得
PAQ= ( Im , 01 )   ( 其中 01 是 m× ( n - m) 零矩阵 ) ,

- 1 - 1
A= P ( Im , 01 ) Q .

Im
B= Q P,
02
其中 : 02 是 ( n - m) × m 零矩阵 , B 是秩为 m 的 n× m 矩阵 .于是

Im
AB = P - 1 ( Im , 01 ) Q - 1 Q P = P- 1 I m P = Im .
02

例 7 ( 习 题 27 )   证明 : 任 何秩 为 r 的矩 阵 A 可 以表示 为 r
个秩为 1 的矩阵之和 , 但不能表示为少于 r 个秩为 1 的矩阵之和 .
证   不妨设 A 为 m× n 矩阵, 证明时要用到:“若 P, Q 分别为 m
阶和 n 阶可逆矩阵, 则 r( PAQ) = r( A)”, 以及矩阵的相抵标准形 .
由于秩( A) = r, 所以存在 m 阶可逆阵 P 和 n 阶可逆阵 Q, 使得
Ir 0
PAQ=   ( 其中 Ir 为 r 阶单位阵 ) ,
0 0 m× n

从而
Ir 0
A = P- 1 Q- 1 .
0 0
令 r 阶对角阵
3 .5 齐次线性方程组有非零解的条件及解的结构 131

0

Λr i = 1 ,

0
其中第 i 个主对角元为 1 , 其余对角元均为零 , 如此则有
Ir = Λr1 + Λr2 + … + Λr r ,
于是 , 取
Λr i 0
Q - 1   ( i = 1 , 2 , … , r) ,
- 1
Ai = P
0 0
则秩 ( Ai ) = 秩 ( Λr i ) = 1 , 且有
A = A1 + A2 + … + Ar .
所以 A 可表示为 r 个秩为 1 的矩阵 A1 , A2 , … , Ar 之和 .
如果 B1 , B2 , … , Bp 的秩均为 1 , 则
r( B1 + B2 + … + Bp ) ≤ r ( B1 ) + r( B2 ) + … + r( Bp ) = p .
因此 , 当 p < r 时 , 秩为 r 的 A 不可能表示为 B1 , B2 , … , Bp 之和 .

3 .5 齐次线性方程组有非零解的条件
   及解的结构

    齐次线性方程组 Ax = 0 有非 零 解的 条 件 , 有 非零 解 时 , 它 的
基础解系的概念 , 求基 础解系 的方 法 , 以及它 的一 般解 的结 构 , 在
内容提要中已经详述 .
这里有两个重要的例题 ( 教材 p137 上的例 3 , 例 4 ) , 要熟悉其
结论并会证明 .
(1 ) 若 A, B 分别是 m× n 和 n× s 矩阵 , 且 AB= 0, 则
r( A) + r( B) ≤ n .
证明时 , 由 AB = 0 可知 , B 的 s 个列 向量 是 齐次 线 性方 程 组
132 第 3 章  线性方程组

Ax = 0 的 解 , 再 利用 Ax = 0 的基 础 解系 含 n - r ( A) 个 解 , 即 可 证
明之 .
* T
( 2 ) 若 A 是 m × n 实 矩 阵 , 则 r ( A A) = r ( A ) .( 证 明 见
教材 ) .
1 1 2 2 7
例 1   设 A= 2 2 1 1 2 .
5 5 1 2 0
求齐次线性方程组 Ax= 0 的基础解系与一般解 .
解   对系数矩阵 A 做初等行变 换 , 将其化 为行 简化阶 梯矩 阵
U, 即
1 1  2  2  7
② + ①× ( - 2 )
A 0 0 -3 -3 - 12
③ + ①× ( - 5 )
0 0 -9 -8 - 35
1 1 2 2 7 1 1 2 0 5
③ + ② ×( - 3 ) ② + ③× ( - 1)
0 0 1 1 4 0 0 1 0 3
1 ① + ③× ( - 2)
②× -
3 0 0 0 1 1 0 0 0 1 1
1 1 0 0 -1
① + ② ×( - 2 )
0 0 1 0  3 =U .
0 0 0 1  1
Ux = 0 与 Ax = 0 是 同解 方 程组 , 求 基础 解系 时 , 取 x2 , x5 为
自由未知量 , 它们分别取 1 , 0 和 0 , 1 时 , 得基础解系中的两个解
x1 = ( - 1 , 1 , 0 , 0 , 0) T ;   x2 = ( 1 , 0 , - 3 , - 1 , 1) T .
方程组的一般解为
x = k1 x1 + k2 x2 = k1 ( - 1 , 1 , 0 , 0 , 0) T + k2 (1 , 0 , - 3 , - 1 , 1 ) T ,
其中 k1 , k2 为任意常数 .
例 2   ( 研 3 -35 .教材 p381 .表示 研 究生 入 学试 题 汇 编 : 3 .线
性方程组中的 35 题 .以后用同样的简称 )
设四元齐次线性方程组 ( Ⅰ ) 为
3 .5 齐次线性方程组有非零解的条件及解的结构 133

2 x1 + 3 x2 - x3 = 0 ,
x1 + 2 x2 + x3 - x4 = 0 .
已知另一个四元齐次线性方程组 ( Ⅱ ) 的一个基础解系为
α1 = ( 2 , - 1 , a + 2 , 1 ) T ,   α2 = ( - 1 , 2 , 4 , a + 8 ) T .
(1 ) 求方程组 ( Ⅰ ) 的一个基础解系 ;
(2 ) 当 a 为何值时 , 方 程 组 ( Ⅰ ) 与 ( Ⅱ ) 有 非零 公共 解 ? 在 有
非零公共解时 , 求全部非零公共解 .
解   (1 ) 对系数 矩阵 做初 等行 变 换 , 将其 化为 行 简化 阶梯 矩
阵,即
2 3 -1  0     1 2  1 -1
1 2  1 - 1 2 3 -1  0
    1  2  1 -1     1  0 - 5 3
0 -1 -3  2 0 -1 - 3 2
    1 0 -5  3
.
0 1  3 -2
所以方程组 ( Ⅰ ) 的同解方程组为
x1 - 5 x3 + 3 x4 = 0 ,
x2 + 3 x3 - 2 x4 = 0 .
取 x3 , x4 为自由未知量 , 它们分别取 1 , 0 与 0 , 1 得基础解系 :
β1 = (5 , - 3 , 1 , 0 ) T ,   β2 = ( - 3 , 2 , 0 , 1) T .
(2 ) 方程组 ( Ⅰ ) 与 ( Ⅱ ) 的一般解分别为 :
x = k1 β1 + k2 β2 , ( 1)
x = k3 α1 + k4 α2 , ( 2)
它们的公共解就是 (1 ) 式和 ( 2) 式中相等的解 , 即公共解为
x = k1 β1 + k2 β2 = k3 α1 + k4 α2 . ( 3)
由 (3 ) 式得
k1 β1 + k2 β2 - k3 α1 - k4 α2 = 0   ( 零向量 ) . ( 4)
134 第 3 章  线性方程组

(4 ) 式等价于一个齐次线性方程组
k1  5 -3 -2  1 k1
k2 -3  2  1 -2 k2
(β1 β2 - α1 - α2 ) =
k3  1  0 - a- 2 -4 k3
k4  0  1 -1 - a- 8 k4
0
0
= . ( 5)
0
0
对方程组 (5 ) 的系数矩阵 A 做初等行 变换 , 将其 化为 阶梯矩 阵 .如
直接用第 1 列的 5 消去 - 3 和 1 , 将会出现分数 , 以后很不方便 , 所
以先把第 2 行乘 2 加到第 1 行 , 得
-1 1  0 -3
    -3 2  1 -2
       A
 1 0 - a-2 -4
 0 1 - 1 - a-8
-1  1  0 -3
② + ①× ( - 3 )  0 -1  1  7
③+ ①  0  1 - a-2 -7
 0  1 -1 - a-8
- 1 1 0 -3
③ +② 0 -1 1  7
=U ( 6)
④ +② 0 0 - a-1  0
0 0 0 - a- 1
当 a≠ - 1 时 , | U | ≠0 , 线性 方程 组 Ak = 0 只有 零 解 , 线性 方
程组 ( Ⅰ ) 与 ( Ⅱ ) 没有非零公共解 .
当 a= - 1 时 ,
3 .5 齐次线性方程组有非零解的条件及解的结构 135

-1 1 0 -3 -1 0 1 4
0 -1 1 7 行 0 - 1 1 7
U= .
0 0 0 0 变换 0 0 0 0
0 0 0 0 0 0 0 0
齐次线性方程组 Uk = 0( 与方程 组 ( 5 ) Ak= 0 是 同解 方 程组 , 其 中
k = ( k1 , k2 , k3 , k4 ) T ) 有非零解 , 其基础解系为
T T
k1 = (1 , 1 , 1 , 0 ) ,   k2 = ( 4 , 7 , 0 , 1) ,
一般解
k= c1 k1 + c2 k2 = ( c1 + 4c2 , c1 + 7c2 , c1 , c2 ) T
( 其中 c1 , c2 为任意常数 ) .将 k3 = c1 , k4 = c2 代入 ( 3 ) 式 , 即 得方 程
组 ( Ⅰ ) 与 ( Ⅱ ) 的全部非零公共解
x = k3 α1 + k4 α2 = c1 α1 + c2 α2 .
T T
当 a = - 1 时 ,α1 = (2 , - 1 , 1 , 1 ) ,α2 = ( - 1 , 2 , 4 , 7 ) .容易 验
证 , 此时 α1 与 α2 也是 方程 组 ( Ⅰ ) 的基 础 解 系 .所 以 , a = - 1 时 ,
方程组 ( Ⅰ ) 与 ( Ⅱ ) 是同解 方程 组 , 它们 的全部 公共 解就是 它们 自
身全部的解 .
求方程组 ( Ⅰ ) 与 ( Ⅱ ) 的非 零公共 解的 另一个 解法 为 : 先根 据
方程组 ( Ⅱ ) 的基础解系 , 反过来求一个方程组 ( Ⅱ ) .
T T
设 α1 = ( 2 , - 1 , a + 2 , 1 ) 与 α2 = ( - 1 , 2 , 4 , a + 8 ) 是方程
a1 x1 + a2 x2 + a3 x3 + a4 x4 = 0 ( 7)
的解 , 于是 , 得
2 a1 - a2 + ( a + 2 ) a3 + a4 = 0 ,
( 8)
- a1 + 2 a2 + 4 a3 + ( a + 8 ) a4 = 0 .
方程组 (8 ) 是关于 a1 , a2 , a3 , a4 的 四元齐次 线性 方程 组 , 容 易求 得
它的基础解系的两个解 :
a1 = 2 a + 8 ,   a2 = a + 10 ,   a3 = - 3 ,   a4 = 0; ( 9)
a1 = a + 10 ,   a2 = 2 a + 17 ,   a3 = 0 ,   a4 = - 3 . (10)
将 (9 ) 和 ( 10 ) 式分别代入方程 ( 7) , 即可得到一个方程组 ( Ⅱ )
136 第 3 章  线性方程组

(2 a + 8) x1 + ( a + 10 ) x2 - 3 x3 = 0 ,
(11)
( a + 10) x1 + ( 2 a + 17) x2 - 3 x4 = 0 .
( 读者不难验证 α1 = (2 , - 1 , a + 2 , 1) T 与 α2 = ( - 1 , 2 , 4 , a + 8 ) T
是方程组 (11) 的基础解系 .) 将方程组 (11) 与题给的方程组 ( Ⅰ ) 联
立起来 , 这个联立方程组的非 零解就 是方 程组 ( Ⅰ ) 与 ( Ⅱ ) 的共 同
的非零解 .此时 , 同样可以得知 :
当 a = - 1 时 , 全部共同的非零解为方程组 ( Ⅰ ) 与 ( Ⅱ ) 的自身
全部非零解 .当 a≠ - 1 时 , 方程组 ( Ⅰ ) 与 ( Ⅱ ) 没有共同的非零解 .
1 1 1
例 3 ( 研 3 -28 )   设   A = a b c , 问:
a2 b2 c2
(1 ) a, b, c 满足什么关系时 , Ax = 0 只有零解 ;
(2 ) a, b, c 满足什么关系 时 , Ax = 0 有 无穷 多组 解 , 并 用基 础
解系表示通解 .
解   (1 ) 当 | A| ≠0 时 , Ax = 0 只有零解 .这 里的 | A| 是三阶 范
德蒙行列式 , 所以
| A| = ( b - a) ( c - a) ( c - b) .
当 a, b, c 互不相等时 , | A| ≠0 , 从而 Ax = 0 只有零解 .
(2 ) 当 | A| = 0 时 , Ax = 0 有无穷多组解 , 所以当 a = b 或 a = c
或 b = c 或 a = b = c 时均有无穷多组解 .
(i) 当 a= b 时 , 有
1 1 1 1 1 1 1 1 1

A= a a c 0 0 c- a 0 0 c- a .
变换
a2 a2 c2 0 0 c2 - a2 0 0 0
取 x2 为自由未知量 , 基础 解 系为 ( - 1 , 1 , 0 ) T , 通解 为 x = k( - 1 ,
T
1 , 0) , k 为任意常数 .
T
同理 , 当 a = c 时 , 通解为 x= k( - 1 , 0 , 1) ; 当 b = c 时 , 通解为
T
x = k(0 , - 1 , 1 ) , k 均为任意常数 .
3 .5 齐次线性方程组有非零解的条件及解的结构 137

( ii ) 当 a = b = c 时 , 有
1 1 1 1 1 1
A= a a a 0 0 0 .
2 2 2
a a a 0 0 0
取 x2 , x3 为自 由未 知量 , 基础 解系 为 ( - 1 , 1 , 0 ) T 与 ( - 1 , 0 , 1) T ,
通解为 x = k1 ( - 1 , 1 , 0) T + k2 ( - 1 , 0 , 1 ) T , k1 , k2 为任意常数 .
λ- 1 1 λ
例 4   设 A= 1 λ- 1 0 .
λ 1 λ- 1
若存在三阶矩阵 B≠0 , 使 AB = 0, 则必有 (   ) .
3
( A ) λ= 0 或 λ= 2 , | B| = 0;   ( B) λ= 0 或 λ= , | B| = 0;
2
2 3
( C) λ= 0 或 λ= , | B| ≠ 0; ( D ) λ= 0 或 λ= , | B| ≠0 .
3 2
解   设 B 的三个列向量为β1 ,β2 ,β3 , 则由
AB = A(β1 ,β2 ,β3 ) = ( Aβ1 , Aβ2 , Aβ3 ) = (0 ,0 ,0)
( 其中 3 个 0 均为三维 零向 量 ) 可知 ,β1 ,β2 ,β3 均为 齐 次线 性方 程
组 Ax= 0 的解向量 .
因此 , 题设中的三阶非零矩阵 B 的列向量至少有一个是 Ax =
0 的非零解 ( 另外的列向量可以是零向量 ) , 所以必有
λ- 1 1 λ
| A| = 1 λ- 1 0 = (λ- 1) 3 + λ- λ2 (λ- 1) - (λ- 1)
λ 1 λ- 1
2
= - 2λ + 3λ= 0 .
3
从而必有 λ= 0 或 λ= .
2
3
当 λ= 0 或 λ= 时 , 秩 ( A) = 2 , 因此 Ax = 0 的基础 解系只 有
2
一个非零向量 .所以 B 的 3 个列向 量的 极大线 性无 关组只 有一 个
138 第 3 章  线性方程组

向量 , 即 B 的列秩 = 秩 ( B) = 1 .从而 | B| = 0 .
综上 , 答案应选 ( B) .
例 5 ( 研 3 -37 )   设齐次线性方程组
ax1 + bx2 + bx3 + … + bxn = 0 ,
bx1 + ax2 + bx3 + … + bxn = 0 ,
( 1)
………………………………
bx1 + bx2 + bx3 + … + axn = 0 .
其中 a≠ 0 , b≠0 , n≥2 .试讨论 a, b 为何值 时 , 方 程组 仅有零 解、有
无穷多组解 ? 在有无穷多组解时 , 求出全部解 , 并用基础解系表示
全部解 .
解   从方程组 (1 ) 中 依次 有 ax1 , ax2 , … , axn 可 见 , n 元 方 程
组 (1 ) 中有 n 个方程 .它的系数行列式
a b b … b
b a b … b
  | A| =
… … … …
b b b … a
1 b b … b
1 a b … b
= [ a + ( n - 1 ) b]
… … … …
1 b b … a
1 b b … b
0 a- b 0 … 0
= [ a + ( n - 1 ) b]
… … … …
0 0 0 … a- b
= [ a + ( n - 1 ) b] ( a - b) n - 1 .
(1 ) 当 | A| ≠ 0 , 即 a≠ b 且 a ≠ ( 1 - n) b 时 , 方 程 组 ( 1 ) 仅 有
零解 .
(2 ) 当 | A| = 0 , 即 a = b 或 a = ( 1 - n) b 时 , 方程组 ( 1 ) 有无 穷
3 .5 齐次线性方程组有非零解的条件及解的结构 139

多组解 .
当 b = a 时 , 秩 ( A) = 1 , 基础解系有 n - 1 个解向量 :
T T T
( - 1 , 1 , 0 , … , 0) , ( - 1 , 0 , 1 , … , 0) , … , ( - 1 , 0 , 0 , … , 1 ) .
其全部解为
T T
      x = k1 ( - 1 , 1 , 0 , … , 0) + k2 ( - 1 , 0 , 1 , … , 0 ) + … +
T
kn - 1 ( - 1 , 0 , 0 , … , 1 ) ,
其中 k1 , k2 , … , kn - 1 为任意常数 .
当 a = (1 - n) b 时 , 方程组 (1 ) 的系数矩阵
(1 - n) b b b … b
b (1 - n) b b … b
A = b b (1 - n) b … b
… … … …
b b b … (1 - n) b
0 1 1 … 1
0 1 - n 1 … 1

b 0 1 1- n … 1 ( 2)
变换
… … … …
0 1 1 … 1- n
0  1  1 …  1
0 - n  0 …  0

0  0 - n …  0 =B.
变换
… … … …
0  0  0 … - n n× n

由于 B 中非零子式最高阶数为 n - 1 , 所以秩 ( B) = 秩 ( A) = n - 1 .
因此 , 方程组 ( 1) 即 Ax = 0 的基础解系含 n - r ( A) = n - ( n - 1 ) = 1
个向量 .由 ( 2) 式的矩阵 A 可 见 , 当 x1 = x2 = … = xn = 1 时 , 满 足
T
方程组 Ax= 0, 所以基础解系为 (1 , 1 , 1 , … , 1) , 其全部解为
T
x = k( 1 , 1 , 1 , … , 1 ) , 其中 k 为任意常数 .
140 第 3 章  线性方程组

例 6 ( 研 3 -38 )   设 α1 , α2 , … ,αs 为 线性 方程组 Ax = 0 的 一


个基础解系 ,β1 = t1 α1 + t2 α2 ,β2 = t1 α2 + t2 α3 , … ,βs = t1 αs + t2 α1 ,
其中 t1 , t2 为实常数 , 试 问 t1 , t2 满 足 什么 关 系时 ,β1 ,β2 , … ,βs 也
为Ax = 0 的一个基础解系 .
解   已知齐次线性方程组 Ax = 0 的基础解系 α1 ,α2 , … ,αs 有
s 个解向量 , 所以 Ax = 0 的 任何 s 个线 性无 关 的 解都 是 其基 础 解
系 .因此 , 只要证明 :β1 ,β2 , … ,βs 是 Ax = 0 的解 , 而且 t1 , t2 满足 某
种关系时 , 它们是线性无关的 .
显然 βi = t1 αi + t2 αi + 1   ( i = 1 , … , s - 1 ) ,   βs = t1 αs + t2 α1 都
是 Ax= 0 的解 , 因为
Aβi = A( t1 αi + t2 αi + 1 ) = t1 Aαi + t2 Aαi + 1 = 0 .
再讨论 t1 , t2 满足什么关系时 ,β1 ,β2 , … ,βs 线性无关 .设
k1β1 + k2 β2 + … + ks - 1 βs - 1 + ksβs = 0 , ( 1)

k1 ( t1 α1 + t2 α2 ) + k2 ( t1 α2 + t2 α3 ) + … + ks - 1 ( t1 αs - 1 + t2 αs ) +
ks ( t1 αs + t2 α1 ) = 0,
( t1 k1 + t2 ks )α1 + ( t2 k1 + t1 k2 )α2 + … + ( t2 ks - 2 + t1 ks - 1 )αs - 1 +
( t2 ks - 1 + t1 ks )αs = 0 . ( 2)
由于 α1 ,α2 , … ,αs 线性无关 ( 因为是基础解系 ) , 所以 (2 ) 式的系 数
必须全为零 , 即
t1 k1       + t2 ks = 0,
t2 k1 + t1 k2       = 0,
………………………… ( 3)
     t2 ks - 2 + t1 ks - 1 = 0 ,
     t2 ks - 1 + t1 ks = 0 .
欲使 β1 ,β2 , … ,βs 线性 无关 , 则 ( 1 ) 式 中的 k1 , k2 , … , ks 必须 全 为
零 , 也就是以 k1 , k2 , … , ks 为元 的 齐次 线 性方 程 组 ( 3 ) 只 有 零解 ,
因此 , 它的系数行列式
3 .5 齐次线性方程组有非零解的条件及解的结构 141

t1 0 … 0 t2
t2 t1 … 0 0
按第 1 行
w w … … t1n + ( - 1) n + 1 t2n ≠ 0 ,
展开
t2 t1 0
t2 t1
即 t1 , t2 满足 t1n + ( - 1 ) n + 1 t2n ≠ 0 时 ,β1 ,β2 , … ,βs 也 是 Ax = 0 的 一
个基础解系 .
例 7 ( 研 3 -36)   设 A 是 m× n 矩阵 , B 是 n× m 矩阵 , 则线 性
方程组 ( AB) x= 0 .
( A ) 当 n > m 时仅有零解 ;   ( B) 当 n > m 时必有非零解 ;
( C) 当 m > n 时仅有零解 ; ( D ) 当 m > n 时必有非零解 .
解   AB 是 m× m 矩阵 , ( AB) x = 0 是 m 元 齐次 线 性方 程组 ;
当 r ( AB) = m 时 , 它 仅 有 零 解 ; 当 r ( AB) < m 时 它 必 有 非 零 解 .
根据
r( AB) ≤ min( r( A) , r ( B) ) ,
可得 :
当 m > n 时 , r ( A) ≤ n, r ( B) ≤ n, 从 而 r ( AB) ≤ n < m, 所 以
( AB) x= 0 必有非零解 .
当 m < n 时 , r( A) ≤ m, r( B) ≤ m, 从而 r ( AB) ≤ m, 此时 , 可 能
有 r( AB) = m, 也可能有 r( AB) < m, 因此不能断言 ( AB) x= 0 仅有
零解或必有非零解 .
综上 , 答案应选 ( D) .
2
例 8   已知 n 阶矩阵 A 满足 A = A + 2 I, 证明
r( A + I) + r ( A - 2 I) = n .
证   由 A2 = A + 2 I 得 A2 - A - 2 I = ( A+ I) ( A - 2 I ) = 0, 所以
r( A + I) + r ( A - 2 I) ≤ n . ( 1)
由于 A - 2 I 与 - ( A - 2 I) = 2 I - A 的秩相等 , 再利用
r ( A + B) ≤ r( A) + r( B) ,
142 第 3 章  线性方程组

又得
      r( A + I) + r ( A - 2 I) = r ( A + I) + r( 2 I - A)
≥r ( A + I + 2 I - A) = r (3 I) = n . ( 2)
由 (1 ) 式和 ( 2) 式即得 r ( A + I) + r( A - 2 I) = n .

3 .6  齐次线性方程组有解的条件
及解的结构

    以 m× n 矩阵 A 为系数矩阵的非齐次线性方程组
Ax = b, ( 3 .1)
如果将 A 按列分块为 A= (α1 ,α2 , … ,αn ) , 则方程组(3 .1) 式即为
x1
x2
(α1 ,α2 , … ,αn ) = b,

xn

x1 α1 + x2 α2 + … + xnαn = b . ( 3 .2)
所谓方 程 组 有 解 , 也 就 是 存 在 x = ( x1 , x2 , … , xn ) T 使 方 程 组
(3 .1 ) , 即向量方程 ( 3 .2 ) 成 立 .所以方程 组 ( 3 .1) 有解 的充分必 要
条件为 , b 可由 A 的列向量组 α1 ,α2 , … ,αn 线性表示 .这个条 件在
方程组 (3 .1 ) 的增广矩阵
( A, b) = (α1 ,α2 , … ,αn , b)
上 , 就是系数矩阵 A 的列向量组的极大 线性无 关组 也是增 广矩 阵
( A, b) 的列向量组的极大线性无关组 , 因此它们的列秩相等 , 即
秩 ( A) = 秩 ( A, b) ( 3 .3)
也是方程组 (3 .1 ) 有解的充要条件 .
用高斯消元法求解方 程组 ( 3 .1 ) 是对 增 广矩 阵 ( A, b) 做 初 等
行变换将其化为阶 梯形 矩阵 ( U, d) , 得 到易 于求 解的 同 解方 程 组
3 .6 非齐次线性方程组有解的条件及解的结构 143

Ux = d, 条件 (3 .3 ) 表现在增广矩阵 ( U , d) 上 , 就是

秩 ( U) = 秩 ( U, d) ,

即 U 与 ( U, d) 有相同的非零行行数 .
如果 Ax= b 有惟一解 , 也就是 b 可由 A 的列向量 组线性表 示
( 如向量方程 ( 3 .2) ) , 且表示法惟一 , 那么 它的必 要条 件是 A 的 列
向量组线性无关 .因此 , Ax = b 有惟一解的充要条件是

秩 ( A, b) = 秩 ( A) = A 的列数 = n . ( 3 .4)

如果

秩 ( A, b) = 秩 ( A) < A 的列数 = n, ( 3 .5)

则 (3 .5 ) 式中的等号意味着 Ax= b 有解 , 而“ < ”号意味着 b 可由 A


的列向量组线 性表 示 的 表 示 法 不 惟 一 , 也 就 是 Ax = b 有 无 穷 多
组解 .
当 Ax= b 有无穷多组解时 , 其一般解的结构为

x = x0 + x, ( 3 .6)

其中 : x0 是 Ax = b 的一个特解 ( 某一个解 ) ;
x = k1 x1 + … + kp x p 是对应齐次方程组 Ax = 0 的一般解 .
证明 (3 .6 ) 式要用到非齐次线 性方程组 Ax = b 的解的 一个 重
要性质 : 若 x1 与 x2 为 Ax = b 的 两 个解 , 则 x1 - x2 为 对应 齐次 线
性方程组 Ax= 0 的解 .
必须注意 , 此时 x1 + x2 不是 Ax = b 的 解 , 而当 k1 + k2 = 1 时
则 k1 x1 + k2 x2 是 Ax = b 的解 .
以上是非齐次线性方程组解的基本理论 , 读者必须认真领会 .
至于当 Ax= b 有无 穷 多组 解 时 , 如 何 求解 的 问题 , 在内 容 提
要中已提及 , 这里不再重复 .
例 1   设非齐次线性方程组
144 第 3 章  线性方程组

 x1 - x2       + 2 x3     = 1 ,
2 x1 - x2       + 3 x3 - x4 = 4 ,
    x2       + p x3 + t x4 = t,
x1 - 3 x2 + ( 3 - p) x3     = - 4 .
试问 : p, t 取何值时 , 方程组有解 , 无解 , 有解时求其解 .
解   对方程组的增广矩阵做初等行变换, 将其化为阶梯形矩阵, 即
1 - 1 2  0  1
2 - 1 3 -1  4
( A, b) =
0  1 p  t   t
1 - 3 3- p  0 -4
1 -1 2  0  1
0  1 -1 - 1  2
0  1 p  t   t
0 -2 1- p  0 -5
1 -1 2  0 1
0  1 -1 -1 2
0  0 1+p t+ 1 t-2
0  0 -1- p -2 - 1
1 -1 2  0 1
0  1 -1 -1 2
. ( 1)
0  0 p+1 t+ 1 t-2
0  0 0 t- 1 t-3
由上面的阶梯形矩阵可见 :
(1 ) 当 p≠ - 1 且 t≠1 时 , 方程组有惟一解 ,
t-3 1 5- t
x4 = ,   x3 = ,
t-1 p+ 1 t - 1
1 5 - t 3t - 5 1 5 - t 2t - 3
x2 = + ,   x1 = - +2 .
p+1 t - 1 t-1 p+1 t - 1 t- 1
3 .6 非齐次线性方程组有解的条件及解的结构 145

(2 ) 当 p + 1 = 0 , 即 p = - 1 且阶梯形矩阵中第 3 , 4 行成比例
t+ 1 t - 2
= - 2t - 3 = - 3t+ 2 t= 5
t- 1 t- 3
时 ( 此时第 3 行可将第 4 行化为全零行 ) , 有
1 -1 2  0 1
0 1 -1 -1 2
( A, b)      
0 0 0  6 3
0 0 0  4 2
1 -1  2  0 1
0  1 -1 -1 2
0  0  0  1 1/ 2
0  0  0  0 0
1 0  1 0 7/ 2
0 1 -1 0 5/ 2
. ( 2)
0 0  0 1 1/ 2
0 0  0 0 0
阶梯形矩阵 (2 ) 所对应的原方程组的同解方程组为
x1   + x3   = 7/ 2 ,
  x2 - x3   = 5/ 2 , ( 3)
        x4 = 1/ 2 .
取 x3 = 0 , 得非齐次方程组的一个特解
x0 = (7/ 2 , 5/ 2 , 0 , 1/ 2 ) T ;
取 x3 为自由未知量 , 方程组 (3 ) 对应的齐次方程组的基础解系为
x1 = ( - 1 , 1 , 1 , 0 ) T .
综上 , 当 p = - 1 , t = 5 时 , 方程组的一般解为
    x = x0 + kx1
T T
= (7/ 2 , 5/ 2 , 0 , 1/ 2 ) + k( - 1 , 1 , 1 , 0) ,
其中 k 为任意常数 .
146 第 3 章  线性方程组

(3 ) 当 p = - 1 , t≠5 时 , 阶梯形矩阵 ( 1 ) 中第 3 行 与第 4 行 不
成比例 , 此时
秩 ( A) = 3≠秩 ( A, b) = 4 ,
所以方程组无解 .
例 2   已知 β1 ,β2 ,β3 是非齐次线性方程组 Ax = b 的 3 个不同
的特解 ,α1 ,α2 , α3 是 对应齐 次方 程组 Ax = 0 的 基础解 系 .试写 出
Ax = b 的 3 种不同形式的一般解 .
解   根据非齐次线性方 程组解 的结 构 , 它的一 般 解的 最简 单
的形式为
x = k1 α1 + k2 α2 + k3 α3 + β1 . ( 1)
由于 α1 + α2 ,α2 + α3 ,α3 + α1 仍是 Ax = 0 的 解 , 且 线性 无关 ;
1
(β1 + β2 ) 仍是 Ax = b 的解 , 所以一般解也可表示为
2
1
  x = k1 (α1 + α2 ) + k2 (α2 + α3 ) + k3 (α3 + α1 ) + (β1 + β2 ) . ( 2)
2
1
再由 α1 + α2 ,α1 - α2 ,α3 也线性无关 , ( 2β1 + 5β2 - 4β3 ) 也 是
3
Ax = b 的解 ( 因为 β1 ,β2 ,β3 的系数和为 1) , 一般解也可表示为
1
x = k1 (α1 + α2 ) + k2 (α1 - α2 ) + k3 α3 + (2β1 + 5β2 - 4β3 ) .
3
( 3)
以上 (1 ) , (2 ) , (3 ) 式中的 k1 , k2 , k3 均为任意常数 .
A α
例 3 ( 研 3 -39)   设 B= , A 是 n 阶 矩阵 ,α是 n 维 列
αT 0
向量 .若秩 ( B) = 秩 ( A) , 则线性方程组
( A ) Ax = α必有无穷多组解 ;
( B) Ax = α必有惟一解 ;
( C) By = 0 仅有零解 ;
( D ) By = 0 必有非零解 ,
3 .6 非齐次线性方程组有解的条件及解的结构 147

T T
其中 x = ( x1 , … , xn ) , y = ( y1 , … , yn , yn + 1 ) .
解   由于 B 是 n + 1 阶矩 阵 , 而秩 ( B) = 秩 ( A) ≤ n, 所以齐 次
线性方程组 By = 0 必 有 非 零 解 .因 此 ( D ) 是 正 确 的 , ( C ) 是 不 正
确的 .
这里 , 读者还应理解 ( A ) , (B) 也是不正确的 , 理由何在 .
由秩 ( B) = 秩 ( A) 可 知 ,α可以 用 A 的列 向量 组线 性 表示 ( 当
T
A 是实对称矩阵时 ,α 也可 由 A 的 行 向量 组线 性表 示 ) , 因 此 , 非
齐次线性方程组
Ax = α
必有解 .但是 A 可能是可逆的 ( 此时 Ax = α只有惟一解 ) , 例如
1 0 0 1
0 1 0 0
B= .
0 0 - 1 1
1 0 1 0
A 也可能是不可逆的 ( 此时 Ax = α有无穷多组解 ) , 例如
1  1  0  1
1  0 - 1  0
B= .
0 -1 - 1 -1
1 0 - 1  0
所以 , 在没有指明 A 是否 可 逆 ( 即秩 ( A) 是 否等 于 n) 的 情 况
下 , 都不能断言题中 ( A ) , (B) 的结论是正确的 .
例 4   设 n 阶矩阵 A = (α1 ,α2 , … ,αn ) 的 行列式 | A| = 0 , 且 A
的任何 n - 1 个列向 量都线 性无关 , 将 A 的 前 n - 1 列构 成的 n×
( n - 1 ) 矩阵记为 A1 = (α1 , α2 , … , αn - 1 ) , 问 方程 组 A1 x = αn 有 解
否 ? 为什么 ?
解   由 | A| = 0 及 A 的任何 n - 1 个列向量线性无关可知
秩 ( A) = 秩 {α1 ,α2 , … ,αn } = n - 1 ,
秩 ( A1 ) = 秩{α1 ,α2 , … ,αn - 1 } = n - 1
148 第 3 章  线性方程组

而矩阵 A 就是方程 组 A1 x = αn 的 增广 矩阵 ( A1 , αn ) , 所 以增 广 矩
阵的秩等于系数矩阵的秩 , 因此 , 方程组 A1 x= αn 有解 .
另一解法 : 由 | A | = 0 可 知其 列向 量 组 α1 , α2 , … , αn 线 性 相
关 , 而 α1 ,α2 , … , αn - 1 线性 无关 , 所 以 αn 可由 α1 ,α2 , … , αn - 1 线 性
表示 ( 否则 α1 ,α2 , … ,αn - 1 ,αn 线性无 关 ) , 且 表示 法惟一 .因 此 , 方
程组 A1 x= αn 有惟一解 .
例 5 ( 研 3 -40 )   已知四阶方阵 A = (α1 ,α2 ,α3 ,α4 ) , 其中四维
列向量 α2 ,α3 ,α4 线性 无 关 ,α1 = 2α2 - α3 .如果 β= α1 + α2 + α3 +
α4 , 求线性方程组 Ax = β的通解 .
T
解   设 x = ( x1 , x2 , x3 , x4 ) , 则 Ax = β即为
x1 α1 + x2 α2 + x3 α3 + x4 α4 = α1 + α2 + α3 + α4 . ( 1)
将 α1 = 2α2 - α3 代入 ( 1) 式 , 整理得
(2 x1 + x2 )α2 + ( - x1 + x3 )α3 + x4 α4 = 3α2 + α4 . ( 2)
向量方程 (2 ) 就是线性方程组 Ax = β.先求它的一个特解 x0 , 由 ( 2)
式,取
x4 = 1 ,
- x1 + x3 = 0 x1 = x3 , 可任取为 x1 = x3 = 1 ,
2 x1 + x2 = 3 x2 = 3 - 2 x1 = 3 - 2 = 1 .
所以 x0 = ( 1 , 1 , 1 , 1) T .
再求非齐次线性方程组 Ax= β对应的齐 次线性方 程组Ax = 0
的通解 x .
Ax = 0 就是 ( 2) 式右端等于零向量 , 即
(2 x1 + x2 )α2 + ( - x1 + x3 )α3 + x4 α4 = 0 . ( 3)
由于 α2 ,α3 ,α4 线性无关 , 所以 ( 3) 式左 端的 3 个系 数必 须全 为 0 ,
因此得到齐次线性方程组
2 x1 + x2 = 0,
- x1 + x3 = 0, ( 4)
x4 = 0 .
3 .6 非齐次线性方程组有解的条件及解的结构 149

T
易得 (4 ) 的通解 为 k( 1 , - 2 , 1 , 0 ) , 这也就 是 ( 3 ) 式即 Ax = 0 的 通
解x.
综上 , 即得 Ax = β的通解为
T T
x = x0 + x = (1 , 1 , 1 , 1 ) + k( 1 , - 2 , 1 , 0) ,
其中 k 为任意常数 .
例 6 ( 补充题 51)   已知下列线性方程组Ⅰ , Ⅱ为 同解线性 方
程组 , 求参数 m, n, t 之值 .
x1 + x2 - 2 x4 = - 6 ,
Ⅰ :   4 x1 - x2 - x3 - x4 = 1 ,
3 x1 - x2 - x3 = 3;
x1 + m x2 - x3 - x4 = - 5 ,
Ⅱ:   nx2 - x3 - 2 x4 = - 11 ,
x3 - 2 x4 = 1 - t .
解   对方程组Ⅰ的增 广 矩 阵 ( A, b) 做 初等 行 变换 , 将其 化 为
行简化阶梯矩阵 , 即
1  1  0 -2 - 6
( A, b) = 4 -1 - 1 -1  1
3 -1 - 1  0  3
1 0 0 -1 -2
0 1 0 -1 -4 ( 1)
0 0 1 -2 -5
T
由此易得 : 方程 组Ⅰ 的一 个特 解 x0 = ( - 2 , - 4 , - 5 , 0 ) , 其 对 应
齐次方程组的通解为 k( 1 , 1 , 2 , 1) T .方程组Ⅰ的通解为
x = ( - 2 , - 4 , - 5 , 0 ) T + k( 1 , 1 , 2 , 1) T   ( k 为任意常数 ) .
由于方程组Ⅱ与Ⅰ是同解方程组 , 所以 x0 也应是方程组Ⅱ 的
解 , 将其代入方程组Ⅱ , 得
- 2 - 4m+ 5 = - 5 m=2,
- 4 n + 5 = - 11 n= 4,
150 第 3 章  线性方程组

    - 5=1 - t t= 6 .
这里必须 注 意 , 如果 题目 改 为 : 是否 存在 m, n, t 使 方程 组 Ⅱ
与方程组Ⅰ为同解方程组 ? 此时解题方法应为 :
将方程组Ⅱ的增广矩阵 ( C, d) 也化为行简化阶梯矩阵
1 m -1 -1 -5
( C, d) = 0 n -1 -2 - 11
0 0 1 -2 1- t
1 m 0 - 3 -4- t
0 n 0 - 4 - 10 - t . ( 2)
0 0 1 - 2 1 - t
如果存在 m, n, t, 使 (2 ) 式中的 行简 化阶 梯阵 与 ( 1 ) 式 中的 行简 化
阶梯阵完全一样 , 那么方程组Ⅰ与Ⅱ肯定是同解方程组 .更一般的
情况 , 要判断两个方程组的解集合是否相同 .
a11 a12 … a1 n y1

a21 a22 … a2 n y2
例 7 ( 补充题 64)   设 A= ,   y= ,
… … … …
am1 am2 … amn yn
b= ( b1 , b2 , … , bm ) T ,   x = ( x1 , x2 , … , xm ) T .
T
(1 ) 证明 : 若 Ay = b 有解 , 则 A x = 0 的任一组 解 x1 , x2 , … ,
xm 必满足方程
b1 x1 + b2 x2 + … + bm x m = 0 . ( 1)
(2 ) 方程组 Ay = b 有解的充要条件是方程组
AT 0
x= ( 2)
bT 1
无解 ( 其中 0 是 n×1 零矩阵 ) .
T
证   (1 ) 由于要证明的 ( 1) 式左端等于 b x, 即要证明
bT x= b1 x1 + b2 x2 + … + bm x m = 0 ,
所以 , 把有解的方程组 Ay = b 表示为转置形式
3 .7 部分疑难习题和补充题的题解 151

T T T
b = y A .
T
再由 A x = 0, 即得
T T T T T T
b x = y A x= y ( A x) = y 0 = 0
( 上式中的 x = ( x1 , x2 , … , x m ) T 为 AT x= 0 的任一个 解 ) , 故 (1 ) 式
成立 .
(2 ) 先证必要性 : 已知 Ay = b 有解 , 即
T
AT ( 3)
秩 ( A) = 秩 ( A, b) ,   秩 ( A ) = 秩 T
.
b
(3 ) 式表明 , bT 可由 AT 的 n 个行 向量 线性 表示 , 于 是 , 方程 组 ( 2)
的增广矩阵
AT 0 初等 AT 0
. ( 4)
T 行变 换
b 1 0 1
T
由此可见 , 方程组 ( 2) 的 增 广矩 阵的 秩为 秩 ( A ) + 1 , 而 其系 数 矩
阵的秩为秩 ( A) , 二者不等 , 所以方程组 (2 ) 无解 .
再证充分性 : 已知方程 组 ( 2 ) 无解 , 即系 数矩 阵的 秩不 等于 其
T T
增广矩阵的秩 .因此 , 系数矩阵中的 b 可由 A 的 n 个行向量线性
T T
表示 ( 因为 : 如果 b 不能由 A 的 n 个行向 量线 性表 示 , 则 系数 矩
AT
阵 T
的秩为秩 ( A) + 1 , 它就 与增广 矩阵 的秩相 同 , 从而 方程 组
b
(2 ) 有解 , 与题设矛盾 ) , 也就是 b 可由 A 的 n 个 列向 量线性 表示 ,
所以方程组 Ay = b 有解 .

3 .7 部分疑难习题和补充题的题解

1 ( 习题 26) *   证明 :若 n 阶方阵 A 的 秩为 r , 则必 有秩为 n - r 的 n 阶


方阵 B, 使 BA = 0 .
证   利用矩阵的相抵标准形 .由于 r( A) = r, 所以 , 存在 n 阶可逆矩阵 P、
Q, 使得
152 第 3 章  线性方程组

Ir 0 Ir 0
PAQ= ,   即 A = P- 1 Q- 1 .
0 0 0 0
于是 , 存在 n 阶矩阵 B(或说取 B)
0 0
B= P
0 In - r

就使
0 0 Ir 0
BA = PP - 1 Q- 1
0 In - r 0 0
0 0 Ir 0
= Q- 1 = 0Q- 1 = 0 .
0 In - r 0 0
2 ( 习题 34) *   设 A* 是 n 阶矩阵 A 的伴随矩阵 , 证明 :
n ,   当 r( A) = n ,
( 1) r( A* ) = 1 ,   当 r ( A) = n - 1 ,
0 ,   当 r ( A) < n - 1 .

( 2) | A | = | A| n - 1 .
证   伴随矩阵 A* 的秩只有 3 种可能——— n, 1 , 0 , 这是一个重要的结论 .
证明时要利用 A* 的定义及由此得到的重要结果
AA* = A* A = | A| I . ( 1)
( 1) 当 r( A) = n, 即 | A| ≠0 时 , 由 (1) 式得
| A A * | = | A| | A* | = | A| n ≠0 ,
所以 , | A* | ≠0 , 即 r( A* ) = n .
当 r( A) = n - 1 时 , | A | = 0 , 但 A 中 存在 n - 1 阶非 零 子 式 , 而 A* =
( A ij ) n× n 的元素 A i j 是 A 中元素 a ji 的代数余子式 , 所以 , 此时 A* ≠0( 即 A* 为
非零矩阵 ) , 因此 r ( A* ) ≥1 ;再由 (1 )式 , 又得
A A* = 0
(利用教材 p137 中例 3 的结论 ) , 于是
r( A) + r( A* ) ≤ n,
从而
r( A* ) ≤ n - r( A) = n - ( n - 1 ) = 1 .
综上 , 即得 r( A* ) = 1 .
当 r( A) < n - 1 时 , A 中非零子式的 最高阶数 ≤ n - 2 , 所 以 A 中元素 a ji
3 .7 部分疑难习题和补充题的题解 153

的代数余子式
Ai j = ( - 1 ) i + j M ji = 0
(因为 Mji 是 a ji 的余子式 , 它是 A 的 n - 1 阶子式 , 所以 Mji = 0) , 因此 , A* =
( A ij ) n× n = 0 , 故 r( A* ) = 0 .
( 2) 证明 | A* | = | A| n - 1 .
注意 , A 可能是可逆的 , 也 可能是 不可逆 的 , 应 证明两 种情 况下 , 结论 都
成立 .
当 A 可逆时 , | A| ≠0 , 由 (1 )式得
| A| | A* | = | A| n , 所以 , | A* | = | A| n - 1 ;
当 A 不可逆时 , r( A)≤ n - 1 , | A | = 0 , 由 ( 1 ) 中结论 又知 r( A* ) = 1 或 0 ,
所以 | A* | = 0 .因此 , 结论也成立 .
3 ( 习题 35)   设 A 是 n 阶可逆矩阵 ( n≥2 ) , 证明 : ( A* ) * = | A| n - 2 A .
证   当 n= 2 时, 设
a1 a2
A= ,
a3 a4

a4 - a2 a1 a2
A* = ,   ( A* ) * = = A( 结论成立) .
- a3 a1 a3 a4
当 n > 2 时 , 显然不能如上那样证明 , 一般的证法如下 :利用
1
A- 1 = A* , 即 A* = | A| A - 1 . ( 1)
| A|
由题可知 , r( A) = n( 即 A 可逆 )时 , r ( A* ) = n( 即 A* 也可逆 ) , 所以 (1 ) 式
中的 A 换为 A * 时 , 等式也成立 , 即
( A* ) * = | A* | ( A* ) - 1 . ( 2)
由( 1) 式可得
1 1
( A* ) - 1
= ( | A| A- 1 ) -1
( A- 1 ) - 1
= = A. ( 3)
| A| | A|
再将上题的结果 | A* | = | A| n - 1 , 连同 (3 )式 , 一起代入 ( 2) 式 , 即得
1
( A* ) * = | A| n - 1 A = | A| n - 2 A .
| A|
4 ( 习题 36)   设 A 是 n 阶矩阵 , 证明 :非齐次线性 方程组 Ax = b 对 任何
154 第 3 章  线性方程组

b 都有解的充分必要条件是 | A| ≠0 .
证   充分性是显然的 , 因为 | A| ≠0( 即 A 可逆 ) , 所以方程组 Ax = b 对任
何 b 都有解
x= A - 1 b .
必要性 : 此时 , 只要用 对 n 个 线性 无 关的 b1 , b2 , … , bn 有 解 , 就 能证 明
| A| ≠0 .最简单的情形就取 n 个单位向量 , 即
bj = ej = ( 0 , … , 0 , 1 , 0 , … , 0 ) T ,   j = 1 , 2 , … , n .
设 Ax= ej 的解为 xj = ( x1 j , x2 j , … , xn j ) T ( j = 1 , 2 , … , n) , 于是就有
A( x1 , x2 ,… , xn ) = ( e1 , e2 , … , en ) = I
(注意其中 ( x1 , x2 ,… , xn ) 是 n 阶矩阵 ) , 从而
| A| | x1 , x2 ,… , xn | = 1 ,
因此 , | A| ≠0 , 必要性得证 .
5 ( 补充题 39)   已知
1 2 3
Q= 2 4 t ,
3 6 9
P 为非零三阶矩阵 , PQ= 0 , 则 :
( A) 当 t = 6 时 , r( P) = 1 ;     (B) 当 t = 6 时 , r ( P) = 2 ;
( C) 当 t≠6 时 , r ( P) = 1; ( D) 当 t≠6 时 , r ( P) = 2 .
解   利用 : 当 PQ= 0 时 , r( P) + r( Q) ≤3 .
当 t≠6 时 , r( Q) = 2 (因为 | Q| = 0 , 且 Q 中有两个 列向量 不成比 例 , 从 而
线性无关 ) , 所以
r ( P)≤3 - r( Q) = 3 - 2 = 1 .
而 P 为非零矩阵 , r( P)≥1 .因此 , r( P) = 1 .答案 应选 ( C ) .其中 ( D ) 显然是 错
误的 , 因为此时 r ( P) + r( Q) = 4 > 3 .
( A) , ( B ) 也 不 是 肯 定 正 确 的 答 案 .因 为 t = 6 时 , r ( Q) = 1 . 此 时 ,
r ( P)≤3 - r( Q) = 3 - 1 = 2 , 所以 r ( P)可以等于 1 , 也可以等于 2 .例如 :
1 1 - 1 1 1 - 1
P1 = 2 2 - 2 ,   P2 = 4 1 - 2
3 3 - 3 2 2 - 2
都能使 P1 Q= 0, P2 Q= 0, 而 r ( P1 ) = 1 , r( P2 ) = 2 .
3 .7 部分疑难习题和补充题的题解 155

6 ( 补充题 40 )   设 α1 = ( a1 , a2 , a3 ) T , α2 = ( b1 , b2 , b3 ) T , α3 = ( c1 , c2 ,
c3 ) T , 则 3 条直线 ai x + bi y + ci = 0( a2i + b2i ≠0 ) ( i = 1 , 2 , 3 )交于一点的充要条
件是(   ) .
( A) α1 ,α2 ,α3 线性相关 ;
( B) α1 ,α2 ,α3 线性无关 ;
( C) r {α1 ,α2 ,α3 } = r {α1 ,α2 } ;
( D ) α1 ,α2 ,α3 线性相关 , 且 α1 ,α2 线性无关 .
解   3 条直线交于一点 P0 ( x0 , y0 ) , 也就是线性方程组
a1 x + b1 y + c1 = 0 ,
a2 x + b2 y + c2 = 0 , ( 1)
a3 x + b3 y + c3 = 0 .
有惟一解 ( x0 , y0 ) .要把方程组 (1 )有惟一解的充要条件归结为 x, y 的系数及
常数项列向量 ( 即 α1 ,α2 ,α3 )的线性相关 性 , 其关 键是要把方程 组 (1 ) 等价 地
表示为 α1 ,α2 ,α3 的向量方程 , 即      
x α1 + y α2 + α3 = 0 . ( 2)
方程组 ( 1)有惟一解 ( x0 , y0 ) , 也就是存在不全为零的 x0 , y0 , 1 使
x0 α1 + y0 α2 + 1・α3 = 0 . ( 3)
所以 α1 ,α2 ,α3 必是线性相关 的 , 而 且 α1 ,α2 是线 性无 关的 , 否 则 , 存 在不 全
为零的 x1 , y1 使
x1 α1 + y1 α2 = 0 . ( 4)
将( 3) , (4 )式相加 , 就有
( x0 + x1 )α1 + ( y0 + y1 )α2 + α3 = 0 .
如此 , ( x0 + x1 , y0 + y1 ) ( ≠ ( x0 , y0 ) ) 也是 方程组 (1 ) 的 解 , 这 与方程 组 ( 1 ) 有
惟一解矛盾 .
所以 , 方程组( 1) 有惟一解 的必要 条件 是 ,α1 ,α2 ,α3 线性 相关 , 且 α1 ,α2
线性无关 .这个条件也是方程组 (1 )有惟一解的充分条件 .
因为 α1 ,α2 ,α3 线性相关 , 即存在不全为零的 k1 , k2 , k3 , 使
k1 α1 + k2 α2 + k3 α3 = 0 . ( 5)
再由 α1 ,α2 线性无关 , 又可知 (5 )式中的 k3 ≠0( 如果 k3 = 0 , 就必有 k1 = k2 =
0 , 这与 k1 , k2 , k3 不全为 0 矛盾 ) , 因此
156 第 3 章  线性方程组

k1 k
α1 + 2 α2 + α3 = 0 . ( 6)
k3 k3
k1 k2
(6) 式表明 , ( x0 , y0 ) = , 是方程组 (1 ) 的解 , 而 且是惟一 解 ( 因为此 时
k3 k3
α3 由 α1 ,α2 的线性表示的表示法是惟一的 , 见教材 p116 中定理 3 .3) .
所以答案应选 ( D ) .注 意 ( C ) 也 是不 正确 的 , 因 为 r {α1 , α2 ,α3 } = r {α1 ,
α2 } 并不意味着 α1 ,α2 线性无关 .
另一解法 : 将方程组( 1) 写成
a1 x + b1 y + c1 z = 0 ,
a2 x + b2 y + c2 z = 0 , (1 )′
a3 x + b3 y + c3 z = 0 .
其中 z = 1 .方程组( 1)′有非零解( x0 , y0 , 1) 的充要条件是系数行列式等于零 ,
即其中 3 个列向量 α1 ,α2 ,α3 线性相关 .此时 (1 )′的系数矩阵的 3 个行向量也
线性相关的 , 不妨设( a3 , b3 , c3 ) 可由 ( a1 , b1 , c1 ) 与 ( a2 , b2 , c2 ) 线 性表示 .于 是
的同解方程组为    
(1)′
a1 x + b1 y = - c1 z = - c1 ,
(2 )′
a2 x + b2 y = - c2 z = - c2 .

非齐次线性方程组 (2 ) 有惟一解 ( x0 , y0 ) ( 即 3 直线有 惟一交点 ) 的充要 条件
为系数行列式
a1 b1
≠0 ,
a2 b2
即其列向量 β1 = ( a1 , a2 ) T ,β2 = ( b1 , b2 ) T 线性无关 , 它们各添分量 a3 , b3 得到
的向量 α1 = ( a1 , a2 , a3 ) T ,α2 = ( b1 , b2 , b3 ) T 也线性无关 .
7 ( 补充题 43)   设 向 量 α,β, γ线 性 无 关 , α,β,δ线 性 相 关 , 下 列 哪 个
成立 ?
( A) α必可由 β,γ,δ线性表示 ;
( B) β必不可由 α,γ,δ线性表示 ;
( C) δ必可由 α,β,γ线性表示 ;
( D ) δ必不可由α,β,γ线性表示 .
解   由 α,β,γ线性无关可知α,β也线性 无关 , 再由 α,β,δ线性 相关 , 又
可知 δ可由 α,β线性表示 , 即
3 .7 部分疑难习题和补充题的题解 157

δ= k1 α+ k2β,
从而 δ= k1 α+ k2β+ 0・γ,
所以 δ也可由 α,β,γ线性表示 .即 (C) 是成立的 , 于是( D )自然就不成立 .
( A) 也不成立 , 因为 α不能由β,γ线性表示 , 如果 δ= - β( 此时 α,β,δ线
性相关) ,α就不能由β,γ,δ线性表示 .而此时 β= 0・α+ 0・γ+ ( - 1)δ,β可
由α,γ,δ线性表示 , 所以 (B)也不成立 .
8 ( 补充题 46)   设 A 为 n 阶矩阵 , 若存在正整数 k ( k≥2 ) 使得 Akα= 0,
但 Ak - 1 α≠0(其中 α为 n 维非零列向量 ) .证明 :α, Aα, … , Ak - 1 α线性无关 .
证   证明的思路 :设
λ0 α+λ1 Aα+ λ2 A2 α+ … +λk - 1 Ak - 1 α= 0 , ( 1)
然后根据已知条件 , 由 ( 1) 式推出 λ0 ,λ1 ,λ2 , … ,λk - 1 必须全为零 .
利用已知条件: Ak - 1 α≠0, Akα= 0, 将 (1)式两端左乘 Ak - 1 , 得
λ0 Ak - 1 α+ λ1 Akα+ λ2 Ak+ 1 α+ … + λk - 1 A2k - 2 α= 0 , ( 2)
其中 Akα= Ak + 1 α= … = A2 k - 2 α= 0 , 因此 , 由 (2 )式得
λ0 Ak - 1 α= 0 , 从而 λ0 = 0   ( 因为 Ak - 1 α≠0) .
所以( 1) 式成立时 ,λ0 必须等 于 0 .再 将 ( 1 ) 式两 端左 乘 Ak - 2 ( 此时 已有 λ0 =
0) , 得
λ1 Ak - 1 α+ λ2 Akα+ … + λk - 1 A2k - 1 α= 0 , ( 3)
其中 Akα= … = A2 k - 1 α= 0 , 因此 , 由( 3) 式得
λ1 Ak - 1 α= 0 , 从而 λ1 = 0   ( 因为 Ak - 1 α≠0) .
如此继续下去 , 就 得: 欲 使 ( 1) 式 成立 , 必须 λ0 = λ1 = λ2 = … = λk - 1 = 0 .所 以
α, Aα, A2 α, … , Ak - 1 α线性无关 .
1 a … a
a 1 … a
9 ( 补充题 49)   设 A= (主对角元全为 1 , 其余全为 a .)
… … w …
a a … 1
为 n 阶矩阵 ( n≥3) , a∈R , 且 r( A) = n - 1 , 求 a .
解   对 矩 阵 A 做 初 等 变 换 时 , A 的 秩 不 变 .现 将 A 的 各 列 都 加 到 第
1列,得
158 第 3 章  线性方程组

1 + ( n - 1) a a … a
1 + ( n - 1) a 1 … a
A = A1 .
… … w …
1 + ( n - 1) a a … 1
易知 | A| = [ 1 + ( n - 1) a] (1 - a) n - 1 .当 r( A) = n - 1 , 即 | A| = 0 时 , 必须 1 +

1
( n - 1 ) a= 0 ,即 a= (注意 , a = 1 时 , | A| = 0 , 但 r ( A) = 1) , 此时 , 将 A1 的
1 - n
第 1 行乘 - 1 加到其余各行 , 得
0 a a … a
0 1- a 0 … 0
A1 0 0 1- a … 0 = A2
… … … w …
0 0 0 … 1 - a
1 n
其中 1 - a = 1 - = ≠0 , 所以秩 ( A) = 秩 ( A2 ) = n - 1 .
1 - n n -1
1
因此 , 当秩 ( A) = n - 1 时 , a = .
1 - n
10 ( 补充题 55 )   设 n 阶矩阵 A 分块为
A11 A12
A= ,
A21 A22
其中 A11 为 k 阶可逆矩阵( k < n) , 证明 : 存在主对角元为 1 的上三角矩阵 U 和
下三角矩阵 L, 使得
A11 0
LAU= . ( 1)
0 B
证   利用 A11 可逆 , 对 A 做初等行变 换把 A21 化 为 0 , 此 时左乘 的分块 初
等矩阵是主对角元为 1 的下三角矩阵 ; 做初等列变换把 A12 化为 0 , 此时 右乘
的分块初等矩阵是主对角元为 1 的上三角矩阵 , 如此就得所要的结果 .即
I 0 A11 A12 I - A11- 1 A12 ( 2)
- 1
- A21 A 11 I A21 A22 0 I
3 .7 部分疑难习题和补充题的题解 159

A11 A12 I - A11- 1 A12


=
0 A22 - A21 A11- 1 A12 0 I
A11 0
= ,
0 A22 - A21 A11- 1 A12
其中( 2) 式的左边矩阵为 L, 右边矩阵为 U, 记 B = A22 - A21 A11- 1 A12 , 上式 就是
(1) 式的结果 .
11 ( 补充题 56 )   设 A, B 均为 n 阶矩阵 , 证明 :
I B
( 1) = | I - AB| ;     ( 2) | I - AB| = | I - BA| .
A I
( 3) det (λI - AB) = det (λI - BA) (λ为任意常数 ) .
证   ( 1) 对式中的分块矩阵做 初等行变换将 A 化为 0 , 得 到一个上三角
块矩阵 , 就可得所要结果 .即
I 0 I B I B
= . ( 1)
- A I A I 0 I - AB
注意: (1 )式中均为 2 n 阶矩阵 , 不是 2 n 阶行列式 .根据矩阵 乘积的行列 式等
于其行列式的乘积 , 由 ( 1) 式得
I 0 I B I B
= . ( 2)
- A I A I 0 I - AB
利用教材 1 .2 节中例 9( p18~20 )的结论 , ( 2) 式 中下三角块 矩阵和上三 角块
矩阵的行列式分别等于
| I| | I | = 1   和   | I| | I - AB| = | I - AB| ,
于是由( 2) 式即得 ( 1) 所要证明的结果 .
( 2) 只要证明 (1) 中的行列式也等 于 | I - BA | .此时 ( 1 ) 式中的 分块矩 阵
做初等行变换将 B 化为 0 , 即
I - B I B I - BA 0
= . ( 3)
0 I A I A I
由( 3) 式即得
I B
= | I - BA| | I | = | I - BA| .
A I
再利用( 1) 的结果 , 即得 | I - BA| = | I - AB| .
( 3) 当 λ= 0 时 , 显然有 | - AB| = | - BA| , 因为
160 第 3 章  线性方程组

| - AB| = ( - 1) n | AB| = ( - 1) n | A| | B| = ( - 1) n | B| | A| = | - BA| .


当 λ≠0 时 , 只要证明
λI B λI B
= |λI - AB|   及   = |λI - BA|
A I A I
证明的思路与 (1 ) , (2) 类似 .由
I 0 λI B
λI B
1 = 1 ,
- A I A I 0 I- AB
λ λ
即得
λI B 1
    = |λI | I- AB = λI I - 1 AB
A I λ λ

= |λI - AB| . ( 4)
同理 , 再由
I - B λI B λI - BA 0
= ,
0 I A I A I
又得
λI B
= |λI - BA| | I | = |λI - BA| . ( 5)
A I
由( 4) , (5 )式即得 : 当 λ≠0 时 , |λI - AB| = |λI - BA| .

12 ( 补充题 57 )   证明 :若 A 是 m× n 矩阵 , r( A) = r, 则存在 m× r 矩阵
B, r× n 矩阵 C , 且 r( B) = r( C) = r, 使得 A = BC .
证   利用相抵标准形 .对于矩阵 A, 存在 m 阶可逆矩 阵 P 和 n 阶可 逆矩
阵 Q, 使得
Ir 0 Ir 0
PAQ= ,   即   A = P- 1 Q- 1 . ( 1)
0 0 0 0
将 P- 1 , Q- 1 分块表示为
C
P - 1 = ( B, S) ,   Q- 1 = , ( 2)
T
其中 : B 为 m× r 矩阵 , r( B) = r( 因为 B 的 r 个列 向量线 性无 关 ) ; C 为 r × n
矩阵 , r ( C) = r(因为 C 的 r 个行向量线性无关 ) .于是由 (1) , ( 2) 式得
3 .7 部分疑难习题和补充题的题解 161

Ir 0 C C
A = ( B, S) = ( B,0) = BC .
0 0 T T
13 ( 补充 题 60)   证 明 : 非 零向 量 组 α1 ,α2 , … , αs 线性 无 关 的充 要 条
件是
i-1

αi ≠ ∑ kα   ( i =
j= 1
j j 2 , 3 , … , s) .

证   必要性是显然的 .因为如果存 在 i ( i∈ { 2 , … , s} ) , 使 得 αi = k1 α1 +
… + ki - 1 αi - 1 , 则 α1 , … ,αi - 1 ,αi 线性相关 , 从而 α1 ,α2 ,… ,αs 线性相 关 .与 题
设矛盾 .

充分性 :α1 ≠0,α2 ≠ k1 α1 , 则 α1 , α2 线性无 关 因为 : 设 λ1 α1 + λ2 α2 = 0,

λ1
则 λ2 = 0 否则 α2 = α1 , 从而 λ1 = 0 ;再由
λ2
α3 ≠ k1 α1 + k2 α2 ,
又得 α1 ,α2 ,α3 线性无关 ;如此继续下去 , 由 α1 ,α2 ,… ,αs - 1 线性无关 , 而
αs ≠ k1 α1 + k2 α2 + … + ks - 1 αs - 1 ,
即得 α1 ,α2 , …,αs - 1 ,αs 线性无关 .
14 ( 补充题 61 )   设向量 组 α1 ,α2 , … ,αr 线性 无关 , 如在向 量组 的前 面
加入一个 向量 β, 证 明 : 在向量 组 β, α1 ,α2 , … ,αr 中 至多有 一个向 量 αi ( 1≤
i≤ r) 可由其前面 的 i 个 向量 β, α1 ,α2 , … ,αi - 1 线 性 表 示 .并 在 R3 中做 几 何
解释 .
证   如果 β,α1 ,α2 , … ,αr 仍然线性无关 , 则任何 αi (1≤ i≤ r)都不能由前
面的 i 个向量线性表示 ;如 果 β,α1 ,α2 ,… ,αr 线性 相关 , 但 β= 0 , 同样 , 任 何
αi 都不能由前面的 i 个向量线性表示 .    
如果 β≠0 , 且 β,α1 ,α2 ,… ,αr 线性相关 .则存在 αi 可 由前面 的 i 个向 量
线性表示 .因为从前往 后考 察 , 如果 β,α1 , … ,αi - 1 仍线 性 无关 , 而 β,α1 , … ,
αi - 1 ,αi 线性相关 , 则 αi 可由β,α1 , … , αi - 1 线 性表示 ( 这 里的极 端情况 是 :β,
α1 , … ,αr - 1 仍线性无关 , 而 β,α1 , … ,αr - 1 ,αr 线性相关) .再证 :至 多有一个 αi
(1≤ i≤ r) 可由前面的 i 个向量线性表示 .
设 αi 与 αj ( j > i)均可由前面的 i 个与 j 个向量线性表示 , 即
αi = k0 β+ k1 α1 + … + k i - 1 αi - 1 , ( 1)
162 第 3 章  线性方程组

αj = l0β+ l1 α1 + … + li - 1 αi - 1 + liαi + … + lj - 1 αj - 1 , ( 2)
其中 k0 ≠0 , l0 ≠0 (否则 α1 , … ,αi , … ,αj 线性相关 ) , 于是由 ( 1) , (2) 式分别得
β= k′
1α1 + … + k′
i - 1 αi - 1 + k′
iαi , ( 3)
β= l′
1α1 + … + l′
i - 1 αi - 1 + l′
iαi + … +

j - 1 αj - 1 + l′
l′ jαj . ( 4)
由( 3) , (4 )式得
1 - k′
( l′ 1 )α1 + … + ( l′
i - 1 - k′
i - 1 )αi - 1 +

( l′ i )αi + … + l′
i - k′ j - 1 αj - 1 + l′
jαj = 0 . ( 5)
由于 α1 , … ,αi , … ,αj 线性无关 , 所以( 5) 式中的系数全等于 0 , 于是 j 不 大于
i , 从而 (2 )式中的系数
l0 = k0 ,   l1 = k1 ,   …   , li - 1 = ki - 1   ,   li = … = lj - 1 = 0
因此 αj = αi , 这就证明了至多有一个 αi 可由前面的 i 个向量线性表示 .
R3 中几何解释 :设 α1 = ( 1 , 0 , 0 ) , α2 = ( 0 , 1 , 0 ) , α3 = ( 0 , 0 , 1 ) .当 β= ( a,
b, 0 )与 α1 ,α2 共面时 ,α1 不能由 β线性表示 , 但 α2 可由 β,α1 线性表示 , 即
1 a
α2 = β- α,
b b 1
而 α3 不能由 β,α1 ,α2 线性表示 ; 当 β= (0 , b, c) 与 α2 ,α3 共面时 ,α2 不能由 β,
α1 线性表示 , 而 α3 可由 β1 ,α1 ,α2 线性表示 , 即
1 b
α3 = β+ 0・α1 - α2 ;
c c
当 β= ( a, b, c)与 α1 ,α2 ,α3 中任意两个都不共面 , 此时只有 α3 可由其前 面的
β,α1 ,α2 线性表示 , 即
1 a b
α3 = β- α - α2 .
c c 1 c
15 ( 补充题 65 )   设 A 是一个 m× n 矩 阵 , m < n, r( A) = m, 齐 次线性 方
程组 Ax= 0 的一个基础解系为 b1 , b2 , … , bn - m , 其中
bi = ( bi1 , bi2 , … , bin ) T ,   i = 1 , 2 , … , n - m .
试求齐次线性方程组
n

∑b
j=1
ij yj = 0 ,   i = 1 ,2 ,… , n - m ( 1)

的基础解系所含解向量的个数 , 并求一个基础解系 .
解   已知 Abi = 0   ( i = 1 , 2 , … , n - m) .记
3 .7 部分疑难习题和补充题的题解 163

b11 b21 … bn - m, 1
b12 b22 … bn - m, 2
B = ( b1 , b2 , … , bn - m ) =
… … …
b1 n b2 n … bn - m, n
则 r ( B) = n - m, AB = 0, 齐次线性方程组 (1 )为
BT y= 0,     其中 y= ( y1 , y2 , … , yn ) T .
它的基础解系所含解向量个数为 n - r( BT ) = n - ( n - m) = m 个 .再由
( AB) T = BT AT = 0 ,
可知 , AT 的 m 个线性无关的列向量( 即 A 的 m 个线性无关的行向量) 是齐次
线性方程组 BT y= 0 的 m 个解 , 从而也就是它的一个基础解系 .
16 ( 补充题 68 )   设 A 是( n - 1 )× n 矩 阵 , | A j | 表示 A 中划去 第 j 列 所
构成的行列式 , 证明:
( 1) ( - | A1 | , | A2 | , … , ( - 1) n | An | ) T 是 Ax = 0 的一个解 ;
( 2) 若 | A j | ( j = 1 , 2 , … , n)不全为零 , 则 (1 )中的解是 Ax= 0 的一个 基础
解系 .
证   ( 1) 在 A = ( ai j ) ( n - 1) × n 的第 1 行 之前添加一 行 ( 添加之 行就是 A 的
第 1 行 ) , 使之成为 n 阶矩阵 , 并记作 B .即
a11 a12 … a1 n
a11 a12 … a1 n
B= a21 a22 … a2 n .
… … …
an - 1 , 1 an - 1, 2 … an - 1, n
于是 , B 的第 2 行元素的代数余子式 :
B21 = - | A1 | ,   B22 = | A2 | ,   … ,
B2 n = ( - 1) 2 + n | An | = ( - 1) n | A n | .
由于 | B | = 0 , 所 以 | B | 中 第 2 行 元 素 分 别 乘其 代 数 余 子 式 之 和 等 于
零 ,即
| B| = a11 ( - | A1 | ) + a12 ( | A2 | ) + … +
a1 n ( ( - 1 ) n | An | ) = 0 . ( 1)
又 | B| 中第 3 , … , n 行元素乘第 2 行对应元素的代数余子式之和也等于零 , 即
164 第 3 章  线性方程组

ai1 ( - | A1 | ) + ai2 ( | A2 | ) + … + ai n ( ( - 1) n | A n | ) = 0 ,
i= 2 ,… , n - 1 . ( 2)
(1) , (2 )两式可统一写成
- | A1 | a11 a12 … a1 n
| A2 | a21 a22 … a2 n
A = ×
… … … …
( - 1 ) n | An | an - 1 , 1 an - 1, 2 … an - 1 , n
- | A1 | 0
| A2 | 0
  = . ( 3)
… …
( - 1) n | An | 0
(3) 式表明 : ( - ( A1 ) , | A2 | , … , ( - 1 ) n | An | ) T 是齐 次线 性方 程组 Ax = 0 的
一个解 .
( 2) 若 | A j | ( j = 1 , 2 , … , n)不全为零 , 由于 | A j | 均为矩阵 A 的 n - 1 阶子
式 , 所以 A 的非零子式的最高阶数为 n - 1 , 即 r( A) = n - 1 .于是 Ax = 0 的基
础解系所含解向量的个数为 n - r( A) = n - ( n - 1 ) = 1 个 , 因此 , 它的非零解
( - | A1 | , | A2 | , … , ( - 1) n | A n | ) T
就是 Ax= 0 的一个基础解系 .
17 ( 补充题 71 )   设 A, B 皆为 n 阶方阵 , 证明 :
r ( AB) ≥r ( A) + r ( B) - n,
并问 :若 A = ( ai j ) s× n , B = ( bij ) n× m , 上述结论是否成立 ?
证   证法 1 : 由 3 .4 节中例 3 可知
A 0 A 0
r ( A) + r ( B) = r ≤r . ( 1)
0 B I B
对( 1) 式右端的分块矩阵做初等行、列变换 , 使之化为“副”对角块矩阵 , 即
I - A A 0 I - B
0 I I B 0 I
0 - AB I - B 0 - AB
= = . ( 2)
I B 0 I I 0
由( 2) 式即得
3 .7 部分疑难习题和补充题的题解 165

A 0 0 - AB
            r =r
I B I 0
= r( I) + r( - AB) = n + r( AB) . ( 3)
由( 3) , (1 )式 , 即得
r ( AB) ≥r ( A) + r ( B) - n .
当 A 为 s× n 矩阵 , B 为 n× m 矩阵时 , 上述结论仍成立 .上面的证法仍适
用 , 要注意的是 , (2 )式中的单位阵 I , 有的是 n 阶 , 有的是 m 阶 .
证法 2 : 设 A = ( aij ) s× n , r( A) = l; B = ( bij ) n× m , r( B) = k; r ( AB) = r .利 用
A 的相抵标准形 .对于 A, 存在 s 阶可逆阵 P 和 n 阶可逆阵 Q, 使得
Il 0
PAQ= .
0 0
于是
b11 b12 … b1 m β1
… … … …
Il 0
      ( PAQ) ( Q- 1 B) = bl1 bl2 … bl m βl ( 4)
0 0
… … … …
bn1 bn2 … bn m βn

b11 b12 … b1 m β1
… … … …
bl1 bl2 … blm βl
= , ( 5)
0 0 … 0
… … …
0 0 … 0

其中 :r ( Q- 1 B) = r ( B) = k, 所 以 ( 4 ) 式 右 边 矩阵 的 行 向量 组 的 秩为 k, 即 秩
{β1 , … ,βl , … ,βn } = k;
r( PAQ) ( Q- 1 B) = r( PAB) = r ( AB) = r, 所 以 ( 5 ) 式矩阵 的行向 量组的 秩
为 r, 即秩{β1 , … ,βl } = r .
由( 4) , (5 )式可 见 , (5 ) 式 中行 向量 的极大 线性 无关 组 ( 含 r 个 向量 ) 与
(4) 式中后 n - l 个向量 βl + 1 , … ,βn 合 在一起 , 可能 线性无 关 , 也 可能 线性 相
关 , 所以它们的向量个数
166 第 3 章  线性方程组

r + ( n - l) ≥秩 {β1 , … ,βl , … ,βn } = k



r( AB) = r≥ l + k - n = r( A) + r( B) - n .
18 ( 补充题 72 )   设向量组 αj = ( a1 j , a2 j , … , anj ) T   ( j = 1 , 2 , … , n) .证
明 :如果
n

| aii | > ∑|
j =1
ai j | ,   i = 1 , 2 , … , n, ( 1)
j≠ i

则向量组 α1 ,α2 , … ,αn 线性无关 .


证   用反证法 , 设 α1 ,α2 ,… ,αn 线性相关 , 则矩阵
A = (α1 ,α2 ,… ,αn )
的秩小于 n( 即 | A| = 0 ) , 于是齐次线性方程组
Ax = 0 ( 2)
有非零解 x= ( x1 , x2 , … , x n ) T , 在 x 的 n 个分量中必有绝对值最大的 .设
| x i | ≥ | x j | ,   ( j≠ i) .
此时 , 在方程组( 2) 中的第 i 个方程为
ai1 x1 + … + ai i x i + … + ai n x n = 0 ,
于是
n n

| aii x i | = - ∑a
j=1
ij xj ≤ ∑|
j=1
aij | | x j | ,
j≠ i j≠ i

从而
n n
| xj |
| aii | ≤ ∑
j=1
| aij |
| xi |
≤ ∑|
j=1
aij | .
j≠ i j≠ i

这与题设 ( 1) 式矛盾 .所以 α1 ,α2 , …,αn 线性无关 .


  第 4 章

向量 空 间 与 线 性变 换

4 .1 基本要求与内容提要

1  基本要求

(1 ) 要理解 n 维实向量空间 Rn 的基 ( 或基底 ) 与向 量关于基 的


坐标的概念 , 会求一组基到另一组基的过渡矩阵 , 以及基变换后向
量的坐标变换 .
n
(2 ) 要熟悉 n 维欧氏空间 R 中 向 量的 内积 运算 及其 性 质 , 会
求向量的长度与向量间的夹角 ; 要理解标准正交基的概念 , 会用施
密特 ( Schmidt ) 正交化方法由一组基求一组标准正交基 .
(3 ) 要熟知正交矩阵及其性质 .
线性代数是研究线性空间 ( 或称向量空间 ) 结构与线性变换理
论的一门学科 .向量的线 性相 关性是 研究 线性 空间结 构与 线性 变
换理论的基础 ; 矩阵是有限维线性空间的线性变换的表示形式 ; 以
m× n 矩阵 A 为 系数 矩 阵的 齐次 和非 齐 次线 性 方程 组 Ax = 0 与
Ax = b 的求解问题 , 是 n 维线性空间到 m 维 线性空间 的线性映 射
求“核”和“全体原象”的问 题 ; 行列 式是 研究这 些问 题的一 个有 力
工具 .关于行列式 , 矩阵 , 向量的 线性 相关性 以及 线性 方程 组解 的
理论等具体问 题 , 我们在前 3 章 已经论述 .而 关于一般 的 ( 或说 抽
象的 ) 线性空间的结构以及线性变换的概念与性质 , 我们以打“ * ”
168 第 4 章 向量空间与线性变换

的形式在本章 4 .3~ 4 .6 节只作 了简要 的介绍 .课时 在 40 左右 的


院校或专业可不作为基本 要求 , 而 课时在 60 ~ 70 的应 有所 要求 .
由于历年来考研试题都不包含这些内容 , 所以 , 我们在“基本要求”
与“内容提要”中都不涉及这些内容 , 也不作分节辅导 , 只在部分疑
难习题和补充题 的 题解 中 对一 些 基 本概 念 题和 较 难的 题 作 了 题
解 .当然 , 必须指出 , 前面的 3 点基本要求应该掌握 , 它们也是考研
的内容 .

2  内容提要

(1 ) R n 的基与向量的 坐标 : 如果 B = {β1 ,β2 , … ,βn } Rn , 且


线性无关 , 又 " α∈Rn 均可由 B 线性表示 , 即
α= a1 β1 + a2 β2 + … + anβn ,
n
则称 B 是R 的一组基 ( 或基底 ) , 有序数组 ( a1 , a2 , … , an ) 称为向量
α关于基 B 的坐标 , 记作
T
αB = ( a1 , a2 , … , an )   或   αB = ( a1 , a2 , … , an ) .
Rn 中任何 n 个 线性 无关的 向量 {ξ1 ,ξ2 , … ,ξn } 都是 R n 的一 组
n
基, 因 为 R 中 任 何 n + 1 个 向 量 都 线 性 相 关 .n 个 单 位 向 量
n
{ ei = ( 0 , … , 1 , … , 0) ( i = 1 , 2 , … , n) }称为 R 的自然基 .
(2 ) 过渡矩阵 , 基变换与坐标变换 .
n
设 B1 = { α1 , α2 , … , αn } 与 B2 = { η1 , η2 , … , ηn } 是 R 的 两
组基 , 且
a11 a12 … a1 n
a21 a22 … a2 n
(η1 ,η2 , … ,ηn ) = (α1 ,α2 , … ,αn ) ,
… … …
an1 an2 … ann
其中右端矩阵 A= ( aij ) n× n 称为基 B1 到基 B2 的过 渡矩阵 ( 或称 基
B1 变为基 B2 的变换矩阵 ) , A 是可逆阵 .
又 α在两组基下的坐标分别为
4 .1 基本要求与内容提要 169

T T
αB1 = x = ( x1 , x2 , … , xn ) ;   αB2 = y = ( y1 , y2 , … , yn ) .
则坐标变换公式为
- 1
Ay = x   或   y = A x.
n
(3 ) R 中向量的内积 , 向量的长度与夹角 .
设 α= ( a1 , a2 , … , an ) T ,β= ( b1 , b2 , … , bn ) T ∈ R n , 则 α与 β的
内积为
(α,β) = a1 b1 + a2 b2 + … + an bn
= αT β=βT α.
2 2 2
向量 α的长度 : ‖α‖ = (α,α) = a1 + a2 + … + an .
向量 α与β的夹角为
(α,β)
〈α,β
〉= ar ccos              
‖α‖‖β‖
a1 b1 + a2 b2 + … + an bn
= ar ccos 2 2 2 2
.
a1 + … + an b1 + … + bn
向量 α与β正交 ( 垂直 ) , 当且仅当 (α,β) = 0 .
向量的内积满足柯西 -施瓦茨 ( Cauchy-Sch war z) 公式
| (α,β) | ≤‖α‖‖β‖ .
向量的内积具有以下性质 :
① (α,β) = (β,α) ;
② (α+β,γ) = (α,γ) + (β,γ) ;
③ ( kα,β) = k(α,β) ;
④ (α,α) ≥0 , 等号成立当且仅当 α= (0 , 0 , … , 0) T .
向量 α与β满足三角不等式
‖α+β‖≤‖α‖ + ‖β‖ ;
当 α⊥β时 , 满足勾股定理
    ‖α+ β‖2 = ‖α‖2 + ‖β‖2 .
n
( 4 ) R 的标准正交基 , 施密特 ( Schmidt ) 正交化方法 .
n
设 ε1 ,ε2 , … ,εn ∈ R , 若
170 第 4 章 向量空间与线性变换

1 ,   i = j,
(εi ,εj ) =   i, j = 1 , 2 , … , n,
0 ,   i≠ j,
n
则{ε1 ,ε2 , … ,εn } 线性无关 , 并称它为R 的一组标准正交基 .
Rn 的自然基{ ei = (0 , … , 1 , … , 0 ) ( i = 1 , 2 , … , n) } 是 最 简单、
最基本的标准正交基 .
n
施密特正交化方法———由R 的 一组 基 {α1 ,α2 , … ,αn } 构造 出
一组标准正交基的方法 : 先正交化 , 即
β1 = α1 ,
(α2 ,β1 )
β2 = α2 - β1 ,
(β1 ,β1 )
(α3 ,β1 ) (α3 ,β2 )
β3 = α3 - β1 - β2 ,
(β1 ,β1 ) (β2 ,β2 )
………………………………………
(αn ,β1 ) (αn ,β2 ) (αn ,βn - 1 )
βn = αn - β1 - β2 - … - βn - 1 ,
(β1 ,β1 ) (β2 ,β2 ) (βn - 1 ,βn - 1 )
其中 {β1 ,β2 , … ,βn }为非零的两两正交的向量组 .
再单位化 , 即
1
εi = βi ,   i = 1 , 2 , … , n .
‖βi ‖
则{ε1 ,ε2 , … ,εn } 为一组标准正交基 .
(5 ) 正交矩阵及其性质
n× n T
设 A∈ R , 若 A A = I, 则称 A 为正交矩阵 .
正交矩阵 A, B 的性质 :
① det A= 1 或 - 1;             ② A- 1 = AT ;
③ AT ( 即 A - 1 ) 也是正交矩阵 ; ④ AB 也是正交矩阵 .
A 为 n 阶正交矩阵的充要条件为 : A 的列 向量 组与行 向量 组
均为Rn 的标准正交基 .
Rn 中的列向量 x, y 由正交矩阵 A 变换为 Ax 和 Ay 时 , 其向量
的内积、长度及向量间的夹角保持不变 .即
4 .2 R n 的基与向量关于基的坐标 171

( Ax, Ay) = ( x, y) ;
‖ Ax‖ = ‖ x‖ ,   ‖ Ay‖ = ‖ y‖ ;
〈Ax, Ay〉=〈x, y〉 .

4 .2 R n 的基与向量关于基的坐标
n
R 的基与向量关于基 B = {β1 ,β2 , … ,βn } 的 坐标的 概念 , 以 及
同一个向量在两组不同基 下的 坐标之 间的 关系 , 在内 容提 要中 已
阐述 .这里要进一步明确以下几个问题 .
(1 ) 一个向量 α关 于 基 B = {β1 ,β2 , … ,βn } 的 坐标 αB = ( a1 ,
T
a2 , … , an ) 是 惟 一 确 定 的 .因 为 α 由 线 性 无 关 的 基 向 量 β1 ,
β2 , … ,βn 线性表示的表示法是惟一的 .
(2 ) 基 B = {β1 ,β2 , … ,βn } 中的向量是有序的 .同样的 基向量 ,
3
如果排序不一样就是不同的基 .例如 , 如果 B1 = {β1 ,β2 ,β3 } 是 R 的
3
一组基 , 则 B2 = {β2 ,β1 ,β3 } 是 R 的 另一组 基 .因此 , 如 果向 量 α关
于基 B1 的坐 标 αB 1 = ( 2 , - 1 , 3 ) T , 则 α关 于 基 B2 的 坐标 αB2 =
( - 1 , 2 , 3 ) T , 因为
α= 2β1 - β2 + 3β3 ,
α= - β2 + 2β1 + 3β3 .

1
同理 , α 关 于 基 B3 = - β3 , 4β1 , - β2 的 坐 标 αB3 =
2
T
1
- 3, ,2 , 因为
2
1 1
α= 3β3 + 2β1 - β2 = ( - 3 ) ( - β3 ) + ( 4β1 ) + 2 - β2 .
2 2
( 3 ) 为什么Rn 中任何 n 个线性无关的向量 {ξ1 ,ξ2 , … ,ξn } 都是
Rn 的一组基 呢 ? 因 为 R n 中任 何 n + 1 个向 量 都是 线性 相 关的 , 因
n
此 , " α∈R , 都有 α,ξ1 ,ξ2 , … ,ξn 是线性相关的 .根据定理 3 .3( 教
172 第 4 章 向量空间与线性变换

材 p116) ,α可由ξ1 ,ξ2 , … ,ξn 线性表示 , 即


α= x1 ξ1 + x2 ξ2 + … + xnξn ,
且表示法惟一 .
(4 ) 基 B1 = {α1 , α2 , … ,αn }到 基 B2 = {η1 , η2 , … ,ηn } 的过 渡
矩阵 A = ( aij ) n × n 是 可 逆 矩 阵 .其 证 明 见 定 理 4 .1 ( 教 材 p160 —
161) .即
若 {α1 ,α2 , … ,αn }线性无关 , 且
(η1 ,η2 , … ,ηn ) = (α1 ,α2 , … ,αn ) ( ai j ) n × n ,
则 { η1 , η2 , … , ηn } 线 性 无 关 的 充 要 条 件 为 A = ( aij ) n× n 可 逆 ,
即 | A| ≠0 .
3
例如 , 设 B = {α1 ,α2 ,α3 } 是线性无关的 ( 或是R 的一组基 ) , 若
β1 = 3α1 - α2 + 2α3 ,
β2 = 2α1 + α2 - α3 ,
β3 = 5α1     + α3 .

3 2 5
(β1 ,β2 ,β3 ) = (α1 ,α2 ,α3 ) -1 1 0 , (*)
2 -1 1
则由
3 2 5
| A| = - 1 1 0 = 3 + 5 - 10 + 2 = 0
2 -1 1
( 即 A 不可逆 ) 可知 ,β1 ,β2 ,β3 是线性相关的 .
一般地 , ( * ) 式中 β1 ,β2 ,β3 与右 边矩阵 A 中 对应的列 向量 组
有相同的线性相关 性 .例 如 ,β1 ,β2 是 线性 无 关的 , 因为 A 中 对 应
T T
的 (3 , - 1 , 2 ) 与 (2 , 1 , - 1 ) 是线性无关的 ( 不成比例的两个非零
向量是线性无关的 ) .
T T
例 1   设 α1 = ( 2 , 1 , a, - 13) ,α2 = ( 0 , 1 , 2 , 2) ,α3 = ( - 2 , 1 ,
4 .2 R n 的基与向量关于基的坐标 173

T T 4
11 , 11) ,α4 = ( 1 , 3 , 1 , 2) , 求 a, 使 {α1 ,α2 , α3 ,α4 } 为 R 的基 , 并 求
β = (3 , 8 , 0 , 6 ) T 关于这组基的坐标 .
解   设 x1 α1 + x2 α2 + x3 α3 + x4 α4 =β.
这个向量方程等价于非齐次线性方程组
x1 2 0 -2 1 x1 3
x2 1 1 1 3 x2 8
    (α1 ,α2 ,α3 ,α4 ) = = . ( 1)
x3 a 2 11 1 x3 0
x4 - 13 2 11 2 x4 6
方程组 (1 ) 有惟一解的充要条件为 α1 ,α2 ,α3 ,α4 线 性无关 ( 即它 是
R4 的基 ) .所以 题 中 的 两 个 问 题 可 以 通 过 求解 方 程 组 ( 1 ) 一 并 回
答 .由于 a 在第 1 列 , 对增广 矩阵做初 等行变换 时 , 在第 3 , 4 行 上
均会出现 a, 很不方便 .为简便起见 , 我们把第 1 , 4 列对换 , 即设
x4 α4 + x2 α2 + x3 α3 + x1 α1 = β,
其对应的非齐次线性方程组的增广矩阵为
1 0 -2 2 3 1 0 - 2 2 3
3 1 1 1 8 0 1 7 -5 -1
  →
1 2 11 a 0 0 2 13 a-2 -3
2 2 11 - 13 6 0 2 15 - 17 0
1 0 -2 2 3 1 0 -2 2 3
0 1 7 -5 -1 0 1 7 -5 -1
 → → . ( 2)
0 0 -1 a+ 8 -1 0 0 1 - a- 8 1
0 0 1 -7 2 0 0 0 a+ 1 1
由 (2 ) 式的阶梯形矩阵可见 , 当 a + 1≠0 即 a≠ - 1 时 , {α1 ,α2 ,α3 ,
4 T
α4 } 为R 的基 ,β关于这组基的坐标 ( x1 , x2 , x3 , x4 ) 为 :
1
x1 =   ( 注意 (2 ) 中第 4 列为 x1 的系数 ) ,
a+ 1
7 44 12
x3 = 2 + ,   x2 = - 15 - ,   x4 = 7 + .
a+ 1 a+ 1 a+ 1
174 第 4 章 向量空间与线性变换

T
例如 , 当 a = - 2 时 ,β的坐标为 ( - 1 , 29 , - 5 , - 5 ) .
3
例 2   在R 中 求非 零向 量使 之 在自 然基 { e1 , e2 , e3 } 和 基 {α1 ,
α2 ,α3 } 下的坐标相同 .其中
α1 = (1 , - 1 , 2 ) ,   α2 = ( 2 , 1 , - 1) ,   α3 = ( - 4 , 1 , 1 ) .
T
解   设所求非零向量为 ( x1 , x2 , x3 ) , 由它在自然基和 基 {α1 ,
α2 ,α3 } 下坐标相同 , 得
x1 α1 + x2 α2 + x3 α3 = x1 e1 + x2 e2 + x3 e3 .
这个向量方程可表示为
x1 x1
(α1 α2 α3 ) x2 = ( e1 e2 e3 ) x2 ,
x3 x3

1 2 -4 x1 1 0 0 x1
-1 1 1 x2 = 0 1 0 x2 .
2 - 1 1 x3 0 0 1 x3
将右边移项到左边 , 得
0 2 -4 x1 0
-1 0 1 x2 = 0 . ( 1)
2 -1 0 x3 0
解方程组 (1 ) , 得所求的非零向量为
( x1 , x2 , x3 ) T = k(1 , 2 , 1 ) T , k 为任意非零常数 .
3
例 3   在R 中 , 求 由基 B1 = {α1 ,α2 ,α3 } 到基 B2 = {β1 ,β2 ,β3 }
的过渡矩阵 , 其中
α1 = (1 , 1 , - 1 ) ,   α2 = ( 1 , - 1 , 1) ,   α3 = ( - 1 , 1 , 1 ) ;
β1 = ( 1 , 1 , 1) , β2 = (0 , 1 , 1 ) , β3 = ( 0 , 0 , 1) .
T T
又已知 ξ在基 B1 下的坐标为 ξB 1 = ( x1 , x2 , x3 ) = ( 1 , 2 , - 1) , 求
ξB2 = ( y1 , y2 , y3 ) T .
解   由基 B1 到基 B2 的过渡矩阵为 A = ( ai j ) 3 × 3 , 即
4 .2 R n 的基与向量关于基的坐标 175

a11 a12 a13


(β1 ,β2 ,β3 ) = (α1 ,α2 ,α3 ) a21 a22 a23 . ( 1)
a31 a32 a33
将基 B1 , B2 中向量按列向量代入 (1 ) 式 , 得
1 0 0 1 1 - 1 a11 a12 a13
1 1 0 = 1 - 1 1 a21 a22 a23 .
1 1 1 -1 1 1 a31 a32 a33
于是 , 过渡矩阵
- 1
1 1 -1 1 0 0
A= 1 - 1 1 1 1 0
-1 1 1 1 1 1
1 1 0 1 0 0 2 1 0
1 1
  = 1 0 1 1 1 0 = 2 1 1 .
2 2
0 1 1 1 1 1 2 2 1
- 1
基变换后 , 坐标变换的公式为 ξB2 = A ξB1 , 即
y1 x1 1 1 -1 1 4
- 1
y2 =A x2 = 0 -2 2 2 = -6 .
y3 x3 -2 2 0 - 1 2
求 ξB 2 的另一方法 :
根据ξB1 = (1 , 2 , - 1 ) T , 得
ξ= α1 + 2α2 - α3 = (4 , - 2 , 0 ) .
T
设 ξB 2 = ( y1 , y2 , y3 ) , 则
y1 β1 + y2 β2 + y3 β3 = ξ,

1 0 0 y1 4
1 1 0 y2 = - 2 ,
1 1 1 y3 0
所以
176 第 4 章 向量空间与线性变换

- 1
y1 1 0 0 4 1 0 0 4 4
y2 = 1 1 0 - 2 = -1 1 0 -2 = -6 .
y3 1 1 1 0 0 -1 1 0 2
例 4   已 知 ξ 在 基 B1 = { α1 , α2 , α3 } 下 的 坐 标 为 ξB1 =
(1 , - 2 , 2 ) T , 求 ξ在基 B2 = {β1 , β2 , β3 } 下 的 坐 标 ξB2 , 其 中 β1
= α1 + α2 ,β2 = α2 + α3 ,β3 = α3 + α1 .
解   法 1: 设 ξB2 = ( y1 , y2 , y3 ) T , 则
ξ= y1 β1 + y2 β2 + y3 β3 = α1 - 2α2 + 2α3 ,
即 y1 (α1 + α2 ) + y2 (α2 + α3 ) + y3 (α3 + α1 ) = α1 - 2α2 + 2α3
( y1 + y3 )α1 + ( y1 + y2 )α2 + ( y2 + y3 )α3 =α1 - 2α2 + 2α3 . ( 1)
( 1 ) 式两边 α1 ,α2 ,α3 的系 数应相 等 ( 因 为 两边 的系 数都 是 ξ关 于
基 B1 的坐标 ) , 于是
y1 + y3 = 1 ,
y1 + y2 = - 2 ,
y2 + y3 = 2 .
3 1 5
由此易得 : y1 = - , y2 = - , y3 = .即
2 2 2
T
3 1 5
ξB 2 = - , - , .
2 2 2
法 2 : 求由基 B1 到基 B2 的过渡矩阵 .由
β1 = α1 + α2 ,
β2 =     α2 + α3 ,
β3 = α1     + α3 .
立即可得
1 0 1
(β1 ,β2 ,β3 ) = (α1 ,α2 ,α3 ) 1 1 0 .
0 1 1
上式右边的矩阵即为由基 B1 到基 B2 的过渡矩阵 A, 于是
4 .3 R n 中向量的内积 标准正交基和正交矩阵 177

1 1 1  1 3
- -
2 2 2 2
1 1 1 1
ξB2 = A - 1 ξB1 = - -2 = - .
2 2 2 2
1 1 1 5
-
2 2 2  2 2

n
4 .3 R 中向量的内积 标准正交基和正交矩阵

除了在 4 .1 节的内容提要 ( 3 ) , ( 4 ) , ( 5 ) 中 所阐 述的基 本概 念


与理论外 , 读者还要进一步明确以下问题 .
(1 ) 我们定义 α= ( a1 , … , an ) 与 β= ( b1 , … , bn ) 的内积为
(α,β) = a1 b1 + … + an bn . ( 1)
n
它只适用于 n 维实向量空间R 中的向量 .此时 ,
2 2
(α,α) = a1 + … + an ≥0 .
因此 , 可利用内积定义向量的长度‖α‖ = (α,α) .
n
对于 n 维复向量空间C 中向量 α,β的 内积 不能如 ( 1 ) 式那 样
2
定义 .如果如 ( 1 ) 式那样定义 , 对于 α= (1 , 2i ) ∈C , 就有
(α,α) = 12 + (2i ) 2 = 1 - 4 = - 3 < 0 .
因此 , 我们就不能用向量自身的内积开方来定义其长度 .
n
关于C 中向量 α与β的内积应定义为 ( 见教材 p351 )
(α,β) = a1 b1 + a2 b2 + … + an bn , ( 2)
其中 , bj 为 b j 的共轭复数 ( j = 1 , 2 , … , n) .此时 , (α,β) ≠ (β,α) , 而
是 (α,β) = (β,α) , 后者为 (β,α) 的共轭复数 .
n
( 2 ) 关于R 中向量α 与β的夹角 .
在空间解析几何的向量代数中讲过 : 几 何向量 a 与 b 的点 积
( 内积 )
a・ b= ‖a‖‖ b‖cos〈a, b〉 .
178 第 4 章 向量空间与线性变换

从而 a 与 b 的夹角
a・ b
〈a, b〉= arccos .
‖a‖‖ b‖
因此 , 我们把R n 中向量 α与β的夹角也定义为
(α,β)
〈α,β
〉= arccos .
‖α‖‖β‖
但是必须注意 | cosθ| ≤1 , 能如上定义夹角〈α,β
〉的前提 是柯西 -施
瓦茨不等式成立 , 即
| (α,β) | ≤‖α‖‖β‖ .
( 3 ) 在R 中两两线性无关的非 零向量 组 α1 , α2 , … ,αs 不一 定
n

是线性无关的 , 例 如 , R3 中 3 个 共 面 而 互 不 共 线 的 非 零 向 量 α1 ,
3
α2 ,α3 , 虽然两两线性无关 , 但整体是线性相关的 .而在 R 中 3 个 两
两正交的非零向量 α1 ,α2 ,α3 一定不共面 , 其中任一个 向量都不 能
用另外两个向量线性表示 , 所以整体是线性无关的 .
一般地 , 若 α1 , α2 , … , αs 是 R n 中 两 两 正 交 的 非 零 向 量 组 , 则
α1 ,α2 , … ,αs 一定线性无关 .( 证明见教材 p168 中定理 4 .5) .附 带
说一句 , 这里为什么要 强调非 零向 量组 呢 ? 因为 零向 量与 任何 非
零向量的内积均为 0, 所以 , 我们也说 零向量与 任何非零 向量都 正
交 .因此 , 如果只说两两正交的向量组 α1 ,α2 , … ,αs , 就不能排除其
中有一个零向量 , 从而不能保证 α1 ,α2 , … ,αs 线性无关 .
(4 ) 对于一组线性无关的向量组 α1 ,α2 , … ,αn , 用施密特正 交
化方法 , 将其正交化时 , 得到的 β1 ,β2 , … ,βn 必 定是 两两正 交的 非
零向量 .因为
(αj ,β1 ) (αj ,β2 )
βj = αj - β1 - β2 - … -
(β1 ,β1 ) (β2 ,β2 )
(αj ,βj - 1 )
βj - 1 ,   j = 1 , 2 , … , n . ( 3)
(βj - 1 ,βj - 1 )
此时 ,β1 ,β2 , … ,βj 两 两正 交 , 且 都 是 非 零向 量 .如 果 βj = 0, 则 由
(3 ) 式 可 见 , αj 可 由 β1 , β2 , … ,βj - 1 线 性 表 示 , 而 后 者 是 由 α1 ,
4 .3 R n 中向量的内积 标准正交基和正交矩阵 179

α2 , … ,αj - 1 线性表示的 , 如此 ,αj 可由 α1 ,α2 , … ,αj - 1 线性 表示 , 从


而 α1 ,α2 , … ,αj - 1 ,αj 线性相关 , 与 α1 ,α2 , … ,αn 线性无关矛盾 .
由此也可 见 , 如果对 线性 相关 的向量 组 α1 ,α2 , … ,αn 来用 施
密特正交化方法 , 那么在正交 化过程 中至 少出现 一个 βj = 0 .如 果
β1 ,β2 , … ,βj - 1 均为非零向量 , 而 βj = 0, 就说明 α1 ,α2 , … ,αj - 1 线 性
无关 , 而 α1 ,α2 , … ,αj - 1 ,αj 线性相关 .
(5 ) 对于正交矩阵 A 需要 明 确 : ①它 是 n 阶实 矩阵 ; ② 其 定
T T
义为 A A = I ( 当然也可定义为 AA = I ) .由这个定义可推出 :
n
( i ) A 为正交矩阵的充要条件 是 A 的列向量 组是R 的 一组 标
准正交基 ( 见教材 p173 定理 4 .6 ) .
- 1 T
( ii ) 正交矩阵 A 的逆矩阵 A = A , 从而
T T T T - 1
( A ) A = AA = AA = I,
即 AT 也是正交矩阵 , 于是 AT 的列向量组也就是 A 的行向量组 也
n
是R 的一组标准正交基 .
T
需要注意 : 对于 n× m 实矩阵 A ( m < n) , 当 A A = Im 时 , 不能
说 A 为正交矩阵 ( 因为这个 A 不是方阵 ) , 此时
A = (β1 ,β2 , … ,βm )
n
的列向量组 β1 ,β2 , … ,βm ( 含 m 个列向量 ) 是 R 中一组 标准正交 的
向量组 ( 即两两正交 , 且每个向量的长度为 1 ) , 但不是 R n 的一组 标
准正交基 ( 因为 m < n) .
(6 ) 以 A = ( ai j ) m × n 为系数矩阵的齐次线性方程组
a11 x1 + a12 x2 + … + a1 n x n = 0 ,
a21 x1 + a22 x2 + … + a2 n x n = 0 ,
( 4)
…………………………………
am1 x1 + am2 x2 + … + am n x n = 0 .
的解 x = ( x1 , x2 , … , xn ) T 的几何意义是 : 每个解向量 x 与 A 的每
个行向量 αi = ( ai1 , ai2 , … , ain ) ( i = 1 , 2 , … , m) 都正 交 .因为 (4 ) 中
第 i 个方程就是 αi 与 x 的内积等于 0, 即
180 第 4 章 向量空间与线性变换

αi x = (αi , x) = ai1 x1 + ai2 x2 + … + ai n x n = 0 ,   i = 1 , … , m .


附带说一句 : 通常把 Ax= 0 的全体解向量构成的解集合称为
Ax = 0 的解空间 , 记 作 N ( A) ; 把 A 的 m 个行 向 量 α1 , α2 , … , αm
所有线性组合
a1 α1 + a2 α2 + … + am αm
( a1 , a2 , … , am 为 任意 常数 ) 形 成的 向量集 合称 为 A 的 行空 间 , 记
作 R ( AT ) .于 是 解 空 间 N ( A) 中 每 个 解 向 量 x 与 A 的 行 空 间
T
R( A ) 中每个向量都正交 , 因为
( a1 α1 + a2 α2 + … + amαm , x)
= a1 (α1 , x) + a2 (α2 , x) + … + am (αm , x) = 0 .
T T
因此 , 就说 N ( A) 与 R( A ) 是正交的 .由于 N ( A) 与 R( A ) 都 包含
于R n 之中 , 也说它们是 R n 的 子空 间 , 所 以 N ( A) 与 R( AT ) 是 R n 的
两个正交的子空间 .
例 1   设 α= ( 1, - 1, 0, - 1 ) ,β= ( 1, - 1 , 1 , - 1 ) ,γ= ( 1 , - 1 ,
- 1 , - 1) .
( 1 ) 求 α的长度及β与γ的夹角 .
( 2 ) 求与 α,β,γ都 正交的 所有 向量 , 并将 这些 向 量表 示为 标
准正交向量组的线性组合 .
解   ( 1 ) ‖α‖ = (α,α) = 12 + ( - 1)2 + 02 + ( - 1)2 = 3 .
〈β,γ
〉 1+1 - 1+1
〈β,γ
〉= a rccos = arccos
‖β‖‖γ‖ 4・ 4
1 π
= a rccos = .
2 3
(2 ) 设 x = ( x1 , x2 , x3 , x4 ) 与 α,β,γ都正交 , 于是
(α, x) = x1 - x2     - x4 = 0 ,
(β, x) = x1 - x2 + x3 - x4 = 0 , ( 1)
(γ, x) = x1 - x2 - x3 - x4 = 0 .
容易求得方程组 (1 ) 的基础解系为
4 .3 R n 中向量的内积 标准正交基和正交矩阵 181

T T
ξ1 = (1 , 1 , 0 , 0 ) ,   ξ2 = (1 , 0 , 0 , 1 ) .
方程组 (1 ) 的解集合 ( 也称解空间 )
x = { k1 ξ1 + k2 ξ2 | k1 , k2 为任意常数 }          
T T
= { k1 (1 , 1, 0 ,0) + k2 (1, 0 , 0, 1) | k1 , k2 为任意常数} ( 2)
中所有向量是与 α,β,γ都正交的全部向量 .
但 ξ1 与 ξ2 不是 标 准 正 交 的向 量 , 所 以 ( 2 ) 式 中 的 线 性 组 合
k1 ξ1 + k2 ξ2 不是标准正交向 量组 的 线性 组合 .由 于 ξ1 ,ξ2 都与 α,
β,γ正交 , 所以 ξ1 与 ξ2 的线性组合也与 α,β,γ正交 .因此 , 用施密
特正交化方法由 ξ1 ,ξ2 构造出 的标准正交向量组 ε1 ,ε2 也都 与 α,
β,γ正交 .
T
令β1 =ξ1 = ( 1 , 1 , 0 , 0) ,
(ξ2 ,β1 ) T 1 T
β2 = ξ2 - β1 = (1 , 0 , 0 , 1 ) - (1 , 1 , 0 , 0 )
(β1 ,β1 ) 2
T
1 1
= , - , 0, 1 ,
2 2
再单位化 , 得
T
1 1 T 1 1
ε1 = ξ1 = ( 1, 1, 0, 0 ) = , ,0 ,0 ,
‖ξ1 ‖ 2 2 2
T T
1 2 1 1 1 1 2
ε2 = ξ2 = , - , 0, 1 = , - , 0, .
‖ξ2 ‖ 3 2 2 6 6 6
于是
x = k1ε1 + k2ε2 k1 , k2 为任意常数                    
T T
1 1 1 1 2
= k1 , , 0,0 + k2 , - , 0, k1 , k2 为任意常数 (3)
2 2 6 6 6

中所有向量也是与 α,β,γ都正交的全部向量 .
严格地讲 , 这里还要证明两个解集合 ( 2 ) 与集合 ( 3 ) 是相等的 .
先证 , " β∈ k1 ξ1 + k2 ξ2 , 均 有 β∈ 集 合 ( 3 ) , 从 而 集 合 ( 2 ) 集合
(3 ) ; 再证 ; " β∈ k1 ε1 + k2ε2 , 均 有 β∈ 集合 ( 2 ) , 从 而集 合 ( 3 ) 集
182 第 4 章 向量空间与线性变换

合 (2 ) .于是集合 (2 ) = 集合 ( 3) .理解了施密特正交化过程 , 这个证


明是不困难的 , 我们略去其证明 .
例2  设
1 -1 1 2 1
2 -2 3 5 4
A= ,
3 -3 4 7 5
4 -4 5 9 6
求齐次线性方程组 Ax= 0 的一个标准正 交的基 础解系 ( 也 称解 空
间的标准正交基 ) .
解   对 A 做初等行变换 , 将其化为行简化阶梯阵 .即
1 -1 0 1 -1
0 0 1 1 2
A→ .
0 0 0 0 0
0 0 0 0 0
取 ( x2 , x4 , x5 ) 为自由未知量 , 依次分别取 (1 , 0 , 0 ) , (0 , 1 , 0 ) , ( 0 , 0 ,
1) , 易得 Ax = 0 的基础解系 {α1 ,α2 ,α3 } 为 :
T
α1 = (1 , 1 , 0 , 0 , 0 ) ,
T
α2 = ( - 1 , 0 , - 1 , 1 , 0 ) ,
α3 = (1 , 0 , - 2 , 0 , 1 ) T .
用施密特正 交 化 方 法 , 由 {α1 , α2 , α3 } 得 到 的 标 准 正 交 的 向 量 组
{ε1 ,ε2 ,ε3 } 仍然是 Ax = 0 的 基础 解系 ( 因为 ε1 ,ε2 ,ε3 都 是 α1 ,α2 ,
α3 的线性组合 , 所以它们仍 然都 是 Ax = 0 的解 , 从 而 也是 基础 解
系 ) .先正交化 , 令
β1 = α1 = ( 1 ,1 ,0 ,0 ,0 ) T ,
(α2 ,β1 ) -1
β2 = α2 - β1 = ( - 1 , 0 , - 1 , 1 , 0 ) T - ( 1 , 1 , 0 , 0 , 0) T
(β1 ,β1 ) 2
T
1 1
  = - , , - 1, - 1, 0 ,
2 2
4 .3 R n 中向量的内积 标准正交基和正交矩阵 183

(α3 ,β1 ) (α3 ,β2 )


  β3 = α3 - β1 - β2
(β1 ,β1 ) (β2 ,β2 )
T
1 3 1 1
= (1,0, - 2, 0,1) - (1,1, 0,0, 0) T -
T
- , , - 1, - 1, 0
2 5 2 2
T
4 4 7 3
= , - , - , ,1 .
5 5 5 5
再单位化 , 得
T
1 1 1 1
ε1 = β1 = β1 = , , 0, 0, 0 ,
‖β1 ‖ 2 2 2
T
1 2 1 1 2 2
ε2 = β2 = β2 = - , , - , - ,0 ,
‖β2 ‖ 5 10 10 10 10
T
1 5 4 -4 7 3 5
ε3 = β3 = β3 = , , - , , .
‖β3 ‖ 23 115 115 115 115 115
于是{ε1 ,ε2 ,ε3 } 就是 Ax = 0 的 解空 间 的标 准正 交基 ( 即 为标 准 正
交的基础解系 ) .
例 3   设 {α1 , α2 ,α3 , α4 } 为 R4 的 一组标 准正 交基 , 证明 : {β1 ,
4
β2 ,β3 ,β4 } 也为R 的一组标准正交基 , 其中 :
1 1
β1 = (α1 + α2 + α3 + α4 ) ,   β2 = (α1 + α2 - α3 - α4 ) ,
2 2
1 1
β3 = (α1 - α2 + α3 - α4 ) ,   β4 = (α1 - α2 - α3 + α4 ) .
2 2
证   需要证明: (βi ,βj ) = 0 ( i≠ j) ; (βi ,βi ) = 1 ( i = 1 , 2, 3 , 4) .
1 1
(β1 ,β2 ) = (α1 + α2 + α3 + α4 ) , (α1 + α2 - α3 - α4 )
2 2
由于当 i≠ j 时 , (αi ,αj ) = 0; (αi ,αi ) = 1( i = 1 , 2 , 3 , 4) , 所以
1
(β1 ,β2 ) = (α1 ,α1 ) + (α2 ,α2 ) + (α3 , - α3 ) + (α4 , - α4 )
4
1
= (1 + 1 - 1 - 1) = 0 .
4
同理 , 可证 当 i≠ j 时 , 所 有 的 (βi , βj ) = 0 .因 此 ,β1 ,β2 ,β3 ,β4 两
184 第 4 章 向量空间与线性变换

两正交 .
1 1
(β1 ,β1 ) = (α1 + α2 + α3 + α4 ) , (α1 + α2 + α3 + α4 )
2 2
1
= (α1 ,α1 ) + (α2 ,α2 ) + (α3 ,α3 ) + (α4 ,α4 )
4
1
= (1 + 1 + 1 + 1) = 1 .
4
同理可证 (βi ,βi ) = 1( i = 2 , 3 , 4 ) .因此 β1 ,β2 ,β3 ,β4 的长度都为 1 .
4
综上 , {β1 ,β2 ,β3 ,β4 } 为 R 的一组标准正交基 .
上述证法比较麻烦 .较为方便的证法为 :
由于已知的 {β1 ,β2 ,β3 ,β4 } 与 {α1 ,α2 ,α3 ,α4 } 的关系可表示为

1 1 1 1
2 2 2 2
1 1 1 1
- -
2 2 2 2
(β1 ,β2 ,β3 ,β4 ) = (α1 ,α2 ,α3 ,α4 ) ( 1)
1 1 1 1
- -
2 2 2 2
1 1 1 1
- -
2 2 2 2

将 (1 ) 式中的 αi ,βi ( i = 1 , … , 4) 都按列排列 , 则

(β1 ,β2 ,β3 ,β4 ) = B,   (α1 ,α2 ,α3 ,α4 ) = A

都是四阶矩阵 .再将 ( 1 ) 式右边的矩阵记为 P, 则

B = AP . ( 2)
4
( 2 ) 式中的 A 为 正 交 矩 阵 ( 因 为 A 的 列 向 量 组 为 R 的 标 准 正 交
基 ) .欲证 β1 ,β2 ,β3 ,β4 为 R4 的标 准正交 基 , 只要 证 B 为正交 矩阵 ,
而欲证 B 为 正交阵 , 只要证 P 为 正交 阵 ( 因为 两个 正交阵 的乘 积
仍为正交阵 ) .由
4 .3 R n 中向量的内积 标准正交基和正交矩阵 185

1 1 1 1 1 1 1 1
2 2 2 2 2 2 2 2
1 1 1 1 1 1 1 1
- - - -
T 2 2 2 2 2 2 2 2
P P=
1 1 1 1 1 1 1 1
- - - -
2 2 2 2 2 2 2 2
1 1 1 1 1 1 1 1
- - - -
2 2 2 2 2 2 2 2
1 0 0 0
0 1 0 0
= = I,
0 0 1 0
0 0 0 1
可知 P 为正交 阵 , 所 以 B = AP 也 为 正 交 阵 , 从 而 B 的 列 向 量 组
4
β1 ,β2 ,β3 ,β4 为R 的一组标准正交基 .
n
例 4   设 α1 ,α2 ,α3 ,α4 ∈R , 证明 : 四阶行列式

(α1 ,α1 ) (α1 ,α2 ) (α1 ,α3 ) (α1 ,α4 )


(α2 ,α1 ) (α2 ,α2 ) (α2 ,α3 ) (α2 ,α4 )
| A| = ≠0 ( 1)
(α3 ,α1 ) (α3 ,α2 ) (α3 ,α3 ) (α3 ,α4 )
(α4 ,α1 ) (α4 ,α2 ) (α4 ,α3 ) (α4 ,α4 )

的充分必要条件为 α1 ,α2 ,α3 ,α4 线性无关 .


证   必要性 : 设 x1 α1 + x2 α2 + x3 α3 + x4 α4 = 0 . ( 2)
将 αi ( i = 1 , 2 , 3 , 4 ) 分别与 ( 2) 式两端的向量做内积 , 得

(αi , x1 α1 + x2 α2 + x3 α3 + x4 α4 )
= x1 (αi ,α1 ) + x2 (αi ,α2 ) + x3 (αi ,α3 ) + x4 (αi ,α4 ) = 0 ,
i = 1, 2, 3, 4 .

于是得到一个四元齐次线性方程组
186 第 4 章 向量空间与线性变换

(α1 ,α1 ) x1 + (α1 ,α2 ) x2 + (α1 ,α3 ) x3 + (α1 ,α4 ) x4 = 0 ,


(α2 ,α1 ) x1 + (α2 ,α2 ) x2 + (α2 ,α3 ) x3 + (α2 ,α4 ) x4 = 0 ,
( 3)
(α3 ,α1 ) x1 + (α3 ,α2 ) x2 + (α3 ,α3 ) x3 + (α3 ,α4 ) x4 = 0 ,
(α4 ,α1 ) x1 + (α4 ,α2 ) x2 + (α4 ,α3 ) x3 + (α4 ,α4 ) x4 = 0 .
由于方程组 (3 ) 的系数行列式 ( 即 (1 ) 式中的行列式 | A| ) 不等 于 0,
所以方程组 ( 3 ) 只有零解 , 即 仅当 x1 = x2 = x3 = x4 = 0 时 ( 2 ) 式 才
能成立 , 所以 α1 ,α2 ,α3 ,α4 线性无关 .必要性得证 .
证必要性还可用反证法 : 设 α1 ,α2 ,α3 ,α4 线性相关 , 不妨设
α1 = k2 α2 + k3 α3 + k4 α4 .
将 ( 1 ) 式中的行 列式 | A | 的 第 2, 3, 4 列分 别 乘 - k2 , - k3 , - k4 加
到第 1 列 , 则 | A| 中第 1 列元素变为
(αi ,α1 - k2 α2 - k3 α3 - k4 α4 ) = (αi ,0) = 0 ,   i = 1 , 2 , 3 , 4 .于是
0 (α1 ,α2 ) (α1 ,α3 ) (α1 ,α4 )
0 (α2 ,α2 ) (α2 ,α3 ) (α2 ,α4 )
| A| = = 0,
0 (α3 ,α2 ) (α3 ,α3 ) (α3 ,α4 )
0 (α4 ,α2 ) (α4 ,α3 ) (α4 ,α4 )
与题设矛盾 .所以 α1 ,α2 ,α3 ,α4 必须线性无关 .
充分性 : 即要证明 α1 ,α2 ,α3 ,α4 线 性无 关时 , | A | ≠0 一定 成
立 .此时如用直接证法 , 难以说清楚 , 所以要用反证法 .
设 | A| = 0, 则矩 阵 A 的 4 个 列 向 量 β1 , β2 ,β3 , β4 线 性相 关 ,
( 因为 A 的列秩小于 4 ) , 于是存在不全为零的 k1 , k2 , k3 , k4 , 使得
k1 β1 + k2 β2 + k3β3 + k4 β4 = 0 . ( 4)
将 | A| 中各列分别乘 k1 , k2 , k3 , k4 加 到第 1 列 , 则第 1 列的 4 个 元
素为
4

αi , ∑ kjαj = 0 ,   i = 1 , 2 , 3 , 4 . ( 5)
j=1

再将 (5 ) 式分别乘 ki ( i = 1 , 2 , 3 , 4) 并相加 , 得
4 .3 R n 中向量的内积 标准正交基和正交矩阵 187

4 4 4 4

∑k
i= 1
i αi , ∑ kjαj
j= 1
= ∑ kα , ∑ k α
i= 1
i i
j= 1
j j = 0 . ( 6)

根据 (ξ,ξ) = 0 当且仅当 ξ= 0, 由 ( 6 ) 式即得


4 4

∑ kα
i=1
i i = ∑ kα
j=1
j j = k1 α1 + k2 α2 + k3 α3 + k4 α4 = 0 . ( 7)

由于 (7 ) 式中的 k1 , k2 , k3 , k4 不全 为零 , 所以 α1 ,α2 , α3 ,α4 线性 相


关 , 与题设矛 盾 .因此 ,α1 , α2 ,α3 , α4 线 性无 关时 , 必 有 | A| ≠ 0 .充
分性得证 .
本题的结论也适用于Rn 中 n 个向量 α1 ,α2 , … ,αn 构成的 n 阶
行列式 | A| .
n
作为练习 .读者应能证明 : ( 对 α1 , … ,αn ∈R 而言 )
| A| = 0 的充分必要条件为 α1 ,α2 , … ,αn 线性相关 .
例 5   设 α1 , α2 , … , αr 线 性 无 关 , 非 零向 量 β与 {α1 , α2 , … ,
αr }中 的 每 个 向 量 都 正 交 , 证 明 : β, α1 , α2 , … , αr 线 性 无 关 , 并
在R3 中作几何解释 .
证   设 kβ+ k1 α1 + k2 α2 + … + krαr = 0, ( 1)
将 β与 ( 1 ) 中左端零向量做内积 , 得
(β, kβ+ k1 α1 + … + krαr ) ( 2)
= k(β,β) + k1 (β,α1 ) + … + kr (β,αr )
    = k(β,β) + 0 + … + 0 = 0( 因为 (2 ) 中后者为零向量 ) .
而 (β,β) > 0 , 所以 k = 0 , 代入 (1 ) 式 , 又得
k1 α1 + k2 α2 + … + krαr = 0 .
因为 α1 ,α2 , … ,αr 线性无关 , 所以 k1 = k2 = … = kr = 0 , 因此 (1 ) 式
成立时 , 必须其系数全为零 .故 β,α1 ,α2 , … ,αr 线性无关 .
3
R 中 几 何 解 释 : α1 , α2 线 性 无 关 是 两 个 非 零 且 不 共 线 的 向
量 ,β与 α1 ,α2 正交 .即 β垂直于 α1 ,α2 所 确定 的平面 , 所 以 β,α1 ,
α2 是不共面的 3 个 向量 , 因而 它们 线性 无关 ( 即 任一 个 向量 都 不
能用另外两个向量线性表示 ) .
188 第 4 章 向量空间与线性变换

例 6   设 α1 ,α2 , … ,αr 线性无关 ,β1 ,β2 也线性无关 , 且后者与


前者 中 的 每 个 向 量 都 正 交 , 证 明 : β1 , β2 , α1 , α2 , … , αr 也 线 性
无关 .
证   设   x1 β1 + x2 β2 + k1 α1 + k2 α2 + … + krαr = 0 . ( 1)
将 β1 ,β2 分别与 (1 ) 式左端的零向量做内积 , 得
(β1 , x1β1 + x2β2 + k1α1 + … + krαr ) = (β1 ,β1 ) x1 + (β1 ,β2 ) x2 = 0,
  ( 2)
(β2 , x1β1 + x2β2 + k1α1 + … + krαr ) = (β2 ,β1 ) x1 + (β2 ,β2 ) x2 = 0 .
齐次线性方程组 (2 ) 的系数行列式
(β1 ,β1 ) (β1 ,β2 )
= ‖β1 ‖2 ‖β2 ‖2 - (β1 ,β2 ) 2 > 0 . ( 3)
(β2 ,β1 ) (β2 ,β2 )
( 3 ) 式是根据 柯西 -施 瓦茨不 等式 | (β1 ,β2 ) | ≤‖β1 ‖‖β2 ‖ ( 等 号
成立当且仅当 β1 ,β2 线性相关 ) .
由 ( 3 ) 式得方程组 ( 2 ) 只 有 零解 , 即 x1 = x2 = 0 .代入 ( 1 ) 式 又
得 k1 = k2 = … = kr = 0 .所以 β1 ,β2 ,α1 ,α2 , … ,αr 线性无关 .
本题可以再推广为 :
设 β1 ,β2 , … ,βm 线 性 无关 , α1 , α2 , … , αr 也 线性 无 关 .若 {β1 ,
β2 , … ,βm }中每个向量与 {α1 ,α2 , … ,αr }中 每个向量 都正交 , 则 β1 ,
β2 , … ,βm ,α1 ,α2 , … ,αr 也线性无关 .
证明时 , 设 x1β1 + … + xmβm + k1 α1 + … + krαr = 0, 会 得到 一
个类似于 (2 ) 的一个 m 元 ( x1 , … , xm ) 的线 性方 程组 , 其系 数行 列
式是例 4 中类型的 | A| ≠ 0, 从 而 x1 = x2 = … = xm = 0 .进而 k1 =
k2 = … = kr = 0 .
例 7   已 知 B = {α1 ,α2 ,α3 } 是 R 3 的一 个 标 准正 交 基 , 求 ξ=
(1 , 2 , 3 ) T 在基 B 下的坐标 .其中 :
T T
1 2 2 2 1 2
α1 = , - , - ,   α2 = - , , - ,
3 3 3 3 3 3
T
2 2 1
α3 = - , - , .
3 3 3
4 .3 R n 中向量的内积 标准正交基和正交矩阵 189

解   如果 B 不是标准正交基 , 求 ξ在基 B 下的坐标ξB = ( x1 ,


x2 , x3 ) T 的方法为 : 求解下列非齐次线性方程组
x1
(α1 ,α2 ,α3 ) x2 = ξ.
x3
而当 B 为标准正交基时 , 可用下列简便的方法 :
设   ξ= x1 α1 + x2 α2 + x3 α3 , 则
(ξ,αi ) = ( x1 α1 + x2 α2 + x3 α3 ,αi )
= x1 (α1 ,αi ) + x2 (α2 ,αi ) + x3 (α3 ,αi ) . ( 1)
(1 ) 式右端的 3 个内积有两个 为零 , 只有 x i 那一 项为非零 , 且 (αi ,
αi ) = 1 , 即
(ξ,αi ) = xi (αi ,αi ) = xi   ( i = 1 , 2 , 3) .
1 4 6
所以   x1 = (ξ,α1 ) = - - = - 3,  
3 3 3
2 2 6
x2 = (ξ,α2 ) = - + - = - 2,
3 3 3
2 4 3
x3 = (ξ,α3 ) = - - + = - 1,
3 3 3
T T
即 ξB = ( x1 , x2 , x3 ) = ( - 3 , - 2 , - 1 ) .
例 8   已知
3
a  - d
7
3 2
P= - c 
7 7
2 3
b  -
7  7
为正交矩阵 , 求 a, b, c, d .
解   根据 P 的 3 个列向量 β1 ,β2 ,β3 为 R3 的标准正交 基 .立 即
可得 :
190 第 4 章 向量空间与线性变换

9 4 6
‖β2 ‖2 = + c2 + = 1 , 所以 c = ± ; ( 1)
49 49 7
4 9 6
‖β3 ‖2 = d2 + + = 1 , 所以 d = ± . ( 2)
49 49 7
再由 (β2 ,β3 ) = 0 , 即
3 2 6
- d+ c- =0 . ( 3)
7 7 49
6 2
当 c= 时 , 由 ( 3 ) 式 得 d = , 这 不符合 ( 2) 式必 须满足的 要
7 7
6
求 .所以 c≠ .
7
6 6
当 c= - 时,得 d = - ( 满足 ( 2) 式的要求 ) .
7 7
6 12
d≠ , 否则由 ( 3) 式得 c= , 这不满足 ( 1) 式 .
7 7
6
综上 , c = d = - .
7
再由 (β1 ,β2 ) = (β1 ,β3 ) = 0 , 得
3 18 2
- a+ + b= 0,
7 49 7
( 4)
6 6 3
- a- - b= 0 .
7 49 7
2 6
求解方程组 (4 ) , 得 a = , b= - .此时 , ‖β1 ‖ = 1 .
7 7
2 6
结论 : 当 a = , b= c= d = - 时 , P 为正 交矩 阵 .检 验结 论
7 7
是否正确 , 可计算 PT P, 看它是否等于单位矩阵 I .
例 9   试问 : 正交矩阵 A = ( aij ) n× n 的元素 a ij 与其代数余子式
A ij 之间是否有某种确定的关系 ?
T - 1 T T T
解   由 于 A A = I, 所 以 A = A , 且 | A A| = | A | | A| =
| A| 2 = 1, 即 | A| = ± 1; 又因为
4 .3 R n 中向量的内积 标准正交基和正交矩阵 191

1 *
A- 1 = A = ± A* , ( 1)
| A|

其中 A 为 A 的元素 a i j 的代数余子式 A ij 构成的矩阵 ( A ij ) n× n 的转
置 .即
* T
A = ( A ij ) n× n . ( 2)
- 1 T T
于是由 (1 ) , (2 ) 式及 A = A = ( aij ) n× n , 即得
a11 a21 … an1 A11 A2 1 … An1
a12 a22 … an2 A12 A2 2 … An2
=± . ( 3)
… … … … … …
a1 n a2 n … ann A1 n A2 n … Ann
由 (3 ) 式可见 , 正交矩阵 A 的元素 a ij 与其代数余子式 A i j 之间有 下
列确定的关系 :
当 | A| = 1 时 , ai j = Aij   ( i, j = 1 , 2 , … , n) ;
当 | A| = - 1 时 , ai j = - Aij   ( i, j = 1 , 2 , … , n) .
例 10   证明 : 如果下三角矩阵 A = ( aij ) n× n 为 正交矩 阵 , 则 A
必为主对角元为±1 的对角矩阵 .
证   用数学归纳法证明之 .当 n = 2 时 , 由

T
a11 a2 1 a11 0 a211 + a221 a21 a22
A A= = 2
= I,
0 a2 2 a21 a22 a2 2 a21 a22
易得 :   a222 = 1 , 所以 a22 = ±1 ;
a21 a22 = 0 , 所以 a21 = 0 ;
2 2 2
a11 + a21 = a11 = 1 , 所以 a11 = ±1 .
因此 , 结论对 n = 2 成立 .假设结论 对 n - 1 阶下 三角矩 阵成 立 .下
面证明对 n 阶下三角矩阵也成立 .
现将 n 阶下三角矩阵 A 分块表示为
A11 0
A= , ( 4)
α an n
其中 α为 1× ( n - 1 ) 矩阵 , A1 1 为 n - 1 阶下三角矩阵 .于是由
192 第 4 章 向量空间与线性变换

T A11T αT A11 0
A A =
0 an n α an n
A11T A1 1 + αT α αT an n In - 1 0
= = ,
an nα a2nn 0 1
即得 : a = 1 , 所以 an n = ± 1; an nα= 0, 所以 α= 0;
2
nn

A1T1 A11 + αT α= A11T A1 1 = In - 1 , 即 A11 为 n - 1 阶 正交矩 阵 .根 据


归纳假设 A11 为主对角元为±1 的对角矩阵 .
综上 , 由 ( 4 ) 式可知 A 也是主对角元为±1 的对角矩阵 .
例 11 ( 补充题 51 )   设 A 为正交矩阵 , I + A 可逆 , 证明 :
- 1
( 1 ) ( I - A) ( I + A) 可交换 ;
( 2 ) ( I - A) ( I + A) - 1 为反对称矩阵 .
2
证   ( 1 ) 因为 I - A = ( I - A) ( I + A) = ( I + A) ( I - A) , 所 以
- 1 - 1
( I + A) ( I - A) ( I + A) = ( I + A) ( I + A) ( I - A) ,
即 ( I + A) - 1 ( I - A) ( I + A) = ( I - A) ,
- 1
在等式两端右乘 ( I + A) , 从而就有
( I + A) - 1 ( I - A) = ( I - A) ( I + A) - 1 .
故 ( I - A) ( I + A) - 1 可交换 .
T
( 2 ) 当 B = - B 时 , 称 B 为反对称矩阵 .于是 , 由
- 1 T - 1 T T
( I - A) ( I + A) = ( I + A) ( I - A)
= ( I + AT ) - 1 ( I - AT ) ( 利用 AT A = I )
T - 1 T - 1 T - 1 T
= A ( A+ I) A ( A - I) = ( A+ I) ( A ) A ( A - I)
- 1 - 1
= ( A + I ) ( A - I ) = - ( I + A) ( I - A)
= - ( I - A) ( I + A) - 1 ( 最后的等号利用 ( 1 ) 的结论 ) .
故 ( I - A) ( I + A) - 1 为反对称矩阵 .


4 .4 部分疑难习题和补充题的题解
1 ( 习题 17 )   检验下列集合对指定的加法和数量乘法运算 ,是否构成实
数域上的线性空间 :

4 .4  部分疑难习题和补充题的题解 193

( 1) 全体 n 阶正交矩阵 , 对矩阵的加法和数量乘法 ;
( 2) 平面上全体向量 , 对通常的向量加法和如下定义的数量乘法
k・α= 0
其中 k∈R ,α为任意的平面向量 ,0 为零向量 .
( 3 ) 全体正实数R + , 加法与数量乘法定义为
a b= ab,     k・ a = ak ,
其中 a, b∈ R+ , k∈ R .
解   检验一个非空集合 V 在数域 F 上 对定义 的加法 和数量 乘法 ( 简 称
数乘) 是否构成一个线性 空间 , 需要检 查 : 集合对 两种运 算是否 封闭 以及 两
种运算是否满足定义中指出的 8 条运 算规则 .只要有一 点不满 足 , V 在 F 上
就不构成线性空间 .
( 1) 两个正交矩阵相加不一 定是正 交矩阵 ( 例如 , 单位 矩阵 I 是 正交 矩
阵 ,但 I + I = 2 I 不是正 交矩阵 ) ; 数 k 与正 交矩阵 A 相 乘 , 当 k≠1 时 , kA 也
不是正交矩阵 .所以全体 正交 矩阵 的集 合对矩 阵的 加法 和数 乘运 算 都不 封
闭 ,因此 ,它不构成一个线性空间 .
( 2 ) 平面上全体向量 , 对定义的数量乘法 k・α= 0,不满足 8 条规则中的
第 5 条 , 即 " α,
1・α= 0≠α.
所以 ,此时平面上的全体向量也不构成线性空间 .
( 3 ) 这里R+ 对定义的加法和数乘在实数域R 上构成一个线性空间 .因为 :

" a, b∈ R + , 和 " k∈ R ,
a b = ab∈R + ;   k・ a = ak ∈ R + .

即R + 对定义的加法和数乘运算是封闭的 .而且两种运算也满 足 8 条规则 , 即


" a, b, c∈R + , k, l∈R , 有
① a b = ab = ba = b a;
② (a b) c = ab c = abc = a bc = a (b c) ;
③ R + 中的 1 为加法零元 θ, 它对 于任意 的 a∈ R + , 均 有 a θ= a 1=
a1 = a;
1
④ " a∈ R + , 它的加法 负元为 ∈R + , 即
a
194 第 4 章 向量空间与线性变换

1 1
a = a = 1 = θ;
a a
⑤ 1・ a = a1 = a;
⑥ k・ ( l・ a) = k・ al = ( al ) k = akl = ( kl)・ a;
⑦ ( k + l)・ a = ak + l = ak a l = ak al = k・ a l・ a;
注意 : 这里“ + ”是实数域R 中的数的加法 , 而“ ”是R + 中定义的加法 .
⑧ k・ ( a b) = k・ ( ab) = ( ab) k = ak bk
= ak bk = k・ a k・ b .
2 ( 习题 18)   全体复数C 在实数域 R 上 和在复数域 C 上 , 对通常的 数的
加法和数乘运算是否都构成线性空间 ? 如构成线性空间 , 其维数是多少 ? 并
给出一组基 .
解   全体复数C 在实数域R 上 (记作C ( R ) ) 和全体复数C 在 复数域C 上
(记作C ( C ) ) , 它们的加法都是复数的加法 , 而其数乘则有区别 :
在C ( R ) 上的数乘为 kz(其中 k∈R , z∈C ) ;
在C (C ) 上的数乘为 kz( 其中 k∈C , z∈C ) .
容易验证 , C ( R ) 和 C ( C ) 对复 数的加 法和 数乘 运算 都封 闭 , 而 且两 种
运算满足定义中规定的 8 条规则 , 所以它们都构成线性空间 .
但是它们的维数是不同的 .
dimC ( R ) = 2 , 因为 C ( R ) 中 数 乘 kz 的 数 k 为 实 数 , 所 以 " a + bi ∈ C
(其中 a , b∈R ) , 它是 a 与 1 相乘加上 b与 i 相乘 , 即它是 1 与 i 的线性组合 ,
而 1 与 i 是线性无关的 .因此{1 , i}是C ( R ) 的一组基 .
dimC ( C ) = 1 , 因为C (C ) 中数乘 k z 的 数 k 为复 数 .所以 " a + bi∈C 均
可视为 a + bi 与 1 相 乘 .因此 , {1}是 C ( C ) 的 一 个 基 .当 然 , {2} 也 可 以 是
C ( C )的基 , 此时 , " a + bi∈C
a + bi a + bi
a + bi = k(2 ) = ・2   即 k= ∈C .
2 2
3 ( 关于线性空间的子空 间 )   设 V 是数 域 F 上的一 个线性 空间 , W 是
V 的一个子集合 , 如何判断 W 是否是域 F 上的一个线性子空间 ?
根据定理 4 .9 (主教材 p178) “
, W 是 V 的一个子空间的充要条件是 W 关
于 V 中的两种运算( 加法与 数量乘 法 ) 封闭”.因 此判断 W 是否 是 V 的子 空
间 , 只要判断 W 关于 V 中的两种运算是否封闭 .例如 :

4 .4  部分疑难习题和补充题的题解 195

( 1) 以 m× n 实矩阵 A 为系数矩阵的齐次线性方程组 Ax = 0 的解集合


S0 = { x| Ax= 0}
是R n的一个子空间 .因为 Ax=0的解是 n 维实向量 ,而且 "α,β∈ S0 与 " k∈R ,均有
A(α+ β) = Aα+ Aβ= 0 + 0 = 0, 即 α+ β∈ S0 ,
A( k α) = kAα= k0 = 0, 即 k α∈ S0 .
所以解集合 S0 关于向量的加法 和数乘 运算封 闭 .因 此 , S0 是 Rn 的一 个子 空
间 , 也称 Ax= 0 的解空间 .Ax = 0 的基础解系就是它的解空间的基 .
而非齐次线性方程组 Ax= b 的解集合
S = { x| Ax = b}
不是Rn 的一个子空间 .因为 " α,β∈ S ,
A(α+ β) = Aα+ Aβ= b+ b= 2 b≠ b,
即 α+ βú S, 所以 S 关于向量加法不封闭 , 因此 S 不是 Rn 的一个子空间 .
( 2) 下列R 3 的子集合 , 哪些是R3 的子空间 ? 并求一组基 .
① W1 = { ( x, y , z) | x - 2 y + 3 z = 0 } ;
② W2 = { ( x, y , z) | x - 2 y + 3 z = 1 } ;
③ W3 = { ( 1 , 2 , z) | z∈R } ;

x y z
④ W4 = ( x , y, z) | = = ;
2 - 3 4

x - 1 y+ 1 z - 2
⑤ W5 = ( x , y, z) | = = .
2 -3 4

解   W1 是R3 的子空间 , 因为 W 1 是齐次线性方程 x - 2 y + 3 z = 0 的 解集


合 .它的基础解系( 即解空间的基) 为 :α= (2 , 1 , 0) T ,β= ( - 3 , 0 , 1 ) T .
W1 的几何意义是空间直角坐标系中过原点的平面 x - 2 y + 3 z = 0 上的
全体向量 ( x, y , z) .显然 , 它关于向量的加 法和数乘运 算是封 闭的 , 即这个 平
面上任意两个向量相加仍在这个平面上 , 任意向量乘常数 k 也在这个平 面上
(注意向量的起点都在原点 , 终点为 ( x, y , z) ) .
W2 不是R3 的子空 间 , 它是 非齐 次 线性 方程 x - 2 y + 3 z = 1 的 解 集合 .
W3 的几何意义是平面 x - 2 y + 3 z = 1 上的全体向量 ( 起点在原点 , 终点坐标
为( x , y, z) ) , 显然这个平面上的向量关于 向量的加法 和数乘 都不封 闭 , 例 如
( x , y, z)在平面上 , 数乘 2( x, y , z) = (2 x, 2 y, 2 z)就不在平面上 .
196 第 4 章 向量空间与线性变换

W3 = {(1 ,2 , z) | z∈R }也不是R 3 的子空间 ,它是过点(1 ,2 ,0)且平行于 z 坐标


轴的直线上的全体向量 .从几 何上易见 , 它关于 向量加法和 数乘均不封 闭 .如
(1 , 2 ,3)∈W3 ,而 k≠1 时 , k(1 ,2 , 3) = ( k,2 k,3 k)ú W3 (因为 k≠1 ,2 k≠2) .
W4 是 R3 的一个子空间 , 它是齐次 线性方程组 3 x + 2 y = 0 , 2 x + z = 0 的
T
1 3
解集合 , 这个解空间的基为 α= - , ,1 .它的几何意义是过原点的直
2 4
x y z
线 = = 上的全体向量 , 显然 , 它关于向量加法和数乘运算是封闭的 .
2 - 3 4
W5 不是R3 的一个子空间 , 它是非齐次线性方程组

- 3 ( x - 1 ) = 2( y + 1) , 3 x + 2 y = 1,
 即 
4 ( x - 1 ) = 2 ( z - 2) . 2x - z= 0 .
x- 1 x+1 z - 2
的解集合 .它的几何意义 是不过 原点的 直线 = = 上的 全体 向
2 - 3 4
量 .它关于向量加法和数乘向量运算不封闭 .
( 3) 在数域 F 上的线性空间 V 中 , 一个向 量组 {α1 ,α2 , … ,αr } 的所有 线
性组合构成的集合 W , 是 V 的一个子空间 , 称为由 α1 ,α2 , … ,αr 生 成的子 空
间 , 记作 W = L(α1 ,α2 ,… ,αr ) , 即
L(α1 ,α2 ,…,αr ) = ξ|ξ= k1 α1 + k2 α2 + … + krαr , k1 ,… , kr ∈ F .
如果向量组 {α1 ,α2 ,… ,αr } 的一个极大线性无关组为
αi1 ,αi2 , … ,αi k   ( 1≤ i1 < i2 < … < ik ≤ r) ,
则 W = L(α1 ,α2 ,… ,αr ) = L (αi1 ,αi2 , … ,αi k ) , 并称 αi1 ,αi2 , … ,αi k 为子 空
间 W 的一组基 , dim W = k, 即 W 为 V 的一个 k 维子空间 .
用施密特正交化方法 , 由 W 的 基 αi1 ,αi2 , … ,αi k 可以 求得 W 的一 组
标准正交基 {ε1 ,ε2 ,… ,εk } .
4 ( 习题 24)   在R4 中 , 求向量组 {α1 ,α2 ,α3 ,α4 } 生成的 子空间 的基和 维
数 ,并求子空间的一组标 准正 交基 .其中 :α1 = ( 2 , 1 , 3 , 1 ) , α2 = ( 1 , 2 , 0 , 1 ) ,
α3 = ( - 1 , 1 , - 3 , 0) ,α4 = (1 , 1 , 1 , 1) .
解   法 1 : 利用第 3 章中求 极大线性 无关组 的方法 .可求 得 {α1 ,α2 ,α3 ,
α4 } 的一个极大线性无关组为{ α1 ,α2 ,α4 } .所以 dim L( α1 ,α2 ,α3 ,α4 ) = 3 .再利
用施密特正交化方法由 {α1 ,α2 ,α4 } 求得子空间的一组标准正交基{ε1 ,ε2 ,ε3 } .
法 2 : 直接对{ α1 ,α2 ,α3 ,α4 } 来用施密 特正交化方 法求 L(α1 ,α2 ,α3 ,α4 )

4 .4  部分疑难习题和补充题的题解 197

的一组标准正交基 .先正交化 , 令
    β1 = α1 = (2 , 1 , 3 , 1) ,
(α2 ,β1 ) 1
    β2 = α2 - β1 = ( 1 , 2 , 0 , 1) - (2 , 1 , 3 , 1 )
(β1 ,β1 ) 3
1 5 2
= , , - 1, ,
3 3 3
(α3 ,β1 ) (α3 ,β2 )
    β3 = α3 - β1 - β2
(β1 ,β1 ) (β2 ,β2 )
- 10 1 5 2
= ( - 1, 1, - 3 ,0 ) - (2 , 1 , 3 , 1 ) - , , - 1,
15 3 3 3
= (0 , 0 , 0 , 0 ) .
这里求得的 β3 = 0, 表示 α3 可由 α1 ,α2 线 性 表示 , 即 α1 ,α2 ,α3 是 线 性相 关
的 .继续作正交化 ,求 β4 , 使 β4 与 β1 ,β2 都正交 .得
( α4 ,β1 ) (α4 ,β2 )
      β4 = α4 - β1 - β2
(β1 ,β1 ) (β2 ,β2 )
7 5 1 5 2
= ( 1, 1,1 ,1 ) - (2 , 1 , 3 , 1) - , , - 1,
15 13 3 3 3
1
= ( - 4 , - 7 , - 1 , 18 ) .
65
再单位化 , 得
1 1
ε1 = β1 = ( 2 ,1 ,3, 1 ) ,
‖β1 ‖ 15

1 3 1 5 2 1
ε2 = β= , , - 1, = ( 1 , 5 , - 3 , 2) ,
‖β2 ‖ 2 13 3 3 3 39
1 65 - 4 - 7 - 1 18 1
ε4 = β= , , , = ( - 4 , - 7 , - 1 , 18) .
‖β4 ‖ 4 390 65 65 65 65 390
所以 , dim L(α1 ,α2 ,α3 ,α4 ) = 3, 它的一组标准正交基为 {ε1 ,ε2 ,ε4 } .
5 ( 习题 33 )   设 V 1 = L(α1 ,α2 ,α3 ) , V 2 = L(β1 ,β2 ) , 求 V1 ∩ V2 , V1 + V2
的基和维 数 , 其 中 : α1 = ( 1 , 2 , - 1 , - 2 ) , α2 = ( 3 , 1 , 1 , 1 ) , α3 = ( - 1 , 0 ,
1 , - 1 ) ;β1 = (2 , 6 , - 6 , - 5 ) ,β2 = ( - 1 , 2 , - 7 , 3) .
解   设 ξ∈ V1 ∩ V 2 , 即 ξ∈ V 1 且 ξ∈ V2 , 于是
ξ= x1 α1 + x2 α2 + x3 α3 = - x4β1 - x5β5 , ( 1)

198 第 4 章 向量空间与线性变换

x1 α1 + x2 α2 + x3 α3 + x4 β1 + x5 β5 = 0 . ( 2)
(2) 式等价于下列齐次线性方程组 Ax= 0, 即
x1 x1
1 3 - 1 2 -1 0
x2 x2
2 1 0 6 2 0
  (α1 ,α2 ,α3 ,β1 ,β2 ) x3 = x3 = . ( 3)
- 1 1 1 -6 -7 0
x4 x4
- 2 1 - 1 -5 3 0
x5 x5
对方程组 ( 3) 中系数矩阵 A 做初等行变换 , 将其化为阶梯形矩阵 U,即
1 3 -1 2 -1 1 0 0 0 - 12
0 1 0 -1 -2 0 1 0 0 2
A→ → =U . ( 4)
0 0 1 -3 -9 0 0 1 0 3
0 0 0 1 4 0 0 0 1 4
由 ( 4 ) 式得 Ax= 0 的一般解为
x= ( x1 , x2 , x3 , x4 , x5 ) T = k( 12 , - 2 , - 3 , - 4 , 1) T . ( 5)
将( 5) 式中的 x4 , x5 代 入 ( 1) 式 右端 ( 或将 x1 , x2 , x3 代 入 ( 1) 式 左端 ) , 即 得
V 1 ∩ V 2 中全部向量 , 即
V 1 ∩ V 2 = {ξ|ξ= k( 4β1 - β5 ) , k 为任意常数 }
= {ξ|ξ= k( 9 , 22 , - 17 , - 23) , k 为任意常数 } .
所以 , dim ( V1 ∩ V2 ) = 1 .它的基向量为 η= ( 9 ,22 , - 17 , - 23 ) .
V 1 + V 2 = L(α1 ,α2 ,α3 ) + L(β1 ,β2 )
= L(α1 ,α2 ,α3 ,β1 ,β2 ) .
由 ( 4 ) 式 可 见, {α1 , α2 , α3 ,β1 ,β2 } 的 一 个 极 大 线 性 无 关 组 为 {α1 , α2 ,α3 ,
β1 } , 所以
dim ( V1 + V2 ) = 4 ,   即   V 1 + V 2 = R4 ,
{α1 ,α2 ,α3 ,β1 } 是它的一组基 .当然 ,既然 V1 + V 2 = R4 , 那么 , R4 的 任何基 ( 例
如自然基 ε1 ,ε2 ,ε3 ,ε4 ) 也都是 V 1 + V2 的基 .
1 0 0
- 1 + 3i
6 ( 习题 30)   设 A = 0 ω 0 , 其中 ω= .
2
0 0 ω2
( 1) 证明 A 的全体实系数多项式 ,对于矩阵多项式的加法和数量乘 法构
成实数域上的线性空间 .

4 .4  部分疑难习题和补充题的题解 199

( 2 ) 求这个线性空间的维数及一组基 .
解   ( 1 ) A 的全体实系数多项式构成的集合 , 记为
P( A) = { p( A) | p( A) = a0 I + a1 A + … + an An , n∈N , ai ∈ R } .
欲证明 P( A) 关于 矩阵 多项 式的 加法 和数 量乘 法构 成实数 域上 的线 性
空间 ,应按定义 ,证明 P ( A) 关 于两 种 运算 封 闭 , 而 且 满足 两 种 运算 的 8 条
规则 .
设 p1 ( A) , p2 ( A) ∈ P( A) , k∈ R , 且
p1 ( A) = a0 I + a1 A + … + am Am ,
p2 ( A) = b0 I + b1 A + … + bm Am + … + bn An
(其中 m, n 为任意自然数 , I = A0 ) ,则
p1 ( A) + p2 ( A) = ( a0 + b0 ) I + ( a1 + b1 ) A + … +
( am + bm ) Am + … + bn An ∈ P ( A) ,
k p1 ( A) = k a0 I + ka1 A + … + k am Am ∈ P( A) .
所以 , P( A) 关于矩阵多项式的加法和数量乘法的两种运算封闭 .而且两 种运
算显然满足定义中规定的 8 条规则 ( 不详述 ) ,其中 :
第 3 条 ,加法的零元素为 A 的零多项式θ( A) ( 即系数全为 0 的多 项式 ) ,
因此 , " p( A) ,均有 p ( A) + θ( A) = p ( A) .
第 4 条 , p ( A ) 的 负 元 为 - p ( A ) , 于 是 " p ( A) , 均 有 p ( A) +
( - p( A) ) =θ( A) .
因此 , P ( A) 关于上述两种运算构成实数域上的一个线性空间 .
( 2 ) 由于 ω3 = 1 , 即 ω是方程 x3 - 1 = 0 的一个根 , 所以
1 0 0 1 0 0
A2 = 0 ω2 0 ,   A3 = 0 1 0 = I.
0 0 ω 0 0 1
因此 , A 的任意实系数多项式 p ( A) 均可由 I, A, A 线性表示 , 而 I, A, A2 是线 2

性无关的 , 即 ,设
k1 I + k2 A + k3 A2 = 0(零矩阵 ) . ( 1)
(1) 式即为
k1 + k2 + k3 0 0
0 k1 + k2 ω+ k3 ω2 0 = 03× 3 .
0 0 k1 + k2 ω2 + k3 ω
200 第 4 章 向量空间与线性变换

于是
k1 + k2 + k3 = 0 ,
k1 + k2 ω+ k3 ω2 = 0 , ( 2)
k1 + k2 ω2 + k3 ω= 0 .
方程组( 2) 只有零解 , 因为系数系列式
1 1 1
1 ω ω2 = 3ω2 - 3ω= 3 - 1 - 3 i 1 + 3 i ≠0 .
-
2 2
1 ω2 ω
综上 , P ( A) 为三维线性空间 , 它的一组基为 { I, A, A2 } .
7 ( 习题 32 )   设
1 0 0
A= 0 1 0 .
3 2 2
求R 3× 3 中全体与 A 可交换的矩阵所组成的子空间的维数及一组基 .
解   设 B = ( bij ) 3 ×3 与 A 可交换 ,即 AB = BA,于是
1 0 0 b11 b12 b13 b11 b12 b13 1 0 0
0 1 0 b21 b22 b23 = b21 b22 b23 0 1 0 ,
3 2 2 b31 b32 b33 b31 b32 b33 3 2 2

b11 b12 b13
  b21 b22 b23
3 b11 + 2 b21 + 2 b31 3 b12 + 2 b22 + 2 b32 3 b13 + 2 b23 + 2 b33
b11 + 3b13 b12 + 2b13 2b13
= b21 + 3b23 b22 + 2b23 2b23 . ( 1)
b31 + 3b33 b32 + 2b33 2b33
由( 1) 式两端矩阵对应元素相等 , 得
b13 = b23 = 0 ,

- b31 + 3 b33 = 3b11 + 2b21 ,


( 2)
- b32 + 2 b33 = 3b12 + 2b22 ,
其中 b11 , b12 , b21 , b22 可任取 .再由第 3 行第 3 列元素相等 , 得 2 b33 = 2b33 , 这表

4 .4  部分疑难习题和补充题的题解 201

明 b33 也可任取 .如此 , 由方程组( 2) 可解得:


b31 = - 3b11 - 2b21 + 3 b33 ,
b32 = - 3b12 - 2b22 + 2 b33 .
综上 , 与 A 可交换的所有矩阵 B 为
b11 b12 0
B= b21 b22 0 . ( 3)
- 3 b11 - 2 b21 + 3b33 - 3 b12 - 2 b22 + 2b33 b33
(3) 式的 B 可表示为
1 0 0 0 1 0 0 0 0
B = b11 0 0 0 + b12 0 0 0 + b21 1 0 0 +
- 3 0 0 0 - 3 0 - 2 0 0
0 0 0 0 0 0
  b22 0 1 0 + b33 0 0 0 , ( 4)
0 -2 0 3 2 1
其中 : b11 , b12 , b21 , b22 , b33 为任意常数 .
结论 : 与 A 可交换的所有矩阵 B, 是 ( 4 ) 式中 5 个矩阵的所 有线性组合 ,
即是由这 5 个矩阵生成的 R 3× 3 的一 个子空间 .这 5 个矩 阵是线性无 关的 , 因
为设 (4)式等于零矩阵 , 即 (3)式为零矩阵 , 则必有 b11 = b12 = b21 = b22 = b33 = 0 .
因此 , 这个子空间的维数为 5 , 而 (4 )式中的 5 个矩阵就是它的一组基 .
8 ( 习题 28)   设 R [ x] 5 的 旧 基 为 B1 = { 1 , x , x2 , x3 , x4 } ; 新 基 B2 =
{1 , 1 + x , 1 + x + x2 , 1 + x + x2 + x3 , 1 + x + x2 + x3 + x4 } .
( 1) 求由旧基到新基的过渡矩阵 ;
( 2) 求多项式 p( x) = 1 + 2 x + 3 x2 + 4 x3 + 5 x4 在 B2 下的坐标 ;
( 3) 若多项式 f ( x) 在基 B2 下 的坐标 为 ( 1 , 2 , 3 , 4 , 5 ) T , 求 它在基 B1 下
的坐标 .
解   ( 1) 记基 B2 中的 5 个多项式依次 为 p0 ( x ) , p1 ( x) , p2 ( x ) , p3 ( x ) ,
p4 ( x ) .则
p0 ( x) , p1 ( x) , p2 ( x ) , p3 ( x ) , p4 ( x )
1 1 1 1 1
0 1 1 1 1
= 1 , x, x2 , x3 , x4 0 0 1 1 1 . ( 1)
0 0 0 1 1
0 0 0 0 1
202 第 4 章 向量空间与线性变换

(1) 式右端矩阵 A 就是旧基 B1 到新基 B2 的过渡矩阵 .


( 2 ) 设 p ( x ) 在 基 B1 , B2 下 的坐 标 依 次为 { p ( x ) } B1 = ( x1 , x2 , x3 , x4 ,
x5 ) T = (1 , 2 , 3 , 4 , 5) T , { p( x) } B2 = ( y1 , y2 , y3 , y4 , y5 ) T , 则
y1 x1 1 - 1 0 0 0 1 - 1
y2 x2 0 1 - 1 0 0 2 - 1
y3 = A- 1 x3 = 0 0 1 -1 0 3 = - 1 .
y4 x4 0 0 0 1 - 1 4 - 1
y5 x5 0 0 0 0 1 5 5
( 3) 法 1: 也 设 { f ( x ) } B1 = ( x1 , x2 , x3 , x4 , x5 ) T , { f ( x ) } B2 =
( y1 , y2 , y3 , y4 , y5 ) T = (1 , 2 , 3 , 4 , 5) T , 则
x1 1 1 1 1 1 1 1 15
x2 2 0 1 1 1 1 2 14
x3 =A 3 = 0 0 1 1 1 3 = 12 .
x4 4 0 0 0 1 1 4 9
x5 5 0 0 0 0 1 5 5
法 2 : 由{ f ( x) } B2 = (1 , 2 , 3 , 4 , 5 ) T , 得
f ( x ) = 1・ p0 ( x) + 2 p1 ( x) + 3 p2 ( x) + 4 p3 ( x ) + 5 p4 ( x )
= 1 + 2 (1 + x ) + 3 (1 + x + x2 ) + 4( 1 + x + x2 + x3 ) +
5( 1 + x + x2 + x3 + x4 )
= 15 + 14 x + 12 x2 + 9 x3 + 5 x4 .
所以{ f ( x) } B1 = (15 , 14 , 12 , 9 , 5 ) T .
9 (习题 25)   设 α,β,γ∈Rn , c1 , c2 , c3 ∈R , 且 c1 c3 ≠0 , 证明 : 若 c1 α+ c2 β+
c3γ= 0,则 L( α,β) = L(β,γ) .
证   由于子空间是向量的集合 , 因 此要证 明两个 子空间 相等 , 必须证 明
它们互相包含 .
先证 : L(α,β) L(β,γ) .
设 ξ∈ L(α,β) 即 ξ= k1 α+ k2β, 由于
c1 α+ c2 β+ c3γ= 0   ( c1 ≠0) ,
c2 c3
所以 α= - β- γ,因此
c1 c1

4 .4  部分疑难习题和补充题的题解 203

c2 c3
ξ= - k1 + k2 β- k1γ∈ L(β,γ) ,
c1 c1
故 L(α,β) L(β,γ) ,
再证 : L(β,γ) L(α,β) .
设 η∈ L(β,γ) ,即 η= k1β+ k2 γ, 再由 c1 α+ c2β+ c3 γ= 0( c3 ≠0) , 得
c1 c2
γ= - α- β,
c3 c3
c1 c2
因此 η= - k2 α+ k1 - k β∈ L(α,β) ,
c3 c3 2
故 L(β,γ) L(α,β) .
综上 , 即得 L(α,β) = L(β,γ) .
10 ( 习题 27 )   设 {α1 ,α2 ,… ,αn } 是 n 维线性空间 V 的一组基 , 又 V 中向
量 αn+ 1 在这组基下的坐标 ( x1 , x2 , … , x n ) 全 不为零 .证明 α1 ,α2 , … ,αn , αn + 1
中任 意 n 个 向 量 必 构 成 V 的 一 组 基 , 并 求 α1 在 基 {α2 , … , αn , αn+ 1 } 下
的坐标 .
证   先证明 α1 ,… ,αi - 1 ,αi + 1 , … ,αn ,αn + 1 (1≤ i≤ n) 线性无关 ( 即是 V 的
一组基) .设
k1α1 + … + ki - 1αi - 1 + ki + 1 αi + 1 + … + knαn + kn + 1 αn+ 1 = 0, ( 1)
- 1
则 kn + 1 = 0 (否则 αn + 1 = ( k1 α1 + … + ki - 1 αi - 1 + 0・αi + ki + 1 αi + 1 + … +
kn+ 1
knαn ) , 这与 αn + 1 在基 {α1 ,… ,αn } 下的坐标全不为零矛 盾 ) , 从而 ( 1) 式中 的其
余系数也必须全为零 (因为 α1 ,… ,αi - 1 ,αi + 1 , … ,αn 线性无关) .故 α1 ,α2 ,… ,
αn ,αn + 1 中任何 n 个向量都是 V 的一组基 .
再由 αn+ 1 = x1 α1 + x2 α2 + … + xnαn ( x i ≠0 , i = 1 , 2 , … , n) , 得
1
α1 = - ( x2 α2 + … + x nαn ) + αn+ 1 ,
x1
x2 xn
所以 α1 在基 {α2 , …,αn ,αn+ 1 } 下的坐标为 - ,… , - ,1 .
x1 x1
1 1 - 1 0 1
2 3 1 -1 0
11 ( 习题 36 )   设 A = .
0 1 3 -1 - 2
4 1 - 13 3 10
204 第 4 章 向量空间与线性变换

( 1 ) 求矩阵 A 的列空间和行空间的基和维数;
( 2 ) 求矩阵 A 的零空间的基和维数 ;
( 3 ) 求 A 的行空间的正交补的维数 .
解   ( 1 ) 矩阵 A 的列空间 R ( A) 和行空间 R( AT ) , 分 别是 A 的列向 量组
{β1 ,β2 ,β3 ,β4 ,β5 } 和行向量组 {α1 ,α2 ,α3 ,α4 } 生成的子空间 .所以 A 列 ( 行 ) 向
量组的极大线性无关组就是 A 的列 ( 行 ) 空间的基 .对 A 做初等 行变换 , 将其
化为阶梯形矩阵 U,即
1 0 - 4 1 3
初等 0 1 3 - 1 - 2
A =U. ( 1)
行 变换 0 0 0 0 0
0 0 0 0 0
由此可见 , 秩 ( A) = 秩 ( U) = 2 .所以 , dim R ( A) = dim R ( AT ) = 2 .由于 A 的 列
向量与 U 的对应的列向量有相同的线性相关性 ,所以 A 的第 1, 2 列 β1 ,β2 是
A 的列向量组的极大线性无关组 , 也就是 A 的列空 间 R ( A) 的 基 .事 实上 , 由
于秩 ( A) = 2 ,而 A 的任意两 个列向 量都 是不 成比 例的 非零 向量 , 所 以 , A 的
任意两个列向量都是 R ( A) 的基 .同理 , A 的任意两个行向量也都是 A 的行空
间 R( AT ) 的基 .
( 2) A 的零空间 N ( A) 就是齐次线性方程组 Ax= 0 的解空间 .由 ( 1 ) 易得
Ax= 0 的基础解系 ,即 N ( A) 的基为 { x1 , x2 , x3 } .其中
x1 = ( 4 , - 3 , 1 , 0 , 0) T ,   x2 = ( - 1 , 1 , 0 , 1 , 0) T ,   x3 = ( - 3 , 2 , 0 , 0 , 1 ) T .
dim N ( A) = 3 .
( 3 ) A 的行空间的正交补 R ( AT ) ⊥ 就是 A 的零空间 , 于是 dim R( AT ) ⊥ =
dim N ( A) = 3 .因为 : Ax = 0 的 解 向量 与 A 的 行 向量 都 正 交 , 所 以 N ( A) ⊥
R( AT ) , 而且秩 ( A) + dim N ( A) = n = 5 , 即 dim R ( AT ) + dim N ( A) = 5 , 于 是
R( AT ) N( A) = Rn .
12 ( 习题 37 )   在 R3 中 , 下列子空间哪些 是正交 子空间 ? 哪些 互为正 交
补 ? 并说明理由 .
( 1) W1 = { ( x , y, z) | 3 x - y + 2 z = 0} ;
( 2) W2 = { ( x , y, z) | x - y - 2 z = 0 } ;

x y z
( 3) W3 = ( x , y , z) | = = ;
3 - 1 2

4 .4  部分疑难习题和补充题的题解 205

x y z
( 4) W4 = ( x , y , z) | = = .
3 5 - 2

解   W1 ⊥ W3 , 且互为正交补 , 其理由有两种说法 :
一是 : W1 与 W3 分别是过原点 的平面 3 x - y + 2 z = 0 与过 原点 的直 线
x y z
= = 上的全体向量 , 而平面的法向量 n1 = ( 3 , - 1 , 2 ) = s3 ( 直线 的方
3 - 1 2
向向量 ) ,即直线垂直 于平 面 ,所 以直 线上 的向 量与 平面 上的 向量 都 互相 垂
直 ,即 W1 ⊥ W3 ; 而且 dim W1 + dim W3 = 2 + 1 = 3 . W1 ∩ W3 = { 0 } , 即 W1
W3 = R3 .因此 , W1 与 W3 互为正交补 .
二是 : W1 是齐次线性方程 3 x - y + 2 z = 0 的解空间 , 它与该方程的系数
矩阵( 3 , - 1 , 2 )的 行 空间 L( 3 , - 1 , 2 ) 互 为正 交 补 ; W3 是齐 次 线 性方 程 组
x y z
= = 的解空间 , 它的基向量为( 3 , - 1 , 2 ) , 它的全部解为 k(3 , - 1 , 2)
3 - 1 2
( k 为任意常数 ) , 所以 W3 = L( 3 , - 1 , 2) 就是 前者的行 空间 , 因此 , W1 与 W3
互为正交补 .
这两种说法 , 前者是从 R3 中子空间的几何 意义上 而言 , 后者是 从齐次 线
性方程的解空间来阐述 .
W3 与 W4 是正交子空间 , 即 W3 ⊥ W4 , 但不 是互 为正 交补 .从几 何意 义
上说 , W3 与 W4 分别是过原点的两条直线上的全体向量 , 而两条直线的 方向
向量 s3 = ( 3 , - 1 , 2 )与 s4 = (3 , 5 , - 2) 是正交的( 因为 ( s3 , s4 ) = 9 - 5 - 4 = 0 ) ,
所以 W3 ⊥ W4 , 但是 dim W3 + dim W4 = 2 , 即 W3 W4 ≠ R3 , 因此 , 它们不 是互
为正交补 .
但要注意 : W1 与 W2 不是正交子空间 .虽然两个过原点的平面 3 x - y +
2 z = 0 与 x - y - 2 z = 0 是垂直的 ( 因为它们的法向量 n1 = (3 , - 1 , 2) 与 n2 =
(1 , - 1 , - 2 )是垂直的 ) , 但这 两个 平面 的交 线上 的全 体向 量 k( 2 , 4 , - 1) ∈
W1 ∩ W2 ( k 为任意 常数 ) , 而 k( 2 , 4 , - 1 ) 与其 自身 是不 正 交的 .所 以 W1 与
W2 不是正交子空间 .
13 ( 习题 38 , 40 )   设 α= ( x1 , x2 , x3 ) ∈R3 , 证明 :
σ(α) = ( x1 , x2 , - x3 )
是线性变换 , 并分别求它在自然基 B1 = {ε1 ,ε2 ,ε3 }和基 B2 = {α1 ,α2 ,α3 } 下的
206 第 4 章 向量空间与线性变换

对应矩阵 .其中 : α1 = (1 , 0 , 0 ) ,α2 = ( - 1 , 1 , 0) ,α3 = (1 , - 1 , 1 ) .


解   证明 σ(α) 为 R 3 的 一 个线 性变 换 , 按 定义 , 需要 证明 : " α,β∈ R 3 ,
k∈R , 恒有
σ( α+ β) = σ(α) + σ(β) ;  σ( kα) = kσ(α) .
设 α= ( x1 , x2 , x3 ) ,   β= ( y1 , y2 , y3 )∈R3 ,   k∈ R , 则
      σ(α+ β) = σ( x1 + y1 , x2 + y2 , x3 + y3 )
= ( x1 + y1 , x2 + y2 , - x3 - y3 )
= ( x1 + x2 - x3 ) + ( y1 + y2 - y3 ) =σ(α) +σ(β) , ( 1)
      σ( k α) = σ( k x1 , k x2 , k x3 )
= ( k x1 , k x2 , - k x3 ) = k( x1 , x2 , - x3 ) = kσ(α) . ( 2)
3
由 ( 1 ) , ( 2 ) 式可见 ,σ(α) = ( x1 , x2 , - x3 ) 是R 的一个线性变换 .
如果 α= ( x1 , x2 , x3 ) 是 α在自然基 {ε1 ,ε2 ,ε3 }下的坐标 ,则
σ( x1 , x2 , x3 ) = ( x1 , x2 , - x3 )
的几何意义是 : σ把向量 α= ( x1 , x2 , x3 ) 变 换为与 ε1 ,ε2 所 确定的 平面相 对
称的向量 ξ= ( x1 , x2 , - x3 ) , 所 以 σ是关 于 {ε1 ,ε2 } 平面的 镜像变 换 ( 或称 镜
面反射 ) .
根据线性变换 σ在一组基 B = {β1 ,β2 ,β3 } 下对应的矩阵的定义
σ(β1 ,β2 ,β3 ) = σ(β1 ) ,σ(β2 ) ,σ(β3 )
a11 a12 a13
= (β1 ,β2 ,β3 ) a21 a22 a23 . ( 3)
a31 a32 a33
(3) 中右端矩阵 A = ( aij )3 × 3 称为 σ在基 B 下 对应的矩阵 .求 A 的 关键是 求σ
关于基 B 的象σ(β1 ) ,σ(β2 ) ,σ(β3 ) ,并将它们表示为基 B 的线性组合 .
先求 σ关于自然基 B 1 = {ε1 ,ε2 ,ε3 } 对应的矩阵 A,由
σ(ε1 ) =σ(1 , 0 , 0) = (1 , 0 , 0 ) = ε1 ,
σ(ε2 ) =σ(0 , 1 , 0) = (0 , 1 , 0 ) = ε2 ,
σ(ε3 ) =σ(0 , 0 , 1) = (0 , 0 , - 1 ) = - ε3 ,
即得
1 0 0
σ(ε1 ,ε2 ,ε3 ) = (ε1 ,ε2 ,ε3 ) 0 1 0 . ( 4)
0 0 -1

4 .4  部分疑难习题和补充题的题解 207

( 4 ) 式右端的矩阵即为所求的矩阵 A .
再求 σ关于基 B 2 = {α1 ,α2 ,α3 } 对应的矩阵 B, 由
σ(α1 ) = σ( 1 , 0 , 0 ) = (1 , 0 , 0) = α1 ,
σ(α2 ) = σ( - 1 , 1 , 0) = ( - 1 , 1 , 0) = α2 , ( 5)
σ(α3 ) = σ( 1 , - 1 , 1) = (1 , - 1 , - 1) = ξ.
这里的关键是要把 ξ表示为 α1 ,α2 ,α3 的线性组合 ,此时容易看出
      ξ = ( 1 , - 1 , - 1 ) = - (1 , - 1 , 1) + ( 2 , - 2 , 0 )
= - 2 ( - 1 , 1 , 0) - ( 1 , - 1 , 1) = - 2α2 - α3 . ( 6)
将 ( 6 ) 式代入 ( 5 ) 式中第 3 式子 ,即得
1 0 0
σ(α1 ,α2 ,α3 ) = (α1 ,α2 ,α3 ) 0 1 - 2 . ( 7)
0 0 - 1
( 7 ) 式右端的矩阵即为所求的矩阵 B .求 B 的一般方法为 :
1 - 1 1
σ(α1 ,α2 ,α3 ) = 0 1 - 1 = (α1 ,α2 ,α3 ) B . ( 8)
0 0 - 1
将 α1 ,α2 ,α3 按列向量代入 ( 8 ) 式 ,即得
- 1
1 - 1 1 1 -1 1
B= 0 1 - 1 0 1 -1
0 0 1 0 0 -1
1 1 0 1 - 1 1 1 0 0
= 0 1 1 0 1 - 1 = 0 1 -2 . ( 9)
0 0 1 0 0 - 1 0 0 -1
( 9 ) 式的结果与 ( 7 ) 式中的 B 是一样的 .
14 ( 习题 42 )   设 B1 = {α1 ,α2 ,α3 }和 B2 = {β1 ,β2 ,β3 }是 R3 的两组基 , 已
知 β1 = 2α1 + α2 + 3α3 ,β2 = α1 + α2 + 2α3 ,β3 = - α1 + α2 + α3 ;σ在 基 B 1下 的对
应矩阵为
5 7 - 5
A= 0 4 - 1 .
2 8 3
208 第 4 章 向量空间与线性变换

试求 : 1 ) σ在基 B3 = { - α2 ,2α1 ,α3 } 下的对应矩阵 ;


( 2 ) σ在基 B2 下的对应矩阵 B .
解   ( 1 ) 已知
5 7 - 5
σ(α1 ,α2 ,α3 ) = (α1 ,α2 ,α3 ) 0 4 - 1 . ( 1)
2 8 3
现在要将 σ( - α2 ) ,σ(2α1 ) ,σ( α3 ) 分别 用基 B3 线 性表示 , 然 后即可 求得 σ在
基 B 3 下的对应矩阵 .由 (1 )式易得 :
          σ( - α2 ) = - σ(α2 ) = - 7α1 - 4α2 - 8α3
7
= 4 ( - α2 ) - (2α1 ) - 8α3 , ( 2)
2
          σ( 2α1 ) = 2σ(α1 ) = 10α1 + 4α3
= 0( - α2 ) + 5 ( 2α1 ) + 4α3 , ( 3)
          σ(α3 ) = - 5α1 - α2 + 3α3
5
= 1( - α2 ) - (2α1 ) + 3α3 . ( 4)
2
于是由 ( 2 ) , ( 3 ) , ( 4 ) 式即得
4  0 1 
7 5
σ( - α2 , 2α1 ,α3 ) = ( - α2 ,2α1 ,α3 ) - 5 - . ( 5)
2 2
- 8  4 3 
( 5 ) 式右端矩阵就是 σ在基 B 3 = { - α2 ,2α1 ,α3 } 下的对应矩阵 .
( 2 ) σ在基 B1 , B2 下的对应矩阵 A, B 间的关系为
B = C - 1 AC, ( 6)
其中 C 为基 B1 到基 B2 的过渡矩阵 , 即
(β1 ,β2 ,β3 ) = (α1 ,α2 ,α3 ) C .
由已知条件易得
2 1 -1
(β1 ,β2 ,β3 ) = (α1 ,α2 ,α3 ) 1 1 1 . ( 7)
3 2 1
( 7 ) 式右端矩阵即为过渡矩阵 C,其逆矩阵为

4 .4  部分疑难习题和补充题的题解 209

- 1 - 3 2
C- 1 = 2 5 -3 .
- 1 - 1 1
于是
- 1 -3 2 5 7 - 5 2 1 - 1
B = 2 5 -3 0 4 - 1 1 1 1
- 1 -1 1 2 8 3 3 2 1
37 24 12
= - 54 - 34 - 18 .
18 12 9
15 ( 习题 43 )   已知R 的线 性变换 对于基 α1 = ( - 1 , 0 , 2 ) T , α2 = (0 , 1 ,
3

1) T ,α3 = (3 , - 1 , - 6) T 的象为
σ(α1 ) = β1 = ( - 1 , 0 , 1) T ,   σ( α2 ) = β2 = (0 , - 1 , 2) T ,
σ(α3 ) = β3 = ( - 1 , - 1 , 3) T .
( 1) 求 σ在基{ α1 ,α2 ,α3 } 下的矩阵表示 ( 即对应矩阵 ) ;
( 2 ) 求 σ(β1 ) ,σ(β2 ) ,σ(β3 ) ;
( 3 ) α在基 {α1 ,α2 ,α3 } 下的 坐标 向量 为 ( 5, 1 , 1 ) T , 求 σ(α) 在 基 {α1 ,α2 ,
α3 } 下的坐标向量 ;
( 4 ) β= (1 , 1 , 1) T , 求 σ(β) ;
( 5 ) σ(γ) 在基 {α1 ,α2 ,α3 } 下的坐标向量 为 ( 2, - 4 , - 2 ) T , 问 : 原 象 γ是
否惟一 ? 如不惟一 ,求所有的原象 γ.
解   ( 1 ) 由已知条件, 得
σ(α1 ,α2 ,α3 ) = (β1 ,β2 ,β3 )
- 1 0 - 1
= 0 - 1 - 1 = (α1 ,α2 ,α3 ) A . ( 1)
1 2 3
( 1 ) 式中的 A 即为σ在基 {α1 ,α2 ,α3 } 下的对应矩阵 .将 α1 ,α2 ,α3 代入 ( 1 ) 式 ,
即得
- 1
- 1 0 3 - 1 0 - 1
A= 0 1 - 1 0 -1 - 1
2 1 - 6 1 2 3
210 第 4 章 向量空间与线性变换

5 - 3 3 -1 0 - 1 - 2 9 7
= 2 0 1 0 - 1 - 1 = - 1 2 1 .
2 - 1 1 1 2 3 - 1 3 2
( 2) σ(β1 ) ,σ(β2 ) ,σ(β3 ) = σ(β1 ,β2 ,β3 )
= σ σ(α1 ,α2 ,α3 ) =σ (α1 ,α2 ,α3 ) A
= σ(α1 ,α2 ,α3 ) A= (α1 ,α2 ,α3 ) A A
2
= (α1 ,α2 ,α3 ) A
2
- 1 0 3 - 2 9 7
= 0 1 - 1 - 1 2 1
2 1 - 6 - 1 3 2
-1 0 3 - 12 21 9 3 - 12 -9
= 0 1 -1 -1 -2 -3 = 2 -5 -3 ,
2 1 -6 -3 3 0 -7 22 15
所以 ,σ(β1 ) = ( 3 , 2 , - 7) T ,σ(β2 ) = ( - 12 , - 5 , 22) T ,σ(β3 ) = ( - 9 , - 3 , 15 ) T .
( 3) 已知
5
α= 5α1 + α2 + α3 = (α1 ,α2 ,α3 ) 1 ,
1
所以
5 5
σ(α) = σ( α1 ,α2 ,α3 ) 1 = (α1 ,α2 ,α3 ) A 1
1 1
因此 ,σ(α) 在基 {α1 ,α2 ,α3 } 下的坐标向量为
5 - 2 9 7 5 6
A 1 = - 1 2 1 1 = - 2 .
1 - 1 3 2 1 0
( 4 ) 先求 β在基 {α1 ,α2 ,α3 } 下的坐标向量, 设
β= x1 α1 + x2 α2 + x3 α3 ,

x1 -1 0 3 x1 1
(α1 ,α2 ,α3 ) x2 = 0 1 - 1 x2 = 1 . ( 2)
x3 2 1 - 6 x3 1

4 .4  部分疑难习题和补充题的题解 211

解方程组 ( 2) , 得 ( x1 , x2 , x3 ) T = (5 , 3 , 2) T , 于是
σ(β)σ(α1 ,α2 ,α3 ) ( x1 , x2 , x3 ) T = (α1 ,α2 ,α3 ) A( x1 , x2 , x3 ) T
- 1 0 3 - 2 9 7 5
= 0 1 - 1 - 1 2 1 3
2 1 - 6 - 1 3 2 2
- 1 0 - 1 5 - 7
= 0 -1 - 1 3 = - 5 .
1 2 3 2 17
( 5) 设 γ在基 {α1 ,α2 ,α3 } 下 的坐 标向 量为 x = ( x1 , x2 , x3 ) T ,σ(γ) 在 基
{α1 ,α2 ,α3 } 下的坐标向量为 y= ( y1 , y2 , y3 ) T = (2 , - 4 , - 2 ) T , 则 Ax = y, 即
- 2 9 7 x1 2
- 1 2 1 x2 = -4 . ( 3)
- 1 3 2 x3 -2
求解非齐次线性方程组 ( 3 ) ,得其全部解为
( x1 , x2 , x3 ) T = (8 , 2 , 0) T + k( - 1 , - 1 , 1) T
= (8 - k, 2 - k, k) T , k 为任意常数 .
所以原象 γ不惟一 , 所有的原象 γ为
γ = x1 α1 + x2 α2 + x3 α3
= ( 8 - k) ( - 1 , 0 , 2 ) T + (2 - k) ( 0 , 1 , 1 ) T + k(3 , - 1 , - 6) T
= ( 4 k - 8 , 2 - 2 k, 18 - 9 k) T , k 为任意常数 .
16 ( 习题 46 )   在 R n× n 中定义线性变换
σ( X) = BXC,
其中

a b
B= C= .
c d

求 σ( X) 在基 { E11 , E12 , E21 , E22 } 下的对应矩阵 .其中 Ei j 为 i 行 j 列元素为 1 ,


其余元素全为 0 的二阶矩阵 .
解   这里先要求出 σ关于基的象 , 并将它表示为基的线性组合 .即

a b 1 0 a b a2 ab
      σ( E11 ) = =
c d 0 0 c d ac bc
212 第 4 章 向量空间与线性变换

= a2 E11 + abE12 + acE21 + bcE22 , ( 1)

a b 0 1 a b ac ad
      σ( E12 ) = = 2
c d 0 0 c d c cd

= acE11 + adE12 + c2 E21 + cdE22 , ( 2)

a b 0 0 a b ab b2
      σ( E21 ) = =
c d 1 0 c d ad bd

= abE11 + b2 E12 + adE21 + bdE22 , ( 3)

a b 0 0 a b bc bd
      σ( E22 ) = =
c d 0 1 c d cd d2

= bcE11 + bdE12 + cdE21 + d2 E22 . ( 4)


由 ( 1 ) , ( 2 ) , ( 3 ) , ( 4 ) 式可得 ( 即将它们形式地表示为矩阵等式 )
a2 ac ab bc
ab a d b2 bd
σ E11 , E12 , E21 , E22 = E11 , E12 , E21 , E22 . ( 5)
ac c2 ad cd
bc cd bd d2
(5) 式右端的矩阵就是 σ( X) 在基{ E11 , E12 , E21 , E22 } 下的对应矩阵 .
17 ( 习题 47 )   设 σ是线 性空 间 V 上 的线 性变 换 , 如果 σk - 1 (ξ) ≠0, 但
σk (ξ) = 0,证明 : ξ,σ(ξ) ,σ2 (ξ) ,… ,σk - 1 (ξ) 线性无关( k > 1) .
证   证明的思路为 : 设
c0 ξ+ c1σ(ξ) + c2 σ2 (ξ) + … + ck - 1 σk - 1 (ξ) = 0 . ( 1)
然后由 ( 1 ) 式成立 , 推出其系数 c0 , c1 , c2 , … , ck - 1 必须全为零 .
先将线性变换 σk - 1 作用于( 1) 式两端的向量 ( 即求 σk - 1 关于 (1 )式两 端向
量的象 , 它们应相等) , 得
σk - 1 c0ξ+ c1σ(ξ) + c2σ2 (ξ) + … + ck - 1σk - 1 (ξ) = σk - 1 (0) ,

c0σk - 1 (ξ) + c1 σk (ξ) + c2σk + 1 (ξ) + … + ck - 1σ2k - 2 (ξ) = 0 . ( 2)
由于 σk - 1 (ξ) ≠0, 而 σk ( ξ) = σk + 1 ( ξ) = … = σ2 k - 2 ( ξ) = 0, 所 以 由 ( 2 ) 式 即
得 c0 = 0 .
再将 σk - 2 作用于 ( 1) 式两端的向量 (此时已有 c0 = 0) , 得

4 .4  部分疑难习题和补充题的题解 213

c1σk - 1 (ξ) + c2σk (ξ) + … + ck - 1σ2 k - 3 (ξ) = 0 . ( 3)


由( 3) 式又得 c1 = 0 .
如此继续作下去 , 在得到 c0 = c1 = c2 = … = ck - 3 = 0 后 , 此时(1) 式已成为
ck - 2σk - 2 (ξ) + ck - 1σk - 1 (ξ) = 0 ( 1 )′
然后用σ作用于 (1 )′两端向量 , 得
ck - 2 σk - 1 (ξ) + ck - 1σk (ξ) = ck - 2σk - 1 (ξ) = 0 .
因为 σk - 1 (ξ) ≠0 ,所以 ck - 2 = 0 , 代入( 1)′后 , 又得 ck - 1 = 0 .
这就证明了 (1 )式成 立时 , 其系数 c0 , c1 , c2 , … , ck - 1 必 须全 为零 , 所以 向
量组 ξ,σ(ξ) ,σ2 (ξ) ,… ,σk - 1 (ξ) 线性无关 .
18 ( 习题 48 )   求 R3 的线性变换
σ( x1 , x2 , x3 ) = ( x1 + x2 + x3 , - x1 - 2 x3 , x2 - x3 ) ( 1)
的象( 值域 )和核以及 σ的秩 .
解   σ的象 (值域 )就是 σ关于R3 中全部向量的象的集合 .由 (1 )式得
σ( x1 , x2 , x3 ) = x1 (1 , - 1 ,0) + x2 (1,0 ,1) + x3 (1 , - 2, - 1) , (1 )′
上式 " ( x1 , x2 , x3 ) ∈R3 都成立 , 即( 1 )′右端的 x1 , x2 , x3 为任意 实数 .将 (1 )′
式右端的 3 个向量依次记为 α1 ,α2 ,α3 , 则 σ的 全部 象的 集合 就是 α1 ,α2 ,α3
的所有线性组合 , 也就是 σ的值域是由α1 ,α2 ,α3 生成的子空间 ,即
σ( R3 ) = L( α1 ,α2 ,α3 ) .
由于 α3 = 2α1 - α2 , 而 α1 ,α2 线性 无关 , 所以 α1 ,α2 是 α1 ,α2 ,α3 的一 个极 大
线性无关组 .因此
σ( R3 ) = L(α1 ,α2 ) .
即σ的值域是由向量α1 = ( 1 , - 1 , 0) 和 α2 = ( 1 , 0 , 1) 所确定 的平面上的 全部
向量 .
秩(σ) = dimσ( R3 ) = 2 .
σ的核是象为零向量的全体原象 .即为满足
σ( x1 , x2 , x3 ) = ( x1 + x2 + x3 , - x1 - 2 x3 , x2 - x3 ) = ( 0 , 0 , 0 )
的全体( x1 , x2 , x3 ) , 也就是齐次线性方程组
  x1 + x2 + x3 = 0 ,
- x1 -   2 x3 = 0 , ( 2)
x2 - x3 = 0
的解空间 .
214 第 4 章 向量空间与线性变换

方程组 ( 2)的基础解系为 ξ= ( - 2 , 1 , 1) T .所以 σ的核为


kerσ= L(ξ) = L( - 2 , 1 , 1 ) .
19 ( 习题 49 )   求 R3 的 一个 线 性 变换 σ, 使 得 σ的象 为 σ( R3 ) = L (α1 ,
α2 ) ,其中 α1 = ( 1 , 0 , - 1 ) ,α2 = ( 1 ,2, 2 ) .
解   σ( R3 ) = L(α1 ,α2 ) 指 的是 : " ( x1 , x2 , x3 ) ∈ R3 ,σ( x1 , x2 , x3 ) 都 是
α1 ,α2 的线性组合 .因此可取
        σ( x1 , x2 , x3 ) = x1 α1 + x2 α2
= x1 (1 , 0 , - 1 ) + x2 (1 , 2 , 2)
= ( x1 + x2 , 2 x2 , - x1 + 2 x2 ) . ( 1)
(1) 式就是值域为 L(α1 ,α2 ) 的一个线性变换 .
此题的答案不是惟一的 ,也可取
        σ( x1 , x2 , x3 ) = x1 α1 + x3 α2
= ( x1 + x3 , 2 x3 , - x1 + 2 x3 ) .
如果 α3 = α1 + α2 = ( 2 ,2, 1 ) ,也可取
      σ( x1 , x2 , x3 ) = x1α1 + x2 α2 + x3 α3
= ( x1 + x2 + 2 x3 , 2 x2 + 2 x3 , - x1 + 2 x2 + x3 ) .
20 ( 习题 50 )   已知R2 的线性变换
σ( x1 , x2 ) = ( x1 - x2 , x1 + x2 ) . (1 )′
( 1) 求 σ2 ( x1 , x2 ) = ?
( 2) 问 σ是否可逆 ? 如可逆 , 求 σ- 1 ( x1 , x2 ) = ?
解   法 1 : ( 1) 由于 σ2 ( x1 , x2 ) = σ σ( x1 , x2 ) , 所以
        σ2 ( x1 , x2 ) = σ( x1 - x2 , x1 + x2 ) (利用 (1 )′式得)
= ( x1 - x2 - x1 - x2 , x1 - x2 + x1 + x2 )
= ( - 2 x2 , 2 x1 ) .
( 2) 由 σ( x1 , x2 ) = ( x1 - x2 , x1 + x2 )
= x1 ( 1 , 1) + x2 ( - 1 , 1 ) ,
可见 r(σ) = 2( 因为 ( 1 , 1 ) , ( - 1 , 1 ) 线性 无关 ) , 因 此 σ是可逆 的 , 于 是存在 线
性变换τ, 使 τσ= I( 恒等变换 ) , 即
τ
σ( x1 , x2 ) = τ( x1 - x2 , x1 + x2 ) = ( x1 , x2 ) . ( 2)
在( 2) 式中 , 令 x1 - x2 = y1 , x1 + x2 = y2 , 则得

4 .4  部分疑难习题和补充题的题解 215

y1 + y2 - y1 + y2
τ( y1 , y2 ) = , .
2 2
所以 σ的逆变换σ- 1 为
x1 + x2 - x1 + x2
σ- 1 ( x1 , x2 ) =τ( x1 , x2 ) = , .
2 2
法 2 : 利用 σ与其对应的矩阵 A 有相同的可 逆性 ,以 及 σ- 1 对应 A - 1 .也
可求解 .
先求 σ在自然基 { e1 , e2 } 下的对应矩阵 A .
σ( e1 ) = σ( 1 , 0) = ( 1 , 1 ) = e1 + e2 ,
σ( e2 ) = σ( 0 , 1) = ( - 1 , 1 ) = - e1 + e2 .
于是

1 - 1
σ( e1 , e2 ) = ( e1 , e2 ) .
1 1

1 -1
σ对应的矩阵 A = 是可逆的 ,所以 σ可逆 .再由
1 1
1 1
2 2
A- 1 = ,
1 1
-
2 2

1 1
2 2
σ- 1 ( e1 , e2 ) = ( e1 , e2 ) .
1 1
-
2 2
从而
1 1
σ- 1 ( e1 ) = e1 - e2 ,
2 2
1 1
σ- 1 ( e2 ) = e1 + e2 .
2 2
于是
σ- 1 ( x1 , x2 ) = σ- 1 ( x1 e1 + x2 e2 )
= x1 σ- 1 ( e1 ) + x2σ- 1 ( e2 )
216 第 4 章 向量空间与线性变换

1 1 1 1
= x1 e1 - e + x2 e1 + e
2 2 2 2 2 2
1 1 x1 + x2 - x1 + x2
= ( x + x2 ) e1 + ( - x1 + x2 ) e2 = , .
2 1 2 2 2
21 ( 补充题 56 )   设 A 是 m× n 矩阵 , B 是 s× n 矩阵 , 两个齐次线性方程
组 Ax= 0, Bx= 0 的解空间的交是什么意义 ? 如何求它们的交 ?
解   两个解空间 S1 与 S2 的交中的向量 x∈ S1 且 x∈ S2 , 即 x 既是 Ax =
0 的解 , 也是 Bx= 0 的解 .因此 ,将两个齐次线性方程组联立起来的方程组
A
x= 0 ( 1)
B
的解空间就是两个解空间的交 S1 ∩ S2 .所以求 S1 ∩ S2 , 就只要求齐次线性方
程组( 1) 的解空间 .
22 ( 补充题 58 )   设 A 为 n 阶 实 矩阵 , 问 : 下 列 命 题 是 否 正 确 ? 并 说
明理由 .
( 1) 若 dim R ( A) = n, 则 R( A) = R( AT ) ;
( 2) 若 dim R ( A) = m < n, 则 R( A) ≠ R( AT ) .
解   (1 ) 命题正确 .因为 : 当 dim R( A) = n 时 , A 的 n 个 n 维列向量 线性
无关 .所以 A 的列空间 R ( A) = Rn .又 因为 : 秩 ( A) = n, A 的 n 个 n 维行向 量
也线性无关 , 从而 A 的行空间 R ( AT ) = Rn .因此 , R( A) = R( AT ) .
( 2) 命题不正确 .反例 :
1 1 1
A= 1 -1 1 ,
1 1 1
此时 , R ( A) = L (1 , 1 , 1) , (1 , - 1 , 1) = R( AT ) .
当然最简单的反例为

1 0
A= .
0 0

记 e1 = ( 1, 0 ) ,此时 R ( A) = L( e1 ) = R( AT ) .
23 ( 补充题 59 )   设 V1 , V2 是 Rn 的两个非 平凡子空间 , 证明 : 在 Rn 中存
在向量α,使 αú V1 , 且 αú V2 , 并在R3 中举例说明此结论 .
证   因为 V1 , V2 是Rn 的两个非平凡子空 间 , 所 以v βú V1 , 如果 βú V2 ,

4 .4  部分疑难习题和补充题的题解 217

命题得证 , 不妨设 β∈ V2 .
又v γú V 2 , 如果 γú V1 , 命题也得证 , 不妨设 γ∈ V1 .
于是 α= β+ γú V 1 , 因为: 如果 α∈ V1 , 由 γ∈ V 1 , 可推 出 α- γ= β∈ V1 ,
与前面的假设矛盾 .
同样 ,α= β+ γú V 2 , 因为 : 如果 α∈ V2 , 由 β∈ V 2 , 可推出 α- β= γ∈ V2 ,
与前面的假设矛盾 .
综上 , v α= β+ γú V1 , 且 α= β+ γú V2 .
R3 中的例子 , 设{e1 , e2 , e3 } 为 R3 的自然基 .
V1 = L( e1 , e2 ) ,   V2 = L( e2 , e3 ) ,
则 α= ( 1 ,1, 1 ) 既不属于 V1 , 也不属于 V 2 .这样的 α有很多很多, 只要既不是
{ e1 , e2 } 所确定的平面上的向量 ,又不是 { e2 , e3 } 所确 定的平面上 的向量 , 都是
符合题意的 α.
  第 5 章

特 征 值 和特 征 向 量   矩阵 的 对 角 化

5 .1 基本要求与内容提要

1  基本要求

(1 ) 准确理解矩阵的特征值与特征向量的概念和性质 ;
(2 ) 熟练掌握矩阵的特征值与特征向量的求法 ;
(3 ) 理解两个矩阵相似的概念和性质 ;
(4 ) 熟练掌握矩 阵可 对角 化的 充 分必 要条 件 .对 可对 角化 矩
阵会求其相似标准形 ;
(5 ) 熟悉 : 实对称 矩阵 必可以 对角 化 , 其特 征值均 为实 数 , 不
同特征值对应的特征向量正交 ;
(6 ) 对 实 对 称 矩 阵 A, 会 求 正 交 阵 T 和 对 角 阵 Λ, 使
- 1
得T AT = Λ .

2  内容提要

(1 ) 特征值和特征向量的概念
① 设 A = ( aij ) ∈C n× n , 若存在 数 λ∈ C ( 复数 域 ) 和向 量 x≠
0, 使得 Ax= λx, 则称数 λ为 A 的一 个特 征 值 , x 为 A 的 属于 ( 对
应于 ) 特征值 λ的特征向量 .
f (λ) = |λI - A| 叫做矩阵 A 的特征多项式 .称 |λI - A| = 0 为
5 .1 基本要求与内容提要 219

A 的特征方程 .线性方程组 (λI - A) x = 0 有 非零 解 x 的充 要条 件


是 |λI - A| = 0 , 即特征值 λ是 n 次代数 方程 |λI - A | = 0 的 根 .特
征向量 x 是 (λI - A) x = 0 的非零解 .
② 矩阵 A 的属于特征 值λ的全 体特 征向 量加 上 零向 量 0 构
成的线性空 间 , 即 (λI - A) x = 0 的解 空间 称为 A 的 关于 特征值 λ
的特征子空间 , 记做 Vλ = { x| Ax =λx} .
(2 ) 特征值与特征向量的性质 :
① 若 λ0 是 A 的一个特征值 , x 是 A 的对应于λ0 的特征向量 ,
即 Ax= λ0 x( x≠0 ) , 则
( i ) λ0 也是 AT 的一个特征值 ;
( ii ) kλ0 为 k A 的一个特征值 ( k 为任意常数 ) ;
m m
( iii ) λ0 是 A 的一个特征值 ( m 为任意正整数 ) ;
( iv) 若 f ( t) 是 t 的 一 个 多 项 式 , 则 f (λ0 ) 是 f ( A) 的 一 个
特征值 ;
( v) 若 A 可逆 , 则λ0 ≠0 , 且λ0- 1 为 A - 1 的一个特征值 λ
. 0- 1 | A|
为 A* ( A 的伴随矩阵 ) 的一个特征值 ; 而且特征向量 x 仍然是矩阵
m - 1 * m
kA, A , f ( A) , A 和 A 的 分 别 对 应于 特 征 值 kλ0 , λ0 , f (λ0 ) ,
- 1 - 1
λ0 和 λ0 | A| 的特征向量 .
② 若 x1 , x2 都是 A 的属于特 征值 λ0 的特征 向量 , 则 k1 x1 +
k2 x2 ( ≠0) ( k1 , k2 为 任 意 常 数 ) 也 是 A 的 属 于 特 征 值 λ0 的 特
征向量 .
③ 不同特征值对应的特征向量线性无关 .
④ k 重特征值至多有 k 个线性无关的特征向量 .
⑤ 若 n 阶矩阵 A = ( ai j ) n× n 的 n 个特征值为λ1 ,λ2 , … ,λn , 则
n n n

( i)   ∑ λ= ∑
i= 1
i
i= 1
aii ,     ( ii)   ∏ λ = | A|.
i= 1
i

⑥ A 可逆的充分必要条件为 0 不是 A 的特征值 .
(3 ) 矩阵的相似关系
220 第 5 章 特征值和特征向量 矩阵的对角化

① 矩阵相似的定义 : 对于矩阵 A, B, 若存在可逆矩阵 P, 使得


AP = B, 则称 A 相似于 B, 记作 A~ B .矩阵 的相 似关系 是一 种
- 1
P
等价关系 , 具有 : 自反性 ( A~ A) ; 对 称性 ( 若 A~ B, 则 B~ A) 和 传
递性 ( 若 A1 ~ A2 , A2 ~ A3 , 则 A1 ~ A3 ) .
② 矩阵的相似有以下性质 :
( i ) 若 A~ B, 则 Am ~ Bm ( m 为任意正整数 ) ;
( ii ) 若 A~ B, 则 f ( A) ~ f ( B) ( 其中 f ( t) 为 t 的多项式 ) ;
- 1 - 1 - 1
( 其证明利用下面性质 : P ( AB) P = ( P AP) ( P BP) ;
- 1 - 1 - 1
P ( kA + lB) P = kP AP + lP BP     ( " k, l∈C ( 复数域 ) )
( iii ) 相似矩阵的特征值都相同 .
(4 ) 矩阵可对角化的条件
① n 阶矩阵 A 与对角阵相似 的充 要 条件 是 A 有 n 个 线性 无
关的特征向量 , 或 A 的每个特征值对应 的线性 无关 的特征 向量 的
最大个数等于该特征值的重数 , 即 A 的 每个特 征值 的重数 等于 其
特征子空间的维数 , 而且不同特征值的重数之和等于 n .
② n 阶矩阵 A 有 n 个互 不相 同的 特 征值 , 则 A 可对 角化 ( 因
为不同特征值对应的特征 向量 线性 无关 ) .此条件 是充 分的 , 但 不
是必要的 .
③ 若 A 存在一个特征值 , 其特征子空间的维数小于特征值 的
重数 , 则 A 不能与对角阵相似 .
(5 ) 对 n 阶可对角化矩阵 A, 求变换矩阵 P, 使 P- 1 AP = Λ( 对
角阵 ) 的解题步骤 :
① 求出 A 的 互异 特 征值 λ1 , … ,λm , 其重 数 分别 为 r1 , … , rm
( 它们的和等于 n) ;
② 求特征子空间 Nλi 的基{ xi1 , xi2 , … , xi r i } , 即 (λi I - A) x = 0
的基础解系 ( i = 1 , 2 , … , m) ;
③ 将 m 个特征子空间的基向量依次按列排成矩阵 P = ( x11 , …,
x1 r1 , … , xm1 , … , xm r m ) ( n 个列向量线性无关 , 变换矩阵 P 可逆 ) , 则
5 .2 矩阵的特征值和特征向量 相似矩阵 221

- 1
P AP = Λ= diag(λ1 , … ,λ1 , … ,λm , … ,λm ).
(6 ) 实对称矩阵 的特 征值 都是 实 数 ; 属于 不同 特 征值 的特 征
向量是正交的 , 一定是线性无关的 .
(7 ) 实 对 称 矩 阵 是 可 对 角 化 的 矩 阵 ( 一 定 和 对 角 阵 相 似 ) ,
即它的 k 重特征值一定对 应有 k 个线 性无关 的特 征 向量 .实 对称
- 1 T
矩阵 A, 必 存 在 正 交 阵 T, 使 得 使 T AT = T AT = diag (λ1 ,
λ2 , … ,λn ).
- 1
(8 ) 对实对称矩阵 A, 求正交阵 T, 使得 T AT 为对角阵的 解
题步骤大致相 同 于求 变 换 矩 阵 P, 使 P- 1 AP 为 对 角 阵 的 解 题 步
骤 , 只是 在求 了特征 向量 后 , 必须 把 k( > 1) 重 特征值 对应 的 k 个
线性无关的特征向量用施 密特 正交化 方法 将其正 交化 , 再 把全 部
n 个特征 向量 单位化 , 然后把 所有 特征值 对应 的标 准正交 特征 向
量按列排成正交矩阵 T( T- 1 = TT ) .

5 .2 矩阵的特征值和特征向量 相似矩阵

1  矩阵的特征 值和特征 向量的概念 和计算

(1 ) 设 A 为 n 阶 方 阵 , 若 存 在 数 λ和 向 量 x ≠0, 使 得 Ax =
λx , 则称数 λ为A 的一个特征值 , x 为 A 的属于 ( 对应于 ) 特 征值 λ
的特征向量 .矩阵的特征 值与 特征向 量在 其他 学科和 工程 技术 中
都有重要的应用 , 且有计算特征值与特征向量的现成的软件 .这里
要求同学会 求一些 二、三 阶和某 些特殊的 n 阶矩阵的 特征值与 特
征向量 .求特征值 , 即求方程 |λI - A| = 0 的 根 , 求特 征向 量 , 即 解
线性方程组 (λI - A) x = 0, 求其基础解系 .
(2 ) 特征值可以是 零 ( 当 矩 阵不 可逆 时 ) , 但是 特 征向 量一 定
不是零向量 ; 每一个特征向量只属 于一个特 征值 , 但一 个特征 值 λ
可以有无数个特征向量 , 因为 属于 λ的 特征向 量的 非 零线 性组 合
222 第 5 章 特征值和特征向量 矩阵的对角化

仍然是属于λ的特征向量 .
(3 ) A 的特征值λ是方程
λ- a11 - a12 … - a1 n
- a21 λ- a2 2 … - a2 n
f (λ) = |λI - A| = =0
… … … …
- an1 - an2 … λ- an n
的根 . f (λ) = 0 是 λ的 n 次 代数 方程 , 在复 数域 中有 n 个 根 ( 可 以
有重根 ) .每个特征值 λ必有 对应 的特 征 向量 , 即 (λI - A) x = 0 必
存在非零解 .若计算中出现矩阵 (λI - A) 满 秩 , 说明 你计算 错误 或
这里的 λ不是 A 的特征值 .
(4 ) 求特征值的方法有两个 : 一是应用定义 : Ax = λx ( x≠0) ,
例如 , 若 Ax = 0 存在非零解 x, 即 ( 0 I - A) x = 0 存在非零解 x, 则 0
是 A 的一个特征值 ; 又例如 , 当 A 的每行的行和都是 k 时 , 则由
a11 … a1 n 1 k 1 1
… … … = … = k … ,   即   Ax = kx,   x= … ,
an1 … an n 1 k 1 1
可见 k 是 A 的一个特征值 , 其对应的特 征向量 是元 素全部 为 1 的
n 维列向量 .
另一求特征值的方法, 也是主要的方法是计算行列式 |λI - A| = 0
的根 .这里特征多项式 |λI - A | 是 n 阶行 列式 , 其展 开式 是 λ的 n
次多项式 .一般情况下 , 它的 因式 分解 是很困 难的 .在 线性 代数 课
程中能求特 征值 的矩阵 A 都是 在行 列式 |λI - A | 的计 算 ( 利用 行
列式的性质和展 开 ) 过程 中 能 够分 解 因式 的 .如 果 随便 给 一 个 矩
阵 , |λI - A| 又难以分解因式 , 那只 好借 助于计 算机 软件或 求特 征
值的近似值 ( 可以查看《计算方法》的教材 ) .

2  特征值和特 征向量的 性质

关于特征值与特征向量的性质 , 以及相似矩阵及其性质 , 在内
5 .2 矩阵的特征值和特征向量 相似矩阵 223

容提要中已经阐述 , 这里再明确以下几个问题 :
(1 ) 矩阵 A 的特征子空间 Vλ 中的 向量都是 A 的对应 于特 征
值λ的特征向量吗 ? 答案是否定的 .
因为特征子空间 Vλ 是齐 次线 性方 程组 (λI - A) x = 0 的解 空
间 , 零向量属于解空间 , 但零向量不是特征向量 , 所以 , 正确的说法
是 : A 的特征子空间 Vλ 中的非零向量都是 A 的对应于特征值λ的
特征向量 .
(2 ) A 的两个不同特征值λ1 ,λ2 的特征子空间的交 Vλ1 ∩Vλ2 = ?
设 x∈ Vλ1 ∩ Vλ2 , 则 Ax= λ1 x, 且 Ax= λ2 x, 于是
λ1 x = λ2 x,   即   (λ1 - λ2 ) x = 0 ,
由于 λ1 - λ2 ≠0 , 所以 , x = 0, 因此 , Vλ1 ∩ Vλ2 = {0} .这 表 明属 于 λ1
的特征向量一定不是属于 λ2 的特征向量 , 也就是说任 何一个非 零
向量不可能是两个不同特征值对应的特征向量 .
(3 ) 相似矩阵 A, B 之间的特征值有何关系 ?
若 A~ B, 则 A, B 具有相同的特征值 .因为 A~ B, 即存在可 逆
矩阵 P, 使得 P- 1 AP = B, 于是
|λI - B| = |λI - P - 1 AP| = | P - 1 (λI - A) P|
= | P - 1 | |λI - A| | P| = |λI - A|.
即 A, B 的 特 征 多 项 式 相 等 , 所 以 特 征 值 也 相 同 .同 时 有 行 列 式
| B| = | A| , 迹相等 , 即 t r ( A) = tr ( B) , 秩相 等 , 即 r ( B) = r ( A) , 但
特征值相同的矩阵不一定是相似矩阵 .例如
2 0 2 1
A= ,   B= .
0 2 0 2
- 1
A, B 的 特 征 值 都 是 2 ( 二 重 ) , 但 对 任 何 可 逆 矩 阵 P, P AP =
- 1
P 2 IP = 2 I≠ B .所以 , A 与 B 不相似 .
0 -2 - 2
例 1   已知矩阵 A = 2 -4 - 2 , 求 A 的特征值及特征
- 2 2 0
224 第 5 章 特征值和特征向量 矩阵的对角化

向量 .
解   将 |λI - A| 中第 1 , 3 行 对换 后 , 第 1 行加 到第 2 行 , 再 将
λ
第 1 行乘 - 加到第 3 行 , 然后按第 1 列展开 , 得
2
λ 2 2 2 -2 λ
|λI - A| = - 2 λ+ 4 2 = - - 2 λ+ 4 2
2 -2 λ λ 2 2
2 -2 λ
1 λ+ 2
0 λ+ 2 λ+ 2
= - = - 2(λ+ 2 ) λ
2
2
λ 1 2-
0 λ+ 2 2- 2
2
λ2
= - 2 (λ+ 2 ) 2 - - λ- 2 =λ(λ+ 2) 2 = 0.
2
另一方法 : |λI - A| 中第 3 行 加到第 2 行 , 提出第 2 行 的公 因
子 (λ+ 2 ) 后 , 再 将 第 2 列 乘 ( - 1 ) 加 到 第 3 列 , 最 后 按 第 3 列 展
开,得
λ 2 2 λ 2 2
|λI - A| = - 2 λ+ 4 2 = 0 λ+ 2 λ+ 2
2 -2 λ 2 -2 λ
λ 2 2 λ 2 0
= (λ+ 2) 0 1 1 = (λ+ 2 ) 0 1 0
2 -2 λ 2 - 2 λ+ 2
2
= λ(λ+ 2) = 0.
从而得 A 的 特征 值为 λ1 = 0 ,λ2 = - 2 ( 二重 特征 值 ) .对于 λ1 = 0 ,
求解 (λ1 I - A) x = 0 , 即
0 2 2 x1 0
-2 4 2 x2 = 0 ,
2 -2 0 x3 0
5 .2 矩阵的特征值和特征向量 相似矩阵 225

T
得基础解系 : x1 = ( - 1 , - 1 , 1 ) . 故 A 的属于 λ1 的 全部特 征向 量
为 k1 x1 ( k1 为任意非 0 常数 ) .
对于 λ2 = - 2 , 求解 (λ2 I - A) x = 0, 即
-2 2 2 x1 0
( 3 行成比例 , 解第 1 个方程
-2 2 2 x2 = 0  
即可 )
2 -2 -2 x3 0
得基础解系 : x2 = ( 1 , 1 , 0 ) T , x3 = ( 1 , 0 , 1 ) T , 则 k2 x2 + k3 x3 ( k2 , k3
为任意不全为 0 的常数 ) 是 A 属于λ2 的全部特征向量 .
例 2   对下列矩阵 A 的特征值 , 能做怎样的断言 ?
2
(1 ) A 不可逆 ;         (2 ) det ( I - A ) = 0;
k
(3 ) A = 0( 幂零矩阵 ) ; (4 ) A = kI - B(λ0 为 B 的特征值 ).
解   ( 1 ) 由 | A | = λ1λ2 …λn = 0 可 知 , A 至 少 有 一 个 特 征
值为零 .
(2 ) 由 | I2 - A| = | I - A| | I + A| = 0 , 得
n
| I - A | = 0, 或 | I + A| = ( - 1 ) | - I - A| = 0.
所以 , 1 或 - 1 是 A 的一个特征值 .
k
(3 ) 设 λ为 A 的特征值 , 即 Ax = λx( x≠0). 由 A = 0 , 得
Ak x = λk x = 0, 于是 λk = 0 , 所以 λ= 0 .
( 4) 设 λ0 为 B 的特征值 , 即 Bx= λ0 x( x≠0) , 由 A = kI - B, 得
Ax = ( kI - B) x = ( kx - Bx) = ( k - λ0 ) x,
所以 , ( k - λ0 ) 是 A 的一个特征值 .
1 1 0
例 3   设 A= 1 0 1 , 则 A 的特征值是 .
0 1 1
( A ) 1 , 0 , 1;   (B) 1 , 1 , 2;   ( C) - 1 , 1 , 2;   ( D) - 1 , 1 , 1 .
3 3 3

答   利用 ∏λ =
i=1
i | A| = - 2 和 ∑λ = ∑
i=1
i
i=1
aii = 2 , 排 除

( A ) , ( B) , ( D ) , 所以 , 选 ( C) .
226 第 5 章 特征值和特征向量 矩阵的对角化

例 4 ( 习 题 6 )   设 λ为 n 阶 矩 阵 A 的 特 征 值 , 且 A 可 逆 ,
证明 :
- 1 - 1
    (1 ) λ 为 A 的特征值 ;
(2 ) | A|λ- 1 为 A 的伴随矩阵 A * 的特征值 .
证   (1 ) 法 1: 利用 Ax =λx   ( x≠0) .
因为 A 可逆 , | A| = λ1 λ2 …λn ≠0 , 故 A 的特征值全不为零 .
在 Ax= λx 的两边左乘 A - 1 得
- 1
x = λA x,
- 1 - 1
所以 A x =λ x   ( x≠0) ,
- 1 - 1
即 λ 为 A 的特征值 , 对应的特征向量仍是 x .
法 2: 利用行列式 .由
- 1 - 1
|λI - A| = |λAA - A| = | A| |λA - I|
- 1 - 1
= | A| | - λ(λ I - A )|
= ( - λ) n | A| |λ- 1 I - A- 1 | = 0.
- 1 - 1 - 1 - 1
因为 | A | ≠ 0 , λ≠ 0 , 所 以 , | λ I - A | = 0, 即 λ 为 A 的
特征值 .
- 1 - 1 * - 1 - 1
证   (2 ) 法 1: 将 A = | A| A 代入 A x = λ x 中 , 得
A* x= λ- 1 x, 所以 , A* x = λ- 1 | A| x   ( x≠0) ,
- 1
| A|
- 1 *
即 λ | A| 为 A 的特征值 , 其对应的特征向量仍是 x .
法 2: 在 Ax = λx 的两边左乘 A * , 利用 A* A = | A | I 得
* *
A Ax =λA x,

即 | A| Ix = λA x,
移项得
* - 1
A x = (λ | A| ) x   ( x≠0).
例 5   设 A, B 均为 n 阶矩阵 , 证明 : 若λ1 ≠0 是 AB 的特征值 ,
则 λ1 也是 BA 的特征值 .
证   设 λ1 为 AB 的非零特征值 , 对应的特征向量是 x1 , 即
( AB) x1 =λ1 x1   ( x1 ≠0 ,λ1 ≠0 ).
5 .2 矩阵的特征值和特征向量 相似矩阵 227

所以 ,λ1 x1 ≠0. 上式两边左乘 B, 得


BA( Bx1 ) =λ1 ( Bx1 )   ( x1 ≠0).
显然 Bx1 ≠0( 否则 , 由 Bx1 = 0, 得 ABx1 = A0 = 0, 与 λ1 x1 ≠0
矛盾 ) , 所以 λ1 为 BA 的一个特征值 , 对应的特征向量是 Bx1 .
更一般的 结论 : AB 与 BA 有 相同 的特征 值 .因为 它们 的特 征
多项式相等 , 即 |λI - AB| = |λI - BA| ( 证明见 第 3 章 3. 7 节中 第
11 题 ) .
例 6   设λ1 ,λ2 为 n 阶矩阵 A 的特征值 , 其对应的特征向量分
别为 x1 , x2 , 则 成立 .
( A ) λ1 = λ2 时 , x1 , x2 一定成比例 ;
( B) λ1 ≠λ2 时 ,λ3 =λ1 + λ2 也是 A 的特征值 , 且对 应的特征 向
量是 x1 + x2 ;
( C) λ1 ≠λ2 时 , x1 + x2 不可能是 A 的特征向量 ;
( D ) λ1 = 0 时 , 有 x1 = 0 .
答   ( C) 正确 .
因为二 重 根 可 能 有 两 个 线 性 无 关 的 特 征 向 量 , 所 以 ( A )
不正确 .
( B) 也 不 正 确 .因 为 若 x1 + x2 是 A 的 对 应 于 λ3 的 特 征
向量 , 即
A( x1 + x2 ) = λ3 ( x1 + x2 ) =λ1 x1 +λ2 x2 ,
则 (λ3 - λ1 ) x1 + (λ3 - λ2 ) x2 = 0,
当 λ1 ≠λ2 时 , x1 , x2 线性无关 , 所以 λ3 = λ1 = λ2 , 矛盾 .所以 x1 + x2
不可能是 A 的特征向量 .因此 ( B) 不正确 .
零向量一定不是特征向量 , 所以 ( D) 也不正确 .
例 7   与可逆矩阵 A 有相同特征值的矩阵是 .

;     ( B) A ;     ( C) A ;     ( D ) A
- 1 2 T
( A) A .
答   ( C) 正确 .因为
|λI - A| = | (λI - A) T | = |λI - AT |.
228 第 5 章 特征值和特征向量 矩阵的对角化

- 1 2 T * - 1
设 A 的特征值为λ, 则 A , A , A , A 的 特征 值分 别为 λ ,
λ2 ,λ, | A|λ- 1 .
T T
例 8 ( 习 题 42 )   已 知 α= ( a1 , … , an ) ,β= ( b1 , … , bn ) 是
n T T
R 中两个向量 , 且 α β= a1 b1 + … + an bn = k .求矩 阵 A = αβ 的 特
征值 .
2 T T T T T
解   A = αβ (αβ ) = α(β α)β = k(αβ ) = k A   ( 其 中 k =
n


T T
β α= α β= ai bi ).
i=1

设         Ax = λx   ( x≠0) , 则
2 2
A x = λ x,
即   k Ax = kλx =λ2 x. 由 x≠0 得
2 T
λ = kλ,   即   λ1 = 0   或   λ2 = k   为   A = αβ 的特征值 .
例 9 ( 习题 18)   设 n 阶矩阵 A 的元素均为 1 , 则 A 的 n 个 特
征值是 .
解   由秩 ( A) = 1 , 得 det A = 0 , 所以 0 是特征值 .
( 0 I - A) x = 0,   即   Ax = 0 ,
有 n - r ( A) = n - 1 个线性无关的特征向量 , 所以 0 至少是 n - 1 重
特征值 , 又 tr ( A) = λ1 + … + λn = n, 所 以有一 个非 0 特 征值为 n, 0
是 n - 1 重特征值 .
由 A 每行的行和都是 n , 也可得
a1 1 … a1 n 1 n 1 1
… … … = … = n … ,   即   Ax= nx,   x= … ,
an1 … an n 1 n 1 1
所以 , A 有一个特征值 为 n , 其对应 的特征向量是元 素全部为 1 的
n 维列向量 .
例 10   设 A, P 都是三阶方阵 , P 可逆 , 已知 A 的特 征值 :λ1 =
3 2 - 1
1 ,λ2 = - 1 ,λ3 = 2 .B = A - 5 A , 求 | B| , | A + 5 I | 和 | 5 I + P AP|.
3 2
解   B 的特征值 μi = λ - 5λ ( i = 1 , 2 , 3 ) .所以 , B 的 3 个特 征
i i
5 .3 矩阵可对角化的条件 229

值为 - 4 , - 6 , - 12 , 故
| B| = ( - 4 ) ( - 6) ( - 12) = - 288.
同理 , A + 5 I 的 3 个特征值为 1 + 5 = 6 , - 1 + 5 = 4 , 2 + 5 = 7 , 所以 ,
| A + 5 I | = 6 ・ 4 ・ 7 = 168 .| 5 I + P - 1 AP | = | 5 P- 1 P + P- 1 AP | =
- 1
|P | | 5 I + A | | P| = | 5 I + A| = 168.
例 11   下列命题哪个是正确的 ?
( A) 若 0 是 某个 矩 阵的 特 征值 , 与 它对 应 的 特征 向 量 是
零向量 .
( B) 若两个矩阵有相 同的特 征值 , 则 它们 对应 的 特征 向量 必
相同 .
( C) 若两个矩阵有 相同 的特征 向量 , 则 它们对 应 的特 征值 必
相同 .
( D ) 不同的矩阵必有不同的特征多项式 .
( E) 不同的矩阵有不同 的特 征值 , 则它 们对 应的 特征 向量 必
不同 .
( F ) 矩阵的一个特征 值可以 对应 多个 特征 向量 , 但一 个特 征
向量只可以属于一个特征值 .
答   ( F ) 正确 , 其余都不正确 .

5 .3 矩阵可对角化的条件

对 n 阶 矩 阵 A, 若 存 在 可 逆 矩 阵 P, 使 P- 1 AP = diag (λ1 ,
λ2 , … ,λn ) , 则称 A 可对角化 ( 即 A 与对角阵 diag (λ1 ,λ2 , … ,λn ) 相
似 ) .关于矩阵可对角化的充要条件 , 相似矩阵的性质 , 以及对可对
角化矩阵 A 如何求可逆阵 P, 使 P- 1 AP = diag(λ1 ,λ2 , … ,λn ) , 在 内
容提要中都已涉 及 , 这 里 不复 述 .读 者 应该 掌 握 教 材中 有 关 定 理
* * *
( 定理5 .4 , 定理5 .5 , 定理5 .6 , 定理5 .7 , 定理 5 .8 , 定理 5 .9) .
例 1   A 是 二 阶实 矩 阵 .( 1) 若 | A | < 0 , 问 A 与 对 角 阵
230 第 5 章 特征值和特征向量 矩阵的对角化

相似否 ?
a b
(2 ) 若 A = , ad - bc = 1 , | a + d | > 2 , 问 A 可 对
c d
角化否 ?
解   (1 ) 由 | A| = λ1λ2 < 0 , 得知 λ1 ≠λ2 , 两 个特 征值都 是一 重
的 , 所以 , A 与对角阵相似 .
λ- a - b 2
(2 ) |λI - A| = =λ - ( a + d)λ+ 1 = 0 ,
- c λ- d
2
此时 , 特征方程的判别式 Δ= ( a + d) - 4 > 0 , 所 以 A 有两 个互 异
的实特征值λ1 ,λ2 , 因此 , A 可对角化 .
2 0 0 2 0 0
例 2 (习题 46)   已知 A= 0 0 1 与 B= 0 y 0 相似 .
0 1 x 0 0 -1
(1 ) 求 x 和 y ;
(2 ) 求一个可逆矩阵 P, 使 P - 1 AP 为对角矩阵 .
解   (1 ) 由 A~ B, 则 A , B 的 特征 值相 同 , 从 而 | A| = | B | =
λ1 λ2 λ3 , t r A = t r B =λ1 +λ2 + λ3 .
由 | A| = - 2 = | B| = - 2 y; 得 y = 1 .
再由 t r A = t r B, 又得 2 + 0 + x = 2 + y + ( - 1 ) , 所 以 , x = 0 .显
然 , A 的特征值为 2 , 1 , - 1 .
(2 ) 当 λ1 = 2 时 , 有

0 0 0 0 1 -2
(2 I - A) = 0 2 -1 → 0 0 1 ,
0 - 1 2 0 0 0
T
得特征向量 x1 = (1 , 0 , 0 ) ;
当 λ2 = 1 时 , 有
5 .3 矩阵可对角化的条件 231

-1 0 0 1 0 0
( I - A) = 0 1 -1 → 0 1 -1 ,
0 - 1 1 0 0 0
得特征向量 x2 = ( 0 , 1 , 1) T ;
当 λ3 = - 1 时 , 有
-3 0 0 1 0 0
( - I - A) = 0 - 1 -1 → 0 1 1 ,
0 - 1 -1 0 0 0
T
得特征向量 x3 = ( 0 , 1 , - 1) .
1 0 0
取 P = ( x1 , x2 , x3 ) = 0 1 1 ,则
0 1 -1
2 0 0
P- 1 AP = 0 1 0 .
0 0 - 1
1 -1 1
例 3 ( 习题 41 )   设 A = x 4 y , 已知 A 有 3 个线 性
-3 -3 5
无关的 特 征 向 量 ,λ= 2 为 A 的 二 重 特 征 值 .求 可 逆 矩 阵 P, 使
P - 1 AP为对角形矩阵 .
解   因为三阶矩阵 A 有 3 个线性无关 的特征向 量 , 所以 A 与
对角矩阵相似 , 二重 特 征 值恰 对 应两 个 线性 无 关 的 特 征向 量 , 即
秩 (2 I - A) = 1.
1 1 -1 1 1 -1
2 I - A= - x -2 - y → 0 x-2 - x- y ,
3 3 -3 0 0 0
由 r (2 I - A) = 1 , 得
x - 2 = 0 ;   - x - y = 0 .   所以 , x = 2;   y = - 2.
232 第 5 章 特征值和特征向量 矩阵的对角化

当 λ1 = 2 时 , 求解 ( 2 I - A) x = 0 .由
1 1 -1 1 1 -1
(2 I - A) = -2 -2 2 → 0 0 0 ,
3 3 -3 0 0 0
得特征向量 x1 = ( - 1 , 1 , 0 ) T , x2 = ( 1 , 0 , 1) T .
再由 λ1 + λ2 + λ3 = t r A, 即 4 + λ3 = 1 + 4 + 5 , 得 λ3 = 6 .求 解
(6 I - A) x = 0. 由
1
1 0 -
5 1 - 1 3
(6 I - A) = - 2 2 2 → 2 ,
0 1
3 3 1 3
0 0 0
得特征向量 x3 = (1 , - 2 , 3 ) T .
-1 1 1
取 P = ( x1 , x2 , x3 ) = 1 0 - 2 ,则
0 1 3
2 0 0
P- 1 AP = 0 2 0 .
0 0 6
例 4 ( 研 5 -18 )   已知三阶矩阵 A 和 三维向量 x, 使得向量 组
2
x, Ax, A x 线性无关 , 且满足
A3 x= 3 Ax - 2 A2 x .
(1 ) 记 P = ( x, Ax, A2 x) , 求三阶矩阵 B, 使得 A = PBP- 1 .
(2 ) 计算行列式 | A + I |.
解   (1 ) 法 1: 由于 x, Ax, A2 x 线 性无 关 , 它 是 R3 的 一 组基 ,
2
且 P = ( x, Ax, A x) 可 逆 . 所 求 的 B 与 A 相 似 , 且 满 足 A =
- 1
PBP ,即
PB = AP = A( x, Ax, A2 x) = ( Ax, A2 x, A3 x).
5 .3 矩阵可对角化的条件 233

3 2
将 A x = 3 Ax - 2 A x 代入上式 , 得
2 2
PB = ( Ax, A x, 3 Ax - 2 A x).
2 2 3 2
再将 ( Ax, A x, 3Ax - 2 A x) 用R 的基 ( x, Ax, A x)线性表示 , 得
0 0 0
PB= ( Ax, A2 x, 3Ax - 2 A2 x) = ( x, Ax, A2 x) 1 0 3 .
0 1 -2
- 1
由于 P 可逆 , 上式两边左乘 P 得
0 0 0
B= 1 0 3 .
0 1 -2
法 2: 令 B = ( bi j ) , 由 AP = PB, 即得
b1 1 b12 b13
2 2
A( x, Ax, A x) = ( x, Ax, A x) b2 1 b22 b23 .
b3 1 b32 b33
2
Ax = b1 1 x + b2 1 Ax + b3 1 A x,             (1 )
2 2
比较上式两边 , 得 A x = b1 2 x + b2 2 Ax + b3 2 A x,             (2 )
A3 x = b1 3 x + b2 3 Ax + b3 3 A2 x.             (3 )
3 2
将 A x= 3 Ax - 2 A x 代入上面的方程 ( 3) , 得
3 Ax - 2 Ax2 = b1 3 x + b2 3 Ax + b3 3 A2 x. ( 4)
再改写方程 (1 ) , (2 ) , (4 ) 为
2
b11 x + ( b21 - 1 ) Ax + b3 1 A x = 0,
2
      b12 x + b22 Ax+ ( b3 2 - 1 ) A x = 0,
b13 x + ( b23 - 3 ) Ax + ( b33 + 2) A2 x = 0.
2
由于 x, Ax, A x 线性无关 , 上式各个系数必全部为 0 , 故得
b1 1 = b1 2 = b1 3 = b2 2 = b3 1 = 0 ,   b21 = b32 = 1 ,   b23 = 3 ,   b33 = - 2 ,
0 0 0
故 B= 1 0 3 .
0 1 -2
234 第 5 章 特征值和特征向量 矩阵的对角化

(2 ) 因为 A~ B, 所以它们的特征值相同 .
λ 0 0
由   |λI - B | = - 1 λ -3 = λ(λ+ 3 ) (λ- 1 ) = 0 , 得
0 - 1 λ+ 2
λ1 = 0 ,λ2 = - 3 ,λ3 = 1.
故 A 的特征值也是 0 , - 3 , 1 , A + I 的特征值为 1 , - 2 , 2 , 于是
| A + I | = 1・ ( - 2 ) ・2 = - 4 .
0 1 - 1
例 5   设 A= 1 0 1 .
- 1 1 0
(1 ) 求变换矩阵 P, 使 P - 1 AP 为对角矩阵 ; ( 2) 求 | 3 I - A|.
解   (1 ) 由
λ -1 1
2
|λI - A| = - 1 λ - 1 = (λ+ 2 ) (λ- 1) = 0 ,
1 -1 λ
得 λ1 = - 2 ,λ2 = 1 ( 二重 ).
T
对 λ1 = - 2 , (λ1 I - A) x = 0 的基础解系为 x1 = ( 1 , - 1 , 1) .
对 λ2 = 1 , (λ2 I - A) x = 0 的 基础 解系 为 x2 = ( 1 , 1 , 0 ) , x3 = T

( - 1 ,0 ,1 )T.
1 1 - 1
取 P = ( x1 , x2 , x3 ) = - 1 1 0 ,
1 0 1
- 1
则有 P AP = Λ= diag( - 2 , 1 , 1) .
(2 ) 3 I - A 的 3 个特征值为 3 - ( - 2 ) = 5 , 3 - 1 = 2 ( 二重 ) , 所
2
以 | 3 I - A| = 5・ 2 = 20 .
例 6 ( 习 题 38 )   设 B = ααT , 其 中 α= ( a1 , … , an ) T , α≠
0, ai ∈R   ( i = 1 , 2 , … , n) .
k
(1 ) 证明 B = tB, 其中 k 为正整数 , t 为常数 , 并求 t;
5 .3 矩阵可对角化的条件 235

- 1
(2 ) 求可逆阵 P, 使 P BP 为对角阵 , 并写出此对角阵 .
解   ( 1) B2 = α(αT α)αT = (αT α)ααT = mααT = m B( 其中 m =
αT α= a21 + … + a2n = t r B) .
B3 = B2 B = mB2 = m2 B, … , 由归 纳 法可 以 得 到 Bk = mk - 1 B =
tB, 其中 t = ( tr B) k - 1 .
(2 ) 先求特征值 .
法 1: 设 Bx = λx( x≠0) , 则
2 2
B x = λ x = mBx = mλx.
由 x≠0 得     λ2 = mλ即λ1 = 0 或 λ2 = m.
T
λ1 = 0 时 , (λ1 I - B) x = - Bx = - αα x = 0. 由 r ( B) = 1 , 得 基
础解系含 n - 1 个线性无关 的解 , 即 λ1 = 0 对 应 n - 1 个线 性无 关
的特征向量 , 所以 ,λ1 = 0 至少是 B 的 n - 1 重特征值 .由
n

∑ λ = tr B= m,   得   λ = m 是一个一重特征值 .
i=1
i 2

法 2: 因 α≠0 , 不 妨 设 a1 ≠ 0 , 于 是 在 | λI - B | 中 第 一 行 乘
ai ai
- 加到第 i 行 ( i = 2 , … , n) , 再将第 i 列乘 都 加到第 1 列
a1 a1
( i = 2 , … , n) , 则有
2
λ- a1 - a1 a2 … - a1 an
- a2 a1 λ- a22 … - a2 an
|λI - B| =
… … w …
2
- an a1 - an a2 … λ- an
λ- a21 - a1 a2 … - a1 an
a2
- λ λ … 0
a1
=
… … w …
an
- λ 0 … λ
a1
236 第 5 章 特征值和特征向量 矩阵的对角化

λ- ∑a …
2
i - a1 a2 - a1 an
i= 1

= 0 λ … 0
… … w …
0 0 … λ
n

∑a
n-1 2
=λ λ- i = 0,
i =1
n

所以 ,λ1 = λ2 = …λn - 1 = 0 , λn = ∑a
2
i = m.
i=1

再求特征向量 .
当 λ1 = 0 时 , (λ1 I - B) x = - Bx = 0, 由 r( B) = 1 , 故只需解第 1
个方程
a21 x1 + a1 a2 x2 + … + a1 an x n = 0.
基础解系含 n - 1 个 线性 无 关的 解 向量 , 所以 ,λ1 = 0 对 应 的
特征向量为 :
T T
x1 = ( - a2 , a1 , 0 , … , 0) ,   x2 = ( - a3 , 0 , a1 , … , 0 ) , … ,
xn - 1 = ( - an , 0 , … , 0 , a1 ) T ,
T T
λ= m 时 , ( mI - B) x= (α αI - αα ) x = 0 , 即
T T
α αx - αα x = 0.
T T T
观察出 x = α满足此方 程 , 因为 (α α)α - α(α α) = 0( 其中 α α=
T
m 是一个数 ) .所以 ,λn = m 对应的特征向量为 xn = ( a1 , … , an ) .
- a2 - a3 … - an a1
a1 0 … 0 a2
令 P = ( x1 , … , xn - 1 , xn ) = 0 a1 … 0 a3 ,
… … w … …
0 0 … a1 an
n


- 1 2
则 P BP = diag 0 , … , 0 , ai .
i= 1
5 .4 实对称矩阵的对角化 237

5 .4 实对称矩阵的对角化

(1 ) 实对称矩阵的特征值都是实数 , 且特征 向量也是 实向量 ;


属于不同特征值的特征向 量是 正交的 ( 正交向 量组 一定是 线性 无
T
关的向量组 ) ; 对于实向量 x≠0, 必有 x x > 0 , 而 对复 向量 x≠0,
未必有 xT x > 0 ( 例如 x = ( 1 , i ) T , 有 xT x = 0 ) , 但 必有 ( x) T x > 0 .在
做证明题时 , 若未知特征值是实数时 , 就必须把特征向量当成复向
量来处理 .
(2 ) 实对称矩 阵一定和对角阵相 似 .对 n 阶实 对称矩阵 A, 它
的 k 重特征值λ恰对 应有 k 个线 性无 关 的特 征向 量 , 即 dimVλ =
- 1
n - r (λI - A) = k .实对称矩阵不仅存在可逆矩阵 P, 使得 P AP =
- 1 T
diag(λ1 ,λ2 , … , λn ) ; 而 且 存 在 正 交 阵 T, 使 得 T AT = T AT =
diag(λ1 ,λ2 , … ,λn ) ; 这里对角阵是惟一的 ( 是 A 的相似标准形 ) , 但
T 不是惟一的 .为什么要求正交阵 T, 使 A 对角化 ? 到 下一章学 习
二次型时 , 可以知道正交 阵对 应的线 性变 换是 保长度 保角 度的 正
交变换 , 在二次曲面化标准形中及其他问题中都有重要的应用 .
(3 ) 对 n 阶实对称矩阵 A, 求正交阵 T, 使得使 T - 1 AT 为对 角
阵的解题步骤 :
① 求出 A 的互异特征 值λ1 , … ,λm , 其 重数 分别 为 r1 , … , rm
( 它们的和等于 n).
② 求特征子空间 Nλi 的基{ xi1 , xi2 , … , xi r i } , 即 (λi I - A) x = 0
的基础解系 ( i = 1 , 2 , … , m).
③ 对应于同一个特征值 λi (λi 的重数 ri > 1) 的特征向量{ xi1 ,
xi2 , … , xi r } , 用施 密 特正 交 化 方法 将 其 正交 化 ; 将 已经 正 交 化 的
i

特征向量单位化 , 再把一重特征值对应的特征向量也单位化 , 得到
n 个相互正交的单位向量 γ1 , … ,γn ( 注意要先正 交化再单 位化 , 不
能先单位化再正交化 , 因为正 交化 后可能 不是 单位 向量了 ) ; 再 排
238 第 5 章 特征值和特征向量 矩阵的对角化

- 1 T
成正交矩阵 T( T = T ) .T 的 任意 两 行 ( 列 ) 向 量 的内 积 为 0 , 每
行 ( 列 ) 向量的长度为 1 .
④ 将 γ1 , … ,γn 依次按列排成矩阵 T = (γ1 , … ,γn ) ( 变换矩 阵
T 一定可逆 ) , 则
T - 1 AP = TT AT = diag (λ1 , … ,λ1 , … ,λm , … ,λm ).
注意 : 对角矩阵的对角元的 排序要 使得 第 i 个 λi 对应 的特 征
向量γi 排在 T 矩阵的第 i 列 .
1 -2 2
例1  设 A= -2 -2 4 ,
2 4 -2
- 1
求正交矩阵 T, 使 T AT 为对角矩阵 .
解   先求特征值 .
λ- 1 2 -2
    |λI - A| = 2 λ+ 2 -4
- 2 -4 λ+ 2
-2 -4 λ+ 2
= - 0 λ- 2 λ- 2
0 - 2(λ- 2) (λ+ 3) (λ- 2 )/ 2
- 2 -4 λ+ 2
= - (λ- 2) 2 0 1 1
0 -2 (λ+ 3)/ 2
2
1 1
= - (λ- 2) ( - 2 )
0 (λ+ 7)/ 2
2
= (λ- 2) (λ+ 7 ) = 0.
得 λ1 = 2 ,λ2 = - 7 .
对于特征值 λ1 = 2( 二重 ) , (λ1 I - A) x = 0, 即
1 2 -2 x1 0
2 4 -4 x2 = 0 .
-2 -4 4 x3 0
5 .4 实对称矩阵的对角化 239

因为实对称矩阵的二重特 征值 必对应 两个 线性无 关的 特征 向量 ,


所以 r ( (λ1 I - A) ) = 1 , 解第 1 个方程可得 Nλ1 的基 , 即由
T T
x1 + 2 x2 - 2 x3 = 0 , 得 : x1 = ( - 2 , 1 , 0 ) , x2 = ( 2 , 0 , 1) .
用施密特正交化方法求 Nλ1 的单位正交基 : 先正交化 , 取
T
β1 = x1 = ( - 2 , 1 , 0 ) ,
T
( x2 ,β1 ) -4 2 4
β2 = x2 - β1 = (2 , 0, 1) T - ( - 2 , 1, 0) T = , ,1 .
(β1 ,β1 ) 5 5 5
再将 β1 ,β2 单位化 , 得
T T
-2 5 5
γ1 = , ,0 ,     γ2 = 2 5 , 4 5 , 5 .
5 5 15 15 3
对于特征值 λ2 = - 7 , 由 (λ2 I - A) x = 0, 即
-8 2 -2 x1 0
2 -5 -4 x2 = 0 ,
-2 -4 -5 x3 0
解得特征子空间 Nλ2 的一组基 x3 = ( 1 , 2 , - 2) T , 将其单位化 ,
得 Nλ2 的单位正交基
γ3 = (1/ 3 , 2/ 3 , - 2/ 3 ) T .
取正交矩阵
2 5 2 5 1
5 15 3

T= (γ1 ,γ2 ,γ3 ) = - 5 4 5 2 ,


5 15 3
5 -2
0
3 3
则有 T- 1 AT= diag (λ1 ,λ2 ,λ3 ) = diag( 2 , 2 , - 7) .
例 2   设 A 是 n 阶正交矩阵 ,λ是 A 的实特 征值 , x 是 A 的 对
T
应于λ的特征向量 .证明 :λ只能是±1 , 且 x 也是 A 的特征向量 .
证   由 A 是正交矩阵知 , AT = A - 1 .
240 第 5 章 特征值和特征向量 矩阵的对角化

设 Ax= λx ( x≠0,λ≠0 , 因为 | A| = ± 1 ≠0 ) , 两 边转置 再右 乘


x( 注意 A - 1 x =λ- 1 x) , 得
T T T
x A x =λx x,
即 xT A- 1 x = xT λ- 1 x = λxT x,
- 1 T
从而 (λ - λ) x x = 0.
因为 λ是 A 的实特 征值 , x 也是实 的特征 向量 , x≠0, 所 以 , xT x >
- 1
0 , 故 λ =λ,λ只能是±1.
当 λ= ± 1 时 , 在 Ax = λx = ( ±1 ) x( x≠0) 的 两边 左 乘 AT ( 注
T
意到 A A = I) , 得
AT Ax = ( ±1 ) AT x,
T T
即 x = ( ±1) A x 或 A x = ± x,
T
所以 , x 也是 A 的特征向量 .
1 1 a 1
例 3 ( 研 5 -19 )   设 A= 1 a 1 ,β= 1 , 已知线性方程
a 1 1 -2
组 Ax= β有解但不惟一 .
(1 ) 求 a 的值 ; ( 2) 求正交矩阵 Q, 使 Q - 1 AQ 为对角矩阵 .
解   ( 1 ) 由 A x = β有解但不惟一 , 知 r ( A,β) = r ( A) < 3 .用
初等行变换求秩 ( A,β) .
1 1 a 1 1 1 a 1
( A,β) = 1 a 1 1 → 0 a- 1 1- a 0 .
a 1 1 -2 0 0 ( a - 1) ( a+ 2) a+ 2
由 r ( A,β) = r ( A) < 3, 得 a = - 2 .
λ- 1 -1 2
(2 ) 由 |λI - A| = -1 λ+ 2 -1
2 -1 λ- 1
=λ(λ- 3 ) (λ+ 3 ) = 0 ,
得 λ1 = 0 ,λ2 = 3 ,λ3 = - 3 .
5 .4 实对称矩阵的对角化 241

T
当 λ1 = 1 时 , 求解 ( 0 I - A) x = 0, 得特征向 量 x1 = ( 1, 1 , 1 ) ,
T
1 1 1
单位化得 ε1 = , , ;
3 3 3
当 λ2 = 3 时 , 求 解 ( 3 I - A) x = 0, 得 特 征 向 量 x2 = ( - 1, 0,
T
T - 1 1
1 ) , 单位化得 ε2 = , 0, ;
2 2
当 λ3 = - 3 时 , 求 解 ( - 3 I - A) x = 0, 得 特 征 向 量 x3 =
T
1 -2 1
( 1 , - 2 ,1 ) , 单位化得 ε3 = , ,
T
.
6 6 6
1 -1 1
3 2 6
1 - 2
取 Q= (ε1 ,ε2 ,ε3 ) = 0 ,
3 6
1 1 1
3 2 6
0
则 Q - 1 AQ= 3 .
-3
例 4   三阶实对称矩阵 A 的特征 值为λ1 = - 1 ,λ2 = λ3 = 1 , 对
T
应于 λ1 = - 1 的特征向量为 α1 = ( 0, 1, 1 ) , 求 A .
解   实对称矩 阵 A 一定 与对 角矩 阵 Λ = diag ( - 1 , 1 , 1 ) 相
似 .不同的特征值对应 的特征 向量 正交 .在与 α1 正 交 的平 面上 取
两个线性无关的向量 , 如 α2 = ( 1, 0, 0 ) T ,α3 = ( 0, 1, - 1 ) T , 则
Aα1 = - α1 , Aα2 = α2 , Aα3 = α3 , 写成矩阵等式得
-1
A(α1 ,α2 , α3 ) = (α1 ,α2 ,α3 ) 1 .
1
- 1 - 1
记 P = (α1 , α2 ,α3 ) , 则 P AP = Λ, 故得 A = PΛP .
242 第 5 章 特征值和特征向量 矩阵的对角化

注意 : α1 , α2 , α3 是正交的向量组 , 只需单位化即得单位正交
1 1
向量组 : γ1 = ( 0, 1, 1 ) T , γ2 = α2 ,γ3 = ( 0 , 1, - 1 ) T .记 T =
2 2
(γ1 , γ2 ,γ3 ) , 则 T = T ( 省略了求逆矩阵的运算 ) , 且 T- 1 AT =
- 1 T

Λ, 从而得
- 1 T
    A = TΛT = TΛT
0 1 0 1 1
0
1 1 -1 2 2
0
= 2 2 1 1 0 0
1 1 1 1 1
0 - 0 -
2 2 2 2
1 0 0
= 0 0 -1 .
0 -1 0
例 5   设 A 是 n 阶实对称矩阵 , 证明 :
T
(1 ) 若存在可逆矩阵 B, 使得 A = B B, 则 A 的主对角线上 的
元素全部大于零 ;
( 2 ) 设 α1 ,α2 , … ,αn 是 A 的 n 个 正交 单位 特征 向量 , 对应 的
特征值是 λ1 ,λ2 , … , λn , 则
A =λ1 α1 α1T +λ2 α2 α2T + … +λnαnαTn ;
(3 ) 当 n = 3 时 , 已知 A 的特征值为λ1 = 1 ,λ2 = 2 ,λ3 = 3 , 对 应
的特征向量为
T T T
α1 = ( 1 , 1 , 1) ,α2 = ( 1 , - 2, 1 ) ,α3 = ( 1 , 0 , - 1 ) , 求 A .
解   令 B= ( bi j ) , 则
b11 b21 … bn1 b1 1 b1 2 … b1 n
b12 b22 … bn2 b2 1 b2 2 … b2 n
A = BT B =
… … … … … …
b1 n b2 n … bn n bn1 bn2 … bn n
5 .4 实对称矩阵的对角化 243

n n n

∑b ∑b … ∑b
2
i1 i1 bi2 i1 bi n
i= 1 i= 1 i=1

n n n

∑b ∑b ∑b
2
i1 bi2 i2 … i2 bi n
= i= 1 i= 1 i=1 ,
… … …
n n n

∑b ∑b ∑b
2
i1 bi n i2 bi n … in
i=1 i= 1 i =1

n n n

A = B B 的对角元 为 ∑ b , ∑ b , … , ∑ b2i n .因为 B 可逆 ,


T 2 2
i1 i2
i =1 i= 1 i= 1

det B≠ 0 .所以 b1 j , b2 j , … , bn j ( j = 1 , … , n) 不全为 0 . 所以 , A 的主


对角线上的元素全部大于零 .
( 2 ) 令 T= (α1 , α2 , … , αn ) , 因为 α1 ,α2 , … ,αn 是单位正交向
量组 , 所以 T 是正交矩阵 , 即
T- 1 = TT = (α1 ,α2 , … ,αn ) T , 且
- 1
T AT = Λ= diag (λ1 ,λ2 , … ,λn ) , 所以
λ1 α1T
λ2 T
- 1 T α2
A = TΛT = TΛT = (α1 ,α2 , … ,αn ) ,
w …
λn T
αn
从而得 A =λ1 α1 α1T +λ2 α2 α2T + … + λnαnαTn .
(3 ) 由于 α1 ,α2 ,α3 是不同的特征 值对应的 特征 向量 , 所以 正
交 , 即 α1 , α2 ,α3 是正交向量组 .
αi
令 γi =   ( i = 1, 2, 3) ,
| αi |
1 T 1 T 1 T
则     γ1 = (1 , 1 , 1) ,γ2 = (1 , - 2 , 1 ) ,γ3 = (1, 0, - 1)
3 6 2
是正交单位特征向量组 , 由 ( 2 ) 得
244 第 5 章 特征值和特征向量 矩阵的对角化

13 - 2 -5
1
A =λ1 γ1 γ +λ2 γ2 γ +λ3 γ3 γ =
T T T
1 2 3 -2 10 -2 .
6
-5 - 2 13
例 6 ( 习题 18)   设 n 阶矩阵 A 的元素均为 1 .
( 1 ) 求 A 的特征值和特征向量 ;
( 2 ) A 可否对角化 ? 若可以 , 求 矩阵 P 使得 P - 1 AP = Λ( 对 角
矩阵 ) ;
(3 ) 若 f ( x) 是 x 的 m 次多项式 , 且常数项为 0 .证明存在 k∈
R , 使得 f ( A) = kA , 并求出 k .
解   (1 ) 由 5 .2 例 9 知 : A 有一 个非 0 特 征值为 行和 n, 0 是
n - 1 重特征值 .
A 的每行行和都是 n , 由
1 … 1 1 n 1
… … … = … =n … ,
1 … 1 1 n 1
可见 λ1 = n 时 , 对应的特 征向量 是元素全 部为 1 的 n 维列 向
T
量 , 记为 xn = ( 1 , 1 , … , 1 ) ;
λ2 = 0 时 , 由 (λ2 I - A) x = - A x = 0 的同解方程
x1 + x2 + … + xn = 0 ,
得 n - 1 个线性无关的特征向量为 :
T T
x1 = ( - 1 , 1 , 0 , … , 0) ,   x2 = ( - 1 , 0 , 1 , … , 0) ,   … ,
T
  xn - 1 = ( - 1 , 0 , … , 0 , 1) .
-1 -1 … -1 1
1 0 … 0 1
(2) 令 P = ( x1 ,…, xn - 1 , xn ) = 0 1 … 0 1 ,
… … w … …
0 0 … 1 1
- 1
则 P AP = diag (0 , … , 0 , n) = Λ .
5 .4 实对称矩阵的对角化 245

- 1
( 3 ) 由 ( 2 ) 的 结果 , A = PΛP , A 的特 征值为 λ, 则 f ( A) 的 特
征值为 f (λ) , 即 f ( n) 和 f ( 0) = 0( n - 1 重 ) .
因为当 k, l∈N 时 , 有
Ak = PΛk P - 1 ,
k l k l - 1
和 aA + bA = P( aΛ + bΛ ) P .
- 1
所以 f ( A) = P f (Λ) P ,
0
0 - 1
即 f ( A) = P P

f ( n)
0
f ( n) 0 - 1
= P P
n w
n
f ( n) - 1
= PΛP = kA .
n
f ( n)
其中 k = , 所以 , f ( A) = kA .
n
例 7   试验性生产线每年一月进行 熟练工 和非熟练 工的人 数
统计 , 然后将 1/ 6 熟练工支援其他部门 , 其缺额由招收新的非熟练
工补齐 .新老非熟练工 经培训 及实 践至年 终考 核有 2/ 5 成 为熟 练
工 .设第 n 年 一 月 份 统 计 熟 练 工 和 非 熟 练 工 所 占 百 分 比 分 别 为
T
x n , yn , 记成向量 xn = ( xn , yn ) .
xn + 1 xn
(1 ) 求 与 的 关 系 式, 并 写 成 矩 阵 形
yn + 1 yn

xn + 1 xn
式 =A ;
yn + 1 yn
246 第 5 章 特征值和特征向量 矩阵的对角化

4 - 1
( 2) 验证 η1 = ,η2 = 是 A 的两个线性无关的特征向
1  1
量 , 并求出相应的特征值 ;
1
x1 2 xn + 1
(3 ) 当 = 时,求 .
y1 1 yn + 1
2
解   (1 ) 第 n 年熟练工和非熟练工所占百分比分别为 x n , yn ,
第 n + 1 年 的 熟 练 工所 占 百 分 比 xn + 1 是 由 上 一 年 留下 的 熟 练 工
5 1
xn 加上新招的 xn 和上一年非熟练 工 y n 两 者经 培训考 核后 的
6 6
5 2 1
2/ 5( 成为 熟 练工 ) 组 成 , 即 xn + 1 = xn + xn + yn ; 第 n + 1
6 5 6
1
年的非熟 练工 所占百 分比 yn + 1 是 由新 招的 xn 和 上一年 非熟 练
6
工 y n 两者经培训考核后余下的 3/ 5( 为非熟 练工 ) 组成 , 即 yn + 1 =
3 1
xn + yn .所以
5 6
5 2 1 9 2
xn + 1 = xn + xn + yn = xn + yn ,
6 5 6 10 5
3 1 1 3
yn + 1 = xn + yn = xn + yn .
5 6 10 5
9 2 9 2
xn + 1 10 5 xn xn 10 5
即  = =A ,   其中 A = .
yn + 1 1 3 yn yn 1 3
10 5 10 5
(2 ) 求特征值 .法 1: 由
9 2
λ- -
10 5 1
|λI - A| = = (λ- 1 ) λ- = 0,
1 3 2
- λ-
10 5
5 .4 实对称矩阵的对角化 247

1 4
得     λ1 = 1 ,λ2 = .可 以求得 λ1 = 1 对应的 特征向 量 η1 = ,
2 1

1 -1
λ2 = 对应的特征向量 η2 = .
2  1
法 2:
9 2
10 5 4 4
Aη1 = = = η1 ,
1 3 1 1
10 5
9 2
10 5 -1 1 -1 1
Aη2 = = = η2 .
1 3  1 2  1 2
10 5
1
所以 ,λ1 = 1 为一 个特 征 值 , 对 应的 特 征向 量 为 η1 ,λ2 = 为
2
另一个特征值 , 对应的特征向量为 η2 .
由于 λ1 ≠λ2 , A 与对角矩阵相似 , 令 P = (η1 ,η2 ) , 则
1
P- 1 AP = diag 1 , .
2

xn + 1 xn 2
xn - 1 n
x1
(3 ) =A =A =…=A ,
yn + 1 yn yn - 1 y1
1
1 1 1
由 (2 ) 的结果得   A = P 1 P - 1 , 其中 P- 1 = ,
5 -1 4
2
n
1
n - 1
所以           A = P 1 P
2
1
1 4 -1 n
1 1
= 1
5 1 1 -1 4
2
248 第 5 章 特征值和特征向量 矩阵的对角化

n n
1 1
4+ 4 -4
1 2 2
= n n .
5 1 1
1 - 1+4
2 2
n
1 1
8 -3
xn + 1 n 2 1 2
于是   =A = n .
yn + 1 1 10 1
2+3
2 2
这里的 3 个小题实际上给 出求 xn + 1 , yn + 1 这 个 应用 问题 的 3
个步骤 .一般来说 , 这类应用 问题 的解 决大都 要经 过这 3 个 步骤 ,
即先列出 xn + 1 , yn + 1 的 表 达式 ; 再 用一 个 向量 αn 的等 式 表 示之 ,
然后找到 αn + 1 与 αn 的递推关 系式 , 一 般是用 矩阵 表示 ; 若 矩阵 与
对角矩阵相似 , 求矩阵的 n 次方就是求特征值 , 特征向量和做矩阵
乘法的运算 .

5 .5 部分疑难习题和补充题的题解
1 ( 习题 4)   设 x1 , x2 , x3 是矩阵 A 的不同特征值λ1 ,λ2 ,λ3 对应的特征
向量 , 证明 x1 + x2 + x3 不是 A 的特征向量 .
证   反证法 . 设 x1 + x2 + x3 是 A 的特征向量, 对应的特征值为 λ, 即
        A( x1 + x2 + x3 ) = λ( x1 + x2 + x3 ) ,
Ax1 + Ax2 + Ax3 = λ1 x1 + λ2 x2 +λ3 x3 =λ( x1 + x2 + x3 ) ,
得 (λ- λ1 ) x1 + (λ- λ2 ) x2 + (λ- λ3 ) x3 = 0 .
因为不同特征值对应的特征 向量 x1 , x2 , x3 线性无 关 ,所 以 λ- λi = 0 ,
即 λ=λ1 = λ2 = λ3 , 这与题设矛盾 , 故 x1 + x2 + x3 不是 A 的特征向量 .
2 ( 习题 5 )   证明对合矩阵 A( A2 = I) 的特征值只能是 1 或 - 1 .
证   设 λ是 A 的一个特征值 ,即
Ax =λx   ( x≠0) ,
则 x= Ix = A2 x= λ2 x   ( x≠0 ) .
所以 ,λ2 = 1 , 即 A 的特征值λ= 1 或 - 1 .
3 ( 习题 10)   设 B = P - 1 A P, x 是 矩阵 A 属于特 征值λ0 的特 征向量 .
证明 : P - 1 x 是矩阵 B 的对应其特征值λ0 的一个特征向量 .
5 .5 部分疑难习题和补充题的题解 249

证   由 Ax =λ0 x ( x≠0, P- 1 x≠0) ,得


      B( P - 1 x) = ( P - 1 A P ) ( P - 1 x)
= P - 1 A x = P - 1 (λ0 x) = λ0 ( P- 1 x) .
所以 , P - 1 x 是矩阵 B 的对应其特征值 λ0 的一个特征向量 .
A 0 B  0
4 ( 习题 12 )   设 A~ B, C~ D, 证明 : ~ .
0  C 0  D
证   由 A~ B, C~ D,可知存在 P, Q,使得 B = P- 1 A P, D= Q- 1 C Q,
所以
- 1
P 0 A  0 P  0 P- 1 AP 0 B  0
= = .
0  Q 0  C 0  Q 0 Q- 1 CQ 0  D
于是命题得证 .
5 ( 习题 13 )   证明 m 阶矩阵
0 1
0 w
J=
w 1
0
只有零特征值 , 其特征子空间是 Rm 的一维子空间 , 并求它的基 .
解   由 |λI - J| =λm = 0 , 可知 λ= 0 , 即 A 只有零特征值 .
由 Jx = 0 x = 0 及 r ( J) = n - 1 , 得 x = ( 1 , 0 , … , 0 ) T 是 λ= 0 对应的特征
子空间的基 .所以 , 特征子空间是R m 的一维子空间 .
6 ( 习题 20)   设 三阶实 矩阵 A 有 二重 特征 值λ1 , 如 果 x1 = ( 1 , 0 , 1 ) T ,
x2 = ( - 1 , 0 , - 1) T , x3 = (1 , 1 , 0) T , x4 = ( 0 , 1 , - 1 ) T 都是 对 应于 λ1 的 特
征向量 , 问 A 可否对角化 ?
解   三阶实矩阵的特征方程是三次方程, 必有一个实根 λ0 .又λ1 是 A 的
二重特征值, 所以 λ0 是单根 .设对应于 λ0 的特征向量为 x0 .λ1 对应的线性无
关( 不成比例) 的特征向量有两个 , 如 x1 , x3 ( 或 x1 , x4 , 或 x3 , x4 , 或 x2 , x3 )
( 注意不可能有多于两个的线性无关的特征向量 ) .不同特征值对应的特 征向
量线性无关 . 所以 ,三阶矩阵 A 有 3 个线性无关的特征向量 , A 可对角化 .
- 3  2 x4 - 1 x
7 ( 习题 21 )   已知 A= .若 f ( x) = , 求 f ( A) .
- 2  2 x3 x6 + 1
解   由 f ( x ) = ( x4 - 1 ) ( x6 + 1 ) - x4 , 得
250 第 5 章 特征值和特征向量 矩阵的对角化

f ( A) = ( A4 - I) ( A6 + I) - A4 = A10 - A6 - I .
λ+ 3 -2
由 |λI - A| = = 0 , 得 A 的特征值λ1 = 1 ,λ2 = - 2 .λ1 ,λ2 对 应的
2 λ- 2
1 2
特征向量分别为 : x1 = , x2 = .
2 1
1  2 1
   令 P= , 则 P - 1 AP = = Λ.
2  1 - 2
于是     P - 1 Ak P = Λk ,   即   Ak = PΛk P - 1 .从而
        f ( A) = PΛ10 P- 1 - PΛ6 P - 1 - PP- 1 = P( Λ10 - Λ6 - I) P- 1
= P f ( Λ) P - 1
1 1 1
=P 10
- 6
- P- 1
( - 2) ( - 2) 1
-1
=P 10 6
P- 1
2 -2 - 1

1 1 2 - 1 1 - 2 1279 - 640
= = .
-3 2 1 959 - 2 1 640 - 321
3 4 0 0
4 - 3 0 0
8 ( 习题 22)   设 A = , 求 Ak ( k 为正整数) .
0 0 2 4
0 0 0 2
解  令
B 0 3 4 2 4
A= , 其中 B = , C= ,则
0 C 4 - 3 0 2
Bk 0
Ak = .
0 Ck
B 的特征值为λ1 = 5 ,λ2 = - 5 , 它们对应的特征向量分别为
2  1
x1 = ,   x2 = .
1 -2
2  1
令 P = ( x1 , x2 ) = ,则
1 - 2
5 .5 部分疑难习题和补充题的题解 251

5  0
P - 1 BP = ,
0 - 5
5k 0
   B =P k
P-1
k
0 ( - 5)

1 2 1 5k 0 -2 -1
= -
5 1 - 2 0 ( - 5) k -1 2
2 1 1 0 - 2 - 1
= - 5k - 1
1 - 2 0 ( - 1 )k - 1 2

k-1
4 + ( - 1 )k 2 - 2( - 1 ) k
=5 .
2 + 2( - 1 ) k 1 + 4( - 1 ) k
0 4
C 不可对角化 , 记 C= 2 I + D, 其中 D = , 则 D2 = 0 .
0 0
用二项式展开计算 Ck 得

k k k 1 k- 1
2k 4 k2 k - 1
C = (2 I + D) = 2 I + C 2 k D+ 0 = .
0 2k
Bk 0
于是 ,     A =k

0 Ck
4・5k - 1 - ( - 5) k - 1 2・5 k - 1 + 2( - 5) k - 1 0 0
k- 1 k- 1 k- 1 k-1
2・5 - 2( - 5) 5 - 4( - 5) 0 0
= .
0 0 2k 4 k2 k - 1
0 0 0 2k
9 ( 习题 26)   设 n 阶实对称矩阵 A 的特征值λi ≥0 ( i = 1 , … , n) .证明 :
存在特征值都是非负数的实对称矩阵 B,使得 A = B2 .
证   A 为实对称矩 阵 , 故 存在 T ( 正交 阵 ) 使 T - 1 AT = diag (λ1 , … ,λn ) .
于是
A = T diag (λ1 , … ,λn ) T - 1 .
注意 : λi ≥0( i = 1 , … , n) , 得

A = Tdiag( λ1 , … , λn ) T - 1 Tdiag( λ1 , … , λn1 ) T - 1 = B2 ,

其中实对称矩阵 B = Tdiag( λ1 , … , λn ) T - 1 的特征值 λi ≥0 ( i = 1 , … , n) .


10 ( 习题 27 )   设 A 为 n 阶实对称幂等矩阵 , r ( A) = r, 求 | A - 2 I | .
252 第 5 章 特征值和特征向量 矩阵的对角化

解   利用 25 题结果 : T - 1 AT = dia g(1 , … , 1 , 0 , … , 0) ( 其中 1 有 r 个 ) .


由     T - 1 ( A - 2 I) T = T - 1 AT - 2 I = diag( - 1 , … , - 1 , - 2 , … , - 2) ,
| A - 2 I | = | T - 1 ( A - 2 I) T| = ( - 1 ) r ( - 2) n - r
得 .
11 (补充题 30)   设 A = ( aij )4 ×4 , 已知 0 是 A 的二重特征值 ,1 是 A 的单
重特征值 , 求矩阵 A 的特征多项式 det (λI - A) .
4 4 4 4

解   利用 ∑λ = ∑ a
i= 1
i
i= 1
ii , 即 0 + 0 + 1 +λ4 = ∑a
i= 1
ii , 得λ4 = ∑a
i= 1
ii - 1.

所以
4

∑a
2
| λI - A | = λ (λ- 1) λ - ii +1 .
i= 1

12 ( 补充题 32)   设λ1 ,λ2 , … ,λn 是矩阵 A = ( aij ) n×n 的 n 个特征值 .证明
n n n

∑λ ∑∑ a
2
i = ik a ki .
i=1 i= 1 k = 1

    证   由 | λI - A | = (λ - λ1 ) (λ- λ2 )… (λ - λn ) , 知
    | λI + A | = ( - 1) n | - λI - A |
= ( - 1) n ( - λ- λ1 ) ( - λ - λ2 ) …( - λ- λn ) ,
于是
    | λI - A | | λI + A | = | λ2 I - A2 |
= (λ2 - λ21 ) (λ2 - λ22 ) …(λ2 - λ2n ) ,

| μI - A2 | = (μ - λ21 ) (μ- λ22 )… (μ - λ2n ) .
n

所以 , λ , … ,λ 是 A 的特征值 .∑λ2i 是 A2 的迹 .即
2 2 2
1 n
i =1
n n n

∑λ ∑∑ a
2 2
i = t rA = ik a ki .
i =1 i= 1 k = 1

13 (补充题 34)   设 A, B 都是 n 阶矩阵 , A有 n 个互不相同的特征值 .证


明 : AB = BA 的充分必要条件是 A 的特征向量也是 B 的特征向量 .
证   必要性 :因 A 有 n 个互不相同的特征值 , 所以 A 有 n 个线性无关的
特征向量 , 故 A 可对角化 .
设         Axi = λi xi   ( xi ≠ 0,   i = 1 , 2 , … , n) ,
则 A( Bxi ) = B( Axi ) = B(λi xi ) = λi ( Bxi ) ,
5 .5 部分疑难习题和补充题的题解 253

即 Bxi ∈ Vλi ( A) .    
因 λi 是 A 的单重特征值 , 对应的 特征子 空间 是一 维的 , 所以 , Vλi ( A) 中
任两个向量成比例 ,于是
Bxi = μi xi   ( i = 1 , 2 , … , n) .
故 xi 也是 B 的对应于特征值μi ( i = 1 , 2 , … , n) 的特征向量 .
充分性 : 因为 A 可对角化 ,所以 , A 有 n 个线性无关的特征向量 , 记 A 的
n 个线性无关的特征向量为 x1 ,… , xn ( 也是 B 的特征向量 ) , 其对应 的 A 和
B 特征值分别为λ1 , … ,λn ;μ1 , … ,μn , 即
Axi =λi xi ,     Bxi = μi xi     ( xi ≠0 ,   i = 1 , 2 , … , n) .
令 P = ( x1 ,… , xn ) , 则
P - 1 AP = dia g(λ1 ,λ2 , … ,λn ) = Λ1 ,
P- 1 BP= diag(μ1 ,μ2 , … ,μn ) = Λ2 .
利用对角矩阵 Λ1 与 Λ2 的乘积可以交换 ,得到
( P- 1 AP) ( P - 1 BP) = Λ1 Λ2 = Λ2 Λ1 = ( P- 1 BP) ( P - 1 AP) .
上式两边左乘 P, 右乘 P - 1 , 即得 AB = BA .
14 ( 补 充 题 35 )   设 A, B 都 是 n 阶 矩 阵 , B 的 特 征 多 项 式 f (λ) =
|λI - B| .证明 : f ( A) 可逆的充要条件为 B 的任一特征值都不是 A 的特征值 .
证   必要性 : 设 B 的特征值为λ1 ,λ2 , … ,λn , 即 B 的特征多项式为
f (λ) = |λI - B| = (λ- λ1 ) (λ- λ2 ) … (λ- λn ) ,
于是 , f ( A) = ( A - λ1 I) ( A - λ2 I) … ( A - λn I) , 由 f ( A) 可逆 , 即
  | f ( A) | = | ( A - λ1 I) ( A - λ2 I) … ( A - λn I ) |
        = | A - λ1 I | | A - λ2 I| … | A - λn I | ≠0 ,
可得   | A - λi I | = ( - 1) n |λi I - A| ≠0   ( i = 1 , 2 , … , n) .
所以 , B 的特征值λi ( i = 1 , 2 , … , n) 都不是 A 的特征值 . 必要性得证 .
其逆亦真 .因为由
|λi I - A| ≠0   ( i = 1 , 2 , … , n) ,
即得 | f ( A) | = | A - λ1 I | | A - λ2 I| … | A - λn I | ≠0 .
所以 , f ( A) 可逆 .
注意   不能把 A 代入 f (λ) = |λI - B| 中的 λ而去求 f ( A) = | AI - B| , 这
样 f ( A) 成了一个数而不是矩阵了 .
15 ( 补充题 36 )   证明反对称实矩阵 A 的特征值λ必是零或纯虚数 .
254 第 5 章 特征值和特征向量 矩阵的对角化

T
证   已知 AT = - A, A = A, 得 A = - A .
设 Ax= λx     ( x≠0) ,
T
上式两边取共轭和转置 ,然后再右乘 x,将 A = - A 代入 ,得
( Ax) T x= (λx) T x,
T
( x) T A x= λ( x) T x,
- ( x) T Ax= λ( x) T x,
- λ( x) T x = λ( x) T x .
T
由于 x≠0 时 , x x> 0, 所以, - λ= λ.因此 ,λ必是零或纯虚数 .
16 ( 补充题 37 )   已知 α= ( a1 , … , an ) ,β= ( b1 , … , bn ) 是R n 中两 个非 零
的正交向量 , 证明 :矩阵 A = αT β的特征值全为零 ,且 A 不可对角化 .
证   由 α,β正交 ,知 βαT = 0 , 所以
A2 = αT (βαT )β= αT 0β= 0 .
设 λ为 A 的任意一个特征值 ,即
Ax =λx   ( x≠0) ,

A2 x = λ2 x = 0 x = 0 .
由于 x≠0 , 故 λ= 0 , 所以 A 的特征值全为零 .
    当 λ= 0 时 , 则 (λI - A)x = - Ax = 0 .
    因为 α,β为非零向量 , 故 A = αT β≠0( 零矩阵 ) , 所以 r ( A) ≥1 . 又 r ( A) =
r (αT β) ≤r (β) = 1 , 故 r( A) = 1 . Ax = 0 的基础解系仅含 n - 1 个线性无关的
解向量 , 即 A 没有 n 个线性无关的特征向量 , 所以 A 不可对角化 .
  第 6 章

二 次 型

6 .1 基本要求与内容提要

1  基本要求

(1 ) 理解实二次 型与 实对 称矩 阵 间的 一一 对应 关系 ; 熟练 掌
握二次型的矩阵表示
f ( x) = xT Ax,   其中 AT = A .
(2 ) 熟悉矩阵 A 合同 ( 或相合 ) 于 B 的 定义 , 理 解合同 关系 是
等价关系 .
( 3 ) 熟练掌握化二次型 xT Ax 为平 方和 ( 标准 形 ) 或求 实对 称
矩阵 A 的相合标准形的 3 种方法 : 正交变换法 ; 配方法 ; 和同型 初
等行、列变换法 .
( 4 ) 了解惯性定理 , 会 求矩阵 A 的正、负惯 性指 数 和符 号差 ,
会求二次型的规范形 .
( 5 ) 熟练掌握正定二次型 ( 正定矩阵 ) 的定义和判别方法 .
( 6 ) 熟悉实 对称 矩 阵 A 正 定 ( 二 次 型正 定 ) 的 各种 等 价 命 题
( 正定的充要条件 ) .
( 7 ) 理解 A 正定的必要条件 : aii > 0 ( i = 1 , … , n) ; det A> 0 .
(8 ) 会利用正交变换化二次型为平方和 及坐标平 移方法判 别
一般二次曲线和曲面的类型 .
256 第 6 章 二次型

2  内容提要

(1 ) 实二次型 xT Ax( 简 称二 次型 ) 与 实对称 矩 阵 A 之 间的 一


一对应 .
n 元二次型 ( n 元二次齐次函数 )
n

∑a ∑
2
f ( x1 , x2 , … , xn ) = ii xi + 2 aij x i x j
i= 1 1 ≤ i < j≤ n

可以用矩阵乘积 xT Ax 来表示 ( 其中 A = ( aij ) n× n 为实对称 矩阵 , 称


T
为二次型 f ( x1 , x2 , … , xn ) 对应的矩阵 , x = ( x1 , x2 , … , xn ) ) .
n

∑a
T 2
若 A 为对角矩阵 , 对应的二次型 x Ax = ii x i 为平方和 .
i=1
T
二次型 x Ax 经过坐标变换 ( 或说非退化线性变换 ) x = Cy ( C
为可逆矩阵 ) , 化为 y1 , y2 , … , yn 的二次型
T T T T T
x Ax = y ( C AC) y = y By,   其中 B = C AC .
( 2 ) 矩阵 A, B 合同 ( 或相合 ) ( AD B) , 即存 在可 逆 矩阵 C, 使
T
得 B= C AC .
合同关系是等价关 系 , 即具 有 : 自 反性 ( " A, AD A) ; 对 称 性
( 若 AD B, 则 BD A ) ; 和传递性 ( 若 AD B, BD C, 则 AD C) .
T
( 3 ) 实二次型 x Ax 化为平方和 ( 标准形 ) , 即求实对称矩阵 A
的相合标 准形 有 3 种方法 : 正交变 换法 ; 配 方法 ; 和 同型初 等行、
列变换法 .
① 正交变换法 ( 主轴定理 )   实对称 矩阵 A, 必存在正 交矩 阵
Q, 使得
- 1 T
Q AQ = Q AQ = diag(λ1 ,λ2 , … ,λn ) .
正交变换法得到的 A 的相合标准 形也 是 A 的 相似标 准形 ( 见第 5
章 5 .4) .若令 x = Qy, 则二次型化为平方和
f ( x1 , x2 , … , xn ) = xT Ax = yT ( QT AQ) y
2 2 2
= λ1 y1 +λ2 y2 + … +λn y n ,
6 .1 基本要求与内容提要 257

2 2 2
其中 λ1 ,λ2 , … ,λn ( y1 , y2 , … , yn 项的系 数 ) 必 是 A 的特 征值 .正 交
变换所得标准形 ( 不计 λ1 ,λ2 , … ,λn 的排序 ) 是惟一的 .
2 2
② 配方法   先将含 x1 及 x1 x i 的项配成完全平方 , 再 将含 x2
及 x2 xi 的项配成完全平方 , 继续之 , 直到化为平方和为止 .即
2
f ( x1 , x2 , … , xn ) = a1 1 ( x1 + b12 x2 + … + b1 n x n ) +
2 2
a′
22 ( x2 + b23 x3 + … + b2 n x n ) + … + a′
nn xn .

如果 f ( x1 , x2 , … , xn ) 中没 有 x21 , … , x2n 的项 , 但有 a12 x1 x2


的项 , 则先令 x1 = y1 + y2 , x2 = y1 - y2 , xi = yi ( i = 3 , … , n) , 将 二
次型 f ( x1 , x2 , … , xn ) 化 为二 次型 g ( y1 , y2 , … , yn ) , 然 后再 用 配
方法 .
③ 同 型 初 等 行、列 变 换 法   设 二 次 型 f ( x1 , x2 , … , xn ) =
T
x Ax, 对 A 做若干同型的初等行、列变换化 A 为对角形 , 对单位矩
阵 I 只做同样的初等列变换 .即
A 同型行列变换 Λ
→ ,
I 列变换 C
T
则     C AC= Λ= diag( d1 , d2 , … , dn ) ( 对角阵 ) .
令 x = Cy, 则 xT Ax = yT Λy = d1 y21 + d2 y22 + … + dn y 2n .
(4 ) 惯性定理 , 二次型的规范形 .
实二次型 xT Ax 化为平方和 ( 实对称矩阵 A 相合于对角 阵 ) 所
得标准形不是惟一的 .但 是对角 元 中正 数的 个数 p( 正惯 性指 数 )
和负数的个数 r - p( 负惯性指数 , 其中 r 是 A 的秩 ) 是由 A 惟一确
定的 .符号差 = p - ( r - p) = 2 p - r .
若 A 的正惯 性指 数 = p , 秩 ( A) = r , 则 A 一 定相 合 于对 角 阵
Λ= diag(1 , … , 1 , - 1… , - 1 , 0 , … , 0 ) , 其 中 ( + 1 ) 是 p 个 , ( - 1)
是 ( r - p) 个 , 0 是 ( n - r) 个 .称此对角矩阵为 A 的合同规范形 .
(5 ) 正定二次型 ( 正定矩阵 ) .
n T
若 " x≠0 ( x∈R ) , n 元实二次 型 f ( x1 , x2 , … , xn ) = x Ax 恒
T
大于 0, 则称 x Ax 为正定二次型 , 称对应的矩阵 A 为正定矩阵 .例
258 第 6 章 二次型

2 2 2
如 , f ( x1 , x2 , … , xn ) = d1 x1 + d2 x2 + … + dn x n 正 定的充要 条件 是
d i > 0 ( i = 1 , 2 , … , n) .A = ( aij ) n× n 正 定 的必 要 条 件 是 a ii > 0 ( i =
1 , … , n) , det A > 0 .
( 6 ) 对于 n 阶实对称矩阵 A = ( ai j ) n × n , 下列命题等价 :
① A 是正定矩阵 , 或 xT Ax 是正定二次型 ;
② A 的正惯性指数为 n , 即 AD I;
③ 存在可逆矩阵 P, 使得 A = PT P;
④ A 的 n 个特征值λ1 ,λ2 , … ,λn 都大于零 ;
⑤ A 的 n 个顺序主子式 ( 左上角主子式 ) 都大于零 , 即
a11 … a1 k
| Ak | = … … > 0,   k = 1, 2, …, n .
ak1 … ak k
n T
(7 ) 若 " x≠0( x∈ R ) , 有 f ( x) = x Ax < 0, 称 f ( x) 为负定 二
次型 , 且 称 对 应 的 矩 阵 A 为 负 定 矩 阵 .若 " x≠ 0 ( x∈ R n ) , 有
T T
f ( x) = x Ax≥ 0 ( ≤ 0 ) 且存在 x0 ≠0 使 得 x0 Ax0 = 0, 称 f ( x) 为 半
正定 ( 半 负 定 ) 二 次 型 , 并 称 对 应 的 矩 阵 A 为 半 正 定 ( 半 负 定 )
矩阵 .

6 .2 二次型的定义和矩阵表示 合同矩阵

1  二次型的定 义和矩阵 表示

( 1 ) n 元二次齐次多项式
n

f ( x1 , x2 , … , xn ) = ∑a
i= 1
ii x2i + ∑
1 ≤ i< j≤ n
2 aij x i x j
2
= a11 x1 + 2 a12 x1 x2 + 2 a1 3 x1 x3 + … + 2 a1 n x1 xn +
2
  a22 x2 + 2 a23 x2 x3 + … + 2 a2 n x2 xn + … +
2 2
  an - 1 , n - 1 xn - 1 + 2 an - 1 , n x n - 1 xn + an n x n
6 .2 二次型的定义和矩阵表示 合同矩阵 259

( 其中 aij ∈ F, i, j = 1 , 2 , … , n) 称为数域 F 上的 n 元二次 型 .F = R


时 , 称为实二次型 , 简 称二次型 .令 A = ( aij ) n× n , 其中 a ji = ai j ( 1 ≤
T
i < j≤ n) , x = ( x1 , x2 , … , xn ) , 则
n n

∑∑ a
T
f ( x1 , x2 , … , xn ) = ij x i x j = x Ax .
i=1 j =1

这是二次型的矩阵表示 , 并称实 对称 矩阵 A 为 二次型 对应 的


矩阵 .这样 , 研究二次型的性质 , 就可 转化 为研究 实对 称矩 阵 A 的
性质 .
( 2 ) 必须注意 , 二次型对应的矩阵是实对称 矩阵 .例如三元 二
次型
2 2 2
f ( x1 , x2 , x3 ) = x1 + 2 x2 - x3 + x1 x2 - 2 x1 x3 + 2 x2 x3
1 x1
1 - 1
2  
= ( x1 , x2 , x3 ) 1 x2
2 1
2  
-1 1 - 1 x3
1 1 -2 x1
= ( x1 , x2 , x3 ) 0 2 2 x2
0 0 -1 x3
1 3 1 x1
= ( x1 , x2 , x3 ) -2 2 3 x2 .
-3 - 1 -1 x3
上式中只有第一个实对称 矩阵 才是二 次型 对应的 矩阵 , 其 余两 个
( 可以 有 无 穷 多 个 ) 都 不 是 此 二 次 型 对 应 的 矩 阵 .给 出 二 次 型
f ( x1 , x2 , … , xn ) , 根据 其系 数可 以 写出 A = ( ai j ) n× n , 得 到矩 阵 表
示 : xT Ax .
当 x = ( b1 , b2 , … , bn ) T ∈ R n 时 , 由 xT x = ( x, x) ( x 与 x 的内 积
或模的平方 ) , 可知 xT x≥ 0 , 而 xT x = 0 的充 分 必要 条 件是 x = 0 .
260 第 6 章 二次型

同样 , 二次型也可以用内积表示为
xT Ax = ( Ax) T x = ( Ax, x) .
( 3 ) n 元二次型 xT Ax 经过坐标 变换 x = Cy( C 为 n 阶 可逆 矩
阵 ) 可以化为 y1 , y2 , … , yn 的二次型
xT Ax = ( Cy) T ACy= yT ( CT AC) y = yT By,
其中 B= CT AC 仍然是实对称矩阵 .
n

若 B = diag( b1 , b2 , … , bn ) , 则对应的二次型 y By =∑ bi y2i 为 T

i= 1

纯平方项的和 .
若 x, y 看成是向量 α在两组基 B 1 , B2 下的坐 标 , 则非退化 线
性变换 x = Cy 就 是 坐 标 变 换 公 式 , C 是 从 基 B1 到 基 B2 的 过 渡
矩阵 .

2  矩阵合同关 系及其性 质

实对称矩阵 A, B 的 合同 概 念 及 其 性 质 , 在 内 容 提 要 中 已 陈
述 , 这里不再重复 .不过还要指出两点 .
( 1 ) 任意的实对称矩阵都与对角阵合同 , 但对角阵不惟一 .
( 2 ) 若实对称矩阵 A, B 合 同 , 则 A, B 一定 等价 ( 相抵 ) .若 实
对称矩阵 A, B 相似 , 则 A, B 必合同 .因为 AD B, 则 A, B 的特征 值
λ1 , … ,λn 相同 , 于是 , 存在正交矩阵 Q1 , Q2 , 使得
Q1- 1 AQ1 = Q1T AQ1 = diag (λ1 , … ,λn ) = Q2T BQ2 ,
T - 1 T - 1 T
从而 ( Q2 ) Q1 A( Q1 Q2 ) = Q AQ = B,
其中 Q= Q1 Q2- 1 ,   QT = ( Q2- 1 ) T Q1T = ( Q2T ) - 1 Q1T .
反之不成立 , 即 A, B 合同 不一定 相似 .A, B 合 同而不 相似 的
例子如下 :
1 1
A= ,   B= .
- 1 - 2
6 .2 二次型的定义和矩阵表示 合同矩阵 261

1 -2 0
例 1   问   A= -2 5 1 对应的二次型是 .
0 1 -1
答   f ( x1 , x2 , x3 ) = x21 + 5 x22 - x23 - 4 x1 x2 + 2 x2 x3 .
例 2   问矩阵
2 1 0 3 0 0 1 0 0
A= 1 2 0 ,   B= 0 1 0 与 D= 0 1 0
0 0 3 0 0 3 0 0 3
中哪些相似 ? 哪些合同 ? 为什么 ?
解   B, D 均为对角阵 , 特 征 值就 是 其对 角 元 .B 和 D 的 特 征
值分别为 3, 1, 3 和 1, 1, 3 .特征值不同的矩阵必不 相似 , 所以 B 和
1
D 不相似 .但 B 与 D 是 合 同 的 , 因 为 若 取 C = diag , 1, 1 , 则
3
CT BC = D .
λ- 2 - 1 0
|λI - A | = -1 λ- 2 0
0 0 λ- 3
= (λ- 3) (λ2 - 4λ+ 3) = (λ- 3 )2 (λ- 1)
= 0,
故知 A 的特征值为 : λ1 = λ2 = 3 ,λ3 = 1 , 所以 A 和 D 不相似 .
因为 A 为实对称矩阵 , 必存在正交阵 Q = (γ1 ,γ2 ,γ3 ) , 使得
- 1 T
Q AQ = Q AQ = diag (λ1 ,λ2 ,λ3 ) = diag( 3 , 3 , 1) .
若取 T1 = (γ1 ,γ3 ,γ2 ) , 则
T1- 1 AT1 = T1T AT1 = diag(3 , 1 , 3 ) = B .
所以 , A 与 B 既相似又合同 .从而 , A 与 D 合同而不相似 .
例 3   设矩阵
262 第 6 章 二次型

1 1 1 1 4
1 1 1 1 0
A=   与   B= ,
1 1 1 1 0
1 1 1 1 0
则 A 与 B(   ) .
( A ) 合同且相似 ;             ( B) 合同但不相似 ;
( C) 不合同但相似 ; ( D ) 不合同且不相似 .
答   选 ( A ) .因 为 A 是 实对 称 矩阵 , 所 以 A~ diag (λ1 ,λ2 ,λ3 ,
λ4 ) , 其中λ1 ,λ2 ,λ3 ,λ4 为 A 的特征值 .A 的各行行和都是 4, 得λ1 =
4; 由 r ( A) = 1 , det A = 0 , 得 λ2 = 0 是 A 的 3 重特 征值 ( 4 - r (λ2 I -
A) = 3 ) , 所以 λ2 =λ3 =λ4 = 0 , A~ diag (4 , 0 , 0 , 0) = B .实对 称矩 阵
A 与 B 相似必合同 , 故 ( A ) 成立 .

6 .3 化二次型为标准形

(1 ) 二次型的标准形
T
n 元二次型 f = x Ax 总可以经过坐标变换 x = Cy( C 为 n 阶可
逆矩阵 ) 化 为 y 的 二 次 型 : f = xT Ax = ( Cy) T ACy = yT ( CT AC) y=
yT Λy, 其中 Λ= CT AC= diag( d1 , d2 , … , dn ) 为对 角矩阵 .这时对 应
的二次型为纯平方项之和 , 即
n

∑d y
T T 2
f = x Ax = y Λy = i i ,
i= 1

称平方和的形式为二次型 f 的标 准形 .求 f 的标准 形就是 要求 与


A 合同的对角矩阵 Λ .
( 2 ) 求实二次型 f = xT Ax 的标 准 形 ( 化为 平 方和 ) 有 3 种 方
法 : 正交变换法 ; 配方法 ; 同型初等行、列变换法 ( 在内 容提要中 已
经说过 ) .
正交变换 x= Qy 是保长度, 保角度的变换 , 即 xT x= ( Qy) T Qy=
6 .3 化二次型为标准形 263

T T T T
y ( Q Q) y = y y   ( Q Q = I ) .这 个性 质 在 几何 问 题中 很 有用 .例
如 , 在R 3 中 , 曲面
T
f ( x1 , x2 , x3 ) = x Ax
2 2 2
= a11 x1 + a22 x2 + a33 x3 + 2 a12 x1 x2 +
2 a1 3 x1 x3 + 2 a23 x2 x3 = 1
经正交变换 x = Qy, 得到标准形
f = xT Ax = λ1 y21 + λ2 y22 +λ3 y23 = 1 .
所以由特征值 λ1 ,λ2 ,λ3 的符 号即 可以判 别曲 面的 类型 .若 λ1 ,λ2 ,
λ3 全为正数 , 则上式 是椭 球 面方 程 ; 若 λ1 ,λ2 ,λ3 中一 个 负的 两 个
正的 , 则表示单叶双曲面 ; 一个正的两个负的 , 则表示双叶双曲面 ;
若 λ1 = 0 ,λ2 ,λ3 均 为 正的 , 则 表示 母 线平 行 于 Ox 轴的 椭 圆 柱 面
等 .在R 2 或R3 空间中 , 判 别曲 线 或曲 面 的 类型 时 , 必 须 用正 交 变
换法 ( 即做坐标系旋转 , 保证图形在变换前后不变形 ) , 而不能用配
方法和初等变换法 .
例 1   求三元二次齐次函数
f ( x) = f ( x1 , x2 , x3 ) = 2 x1 x2 - 2 x1 x3 + 2 x2 x3 ( 1)
在 x = ( x1 , x2 , x3 ) T 满足 : xT x= x21 + x22 + x23 = 1 时的最小值 .
解   所给二次型对应的矩阵为
0 1 -1
A= 1 0 1 ,
-1 1 0
λ -1 1
|λI - A| = - 1 λ - 1 = (λ+ 2 ) (λ- 1) 2 = 0 ,
1 -1 λ
得 λ1 = - 2 ,   λ2 = 1 ( 二重 ) .
T
对 λ1 = - 2 , 对应的特征向量为 (1 , - 1 , 1 ) , 单位化后得
T
1 1 1
ξ1 = , - , ;
3 3 3
264 第 6 章 二次型

T T
对 λ2 = 1 , 基础解系为 : (1 , 1 , 0 ) , ( - 1 , 0 , 1) ; 用施密特正 交
化方法 , 得单位正交的特征向量
T T
1 1 1 1 2
ξ2 = , ,0 ,     ξ3 = - , , .
2 2 6 6 6
取正交矩阵
1 1 1
-
3 2 6
1 1 1
Q = (ξ1 ,ξ2 ,ξ3 ) = - ,
3 2 6
1 2
0
3 6
T - 1
则有 Q AQ= Q AQ= diag( - 2 , 1 , 1 ) .
对二次型 (1 ) , 做正交变换 x = Qy, 得
f ( x) = xT Ax = yT ( QT AQ) y = - 2 y21 + y22 + y23 = φ( y) .
T 2 2 2
相应地 , 条件 x x = x1 + x2 + x3 = 1 化为
T T T 2 2 2
y ( Q Q) y = y y= y1 + y2 + y3 = 1 . ( 2)
2 2 2
原题化为 φ( y) = - 2 y1 + y2 + y3 在条件 ( 2) 下求最小值 .由
2 2 2 2 2 2
φ( y) = - 2 y1 + y2 + y3 ≥ - 2( y1 + y2 + y3 ) = - 2 ,
T
得当 y1 = ( ±1 , 0 , 0 ) 时 ,φ( y1 ) = - 2 是 φ( y) 的最小值 .所以
1 1 1 1
-
3 2 6 3
±1
1 1 1 1
x1 = Qy1 = - 0 =± -
3 2 6 3
0
1 2 1
0
3 6 3
时 , f ( x) 取得最小值 fm in = f ( x1 ) = - 2 .
例 2   对例 1 中的 f ( x1 , x2 , x3 ) 用 配方 法 , 将其 化 为标 准形 ,
并求变换矩阵 C .
6 .3 化二次型为标准形 265

解   因为所有的平方项的系数全部 为 0, 所以先 做一个 变换 ,


使之出现非 0 系数的平方项 .令
x1 = y1 + y2 ,
x2 = y1 - y2 ,
x3 = y3 .

x1 1 1 0 y1
x2 = 1 - 1 0 y2 ,
x3 0 0 1 y3
记作
x = C1 y . ( 3)
将 ( 3 ) 式代入 ( 1 ) 式 , 得
      f ( x1 , x2 , x3 ) = 2 x1 x2 - 2 x1 x3 + 2 x2 x3
= 2( y1 + y2 ) ( y1 - y2 ) - 2( y1 + y2 ) y3 +
  2( y1 - y2 ) y3
= 2 y21 - 2 y22 - 4 y2 y3 .
再将含 y22 及 y2 y3 的项配成完全平方 , 得
f ( x1 , x2 , x3 ) = 2 y21 - 2 y22 - 4 y2 y3 = 2 y21 - 2( y2 + y3 ) 2 + 2 y23 .
z1 = y1 , y1 = z1 ,
令 z2 = y2 + y3 ,   即 y2 = z2 - z3 ,
z3 = y3 . y3 = z3 .
y1 1 0 0 z1
或 y2 = 0 1 -1 z2 ,
y3 0 0 1 z3
记作
y = C2 z . ( 4)
则得到二次型的标准形为
266 第 6 章 二次型

2 2
f ( x1 , x2 , x3 ) = 2 z 1 - 2 z2 + 2 z3 .
将 (4 ) 式代入 ( 3) 式 ( 注意 C1 和 C2 都是可逆矩阵 ) , 得到
x = C1 y = C1 ( C2 z) = ( C1 C2 ) z = C z,
其中
1 1 0 1 0 0 1 1 -1
C = ( C1 C2 ) = 1 -1 0 0 1 -1 = 1 -1 1 .
0 0 1 0 0 1 0 0 1
可逆矩阵的乘积仍然可逆 , 所以变换矩阵 C 可逆 .
例 3   用初等变换法重做例 1 .
f ( x) = f ( x1 , x2 , x3 ) = 2 x1 x2 - 2 x1 x3 + 2 x2 x3 . ( 1)
A 同型行列变换 Λ
解 若 → , 则
I 列变换 C
CT AC= Λ= diag( d1 , d2 , … , dn ) .
以下[ i]和 ( j) 分别表示第 i 列和第 j 行 , 则
01 - 1 2 10
10 1 1 01
[ 1] + [2 ] [2] + [1] × ( - 1/ 2)
A - 11 0 ( 1) + (2 ) 0 10 (2) + (1) × ( - 1/ 2)
= [ 1] + [2 ] [ 2 ] + [ 1] × ( - 1/ 2)
I 10 0 1 00
01 0 1 10
00 1 0 01
2 0 0 2 0 0
0 - 1/ 2 1 0 - 1/ 2 0
[ 3] + [2 ] × 2
0 1 0 ( 3) + (2 ) × 2 0 0 2
.
[ 3] + [2 ] × 2
1 - 1/ 2 0 1 - 1/ 2 - 1
1   1/ 2 0 1   1/ 2   1
0 0 1 0 0  1
于是 , 取
6 .3 化二次型为标准形 267

1 - 1/ 2 -1
C= 1 1/ 2 1 ,
0 0 1
1
就有 CT AC = diag 2 , - , 2 , 相应 地做 坐 标变 换 x = Cy, 则
2
二次型化为标准形
1 2
xT Ax= 2 y21 - y2 + 2 y23 .
2
从例 1、例 2 和例 3 可见 , 用 3 种不 同的方法 把同一个 二次 型
化成了 3 个不同的标准形 , 即 同一个 三 阶实 对称 矩阵 A 相合 于 3
个不同的对角阵 :
1
diag( - 2, 1 , 1) ;     diag(2 , - 2, 2) ;     diag 2 , - ,2 .
2
但是它们的对角元中都有两个正数 , 一个负 数 .3 个对角 阵都相 合
于对角阵 diag( 1 , 1 , - 1) .
例 4   求一个正交变换 , 化二次型
2 2 2
f ( x1 , x2 , x3 ) = x1 + 4 x2 + 4 x3 - 4 x1 x2 + 4 x1 x3 - 8 x2 x3 为 标
准型 .
解   所给二次型对应的矩阵为
1 -2 2
A= -2 4 -4 .
2 -4 4

λ- 1 2 -2
2
|λI - A| =  2 λ- 4  4 =λ (λ- 9 ) = 0 ,
-2 4 λ- 4
得 λ1 = 0( 二重 ) ,   λ2 = 9 .
λ1 = 0( 二重根 ) 有两个线性无关的特征向量 .由 (0 I - A) x = 0,

268 第 6 章 二次型

- x1 + 2 x2 - 2 x3 = 0 ,
T T
得基础解系为 : α1 = (2 , 1 , 0 ) ,α2 = ( - 2 , 0 , 1) ; 用施 密特正交 化
方法 , 得正交的特征向量 :
    β1 = (2 , 1 , 0 ) T ,
- 2
-2 2 5
α2 ,α1 -4
β2 = α2 - α2 = 0 - 1 = 4 ,
α1 ,α1 5
1 0 5
1
单位化得
T T
2 1 2 4 5
ξ1 = , ,0 ,     ξ2 = - , , .
5 5 3 5 3 5 3 5
T
λ2 = 9 的特征向量为 ( 1 , - 2 , 2) , 单位化后得
T
1 2 2
ξ3 = , - , .
3 3 3
取正交矩阵
2 -2 1
5 3 5 3
1 4 -2
Q = (ξ1 ,ξ2 ,ξ3 ) = 3 ,
5 3 5
5 2
0
3 5 3
T - 1
则有 Q AQ = Q AQ= diag(0 , 0 , 9 ) .
做正交变换 x = Qy, 得
T T T 2
f ( x1 , x2 , x3 ) = x Ax= y ( Q AQ) y = 9 y3 .
此为所求的标准型 .
例 5   已知二次型
2 2 2
f ( x1 , x2 , x3 ) = 5 x1 + 5 x2 + cx3 - 2 x1 x2 + 6 x1 x3 - 6 x2 x3
的秩为 2 .(1 ) 求参数 c 及二次型对应 矩阵的特 征值 ; ( 2 ) 指 出方 程
6 .3 化二次型为标准形 269

f ( x1 , x2 , x3 ) = 1 表示何种二次曲面 .
解   (1 ) 所给二次型对应的矩阵为
5 -1 3
A= -1 5 -3 .
3 -3 c
由秩 ( A) = 2 , 知 det A = 0 .即 | A| = 24 ( c - 3 ) = 0 , 故得 c = 3 .

λ- 5 1 -3
|λI - A | = 1 λ- 5 3 =λ(λ- 4) (λ- 9 ) = 0 ,
-3 3 λ- 3
得 λ1 = 0 ,λ2 = 4 ,λ3 = 9 .
(2 ) 对于实对称矩阵 A, 必存在正交矩阵 Q, 使得
- 1 T
Q AQ = Q AQ = diag (λ1 ,λ2 ,λ3 ) = diag( 0 , 4 , 9) .
做坐标变换 x = Qy, 则方程 f ( x1 , x2 , x3 ) = 1 化为
xT Ax = yT ( QT AQ) y = λ1 y21 +λ2 y22 +λ3 y23 = 4 y22 + 9 y23 = 1 .
所以 , f ( x1 , x2 , x3 ) = 1 表示母线平行于 Oy1 轴的椭圆柱面 .
例 6   已知二次曲面
2 2 2
x + ay + z + 2 bx y + 2 x z + 2 y z = 4
可以经过正交变换
x ξ
y =Pη
z ζ
2 2
化为椭圆柱面方程 η + 4ζ = 4 .求 a, b 的值和正交矩阵 P .
T T
解   所给二次曲面可以表示为 x Ax = 4 , 其中 x = ( x, y, z) ,
对应的矩阵为
1 b 1
A= b a 1 .
1 1 1
270 第 6 章 二次型

T
已知经过正交变换 x = Py( 其中 y = (ξ,η,ζ) ) 化为 椭圆柱 面方 程
η2 + 4ζ2 = 4 , 即
xT Ax = ( Py) T APy= yT Λy = η2 + 4ζ2 = 4 .
所以 , 矩阵 A~ diag( 0 , 1 , 4) = Λ .利用相似矩阵有相同 的迹和相 同
的行列式 , 得
    t r A = 1 + a + 1 = 0 + 1 + 4 = t rΛ, 得 a = 3 ,
    det A = - ( b - 1 ) 2 = detΛ= 0 , 得 b = 1 .
所以
1 1 1
A= 1 3 1 .
1 1 1

λ- 1 - 1 -1
|λI - A| = -1 λ- 3 -1 =λ(λ- 1) (λ- 4 ) = 0 ,
-1 - 1 λ- 1
得 λ1 = 0 ,λ2 = 1 ,λ3 = 4 .
T T
λ= 0 , 1 , 4 对应的特征向量分别为 (1 , 0 , - 1 ) , ( 1 , - 1 , 1 ) 和
(1 , 2 , 1 ) T .由于不同的 特征 值对应 的特 征向 量正交 , 只 须单 位化 ,

T T T
1 1 1 1 1 1 2 1
ξ2 = , 0, - ,   ξ2 = , - , ,   ξ3 = , , .
2 2 3 3 3 6 6 6
所求正交矩阵为
1 1 1
2 3 6
-1 2
P = (ξ1 ,ξ2 ,ξ3 ) = 0 .
3 6
-1 1 1
2 3 6
6 .3 化二次型为标准形 271

T
例 7   设四元二次型 f ( x1 , x2 , x3 , x4 ) = x Ax, 其中
0 1 0 0
1 0 0 0
A= .
0 0 y 1
0 0 1 2
(1 ) 已知 A 的一个特征值为 3, 求 y ;
(2 ) 求矩阵 P, 使 ( AP) T ( AP) 为对角矩阵 .
解   ( 1 ) 已知 3 是 A 的特征值 , 则 | 3 I - A| = 0 , 即
3 - 1 0 0
-1 3 0 0 3 -1 3- y -1
| 3 I - A| = =
0 0 3- y -1 -1 3 - 1 1
0 0 - 1 1
= 8 (2 - y) = 0 ,
从而得 y = 2 .
(2 ) 因为 AT = A, 所以 , ( A2 ) T = A2 为实对称矩阵 .由于
( AP) T ( AP) = PT ( AT AP) = PT A2 P,
2
所以题目就是求 A 的合同标准形 .
利用分块矩阵的运算 :
A1 0 1 2 1
A= ,   其中 A1 = ,   A2 = ,
A2 1 0 1 2

A21 1 0 5 4
,   其中 A21 = = I2 ,   A22 =
2
A = 2
.
A 2 0 1 4 5
2
用配方法或同型初 等行、列 变换 法 或正 交变 换 法 , 都 可求 A2
的合同标准形 .
( 1 ) 配方法
2
T 2 2 4
2 9 2
x A x = 5 x + 5 x + 8 x1 x2 = 5 x1 + x2
2 1 2 + x2
5 5
272 第 6 章 二次型

9 2
= 5 y21 + y2 ,
5
4
其中   y1 = x1 + x2 , y2 = x2 , 即
5
4
x1 1 - y1 记作
x= = 5 C1 y,
x2 y2
0 1
5
T 2
则 C A C1 =
1 2 9 = Λ2 .
5
1 0
0 1
I2
取 P= = 4 ,
C1 1 -
5
0 1

2
T T 2
I2 A1 I2
( AP) ( AP) = P A P =
C1T A22 C1
I2 I2 9
= T 2
= = diag 1, 1, 5 , .
C A C1
1 2 Λ2 5
(2 ) 同型初等行、列变换法

-
4
[ 1 ] + [ 2]
5  0
5
5 4 9
2 -
4
( 1 ) + ( 2) 0  
A 2 4 5 5 5
= .
1 0 4 4
I - [ 1 ] + [ 2]
5 1 -
0 1 5
0  1
4 5
1 - T 2
取     C1 = 5 ,   则 C A C1 = 1 2 9 .
0 1 5
6 .3 化二次型为标准形 273

以下同配方法 .
T
(3 ) 用正交变换法 求 A 的 合同 标 准形 , 得 P AP = Λ( P 为 正
交矩阵 ) , 则
T T 2 2
( AP) ( AP) = ( P AP) = Λ .
λ -1 2
由     |λI - A1 | = = λ - 1 = 0,   得 λ1 = 1 ,λ2 = - 1 .
-1 λ
对应于 λ1 = 1 和 λ2 = - 1 的单位特征向量为
1 1 1 1
x1 =   和   x2 = .
2 1 2 -1
令 P1 = ( x1 , x2 ) , 则
T 1
P1 A1 P1 = = Λ1 .
-1
λ- 2 -1 2
由     |λI - A2 | = =λ - 4λ+ 3
- 1 λ- 2
= (λ- 3) (λ- 1 ) = 0 ,
得   λ3 = 3 ,λ4 = 1 .对应于 λ3 = 3 和 λ4 = 1 的单位特征向量为
1 1 1 1
x3 =   和   x4 = . 
2 1 2 -1
令          P2 = ( x3 , x4 ) ,
T 3
则          P2 A2 P2 = = Λ2 .
1
1 1
0 0
2 2
1 1
- 0 0
P1 2 2
取 P= = ,
P2 1 1
0 0
2 2
1 1
0 0 -
2 2
274 第 6 章 二次型


T
T
P1 A1 P1
P AP =
P2T A2 P2
T
P1 A1 P1 Λ1
= = ,
P2T A2 P2 Λ2
T T 2 T 2
( AP) ( AP) = P A P = ( P AP)
2 2
Λ1 Λ1
= =
Λ2 2
Λ2
= diag( 1 , 1 , 9 , 1) .

6 .4 惯性定理和二次型的规范形
T
(1 ) 实二次型 x Ax 化 为平 方和 ( 实 对 称矩 阵 A 合 同于 对 角
阵 ) 所得标准形不是惟 一的 .但 据惯性 定理 , 对 角元中 正数 的个 数
p( 称为正惯性指数 ) 和负数的个数 r - p( 称 为负惯 性指 数 , 其中 r
是 A 的秩 ) 是由 A 惟一确定的 .
实对称矩阵 A 的合同标准形 C1T AC1 = diag( d1 , d2 , … , dn ) 中 ,
对角元依正数 , 负数和零排序 , 即
T
C1 AC1 = diag( d1 , … , dp , - dp + 1 , … , - dr , 0 , … , 0) ,
其中 , di > 0( i = 1 , … , r) .若取
1 1
C2 = diag ,…, , 1, …, 1 ,
d1 dr
T T T
则 C2 ( C1 AC1 ) C2 = C AC= diag(1 , … , 1 , - 1 , … , - 1 , 0 , … , 0) ,
其中 C = C1 C2 .称 diag (1 , … , 1 , - 1 , … , - 1 , 0 , … , 0 ) 为 A 的合 同
( 相合 ) 规范形 , 其中有 p 个 ( + 1 ) , ( r - p ) 个 ( - 1 ) 和 ( n - r) 个 0
( r = 秩 ( A) = A 的正、负惯性指数之和 ) .
( 2 ) 任意两个 n 阶实对称矩阵合同的充分必要条 件是它们 有
相同的规范形 , 或相等的正惯性指数和相等的秩 , 或有相等的正惯
性指数和相等的负惯性指数 .

6 .4  惯性定理和二次型的规范形 275

(3) 全体 n 阶实对称矩阵按其规范型 (不考虑 1 , - 1, 0 的排序) 分


1
类, 共有 ( n + 1)( n + 2)类 .因为规范型中秩 r = i 时, 正惯性指数 p 可
2
以取 0, 1 ,… , i, 共有 i + 1 类 ( 负惯性指数 = r - p) , ( i = 0, 1, …, n) , 所
n

以, 共有 ∑ ( i + 1) = 1 + 2 + … + ( n + 1) = 1 ( n + 1) ( n + 2) 类 .
i=0 2
    例 1   设 A 为 n 阶实对称矩阵 , r( A) = n,   Ai j 是 A = ( aij ) n× n
中元素 a i j 的代数余子式 ( i, j = 1 , 2 , … , n) , 二次型
n n
A ij
f ( x1 , x2 , … , xn ) = ∑∑
i=1 j =1 | A|
xi x j .

(1 ) 记 x = ( x1 , x2 , … , xn ) , 把 f ( x1 , x2 , … , xn ) 写 成 矩 阵 形
- 1
式 , 并证明二次型 f ( x) 的矩阵是 A .
T
(2 ) 二次型 g( x) = x Ax 与 f ( x) 的规 范 形是 否 相同 ? 说 明
理由 .
解   ( 1 ) 设二次 型 f ( x) 的 矩 阵是 B, 则 f ( x) = xT Bx . 已 知
- 1
bi j = | A| A ij , 利用 A 的伴随矩阵是 A 的代数 余子 式矩阵 的转 置
* T - 1 - 1 *
A = ( A ij ) 和 A = | A| A ,得
BT = ( | A| A* = A - 1 .
- 1
A ij ) T = | A| - 1

所以
- 1 T T - 1 - 1
B= ( A ) = ( A ) =A .
- 1 - 1
(2 ) 法 1: 利用定义 .由于 | A | ≠ 0 , 故 A 可逆 , 令 x = A y,
" x≠0, 则 y≠0, 且
g( x) = xT Ax = ( A - 1 y) T A( A - 1 y) = yT A - 1 y = f ( y) ,
故 g( x) 与 f ( x) 合同 , 它们有相同的规范形 .
- 1 - 1 - 1 - 1 T - 1 - 1
法 2: A = A AA = (A ) AA , 所以, A 与 A 合同,
f ( x) 与 g( x) 合同 , 它们有相同的规范形 .
法 3: 利用特征值 .设 Ax =λx   ( x≠0) , 则 A- 1 x =λ- 1 x,λ与
λ- 1 同为 正 或 负 .所 以 , xT Ax 和 xT A - 1 x 的 标 准形 中 有 相 同 的 正
276 第 6 章 二次型

( 负 ) 惯性指数 , 即 f ( x) 与 g( x) 有相同的规范形 .

6 .5 正定二次型和正定矩阵

关于正定二次 型 ( 正定 矩 阵 ) 、负 定二 次 型 ( 负 定 矩阵 ) 、半 正
( 负 ) 定二次型 ( 半正 ( 负 ) 定矩 阵 ) 的 定义 , 以及 实对 称矩阵 正定 的
等价命题 ( 或说充分必要条件 ) , 在内容提要中已陈述 , 这里不再重
T
复 .需要指出 : 一个二次型 f ( x) = x Ax, 经坐标变 换 x = Cy( C 为
可逆矩阵 ) , 化为 y 的二次型 g ( y) , 则 f ( x) 和 g( y) 有相同 的正 定
性 .因为
T T T T
f ( x) = x Ax = ( Cy) A Cy = y ( C AC) y = g( y) .
由 f ( x) 正定 , 可 知 : " y≠0, x = Cy≠0, g( y) = f ( x) > 0 , 所
- 1
以 g( y) 正 定 ; 再 由 g ( y) 正 定 又 可 知 , " x≠0, y = C x≠ 0,
f ( x) = g( y) > 0 , 所以 , f ( x) 正定 .
同理 , f ( x) 和 g( y) 的负 定 性、半 正 ( 负 ) 定 性也 相 同 .也 就 是
说 , 合同矩阵的正 ( 负 ) 定性、半正 ( 负 ) 定性是相同的 .
此外 , 对于 n 阶实对称正 定矩 阵 A ( 或正 定 二次 型 xT Ax) , 一
定存在正定矩阵 B, 使得
A = B2 .
- 1 T
因为存在正交矩阵 Q( 注意 Q = Q ) , 使得
QT AQ = diag (λ1 ,λ2 , … ,λn )   (λi > 0 , i = 1 , 2 , … , n) .
则     A = Q diag (λ1 ,λ2 , … ,λn ) QT
= Q diag ( λ1 , … , λn ) QT Q diag( λ1 , … , λn ) QT
= B2 ,
T
其中 B= Q diag ( λ1 , … , λn ) Q 仍然是正定矩阵 .
例 1   判断下列三元二次型
2 2 2
f ( x1 , x2 , x3 ) = x1 + x2 + x3 - x1 x2 + x2 x3
6 .5 正定二次型和正定矩阵 277

是不是正定二次型 .
解   法 1: 二次型的对应矩阵为
1
 1 - 0
2
1 1
A= -  1 .
2 2
1
 0   1
2
2 1
A 的特征多项式为 |λI - A | = (λ- 1 ) (λ- 1) - , 从而 得其 特
2
2 2
征值为 λ1 = 1 ,λ2 = 1 + ,λ3 = 1 - 都大于零 , 所以二次型正定 .
2 2
法 2: 用配方法得
2 2
x2 3 2 2 2
    f ( x1 , x2 , x3 ) = x1 - + x2 + x3 + x3
2 4 3 3
2 3 2 2 2
= y1 + y2 + y3 ≥0 .
4 3

x2 2
f ( x1 , x2 , x3 ) = 0 x1 - = 0,   x2 + x3 = 0 和 x3 = 0 ,
2 3
即   x1 = x2 = x3 = 0 .故 x≠0 时 , f ( x1 , x2 , x3 ) > 0 , 所以此 二次 型
正定 .
法 3: 利用 A 的 3 个顺序主子式都大于零 .
-1
1
2 3 1
det A1 = 1 > 0,   det A2 = = > 0,   det A3 = | A| = >0 .
-1 4 2
1
2
所以此二次型正定 .
法 4: 利用同型初等行、列变换化 A 为对角形 .
278 第 6 章 二次型

-1 1 0 0
1 0
2 3 1
1 0
-1 1 [2 ] + [ 1] × 4 2
1 2
2 2 1 1
(2 ) + ( 1) ×
A 2 0 1
= 1 2
I 0 1 1
2 [2 ] + [ 1] ×
2 1
1 0
1 0 0 2
0 1 0 0 1 0
0 0 1 0 0 1
2 0  0
3
0  0
-2 4
[3 ] + [ 2] ×
3
2
(3 ) + ( 2) ×
-2 0 0  
3 3
  .
-2 1 1
[3 ] + [ 2] ×
3 1 -
2 3
2
1 1 -
3
0 0  1
1 1 2
1 -
2 3 3
T
取 C= 2 ,   则 C AC= 4 .
1 1 -
3 2
0 0 1 3
A 的正惯性指数为 3 , 所以此二次型正定 .
例 2 ( 研 6 -2)   设 A 为 n 阶正定 矩阵 , I 是 n 阶 单位矩 阵 .证
明 A+ I 的行列式大于 1 .
T
解   因为 A 为正定矩 阵 , 则存在 正交矩阵 Q( 注 意 Q Q = I) ,
使得
T
Q AQ= diag(λ1 ,λ2 , … ,λn ) ,
6 .5 正定二次型和正定矩阵 279

其中 λi > 0   ( i = 1 , 2 , … , n) 是 A 的特征值 .于是


T
Q ( A + I ) Q = diag(λ1 ,λ2 , … ,λn ) + I
= diag(λ1 + 1 ,λ2 + 1 , … ,λn + 1 ) .
上式两边取行列式得
T T T
| Q ( A + I ) Q| = | Q | | A + I| | Q| = | Q Q | | A + I|
= (λ1 + 1 ) (λ2 + 1 ) … (λn + 1 ) ,
所以 , | A + I | = (λ1 + 1 ) (λ2 + 1) … (λn + 1) > 1 .
例 3 ( 补充题 49 , 研 6 -7)   设有 n 元二次型
2 2
f ( x1 , x2 , … , xn ) = ( x1 + a1 x2 ) + ( x2 + a2 x3 ) + … +
( xn - 1 + an - 1 xn ) 2 + ( xn + an x1 ) 2 ,
其中 ai ( i = 1 , 2 , … , n) 为实数 .试问当 a1 , a2 , … , an 满足何 种条 件
时 , 二次型为正定二次型 .
解   因 为 f ( x) 是 完全 平 方项 的和 , 所 以 f ( x) ≥ 0 . f ( x) = 0
的充要条件是
x1 + a1 x2 = x2 + a2 x3 = … = xn - 1 + an - 1 xn = xn + an x1 = 0 . (1)
若方程组 (1 ) 只有零解 , 则 " x≠0   f ( x) > 0 , 即 二次 型正定 .方 程
组 (1 ) 只有零解的充要条件是其系数行列式不等于零 , 即
1 a1
1 a2
w = 1 + ( - 1) n + 1 a1 a2 … an ≠ 0
1 an - 1 ( 对第一列展开 ) ,    
an 1
n
因此 , 当 a1 a2 … an ≠ ( - 1) 时 , 二次型正定 .
例 4 ( 补充题 50 , 研 6 -8 )   设 A 为 m 阶 实对 称 正定 矩 阵 , B
是 m× n 实矩阵 , BT 为 B 的转置 , 试证 : BT AB 为 正定矩阵 的充 分
必要条件是秩 r ( B) = n .
证   充分性: 首先 BT AB是实对称矩阵, 因为( BT AB) T = BT AB .
由于 B 不 是方 阵 , 判 别矩 阵正 定 的一 些定 理都 不好 用 , 要 利
280 第 6 章 二次型

用二次型正定的定义来判别 .由于
T T T T
x ( B AB) x = ( Bx) A( Bx) = y Ay,
当秩 ( B) = n 时 , " x≠0, 均有 y = Bx≠0( 因为秩 ( B) = n, Bx = 0 只
有零解 ) , 从而 yT Ay > 0( 因为 A 正定 ) .所以 , BT AB 正定 .
T T T
必 要 性 : 由 B AB 正 定 , 即 " x ≠ 0, x ( B AB ) x =
T
( Bx) A( Bx) > 0 , 可知 必 须 有 Bx≠0, 即 Bx = 0 没 有 非 零 解 , 所
以 , r( B) = n( 此时也必有 m≥ n) .
注意   下 列证 明是错 误的 :“由 A 正定 , 存在 可逆 矩阵 P, 使
T T T T T
得 A = P P, 于 是 , ( B AB) = ( B P PB ) = ( PB) ( PB) , 所 以
( BT AB) 正定 .”错在 ( PB) m × n 不是方阵 , 从而不是可逆矩阵 .如果 B
是 m 阶方阵 , 这样证明是可以的 ( 即 BT AB 正定 的充要 条件 是 PB
可逆 , 而 PB 可逆的充要条件是 B 可逆 , 即 r( B) = m) .
例 5   设 A 是 m× n 实矩阵 , I 是 n 阶 单 位阵 .已 知 B = λI +
AT A, 试证 : 当 λ> 0 时 , 矩阵 B 为正定矩阵 .
证   首先 B=λI + AT A 是实对称矩阵, 因为 BT = (λI + AT A) T =
T
λI + A A = B .
法 1: 利用定义 .由于二次型
xT Bx = xT (λI + AT A) x =λxT x + xT AT Ax = λxT x + ( Ax) T Ax,
T T
而 " x≠0, Ax 的内积 ( Ax, Ax) = ( Ax) Ax≥ 0 , 且 λ> 0 时 ,λx x >
0 , 因此 , " x≠ 0 , 有
xT Bx = λxT x + ( Ax) T Ax> 0 .
所以 B 为正定矩阵 .
法 2: 利用特征值全部大于 零 .因为 AT A 是实对 称矩 阵 , 设 μ
T
是 A A 的任一特征值 , 对应的特征向量是 x, 即
T
A Ax = μx   ( x≠0) .
两边左乘 xT , 有
xT AT Ax = μ xT x, 即   ( Ax) T Ax = μxT x,
6 .5 正定二次型和正定矩阵 281

T
( Ax) Ax
所以 ,μ= T ≥0   ( 因为 : x≠0, xT x > 0;   ( Ax) T Ax≥0 ) .
x x
(λI + AT A) 的特征值 为 (λ+ μ) .当 λ> 0 时 , B 的 任 一特 征 值
(λ+ μ) > 0 , 所以 B 为正定矩阵 .

例 6   设 A 为 n 阶 正定 矩阵 , x = ( x1 , … , xn ) T ∈ Rn , b 是 一
固定的实 n 维列向量 .证明 :
1 T
p( x) = x Ax - xT b
2
1 T -1
在 x0 = A- 1 b 处取得最小值 , 且 pmin = - b A b.
2
证   这是把一元二次函数的最小值问题推广到 n 元二次函数
( 二次项部分是正定二次型 ) .当 n = 1 时 , A = ( a) , a > 0 , 一元二 次
1 2 b
函数 p( x) = ax - bx 的图形 是条抛物 线 , 在顶点 x0 = 处 , 取
2 a
2
b
得最小值 pmin = - .
2a
对于一般的 n, 欲证 p( x0 ) 是 p( x) 的最小值 , 只需证 " x≠ x0 ,
p( x) - p( x0 ) > 0 .
1 T T
将 b = Ax0 代入 p( x) = x Ax - x b, 得
2
1 T T 1 T T
p( x) - p( x0 ) = x Ax - x b - x0 Ax0 + x0 b
2 2
1 T T 1 T
= x Ax - x Ax0 + x0 Ax0 . ( 1)
2 2
T T T T
因为 x Ax0 = ( x Ax0 ) = x0 Ax,
1 T 1
所以 xT Ax0 = x Ax0 + x0T Ax . ( 2)
2 2
将 (2 ) 式代入 ( 1) 式 , 得
1 T T T T
    p( x) - p( x0 ) = ( x Ax - x Ax0 - x0 Ax + x0 Ax0 )
2
282 第 6 章 二次型

1 T
= [ x A( x - x0 ) - x0T A( x - x0 ) ]
2
1 T
= ( x - x0 ) A( x - x0 ) .
2
由于 A 正定 , 所以 " ( x - x0 ) ≠0, 即 x≠ x0 , 恒 有 p( x) - p( x0 ) =
1
( x - x0 ) T A( x - x0 ) > 0 , 因此
2
1 T T
pm in = p( x0 ) = x0 Ax0 - x0 b.
2
- 1
将 Ax0 = b 与 x0 = A b 依次代入上式 , 得
1 T T 1 T 1 - 1 T
pm in = x0 b - x0 b = - x0 b = - ( A b) b
2 2 2
1 T -1
= - b A b.
2

例 7   设 A 为 n 阶实对称矩阵 , A 的 n 个特征值λ1 ≤λ2 ≤ …
≤λn .证明 : " x∈ R n ,
λ1 ( x, x) ≤ ( Ax, x) ≤λn ( x, x)
( 其中 ( x, y) = xT y 表示 x 与 y 的内积 ) , 并指出分别取怎样的非 零
向量 x 使两个等号成立 .
T
证   对 于 实 对 称 矩 阵 A, 存 在 正 交 阵 Q, 使 得 Q AQ =
diag (λ1 , … ,λn ) = Λ.令 x= Qy, 则
( x, x) = xT x= yT QT Qy= yT y,
即 ( x, x) = x21 + … + x2n = y21 + … + y2n = ( y, y) .
于是
T T T T 2 2
( Ax, x) = ( Ax) x = x Ax = y Q AQy =λ1 y1 + … +λn yn ,
λ1 ( x, x) =λ1 ( y21 + … + y2n ) ≤λ1 y21 + … +λn y2n
2 2
≤λn ( y1 + … + yn ) =λn ( y, y) =λn ( x, x) .
所以 λ1 ( x, x) ≤ ( Ax, x) ≤λn ( x, x) .
当 Ax =λn x, 即 x 是 A 的 对应 于 λn 的 特征 向 量时 , ( Ax, x) =
6 .5 正定二次型和正定矩阵 283

λn ( x, x) 成 立 ; 当 Ax = λ1 x, 即 x 是 A 的 对 应 于 λ1 的 特 征 向 量 时 ,
( Ax, x) =λ1 ( x, x) 成立 .

例 8   设 A, B 为 n 阶正定矩阵 .证明: A + B 的最 大特征值 ρ
大于 A 的最大特征值 .
证   设 μ1 和 λn 分别是 B 的最小和 A 的最大特征值 ( 因为 A, B
正定 ,μ1 ,λn 都大于零 ) , xn 是 A 的对应于特征值 λn 的特征向量, 则
Axn =λn xn ,   ( xn ≠0) ,

T T
xn Axn =λn xn xn , (1)
xTn Axn
故 λn = T . (2)
xn xn
由于正定矩阵是实对称矩阵 , 所以 A + B 为实对称矩阵 .由例 7
知, xn ≠0 时 , 有
( ( A+ B) xn , xn )≤ρ( xn , xn ) ,
μ1 ( xn , xn ) ≤ ( Bxn , xn ) ,

xTn ( A+ B) xn ≤ρxTn xn ,
T T
μ1 xn xn ≤ xn Bxn .
所以
xnT ( A+ B) xn xTn Axn + xTn Bxn
ρ≥ T = T
xn xn xn xn
λn xnT xn + μ1 xTn xn
≥ = λn + μ1 >λn .
xnT xn

例 9   设 λ1 和 μ1 分别是 n 阶实对称矩阵 A 和 B 的最小特征
值 .证明 : A+ B 的最小特征值 ω大于或等于λ1 + μ1 .
证   设 ω为 A + B 的最小特征值, 对应的特征向量为 x0 , 即
( A+ B) x0 = ωx0   ( x0 ≠0) ,
于是
284 第 6 章 二次型

T T
x0 ( A+ B) x0 = ωx0 x0 .
显然 , A+ B 为实对称矩阵 , 由例 7 知, x0 ≠0 时 , 有
λ1 ( x0 , x0 ) ≤ ( Ax0 , x0 ) ,   μ1 ( x0 , x0 ) ≤( Bx0 , x0 ) .
所以
x0T ( A+ B) x0 x0T Ax0 + x0T Bx0
ω= T = T
x0 x0 x0 x0
T T
λ1 x0 x0 + μ1 x0 x0
≥ T =λ1 + μ1 .
x0 x0
例 10   设 A, B 皆是正定矩阵, 且 AB= BA .证明 : AB 也是正定
矩阵 .
证   由 AB= BA, 得 AB 是实对称矩阵 , 因为
T T T
( AB) = B A = BA = AB .
证明 AB 也是正定矩阵 .有以下方法 .
法 1: 证明 AB 的特征值全部大于零 .设 λ为 AB 的任意一个特
征值 , 其对应的特征向量为 x, 即
ABx=λx   ( x≠0) ,
- 1
于是 Bx=λA x.
两边左乘有 xT , 得
T T - 1
x Bx =λx A x .
- 1 - 1
由于 B正定 , A 也正 定 ( A 正定 , 则 A 也正 定的 证明见 教材 6 .4
T T - 1
例 1) , 所以, " x≠0, x Bx > 0, x A x> 0, 因此
T
x Bx
λ= T - 1 > 0 .
x A x
由 λ的任意性得 AB 为正定阵 .
法 2: 证明 AB 与正定矩阵合同 .
由 A, B 皆正定 , 存在可逆阵 C, 使得
CT AC= I,   即 CT A= C - 1 .
两边右乘 BC, 得

6 .6  其他有定二次型 285

T - 1
C ( AB) C= C BC,
从而得知 : C ( AB) C 与 B 相 似 , CT ( AB) C 和 B 有相 同的 全都 大
T

T T
于零的特征值 , 故 C ( AB) C 正定 .而 AB 与 C ( AB) C 合同 , 所 以
AB 也正定 .
T
法 3: 证明存在可逆 D, 使得 ABD D D .
由于 A, B 皆 正定 , 所以 存在 可逆 阵 P, Q, 使 得 A = PT P, B =
QT Q, 于是
AB = ( PT P) ( QT Q) = ( Q- 1 Q) ( PT PQT Q)
- 1 T T
= Q ( QP PQ ) Q,
即 AB 与 QPT PQT 相似 , 必合同 .而 QPT PQT 正定 ( 因为 QPT PQT =
T T T T T
( PQ ) ( PQ ) = D D, 其中 D = ( QP ) 可逆 ) , 所以 AB 也正定 .
T T
法 4: 同法 3, 由 A = P P, B = Q Q( 其中 P, Q 为可逆矩阵 ) , 得
T T
AB = ( P P) ( Q Q) .
两边左乘 ( PT ) - 1 , 右乘 PT 得
T - 1 T T - 1 T T T T T
( P ) ( AB) P = ( P ) ( P P) ( Q Q) P = ( PQ ) ( QP )
= ( QPT ) T ( QPT ) = DT D   ( 其中 D = QPT 可逆 ) .
T - 1 T T - 1 T
所以 , ( P ) ( AB) P 正 定 , 而 AB 与 ( P ) ( AB) P 相 似 , 必 合
同 .所以 , AB 也为正定阵 .
法 5: 利用 : 对实对称正定矩阵 A, B, 分别存在实对称正定 矩
2 2
阵 C, D, 使得 A = C , B = D .从而得
C - 1 ( AB) C = C - 1 ( CCDD) C = CDDC
T T T T
= C D DC= ( DC) DC= F F,
( 其中 F = DC 可逆 ) .所以 C - 1 ( AB) C 正定 , 而 AB 与其相 似 , 因 此
AB 也正定 .


6 .6 其他有定二次型
T
( 1 ) 定 义   如 果 n 元 实 二 次 型 f ( x) = x Ax, " x≠0 ( x∈
R n ) , 恒有 :
286 第 6 章 二次型

T
① x Ax < 0 , 则称 f ( x) 为负定二次型 ( A 为负定矩阵 ) ;
② xT Ax≥0, 且存在 x0 使得 f ( x0 ) = 0 , 则称 f ( x) 为半正定二
次型 ( A 为半正定矩阵 ) ;
③ xT Ax≤0, 且存在 x0 使得 f ( x0 ) = 0 , 则称 f ( x) 为半负定二
次型 ( A 为半负定矩阵 ) .
正定、负定、半正定、半负定二次型统称为有定二次型 .不是有
定的二次型叫做不定二次型 .
( 2 ) n 元二次型 xT Ax 负定与以下各命题等价 :
① ( - A) 正定 ;
② A 的负惯性指数为 n , 即 AD ( - I ) ;
T
③ 存在可逆矩阵 P, 使得 A = - P P;
④ A 的 n 个特征值λ1 ,λ2 , … ,λn 都小于零 ;
⑤ A 的奇数阶顺序主子式 det Ak 都小 于零 , 偶 数阶顺 序主 子
式 det Ak 都大于零 .
(3 ) n 元二次型 xT Ax 半正定 ( 或半负定 ) 与以下各命题等价 :
① A 的正 ( 或负 ) 惯性指数为 r ( A) < n, 即
AD diag( 1 , … , 1 , 0 , … , 0 ) ( 或 AD diag ( - 1 , … , - 1 , 0 , … ,
0) ) , 其中 1( 或 - 1) 有 r 个 .
② A 的 n 个特 征值 λ1 ,λ2 , … ,λn 都 ≥ 0 ( 或 ≤ 0) , 至少 有一 个
等于零 .
③ A 的各阶主子式都≥0 ( 或 : 奇数阶的主子式≤0; 偶数阶主
子式≥0 ) , 至少有一个等于零 ( 注意 , 这里是“ 各阶主子式”不是“ 各
阶顺序主子式”) .
T T
例 1   设 x Ax 为半 负定 二 次 型 , 问 : ( 1 ) x ( - A) x 是 否 半
正定 ? ( 2 ) A 的各阶主子式是否都≤ 0 ?
T T
答   ( 1 ) x Ax 为半负定 , 则 x ( - A) x 半正定 ;
( 2 ) 否 .( - A) 的 各 阶 主 子 式 都 ≥ 0, 即 A 的 偶 数 阶 主 子 式
都≥0; A 的奇数阶主子式都≤ 0 .
6 .7 部分疑难习题和补充题的题解 287

T
例 2   设 B 是 n 阶 实 矩 阵 , r ( B) < n .证 明 B B 是 半 正 定
矩阵 .
T T T T
证   由于 ( B B) = B B, 所以 B B 是实对称矩阵 .
因为 " x≠0 ( x∈ R n ) , 恒 有 内 积 ( Bx, Bx) = ( Bx) T ( Bx) =
T T
x B Bx≥ 0 .又 Bx = 0 有 非 零 解 , 即 存 在 x≠0, 使 Bx = 0 , 从 而
xT BT Bx = 0 .所以 , 二次型 xT BT Bx 半正定 , 即 BT B 是半正定矩阵 .

6 .7 部分疑难习题和补充题的题解

1 ( 习题 5 )   若二次型 f ( x1 , … , xn ) = xT Ax 对一切 x = ( x1 , … , xn ) T 恒
有 f ( x1 , … , x n ) = 0 , 证明 A 为 n 阶零矩阵 .
证   取 xi = (0 , … , 1 , … , 0 ) T ( 其 中第 i 个 分量 为 1 , 其余 分量 全为 零 ) ,
则有
n n

∑∑ a
T
f ( xi ) = x Axi =
i ij x i x j = aii = 0 ,   i = 1 , 2 , … , n .
i= 1 j= 1

再取 xij = ( 0 , … , 1 , … , 1 , … , 0) T ( 其中第 i 和第 j 个分量为 1 , 其余分量 全为


零) , 则有
f ( xij ) = xTij Axij = 2 aij = 0 ,   i, j = 1 , 2 , … , n .
所以 , A 的 n2 个元素全为 0 , 即 A 为 n 阶零矩阵 .
2 ( 习题 6 )   设 A, B 均为 n 阶对称矩阵 , 且对一切 x 有 xT Ax = xT Bx, 则
A= B .
提示 : 由 f ( x1 , … , x n ) = xT ( A - B) x, 对 一 切 x = ( x1 , … , xn ) T 恒 有
f ( x) = 0 .利用上题结果得 A - B = 0 .
3 ( 习题 7 )   设 AD B, CD D, 且它们都 是 n 阶实 对称矩阵 , 下列结论 成
立吗 ?
A 0 B 0
( 1) ( A + C) D ( B + D) ;   ( 2 ) D .
0 C 0 D
答   ( 1 ) 不成立 ;如
1 0 0 0
A= ,   B= ,   C= ,   D= .
0 1 - 1 -1
288 第 6 章 二次型

此时 , A + C 与 B + D= 0 不合同 .
( 2) 成立 .   由 C1T AC1 = B, C2T CC2 = D( 其中 C1 , C2 为可逆矩阵 ) ,得
T
C1 A 0 C1 C1T AC1 0 B 0
= = ,
C2 0 C C2 0 C2T C C2 0 D

C1
其中 仍然可逆 .所以结论成立 .
C2

4 -2 0 0 0
- 2 1 0 0 0
4 (习题 9)   设 A = 0 0 5 0 0 , 求 正 交 矩 阵 Q, 使 得
0 0 0 - 4 6
0 0 0 6 1
Q AQ 为对角矩阵 .
T

解   利用上题分块矩阵合同的结论 .令

A1
A= A0 ,
A2

4 -2 - 4 6
其中 A1 = , A0 = (5) , A2 = .
- 2 1 6 1
4 -2 1 2 0
对 A1 = , 存在 P1 = , 使得 P1- 1 A1 P1 = = Λ1 ;
- 2 1 2 -1 5
- 4 6 2 3 5
对 A2 = , 存在 P2 = , 使得 P2- 1 A2 P2 = = Λ2 .
6 1 3 -2 - 8
不同特征值对应的特征向量已经正交 , 只需单位化 .取

1 2 2 3
5 5 13 13
Q1 = ,   Q2 = ,
2 1 3 2
- -
5 5 13 13
6 .7 部分疑难习题和补充题的题解 289

1 2
5 5
2 1
-
Q1 5 5
令 Q= 1 = 1 .
Q2 2 3
13 13
3 2
-
13 13
0
5
则有 Q- 1 AQ= Λ= 5 .
5
-8
5 ( 习题 13)   设 n 阶实对称矩阵 A 的秩为 r( r < n) , 试证明 :
( 1) 存在可逆矩阵 C,使得 CT AC = diag( d1 , … , dr , 0 , … , 0 ) ( di ≠0 , i = 1 ,
2 , … , r) ,
( 2) A 可以表示为 r 个秩为 1 的实对称矩阵之和 .
证   ( 1) 实对称 矩阵 A 存 在正 交 矩 阵 Q, 使 得 Q- 1 AQ = QT AQ 为 对 角
矩阵
Λ= diag(λ1 , … ,λr , 0 , … , 0) ,
其中 λi ≠0 ( i = 1 , 2 , … , r)为 A 的非零特征值 .
( 2 ) 由 ( 1 ) 的结论 ,有 A = QΛQ - 1 = ( Q- 1 ) T ( Λ1 + Λ2 + … + Λr ) Q- 1 , 其中
Λi = diag( 0 , … ,λi , … , 0) (第 i 个元素为λi 其余元素全为 0 , i = 1 , … , r) ,
则 A = ( Q- 1 ) T Λ1 Q- 1 + ( Q- 1 ) T Λ2 Q- 1 + … + ( Q- 1 ) T Λr Q - 1
= D1 + D2 + … + Dr ,
其中 Di = ( Q- 1 ) T Λi Q - 1 = DTi ( i = 1 , … , r) 是 秩为 1 的实 对称矩 阵 .因 为
r ( Di ) = r(Λi ) = 1 .
6 ( 习题 14)   设 n 阶实对称幂等矩阵 A( A2 = A) 的秩为 r, 试求 :
( 1) 二次型 xT Ax 的一个标准形 ;
( 2 ) det ( I + A + A2 + … + An ) .
290 第 6 章 二次型

解   ( 1) ( 见主教材第 5 章 5 .2 中例 4) 对 n 阶实对称幂等矩阵 A 存在正


交阵 P( PT = P - 1 ) , 使得 PT AP= diag( 1 , … , 1 , 0 , … , 0) = Λ( 1 是 r 重特征值 , 0
是 n - r 重特征值 ) .
令 x= Py, 则 xT Ax= yT ( PT AP) y= y21 + … + y2r 为二次型 xT Ax 的一个标
准形 .
( 2 ) 由 A2 = A, 得 Ak = A( k = 1 , … , n) , 于是 , Ak = A = PΛP- 1 , 且
det ( I + A + A2 + … + An ) = det ( I + nA)
= | PP - 1 + nPΛP - 1 | = | P( I + nΛ) P- 1 |
= | P| | I + nΛ| | P - 1 | = | P- 1 P| | I + nΛ|
n+1

n+ 1
= = (n + 1)r .
1

1
7 ( 习题 15)   设 n 阶实对称矩阵 A的正负惯性指数都不为零 .证明 : 存在
非零向量 x1 , x2 和 x3 ,使得 x1T Ax1 > 0 , x2T Ax2 = 0 和 x3T Ax3 < 0 .
证   设 A 的正、负惯性指数分别为 p , r - p .则存在可逆阵 C,使得
CT AC= diag(1 , … , 1 , - 1 , … , - 1 , 0 , … , 0) ,
其中 1 的个数为 p > 0 , ( - 1) 的个数为 r - p > 0 .令 x= Cy,则
xT Ax= yT ( CT AC) y= y21 + … + y2p - y2p + 1 - … - y2r .
取 y1 = (1 , 0 , … , 0) T , y3 = (0 , … , 0 , 1 , … , 0 ) T ( 第 r 个分量 为 1 , 其余 分量 全
部为 0 ) , y2 = (1 , 0 , … , 0 , 1 , 0 , … , 0 ) T ( 第一和第 r 个分量为 1 , 其余分量 全部
为 0) , 代入 xi = Cyi ( i = 1 , 2 , 3) , 则得
x1T Ax1 = 1 > 0 ,   x3T Ax3 = - 1 < 0 ,   x2T Ax2 = 0 .
8 (习题 16 )   设 A 是奇数阶实对称矩阵 ,且 det ( A) > 0 .证明 : 存在非零
的向量 x0 ,使 x0T Ax0 > 0 .
证   若矩阵 A 的全部 ( 奇 数个 ) 特征 值都小 于或 等于 零 , 则 A 的 行列 式
( 等于所有特征值的乘积 ) 也小于或等于 零, 这 与已知 的 det ( A) > 0 矛盾 .所
以存在特征值 ( 不妨设 )λ1 > 0 .
由于 A 是实对称矩阵 ,存在正交阵 Q,当 x= Qy 时 ,有
6 .7 部分疑难习题和补充题的题解 291

xT Ax = yT ( QT AQ) y= λ1 y21 + … + λn y2n .


取 y0 = (1 , 0 , … , 0) T , 代入 x0 = Qy0 ≠0,则 x0T Ax0 = λ1 > 0 .
或   设特征值 λ1 > 0 所对应的特征向量为 x0 , 即
Ax0 =λ1 x0 ,
则 x Ax0 = x0Tλ1 x0 = λ1 xT0 x0 > 0 .
T
0

9 ( 习题 23 ,24 )   用正交变换法化二次型 ∑x ∑ x i x j 为标准型 ,


2
i +
i= 1 1≤ i< j≤ n

并说明它是否正定 .在 n = 3 的情况下求正交变换的矩阵 Q .
解   二次型对应的矩阵为
1 1
1 …
2 2
1 1
1 …
A= 2 2 .
… … w …
1 1
… 1
2 2
1 1
λ- 1 - … -
2 2
1 1
- λ- 1 … -
由   |λI - A| = 2 2
… … w …
1 1
- - … λ- 1
2 2
1 1
1 - … -
2 2
1
n+ 1 1 λ- 1 … -
= λ- 2
2
… … w …
1
1 - … λ- 1
2
292 第 6 章 二次型

1 1
1 - … -
2 2
1
n+ 1 0 λ- … 0
= λ- 2
2
… … w …
1
0 0 … λ-
2
n+ 1 1 n- 1
= λ- λ- = 0,
2 2
1 1
得 : λ1 = ( n + 1) ,λ2 = ( n - 1 重 ) .由此可得 A 正定 .
2 2
1
由于 A的每行行和均为 ( n + 1) , 由 (λ1 I - A) x= 0 其中 x= (1 ,… ,1)T 得
2
1
λ1 = ( n + 1)对应的特征向量为 x1 = (1 ,… ,1)T .
2
1
当 λ2 =
( n - 1 重根 )时 , 由(λ2 I - A) x= 0 的同解方程组
2
x1 + x2 + … + x2 = 0 ,
得 λ2 对应的 n - 1 个线性无关的特征向量 :
x2 = ( 1 , - 1 , 0 , … , 0 ) T ,   x3 = ( 1 , 1 , - 2 , 0 , … , 0 ) T ,   … ,
xn = ( 1 , … , 1 , - n) T .
x1 ,… , xn 已经为正交向量组 , 只须单位化 .令
xi
γi =   ( i = 1 , 2 , … , n) ,   Q= (γ1 ,… ,γn ) ,
‖ xi ‖
则 Q 为正交阵 ,且
1 1 1
QT AQ= diag ( n + 1) , , …,
2 2 2
为所求的标准形 .
( 习题 24 )   当 n = 3 时 , 求 正 定 矩 阵 B, 使 得 A = B2 ( 其 中 A 为 上 题
的 A) .
1 1 1
2
3 2 6
1 1 1 1
-1
当 n= 3 时,取 Q= - ,   则 Q AQ= 2 .
3 2 6
1
1 -2
0 2
3 6
6 .7 部分疑难习题和补充题的题解 293

2
1
所以 A=Q 2 Q- 1
1
2

2 2
1 1
-1
=Q 2 Q Q 2 Q- 1 .
1 1
2 2

2
1
令正定矩阵 B = Q 2 Q- 1 , 则 B2 = A .
1
2
10 ( 习题 27 )   若 P 是可逆矩阵 .用定义证明 : PT P 是正定矩阵 .
证   " x≠0, 因为 P 可逆 ,所以 , y= Px≠0, 且
xT ( PT P) x= ( Px) T Px = yT y > 0 ,
因此 ,矩阵 PT P 正定 .
11 ( 习题 38 )   若 对 于任 意的 全 不为 0 的 x1 , … , x n , 二次 型 f ( x1 , … ,
xn ) 恒大于 0 , 问二次型 f 是否正定 ?
. " x≠0”指 的是 任意 的 x = ( x1 , … , x n ) T 中 x1 , … , xn 不全 为
答   否“
0 , 而不是“全不为 0”的 x1 , … , xn .例如 , 二次型
f ( x1 , x2 , x3 ) = x21 + ( x2 - x3 ) 2 .
对于任意的全不为 0 的 x1 , x2 , x3 , 恒有 f ( x1 , x2 , x3 ) > 0 , 但 f ( x) 不是正定 , 而是
半正定(因为 , x1 = 0 , x2 = x3 = 1 时 , f ( x1 , x2 , x3 ) = 0) .
12 ( 习题 41 )   设 A 是实对称矩阵 , B 是正定矩阵 .证明 : 存在可逆阵 C,
使得 CT AC 和 CT BC 都成对角形 .
证   因为 B 正定 ,所以 ,存在可逆阵 C1 ,使得 C1T BC1 = I .
因为 ( C1T AC1 ) T = C1T AC1 , 所 以, C1T AC1 为实 对 称 阵 .于 是 , 存 在 正 交 阵
C2 ( 注意 C2T C2 = I) ,使得
294 第 6 章 二次型

C2T ( C1T AC1 ) C2 = dia g(λ1 ,λ2 , … ,λn ) ,


其中 λ1 ,λ2 , … ,λn 为 ( C1T AC1 ) 的特征值 .
令 C= C1 C2 ,则
CT AC = C2T ( C1T AC1 ) C2 = diag(λ1 ,λ2 , … ,λn ) ,
CT BC = C2T CT1 BC1 C2 = C2T IC2 = I .
于是 , CT AC 和 CT BC 都成 对 角形 .
13 ( 习题 43 )   设 A 是 n 阶正定矩阵 , x= ( x1 , x2 , … , x n ) T , 证明 :
0 xT
f ( x) = det
x A
是一个负定二次型 .
证   法 1 : 对分块矩阵做初等行变换 ,将其化为下三角块阵 , 即
1 - xT A - 1 0 xT - xT A - 1 x 0
= .
0 In x A x A
两边取行列式 , 得
1 - xT A - 1 0 xT - xT A - 1 x 0
= ,
0 In x A x A
0 xT
即 f ( x) = = - xT A- 1 x | A|
x A
因为 A 正 定 , 所 以 | A| > 0, 且 A - 1 也 正定 , 于是 , " x≠0, xT A - 1 x > 0 ,
从而
f ( x) = - xT A - 1 x | A| < 0 ,
即 f ( x) 负定 .
法 2 : 因为 A 可逆 , 做非退化线性变换 x= Ay( y= A - 1 x) ,即
n n

xi = ∑a
j=1
ij y j , 于是 x i - ∑a
j =1
ij y j = 0   ( i = 1 , 2 , … , n) .

0 x1 x2 … xn
x1 a11 a12 … a1n
在 | B| = x2 a21 a22 … a2n
… … … …
xn an1 an2 … ann
6 .7 部分疑难习题和补充题的题解 295

中 , 第 i + 1 列 ( xi 所 在 列 ) 乘 ( - yi ) 都 加 到 第 1 列 , 则 第 1 列 变 换 成
n n
T
- ∑ x y , 0 ,… ,0
i= 1
i i , 再对第 1 列展开 , 注意 : ∑x y
i= 1
i i = xT y,而 y = A- 1 x ,


n

| B| = - | A| ∑
i=1
x i y i = - | A| xT y= - | A| xT A - 1 x .
- 1
由于 | A| > 0 和 A 正定 , 所以 , " x≠0, f ( x) = - | A| xT A- 1 x < 0 , 即
f ( x) 负定 .
14 ( 习题 44 )   设 A= ( aij ) n× n 是 n 阶正定矩阵 , 证明: | A| ≤ a11 a22 …ann .
证   将 A 分块为
An - 1 α
A= T
,   其中 α= a1n , a2 n , … , an - 1, n T
.
α ann
因为 A 正定 , A 的 k 阶 顺序主 子式 det Ak 大 于零 ( k = 1 , 2 , … , n) , 所以 ,
矩阵 An - 1 正定( An- -11 也正定 ) , 且 An - 1 可逆 .对分 块矩 阵做 初等 行变换 , 将 矩
阵 A 化为上三角块阵 ,即
In - 1 0 An - 1 α An - 1 α
= .
- αT An- -1 1 1 αT ann 0 an n - αT An- -1 1 α
上式两边取行列式 ( 上三角块矩阵的行列式等于对角块的行列式的乘积 ) ,得
| A| = | An - 1 | ann - αT An- -11 α .
因为 An- -11 也正定 , 当 α≠0 时 , 有 αT An- -11 α> 0 , ann - αT An- -1 1 α< ann , 所以
| A| ≤ ann | An - 1 | .
同理 , 在 An - 1 中 , 有 | An - 1 | ≤ an - 1 , n - 1 | An - 2 | .如此类推 , 得
| A| ≤ ann | An - 1 | ≤ ann a n - 1, n - 1 | An - 2 | ≤…≤ ann an - 1 , n - 1 … a22 | A1 |
其中 | A1 | = a11 , 所以 | A| ≤ a11 a22 … ann .
历 年 硕士 研 究 生 入 学考 试 中
线性 代 数 试 题 的题 解

1 行列式

1. 设 A = (α,γ2 ,γ3 ,γ4 ) , B = (β,γ2 ,γ3 ,γ4 ) , 其中 α,β,γ2 ,γ3 ,γ4 为四维列
向量 ,已知 | A| = 4 , | B| = 1 , 求 | A + B| .
解 | A + B| = |α+ β,2γ2 ,2γ3 , 2γ4 |              
= |α,2γ2 ,2γ3 ,2γ4 | + |β,2γ2 ,2γ3 ,2γ4 |
= 23 |α,γ2 ,γ3 ,γ4 | + 23 |β,γ2 ,γ3 ,γ4 |
= 8 | A| + 8 | B| = 8×4 + 8×1 = 40 .
a1 0 0 b1
0 a2 b2 0
2. 计算行列式 D = .
0 b3 a3 0
b4 0 0 a4
解   先将第 4 行依次与第 3、第 2 行 对换 两次 , 再 将第 4 列 与第 3、第 2
列依次对换两次 , 得
a1 b1 0 0
b4 a4 0 0 a1 b1 a2 b2
D = ( - 1)4 =
0 0 a2 b2 b4 a4 b3 a3
0 0 b3 a3
= ( a1 a4 - b1 b4 ) ( a2 a3 - b2 b3 ) .
3. 设 A∈R m× n , B∈Rn× m , 则下列命题必成立的是 (   ) .
① 若 m > n, 则 | AB| ≠0 ;     ② 若 m > n, 则 | AB| = 0 ;
③ 若 n > m, 则 | AB| ≠0 ;     ④ 若 n > m, 则 | AB| = 0 .
1 行列式 297

答   命题②必成立 .因为 : AB 是 m 阶矩阵 , 且


秩 ( AB) ≤min( 秩 ( A) , 秩 ( B) ) ≤ n < m .
所以 | AB| = 0 .
x- 2 x- 1 x- 2 x- 3
2x - 2 2x- 1 2x- 2 2x- 3
4. 方 程 f ( x ) = = 0 的根的个数有
3x - 3 3x- 2 4x- 5 3x- 5
4x 4x- 3 5x- 7 4x- 3
几个 ?
解   法 1 : 第 1 列乘 ( - 1 )分别加到第 2 , 3 , 4 列 , 然后再将第 2 列加到第
4列,得
x-2 1 0 -1 x-2 1 0 0
2x - 2 1 0 -1 2x - 2 1 0 0
f ( x) = =
3x - 3 1 x-2 -2 3x - 3 1 x-2 -1
4x -3 x-7 -3 4x -3 x-7 -6
x- 2 1 x- 2 - 1
=
2x- 2 1 x- 7 - 6
= ( x - 2 - 2 x + 2) ( - 6 x + 12 + x - 7) = - x( - 5 x + 5) = 0 ,
从而 x = 0 , 1 , 所以方程 f ( x) = 0 有两个根 .
法 2: 将第 3 列加到第 1 列 , 第 4 列加到第 2 列 , 然后 , 第 2 列乘( - 1)加到
第 1 列 , 第 3 列乘 ( - 2)加到第 2 列 , 得
2x - 4 2x - 4 x-2 x-3
4x - 4 4x - 4 2x - 2 2x-3
f ( x) =
7x - 8 6x - 7 4x - 5 3x-5
9x - 7 8x - 6 5x - 7 4x-3
0 0 x- 2 x- 3
0 0 2x - 2 2x - 3
= =0 .
x- 1 - 2x+3 4x - 5 3x - 5
x- 1 - 2x+8 5x - 7 4x - 3
看得出 f ( x)是 x 的二次多项式 , 且 x = 1 时 , 第 1 列全部 为 0 , 所以 , 1 是
f ( x) = 0 的一个根 ;当 x = 0 时 , 行列式的第 1 与第 2 行相同 , 所以 , 0 是 f ( x) =
0 的又一个根 .
298 历年硕士研究生入学考试中线性代数试题的题解

法 3 : 第 1 行乘( - 1)加到第 2 行 , 然后第 2 行乘 ( - 1) , ( - 3) , ( - 4) 分别


加到第 1 , 3 , 4 行 , 再提出第 2 行的公因子 x 得
x- 2 x-1 x-2 x-3
x x x x
f ( x) =
3x - 3 3x - 2 4x - 5 3x-5
4x 4x - 3 5x - 7 4x-3
- 2 -1 -2 - 3
1 1 1 1
= x =0 .
- 3 -2 x- 5 - 5
0 -3 x- 7 - 3
看得出 f ( x)是个二次多项式 , 在复数域上有两个根 .
5. 设 A, B 均为 n 阶矩阵 , | A| = 2 , | B| = - 3 ,求 | 2A* B - 1 | .
解   由 AA* = | A| I,得 | A| | A* | = | A| n .而 | A| = 2≠0 , 所以
| A* | = | A| n - 1 .
于是
| 2A* B - 1 | = 2n | A* | | B- 1 | = 2n | A| n - 1 | B| - 1

1 1 2 n- 1
= 2n ・2 n - 1 = - 2 .
- 3 3
6. 设 α= ( 1 , 0 , - 1 ) T ,矩阵 A = ααT , n 为正 整数 , I 为三 阶单位 矩阵 , 则
| a I - An | = .
解   由于 αT α= 2 , 所以 , 有
A2 = α( αT α)αT = 2ααT = 2 A .
由归纳法得
An = 2 n - 1 A,

 1
| aI - An | = | aI - 2 n - 1 ααT | = aI - 2n - 1   0 (1 , 0 , - 1)
- 1
a 1 0 - 1
= a - 2n - 1 0 0 0
a - 1 0 1
2 矩阵 299

a - 2n- 1 0 2n - 1
= 0 a 0 = a( a - 2 n - 1 ) 2 - (2 n - 1 ) 2
n-1
2 0 a - 2n- 1
= a2 ( a - 2n ) .
7. 设四阶矩阵 A~ B, A 的特征值为 2 ,3 ,4,5,计算 | B - I| .
解   四阶矩阵 A 有 4 个互不相同的特征值 ,所以 A 相似于对角阵 diag(2 ,
3 , 4 , 5 ) .由 于 B~ A, 所 以 B 也相 似 于 diag ( 2 , 3 , 4 , 5 ) , 因 此 , 存 在 可逆 矩 阵
P,使
  P- 1 BP= diag( 2 , 3 , 4 , 5) ,          
- 1
从而 B = P diag(2 , 3 , 4 , 5) P ,
故   | B - I | = | P dia g(2 , 3 , 4 , 5 ) P - 1 - PP- 1 |
= | P( dia g(2 , 3 , 4 , 5 ) - I) P - 1 |
= | P| | diag(1 , 2 , 3 , 4) | | P- 1 | = 4 ! = 24 .
8. 设行列式
3 0 4 0
2 2 2 2
D= ,
0 - 7 0 0
5 3 - 2 2
则第 4 行各元素代数余子式之和的值为 .

0 4 0 3 4 0
    A41 = - 2 2 2 = 56;     A42 = 2 2 2 = 0;
-7 0 0 0 0 0
3 0 0 3 0 4
A43 = - 2 2 2 = - 42; A44 = 2 2 2 = - 14 .
0 - 7 0 0 - 7 0
第 4 行各元素代数余子式之和为 A41 + A42 + A43 + A44 = 0 .

2 矩阵

1. 给定 A,且 A - 2 I 可逆 ,已知 AB = A + 2B, 求 B .


解   由 AB = A + 2 B 得   AB - 2 B = A,即
300 历年硕士研究生入学考试中线性代数试题的题解

( A - 2 I) B = A,
所以 B = ( A - 2 I) - 1 A .
2. 设 n 阶矩阵 A 的行列式 | A| = a≠0 , 求 | A* | .
解   | A* | = | A| n - 1 = an - 1   ( 见行列式中第 5 题 ) .
3. 已知 AP = PB, B 为对角矩阵 , P 为下三角形可逆矩阵 ,求 A, A5 .
解   由 AP = PB 得 A = PBP - 1 , 从而
A5 = ( PBP - 1 ) ( PBP- 1 ) ( PBP - 1 ) ( PBP- 1 ) ( PBP - 1 )
= PB5 P - 1 .
3 0 0
4. 设 A = 1 4 0 , 求 ( A - 2 I) - 1 .
0 0 3
1  0 0
1 0 0
-1 1 1
解  A- 2I= 1 2 0 ,   ( A - 2 I) = - 0 .
2 2
0 0 1
0  0 1
5 2 0 0
2 1 0 0
5. 设 A = , 求 A- 1 .
0 0 1 - 2
0 0 1 1
解   用分块的方法求其逆矩阵 .即
- 1
- 1
A1 0 A1- 1 0
A = =
0 A2 0 A2- 1
 1 - 2  0 0
- 2  5  0 0

= 1 2 .
 0  0  
3 3
1 1
 0  0 -
3 3
1 - 1  0  0 2 1 3 4
0  1 - 1  0 0 2 1 3
6. 设 B = ,   C= ,
0  0  1 -1 0 0 2 1
0  0  0  1 0 0 0 2
2 矩阵 301

且矩阵 A 满足 A( I - C - 1 B) T CT = I, 试化简方程 , 并求 A .
解   思路 : 先利用转置和逆的运算规律 :
CT AT = ( AC ) T ,   ( A - B) T = AT - BT , ( AB) - 1 = B- 1 A - 1 ,
化简左式 .再利用 AP = I, 则 A - 1 = P .
左 = A( I - C - 1 B) T CT = A[ C( I - C - 1 B) ] T .
= A( C - B) T = 右边 = I,
从而得 A - 1 = ( C - B) T , A = ( ( C - B) T ) - 1 .故
1 0 0 0  1  0  0  0
2 1 0 0 -2  1  0  0
A- 1 = (C - B)T = , A= ( A- 1 ) - 1 = .
3 2 1 0  1 -2  1  0
4 3 2 1  0  1 -2  1
7. 设 A, B, C 均为 n 阶矩阵 , 且 ABC = I,则必有 (   ) .
① ACB = I;   ② CBA = I;   ③ BAC = I;   ④ BCA = I .
解   必有 BCA = I .因为
ABC= A( BC) = I,
-1
所以 BC = A , 从而有 BCA = ( BC) A = A - 1 A = I .
8. 设 A, B 为三阶矩阵, 且 AB + I = A2 + B,其中
1 0 1
A= 0 2 0 .
- 1 0 1
求 B.
解   由 AB + I = A2 + B 得 AB - B = A2 - I,即 ( A - I) B = A2 - I, 所以
 0 0 1 - 1
- 1 0  2
-1
B = ( A - I) ( A2 - I) =  0 1 0  0 3  0
-1 0 0 - 2 0 -1
0 0 - 1 -1 0  2  2 0 1
= 0 1  0  0 3  0 =  0 3 0 .
1 0  0 -2 0 - 1 - 1 0 2
a1 b1 a1 b2 … a1 bn
a2 b1 a2 b2 … a2 bn
9. 设 A = ,
… … …
an b1 an b2 … an bn
302 历年硕士研究生入学考试中线性代数试题的题解

其中 ai bi ≠0( i = 1 , 2 , … , n) , 求 r( A) .
解   设 α= ( a1 , a2 , … , an ) T ,β= ( b1 , b2 … , bn ) , 则
A = αβ.
于是 ,r ( A) ≤ min( r (α) , r (β) ) .由于 α≠0,β≠0, 所以 , r (α) = 1 , r (β) = 1 .又 A
为非零矩阵 , 故 r ( A) ≥1 .因此
1≤ r ( A) ≤ min( r ( α) , r (β) ) = 1 .
从而即得 r ( A) = 1 .
1 1
10. 设 α= (1 , 2 , 3) ,β= 1, , , A = αT β, 则 An = .
2 3
解   因为 A2 = (αT β) (αT β) = αT (βαT )β= (βαT )αT β= (βαT ) A, 所以
    An = (αT β) (αT β) … (αT β) (αT β)
= αT (βαT ) (βαT ) … (βαT )β= (βαT ) n - 1 αT β= (βαT ) n - 1 A .

1
1 1
βαT = 1    2 = 3,
2 3
3
1 1
1
2 3
1
1 1 2
故 An = 3 n - 1 αTβ = 3n - 1 2 1    = 3n - 1 2 1 .
2 3 3
3
3
3 1
2
11. 设 A 为 n 阶非零实矩阵 , 当 A* = AT 时 , 证明 | A| ≠0 .
证   法 1 : 由 A* = AT 得 aij = Ai j ( A ij 是 a ij 的代数余子式 ) .因为 A 为非
零实矩阵 , 所以存在 ai j ≠0 , 将行列式 | A| 对第 i 行展开 , 即得
n n

∑a ∑a
2
        | A| = ik A ik = ik
k= 1 k= 1

= a + … + a + … + a2in > 0   (因为 aij ≠0) .


2 2
i1 ij

法 2 : 由 A* A = AT A = | A | I 可 知 , 欲 证 | A | ≠ 0 , 只 要 证 AT A≠0 ( 零
矩阵) .
设 A = (α1 ,α2 ,… ,αn ) , 其中 αj = ( a1 j , a2 j , … , an j ) T ( j = 1 , … , n) , 则
2 矩阵 303

α1T
α2T
T
A A= (α1 ,α2 , … ,αn ) = (αTi αj ) n× n .

αTn
由于 A = (αij ) n× n ≠0 , 所以存在 aij ≠0 , 即存在 αj ≠0, 从而有
n

α αj = ∑a = ‖αj ‖2 ≠0 .
T 2
j ij
i=1

因此 , A A≠0 , 于是 | A| ≠0 得证 ( 因为若 | A| = 0 , 则 A* A = AT A = | A| I = 0 ) .
T

1 1 1
12. 设 A = diag , , , 且 A - 1 BA = 6 A + BA, 求 B .
3 4 7
解   由 A - 1 BA = 6 A + BA, 得
( A - 1 - I) BA = 6 A .
注意到 A 可逆 ,等式两边右乘 A - 1 , 又得
( A - 1 - I) B = 6 I .
于是
    B = 6 ( A - 1 - I) -1 - 1
= 6 diag(3 , 4 , 7) - I
-1 1 1 1
= 6 diag(2 , 3 , 6) = 6diag , , = diag( 3 , 2 , 1 ) .
2 3 6
1 1 - 1
13. 设 A = 0 1   1 , 且 A2 - AB = I, 求 B .
0 0 - 1
解   由 A2 - AB = I, 即 AB = A2 - I,得
      B = A - 1 ( A2 - I) = A - A - 1 .
1 1 - 1 1 - 1 -2 0 2 1
= 0 1  1 - 0  1  1 = 0 0 0 .
0 0 - 1 0  0 -1 0 0 0
1 2 -3 - 2 1 2 0 1
0 1  2 - 3 0 1 2 0
14. 设 B = ,   C= ,
0 0  1  2 0 0 1 2
0 0  0  1 0 0 0 1
且( 2 I - C - 1 B) AT = C - 1 , 求 A .
304 历年硕士研究生入学考试中线性代数试题的题解

解   由 ( 2 I - C - 1 B) AT = C - 1 , 得
AT = ( 2 I - C - 1 B) - 1
C-1 = C( 2 I - C - 1 B) - 1
= ( 2 C - B) - 1 .
计算过程略去 , 答案为
1 - 2 1 0 1 0 0 0
0 1 -2 1 - 2 1 0 0
AT = ,   A= .
0 0 1 - 2 1 - 2 1 0
0 0 0 1 0 1 - 2 1
 1  1 -1
15. 设 A = - 1  1  1 ,
 1 -1  1
* -1
求矩阵 X,使之满足 A X= A +2X.
解   由 A* X= A - 1 + 2 X,即 ( A* - 2 I) X= A - 1 , 得
X = ( A* - 2 I) -1
A - 1 = ( A( A* - 2 I) ) - 1
= ( | A| I - 2 A) - 1
-1
2 -2 2 1 1 0
1
= 2 2 -2 = 0 1 1 .
4
- 2 2 2 1 0 1
16. 设 A = diag( 1 , - 2 , 1 ) , 且 A* BA = 2 BA - 8 I, 求 B .
解   由 A* BA = 2 BA - 8 I, 即 ( A* - 2 I) BA = - 8 I, 得
B = - 8 ( A* - 2 I) - 1 A - 1 = - 8 ( A( A* - 2 I) ) - 1

= - 8( AA * - 2 A) - 1 = - 8( | A| I - 2A) - 1   ( | A| = - 2 )
1
= - 8 ( - 2 I - 2 A) - 1 = - 8× ( I + A) - 1
- 2
1 1
= 4 ( diag( 2 , - 1 , 2 ) ) - 1 = 4dia g , - 1,
2 2
= diag (2 , - 4 , 2 ) .
17. 设矩阵 A 的伴随矩阵为
1 0 0 0
0 1 0 0
A* = ,
1 0 1 0
0 - 3 0 8
2 矩阵 305

且 ABA - 1 = BA - 1 + 3 I,求 B .
解   由   | A* | = 8 = | A| 4 - 1 , 得 | A| = 2( 后面要用 ) .
再由   ABA - 1 = BA - 1 + 3 I,得
( A - I) BA - 1 = 3 I,
于是 ,
          B = 3 ( A - I) - 1 A ( 1)
= 3 ( A - I) - 1 ( A - 1 ) - 1 = 3 ( A - 1 ( A - I) ) - 1
= 3 ( I - A- 1 ) - 1 ( 2)
1 *
法 1 : 将 A - 1 = | A| - 1
A* = A 代入 ( 2 ) 式 ,得
2
A* -1
2 I - A* - 1
B= 3 I - =3 = 6 ( 2 I - A* ) -1
.
2 2
所以

- 1 1 0 0 0
1 0 0 0 6 0 0 0
0 1 0 0
0 1 0 0 0 6 0 0
B= 6 =6 1 0 1 0 = .
- 1 0 1 0 6 0 6 0
1 1
0 3 0 - 6 0 0 - 0 3 0 - 1
2 6
1 *
法 2 : 由 A- 1 = A ,得
| A|
A = | A| ( A* ) - 1 = 2 ( A* ) -1
.

- 1 2 0 0 0
1 0 0 0
0 2 0 0
0 1 0 0
将 A= 2 = - 2 0 2 0 代入 (1 ) , 得
1 0 1 0
3 1
0 -3 0 8 0 0
4 4
- 1
1 0 0 0 2 0 0 0
0 1 0 0 0 2 0 0
- 1
B = 3 ( A - I) A= 3 -2 0 1 0 - 2 0 2 0
3 -3 3 1
0 0 0 0
4 4 4 4
306 历年硕士研究生入学考试中线性代数试题的题解

6 0 0 0
0 6 0 0
= .
6 0 6 0
0 3 0 1
18. 设
1 0 0 0
-2 3 0 0
A= ,
0 - 4 5 0
0 0 - 6 7
-1 - 1
且 B = ( I + A) ( I - A) , 则 ( I + B) = .
解   由 B = ( I + A) - 1 ( I - A) ,即 ( I + A) B = I - A,得
( I + A) B + ( I + A) = 2 I .
于是 ( I + A) ( I + B) = 2 I,
1 0 0 0

1 -1 2 0 0
所以 ( I + B) - 1 = ( I + A) = .
2 0 -2 3 0
0 0 - 3 4
a21 a22 a23
19. 设 A= ( aij )3× 3 ,B = a11 a12 a13 ,
a31 + a11 a32 + a12 a33 + a13
0 1 0 1 0 0
P1 = 1 0 0 ,   P2 = 0 1 0 ,
0 0 1 1 0 1
则必有(   ) .
① AP1 P2 = B;     ② AP2 P1 = B;
③ P1 P2 A = B;     ④ P2 P1 A = B .
解   B 是由 A 经初等行变换——— ( 1 ) 第 1 行加到第 3 行 , ( 2 ) 第 1 与第 2
行对换而得到的 .这里 P2 是倍加初等阵, 它左乘 A( 即 P2 A) 是把 A 的 第 1 行
乘 1 加到第 3 行 ; P1 是对换初等阵 , 它左乘 P2 A 是将 P2 A 的第 1 与第 2 行对
换 .所以③成立 , 即
P1 ( P2 A) = P1 P2 A = B .
2 矩阵 307

20. 设 A 是 4×3 矩阵 ,且 A 的秩 r ( A) = 2 , 而
1 0 2
B= 0 2 0 ,
- 1 0 3
则 r ( AB) = .
解   由于 | B| = 10≠0 , 所以 B 可逆 ,即 r ( B) = 3 .
法 1 : 因为 B 满秩 ,所以 , r ( AB) = r ( A) = 2 .
法 2 : 利用 r ( A) + r ( B) - 3≤ r ( AB) ≤ min{r ( A) , r ( B) } ,
2 + 3 - 3≤ r ( AB) ≤ min{2 , 3} = 2 ,
所以 , r ( AB) = 2 .
21. 设 A 为 n 阶矩阵 ( n≥3) , k 为常数( k≠0 , ±1) , 则 ( kA) * = (   ) .
① k A * ;   ② kn - 1 A* ;   ③ kn A * ;   ④ k - 1 A* .

解   ( k A) = kn - 1 A* .因为 : k A 的第 i 行第 j 列元 素 kai j 的代 数余 子
式( k A) ij (是 n - 1 阶行列式 ) , 是 A 的元素 ai j 的代数余子式 A i j 的 k n - 1 倍 , 即
ka11 … ka1 j - 1 ka1 j + 1 … ka1n
… … … …
kai - 1, 1 … kai - 1, j - 1 kai - 1, j + 1 … kai - 1, n
( kA) ij = = kn - 1 Aij .
kai + 1, 1 … kai + 1, j - 1 kai + 1, j + 1 … kai+ 1, n
… … … …
kan1 … kan, j - 1 kan, j + 1 … kann

于是   ( k A) = ( ( kA ) ij ) Tn× n = ( kn - 1 Ai j ) Tn× n = kn - 1 ( A ij ) n×
T
n = k
n-1
A* .故
选② .
22. 设 A 为 n 阶可逆矩阵 , A 的第 i , j 行对换后得 B .
( 1 ) 证明 B 可逆 ;     ( 2 ) 求 AB- 1 .
解   ( 1 ) 因为 A 可逆 | A| ≠0 ,又 | B| = - | A| ≠0, 所以 B 可逆 .
( 2 ) 第 i, j 行对换的初等矩阵为 E ij , 因为
Eij E ij = I,   所以 Eij- 1 = Eij .
由于 B = Eij A,   B - 1 = A - 1 Eij- 1 = A - 1 Eij , 所以
AB- 1 = A( A - 1 Eij ) = Eij .
23. β= ( x1 , … , xn ) T ≠0 , A = I - ββT , 证明 :
308 历年硕士研究生入学考试中线性代数试题的题解

( 1) A2 = A 的充分必要条件是βTβ= 1 ;
( 2) 当 βT β= 1 时 , A 不可逆 .
证   思路 : 注意   β
βT 是 n× n 矩阵 , 而 βT β是 1×1 矩阵 , 可以看成 一个
数 k.
(ββT ) 2 = (ββT ) (ββT ) = β(βTβ)βT = kββT .
( 1 ) 设 βT β= k, 则
A2 = ( I - ββT ) 2
= I - 2ββT + β(βTβ)βT
= I + ( k - 2)ββT .
由于 A2 = A,即得 I + ( k - 2 )ββT = I - β
βT 的充分必要条件为
k - 2 = - 1 ,   即 ββT = k = 1 .
( 2) 由 (1 )的结论 , A2 = A 的充分必要条件是βT β= 1 .
反证法 : 若 A 可逆, 在 A2 = A 等式两边左乘 A - 1 , 得
A = I,这与 A = ( I - ββT ) 矛盾 ( 因为 ββT ≠0 ) , 所以 A 不可逆 .
另一证法 :   由于 βT β= 1 时 , A2 = A,即
A2 - A = ( A - I) A = 0,
所以 r ( A - I) + r ( A) ≤ n .
又 A- I= -β
βT ,故 r ( A - I) = 1,
所以 r ( A) ≤ n - 1 , 从而 A 不可逆 .
24. 设 A, B 为 n 阶矩阵 , A* , B* 分别为 A, B 对应的伴随矩阵 ,分块矩阵
A 0
C= ,则 C 的伴随矩阵 C * = .
0 B
| A| A* 0 | B| B* 0
( A) *
;     ( B) ;
0 | B| B 0 | A| A*
| A| B* 0 | B| A* 0
( C) ;     (D ) .
0 | B| A* 0 | A| B*
解   C 矩阵在 A 的第 i 行、第 j 列处的代数余子式为
( C( A) ) ij = Aij | B| ,
其中 Ai j 是 A 的元素 ai j 的代数余子式 .
C 矩阵在 B 的第 i 行、第 j 列处的代数余子式为
( C( B) ) ij = Bi j | A| ,
2 矩阵 309

其中 Bij 是 B 的元素 bij 的代数余子式 .所以


| B| A* 0
C = .( 答案为 ( D) )
0 | A| B*
k 1 1 1
1 k 1 1
25. 设矩阵 A= ,
1 1 k 1
1 1 1 k
且 r ( A) = 3, 则 k = .
解   做初等变换 : 先将第 1 , 4 列 对换 , 再做 行变 换 , 将其化 为阶 梯形 矩
阵 ,即
k 1 1 1 1 1 1 k
1 k 1 1 0 k- 1 0 1- k
A= →
1 1 k 1 0 0 k- 1 1- k
1 1 1 k 0 1 - k 1 - k 1 - k2
1 1 1 k 1 1 1 k
k≠1 0 1 0 -1 0 1 0 -1
→ .
0 0 1 -1 0 0 1 -1
0 1 1 1+ k 0 0 0 3+ k
显然 , k≠1( 若 k = 1 , 则 r( A) = 1 ) .已知 r( A) = 3 ,所以 ,3 + k = 0 , 即 k = - 3 .
26. 设
a11 a12 a13 a14 a14 a13 a12 a11
a21 a22 a23 a24 a24 a23 a22 a21
A= ,     B= ,
a31 a32 a33 a34 a34 a33 a32 a31
a41 a42 a43 a44 a44 a43 a42 a41
0 0 0 1 1 0 0 0
0 1 0 0 0 0 1 0
P1 = ,     P2 = ,
0 0 1 0 0 1 0 0
1 0 0 0 0 0 0 1
其中 A 可逆 , 则 B - 1 等于 .
( A) A - 1 P1 P2 ;             ( B ) P1 A - 1 P2 ;
310 历年硕士研究生入学考试中线性代数试题的题解

( C ) P1 P2 A- 1 ; ( D) P2 A - 1 P1 .
解   矩阵 B 是 A 经过第 1 ,4 列对换和第 2 ,3 列对换而得到的 , 所以
B = AP1 P2 ,   或   B = AP2 P1 .
而初等对换阵 P1 , P2 的逆矩阵还是自身 ,即 P1- 1 = P1 , P2- 1 = P2 , 于是
B - 1 = ( AP1 P2 ) - 1 = P2- 1 P1- 1 A - 1 = P2 P1 A - 1 ,
或 B - 1 = ( AP2 P1 ) - 1 = P1- 1 P2- 1 A - 1 = P1 P2 A - 1 .
答案为 ( C) .
27.   已知 A, B 为三阶矩阵 , 且满足 2A - 1 B = B - 4 I .
( 1 ) 证明 : 矩阵 A - 2 I 可逆 ;
1 - 2 0
( 2) 若 B= 1 2 0 , 求矩阵 A .
0 0 2
解   ( 1 ) 在 2 A - 1 B = B - 4 I 两边左乘 A, 得
2 B = AB - 4 A,   即   ( A - 2 I) B = 4 A . ( 1)
已知条件中有 A ,表示 A 可逆, 在 ( 1 ) 式两边右乘 A - 1 ,得
-1

1
( A - 2 I) BA - 1 = 4 I,   即   ( A - 2 I) BA - 1 = I . ( 2)
4
1
由 ( 2 ) 式可知 : A - 2 I 可逆 ,且 ( A - 2 I) - 1 = BA - 1 .
4
1
( 2 ) 在 2 A - 1 B = B - 4 I 两边右乘 B - 1 ,得
2
1
A- 1 = I - 2 B- 1 .
2
从而得 ( 计算过程略去 )
1
- - 1  0
2 0 2 0
- 1 1 - 1 -1
A =   0  0 ,   A= ( A ) = - 1 -1 0 .
2
1 0 0 - 2
 0 0 -
2

3 线性方程组

1. n 维向量 α1 ,α2 ,… ,αs ( s≥3 )线性无关的条件是什么 ?


解   α1 ,α2 ,… ,αs 线性无关 , 即 : 若
3 线性方程组 311

x1 α1 + x2 α2 + … + xsαs = 0 , ( 1)
则 x1 , x2 , … , xs 必须全为 零 .这里 ( 1 ) 式 的 向量 方程 可 以表 示为 ( 或 说等 价
于) 一个齐次线性方程组
x1 0
x2 0
Ax= 0   即   (α1 ,α2 ,… ,αs ) = , ( 2)
… …
xs 0
其中 A 是将 α1 ,α2 ,… ,αs 按列排成的 n× s 矩阵 .
所以 ,α1 ,α2 , …,αs 线性无关的充要条件是齐 次线性方程组 Ax = 0 只 有
零解 .
2. 设四阶矩阵 A 的行列式 | A| = 0 ,则 A 中必有 (   ) .
① A 的列向量线性相关 ,且任意 3 个列向量也线性相关 ;
② A 的 4 个列向量两两线性相关 ;
③ A 中必有一个列向量是其余向量的线性组合 ;
④ A 中任意 3 个行向量线性无关 , 但其 4 个行向量线性相关 .
答   答案为③ .这里要注意 : 题中 4 个条件 都能保证 | A| = 0 ,但 | A| = 0
时 ,并不必须是① , ② , ④的情况 , 而必须是③的情况 .
3. 已知 α1 ,α2 ,α3 ,α4 线性无关 ,则 (   ) .
① α1 + α2 ,α2 + α3 ,α3 + α4 ,α4 + α1 也线性无关 ;
② α1 - α2 ,α2 - α3 ,α3 - α4 ,α4 - α1 也线性无关 ;
③ α1 + α2 ,α2 + α3 ,α3 + α4 ,α4 - α1 也线性无关 ;
④ α1 + α2 ,α2 + α3 ,α3 - α4 ,α4 - α1 也线性无关 .
解   ① (α1 + α2 ) - (α2 + α3 ) + (α3 + α4 ) - (α4 + α1 ) = 0, 所以 ①组向 量
线性相关 ;
② (α1 - α2 ) + (α2 - α3 ) + (α3 - α4 ) + (α4 - α1 ) = 0,所 以②组 向量线 性
相关 ;
④ (α1 + α2 ) - (α2 + α3 ) + (α3 - α4 ) + ( α4 - α1 ) = 0,所 以④组 向量线 性
相关 ;
作为考题 , 排除了① , ② , ④ , 则③组向量必是线性无关的 .其证明为 : 设
x1 (α1 + α2 ) + x2 (α2 + α3 ) + x3 (α3 + α4 ) + x4 (α4 - α1 ) = 0,

312 历年硕士研究生入学考试中线性代数试题的题解

( x1 - x4 )α1 + ( x1 + x2 )α2 + ( x2 + x3 )α3 + ( x3 + x4 )α4 = 0 .


由于 α1 ,α2 ,α3 ,α4 线性无关 , 所以上式中它们的系数必须全为 0 ,得齐次线性
方程组:
x1 - x4 = 0 ,   x1 + x2 = 0 ,   x2 + x3 = 0 ,   x3 + x4 = 0 . ( 1)
方程组( 1) 的系数行列式
1 0 0 -1
1 0 0 1 1 0
1 1 0 0
= 1 1 0 - ( - 1) 0 1 1 = 2≠0 ,
0 1 1 0
0 1 1 0 0 1
0 0 1 1
所以方程组 ( 1)只有零解 , 因此向量组③线性无关 .
4.   设 α1 = ( 1, 0 , 2 , 3 ) ,α2 = ( 1, 1, 3 , 5 ) ,α3 = ( 1 , - 1 , a + 2 , 1 ) , α4 =
( 1 ,2,4 , a + 8) ,β= ( 1 ,1, b+ 3 , 5) .试问 :
① a, b为何值时 ,β不能由 α1 ,α2 ,α3 ,α4 线性表示 ;
② a, b为何值时 ,β可由 α1 ,α2 ,α3 ,α4 惟一地线性表示 .
解   这个问题是讨论 : 向量 方程 x1 α1 + x2 α2 + x3 α3 + x4 α4 = β, 何时 无
解 ,何时有惟一解 .这个向量方程可 表示为 一个非 齐次线 性方程 组 Ax = βT ,
其增广矩阵为
1 1 1 1 1
初等 0 1 - 1 2 1
(α1T , α2T , α3T , α4T βT ) .
行 变换 0 0 a+1 0 b
0 0 0 a+ 1 0
由此可见 : ① a + 1 = 0 即 a = - 1 , 且 b≠0 时 , 方程组 Ax= βT 无解 ,即 β不能
由α1 ,α2 ,α3 ,α4 线性表示 ;
② a + 1≠0 , 即 a≠ - 1 时 , Ax= βT 有惟 一解 , 即 β可 由 α1 ,α2 ,α3 ,α4 惟
b a + b+ 1
一地线性表示 .此时, 方程组的解为 : x4 = 0 , x3 = , x2 = 1 + x3 = ,
a+1 a+ 1
- 2b
x1 = 1 - x2 - x3 - x4 = 1 - (1 + x3 ) - x3 - 0 = - 2 x3 = .所以
a+ 1
1
β= [ - 2bα1 + ( a + b + 1)α2 + bα3 ] .
a+ 1
5. 题型与 4 相同 ( 略去 ) .
6. 设 α1 = ( 1, 1 ,1 , 3 ) T ,α2 = ( - 1 , - 3 , 5 , 1 ) T ,α3 = (3 , 2 , - 1 , p + 2 ) T ,
3 线性方程组 313

α4 = ( - 2 , - 6 , 10 , p) T , 试求 :
① p 为何值时 ,α1 ,α2 ,α3 ,α4 线性无关 ,并将 α= ( 4 ,1 ,6 ,10 ) 用它们线性
表示 .
② p 为何值时 ,α1 ,α2 ,α3 ,α4 线性相关 ,并求一个极大线性无关组 .

初等
( A,α) = (α1 , α2 , α3 , α4 α)
行 变换

1 - 1 3 -2 4
0 - 2 -1 -4 -3
. ( 1)
0 0 1 0 1
0 0 0 p-2 1- p
① α1 ,α2 ,α3 ,α4 线性无关的充要条件 是齐次 线性方 程组 Ax = 0 只有 零
解 , 由 (1 )式可见 , p - 2≠0 , 即 p≠2 时 , Ax = 0 只 有零 解 , 从而 α1 ,α2 ,α3 ,α4
线性无关 .此时 ,非齐次线性方程组 Ax = α的解为
1 - p 3p- 4
x4 = ,   x3 = 1 ,   x2 = ,   x1 = 2 .
p - 2 p- 2
3p - 4 1 - p
所以 α= 2α1 + α2 + α3 + α4 .
p- 2 p - 2
② 当 p - 2 = 0 , 即 p = 2 时 ,α1 ,α2 ,α3 ,α4 线性相 关 .上面矩 阵经 初等 行
变换后 , 对应的列向量组有相同的线 性相关 性 .在 阶梯形 矩阵中 前 3 列线 性
无关 ,所以 {α1 ,α2 ,α3 } 是 α1 ,α2 ,α3 ,α4 的一个极大线性无关组 .
7. 设 β可由 {α1 , …,αm }线性表 示 , 记 (Ⅰ) : {α1 , …,αm - 1 } , ( Ⅱ ) : {α1 , … ,
αm - 1 ,β} ,若 β不能由 ( Ⅰ ) 线性表示 , 则 (   ) .
① αm 不能由 (Ⅰ ) 表示 , 也不能由( Ⅱ )表示 ;
② αm 不能由 (Ⅰ ) 表示 , 但可由( Ⅱ )表示 ;
③ αm 可由 (Ⅰ ) 表示 , 也可由( Ⅱ )表示 ;
④ αm 可由 (Ⅰ ) 表示 , 但不可由( Ⅱ )表示 .
解   题设为 : 存在 x1 , … , x m - 1 , xm , 使
β= x1 α1 + … + x m - 1 αm - 1 + xmαm . ( 1)
若 β不能由 ( Ⅰ ) : {α1 , … ,αm - 1 } 线性表示 ,则由 ( 1 ) 式可见 x m ≠0 , 且 αm 也不
能由( Ⅰ )表示 (否则 β可由 ( Ⅰ) 表示 ) ; 此时 , 由 (1) 式又可得
314 历年硕士研究生入学考试中线性代数试题的题解

1
αm = (β- x1 α1 - … - xm - 1 αm - 1 ) .
xm
因此 αm 可由 (Ⅱ ) 表示 .所以答案为② .
8. n 维向量 α1 ,… ,αm ( m < n) 线 性无关 , 则 n 维向 量β1 ,β2 , … ,βm 线 性
无关的充要条件为 (   ) .
① 向量组 α1 , … ,αm 可以由向量组β1 ,β2 ,… ,βm 线性表示 ;
② 向量组 β1 ,β2 ,… ,βm 可以由向量组 α1 ,… ,αm 线性表示 ;
③ 向量组 α1 , … ,αm 与向量组β1 ,β2 ,… ,βm 等价 ;
④ 矩阵 A = (α1 , … ,αm ) 可 以经 过 初 等 变 换 化 为 矩阵 B = (β1 ,β2 , … ,
βm ) , 即 Aǖ B .
解   答案为④ .因为初等变换不改变矩阵的秩 , 所以 ,
r ( B) = r ( A) = m( 即 β1 ,β2 ,… ,βm 线性无关 )的充要条件为 Aǖ B .
① , ② , ③都不正确是因为 : 两组线 性无关 的向 量相 互间并 不一 定可 以
线性表示 .例如 :
α1 = ( 1 ,0 ,0 ) T ,   α2 = ( 1 ,1 ,1 ) T ;   β1 = ( 0 ,1 ,0 ) T ,   β2 = (0 , 0 , 1) T .
这里 { α1 ,α2 } , {β1 ,β2 } 都 是 线性 无关 的 向量 组 , 它 们相 互 间 不能 线 性 表示 .
然而
1 1 1 0 0 0
A= 0 1 ǖ 0 1 ǖ 1 0 = B.
0 1 0 0 0 1
9. 设 α1 = ( 1 ,2 ,3 ,4 ) ,α2 = ( 2 ,3 ,4 ,5 ) ,α3 = ( 3 ,4 ,5 , 6 ) ,α4 = ( 4 ,5, 6,7 ) ,
求秩 { α1 ,α2 ,α3 ,α4 } = ?
1 2 3 4
初等 0 - 1 -2 - 3
解   (α1 , α2 , α3 , α4 ) ,
行变 换 0 0 0 0
0 0 0 0
所以 , 秩 {α1 ,α2 ,α3 ,α4 } = 2 ( 上列阶梯阵的非零行行数 ) .
10. 设 α1 = ( 1, 2, - 1 , 1 ) , α2 = ( 2 , 0 , t, 0 ) ,α3 = ( 0 , - 4 , 5 , - 2 ) , 若 秩
{α1 ,α2 ,α3 } = 2 , 则 t = .
解   方法同上 , 得 t = 3 .
11. 第 3 章 3 .7 节中第 9 题 .
3 线性方程组 315

12. 第 3 章 3 .7 节中第 8 题 .
T
1
13. (补充题 3 -52 )   设 α= ( 1 , 2, 1 ) T ,β= 1, ,0 ,γ= ( 0 , 0 , 8 ) T ,
2
A = αβT , B = βT α,求解方程 2 B2 A2 x= A4 x+ B4 x+ γ.
1
1 0
1 2
1
解 A= 2 1, ,0 = 2 1 0 ,
2
1 1
1 0
2
1
1
B= 1, ,0 2 =1 + 1= 2,
2
1
    A = (αβ ) (αβ ) = (β α) ( αβT ) = BA = 2 A,
2 T T T

    A4 = (αβT ) (αβT ) (αβT ) (αβT ) = (βT α) 3 (αβT ) = B3 A = 8 A .


由方程 2 B2 A2 x = A4 x+ B4 x + γ, 得
( 2 B2 A2 - A4 - B4 I) x= γ,
即 ( 8 A - 16 I) x= γ,
从而得
-8 4 0 x1 0
16 -8 0 x2 = 0 .
8 4 - 16 x3 8
T
1
该方程有惟一解 x= ( x1 , x2 , x3 ) T = 0,0 , - .
2
14. 已知 α1 ,α2 ,α3 是 Ax = b 的 3 个解 ( A∈ R4 × 4 ) , r ( A) = 3 .α1 = (1 , 2 ,
3 , 4 ) T ,α2 + α3 = ( 0 , 1 , 2 , 3) T , c 为任意常数 , 则 Ax= b 的通解 x = .
T T
① α1 + c( 1 , 1 , 1 , 1) ;     ② α1 + c( 0 , 1 , 2 , 3 ) ;
③ α1 + c( 2 , 3 , 4 , 5) T ;     ④ α1 + c( 1 , 3 , 5 , 7 ) T .
解   答案为③ .因为 :
对应齐次 方程组 Ax = 0 的基 础解 系含 4 - r ( A) = 4 - 3 = 1 个 向 量 .由
Aαi = b( i = 1 , 2 , 3 ) , 得
Aα1 = b,   A(α2 + α3 ) = 2b .
从而
316 历年硕士研究生入学考试中线性代数试题的题解

A( 2α1 - α2 - α3 ) = 2b - 2 b= 0 ,
故 x1 = 2α1 - α2 - α3 = ( 2, 3, 4 ,5 ) T 是 Ax = 0 的基 础解系 , 因此 , Ax = b 的 通
解为α1 + c( 2 , 3 , 4 , 5) T .
15. 已知向量组 α1 = ( 1 ,2 , - 3) T ,α2 = ( 3,0 ,1 ) T ,α3 = ( 9, 6, - 7 ) T ,β1 =
( 0 ,1, - 1 ) T ,β2 = ( a , 2 , 1) T ,β3 = ( b, 1 , 0 ) T , 且秩 (α1 ,α2 ,α3 ) = 秩 (β1 ,β2 ,β3 ) .
又 β3 可由 α1 ,α2 ,α3 线性表示 .求 a, b 的值 .
解   β3 可由 α1 ,α2 ,α3 线性表示 , 向量组 {β3 ,α1 ,α2 ,α3 } 与 {α1 ,α2 ,α3 } 等
价 ,即等秩 .
由观察法看出 α1 ,α2 线性无关 ( 因为 α1 ,α2 不成比例 ) ,α1 ,α2 ,α3 线 性相
关 .事实上 α3 = 3α1 + 2α2 .因为若 令 k1 ( 1 , 2 , - 3 ) T + k2 (3 , 0 , 1 ) T = ( 9 , 6 , -
7) T , 从第 2 个分量看得出 2 k1 + 0 = 6 , k1 = 3 ; 再从第一个分量看得出 3 + 3 k2
= 9 , k2 = 2 .从而得秩(α1 ,α2 ,α3 ) = 秩 (β1 ,β2 ,β3 ) = 2 .
由 β1 = (0 , 1 , - 1) T ,β2 = ( a, 2 , 1 ) T ,β3 = ( b, 1 , 0 ) T , 看出 β1 ,β2 线性无 关
( 不成 比 例 ) .而 秩 (β1 ,β2 ,β3 ) = 2 , 故 β1 ,β2 ,β3 线 性 相 关 .令 β3 = k1 β1 +
k2β2 ,即
( b, 1 , 0 ) T = k1 (0 , 1 , - 1 ) T + k2 ( a, 2 , 1) T ,
b= k2 a,
从而得 k1 + 2 k2 = 1 ,
- k1 + k2 = 0 .
1 a
解得 k1 = k2 = ;b= .
3 3
    已知 β3 可由 α1 ,α2 ,α3 线性表示 , 所以,β3 也可由 α1 ,α2 线性表示 .令
( b, 1 , 0) T = k1 (1 , 2 , - 3) T + k2 (3 , 0 , 1) T ,
1 3
解得 k1 = ,k = , b= 5 , 所以 a = 3 b= 15 .
2 2 2
16. 已知 R3 中 向量 组 α1 = ( a, 2 , 10 ) T ,α2 = ( - 2 , 1 , 5 ) T ,α3 = ( - 1 , 1 ,
4) T ,β= ( 1 , b, c) T .试问 : a, b, c 满足什么条件时 , 有
( 1 ) β可由 α1 ,α2 ,α3 线性表示 , 且表示法惟一 ;
( 2 ) β不能由 α1 ,α2 ,α3 线性表示 ;
( 3 ) β可由 α1 ,α2 ,α3 线性表示 , 但表示法不惟一 ,并求出一般表达式 .
解   ( 1 ) 设 β= x1 α1 + x2 α2 + x3 α3 .此方程组的增广矩阵
3 线性方程组 317

2 1 1 b
a - 2 - 1 1
0 0 - 1 c- 5b
2 1 1 b
a a ab
10 5 4 c 0 - 2- - 1- 1 -
2 2 2
2 1 1 b
a a ab
0 - 2- - 1- 1 - .
2 2 2
0 0 - 1 c- 5b
a
当- 2 - ≠0 , 即 a≠ - 4 时 , 得 惟一 解 ,β可 经 α1 ,α2 ,α3 线性表 示 , 且
2
表示法惟一 .
( 2 ) 当 a = - 4 时 , 此时增广矩阵变换为
2 1 1 b 2 1 0 - 1- b
0 0 1 1 + 2b 0 0 1 1 + 2b .
0 0 - 1 c - 5b 0 0 0 1 - 3 b+ c
当 1 - 3b + c≠0 时 ,β不能由α1 ,α2 ,α3 线性表示 .
( 3) 当 a = - 4 , 1 - 3 b+ c = 0 时 , 方程 组有无 穷多组 解 ,β可由 α1 ,α2 ,α3
线性表示 , 且表示法不惟一 .其一般表达式为
β= k α1 - (1 + b+ 2 k)α2 + (1 + 2 b)α3 , k 为任意常数 .
17. (习题 3 -45)   设向 量组 {α1 ,α2 ,α3 } 线性 相关 , {α2 ,α3 ,α4 } 线 性 无
关 .回答下列问题 ,并证明之 .
( 1 ) α1 能否由{α2 ,α3 } 线性表示 ?
( 2 ) α4 能否由{α1 ,α2 ,α3 } 线性表示 ?
解   ( 1 ) 因为 α2 ,α3 线性无关 ,而 α1 ,α2 ,α3 线性相关 , 所以 α1 可由 α2 ,
α3 线性表示 ( 证明见定理 3 .3 ) .
( 2 ) α4 不能由{α1 ,α2 ,α3 } 线性表 示 .因为如 果 α4 能由 {α1 ,α2 ,α3 } 线 性
表示, 则 α4 就能由 α2 ,α3 线性表示 ( 因为 α1 能由 α2 ,α3 线性表示 ) ,这与 α2 ,
α3 ,α4 线性无关矛盾 .
18. 设 A∈ R n× m , B∈R m× n , n < m, 证 明 : 若 AB = I, 则 B 的 列向 量组 线
性无关 .
证   AB 是 n 阶 矩 阵 , 已 知 r ( AB) = r ( I) = n, 而 r ( AB) ≤ min ( r ( A) ,
r ( B) ) ,所以 r ( B) = n, 因此 , B 的 n 个列向 量的秩 为 n , 即 B 的列向 量组线 性
318 历年硕士研究生入学考试中线性代数试题的题解

无关 .
19. 设非 齐次 线性方 程组 Ax = b 的增广 矩阵 如下 , 问 其中 参数 ( a, b,λ
等) 取何值时 , 方程无解 , 有惟一解 , 有无穷多组解 ? 当有无穷多组解时 , 求其
一般解 .
1 1 1 1 0
1 0 1 λ
0 1 2 2 1
( 1) ;   (2 ) 4 1 1 λ+ 2 ;
0 - 1 a- 3 - 2 b
6 1 4 2λ+ 3
0 2 1 a - 1
1 3 2 4 1 2 λ - 1 1
( 3) 0 1 a - a - 1 ; (4 ) λ -1 1 2 ;
1 2 0 3 3 4 5 - 5 - 1
1 2 1 1
( 5) 2 3 a+ 2 3 .
1 a - 2 0
这些题与我们在第 3 章 3 .6 节中的例 1 , 以 及教材第 3 章 3 .5 节中的 例
2 都是同样类型的 .这里不再给出题解 .留给读者自己独立完成 .
20. (补充题 3 -38 )   已知 β1 ,β2 是 方程 Ax = b 的两 个不同 解 ,α1 ,α2 是
对应齐次方程 Ax= 0 的基础解系 ,则 Ax= b 的一般解是 (   ) .
β1 - β2
( A) k1 α1 + k2 (α1 + α2 ) + ;
2
β1 + β2
( B) k1 α1 + k2 (α2 - α1 ) + ;
2
β1 - β2
( C) k1 α1 + k2 (β1 + β2 ) + ;
2
β1 + β2
( D ) k1 α1 + k2 (β1 - β2 ) + .
2
解   答案为 ( B) .因为 α1 ,α2 - α1 是 Ax= 0 的两个线性无关的解 ,从而是
β1 + β2 β1 + β2 1 1
它的基础 解 系; 是 Ax = b 的一 个 解 , 即 A = Aβ1 + Aβ2 =
2 2 2 2
b b
+ = b .所以 Ax= b 的一般解为 ( B) .
2 2
21. 见第 3 章 3 .7 节中第 5 题 .
3 线性方程组 319

22. 见第 3 章 3 .7 节中第 6 题 .
23. (补充题 3-50 )   设 n 阶 矩阵 A 的每行 元素之 和均 为零 , 又 r ( A) =
n - 1 , 求齐次线性方程组 Ax= 0 的通解 .
解   由 r ( A) = n - 1 可知 , Ax = 0 的 基础解 系只含 n - r ( A) = 1 个解 向
量 ;再由 A 的每行元素之和均为零又 可知 , x = ( 1 , 1, … ,1 ) T 是 Ax = 0 的解 ,
从而也是其基础解系 ,因此 Ax= 0 的通解为
x= k( 1 , 1 , 1 , 1) T ,   k 为任意常数 .
24. 若 ξ1 = ( 1, 0,2 ) T ,ξ2 = (0 , 1 , - 1) T 是齐次线性 方程组 Ax = 0 的解 ,
则系数矩阵 A 为 (   ) .
2 0 - 1
① ( - 2, 1,1 ) ;     ② .
0 1  1
解   答案是① .因为 ξ1 与 ξ2 是
- 2 x1 + x2 + x3 = 0
的两个解 .而 ξ1 与 ξ2 不是以②中矩阵作为系数矩阵的齐次线性方程组的解 .
一般由齐次线性方程组的解 , 反过来 求这个 方程组 的方法 , 请 见第 3 章
3 .5 节中例 2 的另一解法 .
1 1 0 0 5
25. 设 A= 1 1 -1 0 0 ,求齐次线性方程组 Ax= 0 的基础解系 .
0 0 1 1 1
解   求解过程略去 .基础解系为
ξ1 = ( - 1,1 ,0 ,0, 0 ) T ;   ξ2 = ( - 5 , 0 , - 5 , 4 , 1 ) T .
26. 设 A = ( ai j ) 是 n× (2 n) 矩 阵 .已 知 Ax = 0 的基 础解系 为 : ξ1 = ( b11 ,
b12 , … , b1, 2 n ) T ,ξ2 = ( b21 , b22 , … , b2 , 2 n ) T , … ,ξn = ( bn1 , bn2 , … , bn, 2n ) T , 试求 齐
次线性方程组 By= 0,即
b11 y1 + b12 y2 + … + b1, 2 n y 2n = 0 ,
b21 y1 + b22 y2 + … + b2, 2 n y 2n = 0 ,
( 1)
  ……………………………
bn1 y1 + bn2 y2 + … + bn, 2 n y 2 n = 0 .
的通解 , 并说明理由 .
解   由 ( 1)知 :
320 历年硕士研究生入学考试中线性代数试题的题解

ξ1T
ξ2T
B= ,   从而 BT = (ξ1 ,ξ2 ,… ,ξn ) .

ξTn
方程组 ( 1 ) 可 表示为 By = 0 .由于 ξ1 ,ξ2 , … ,ξn 是 Ax = 0 的基 础解 系 , 所 以
2n - r ( A) = n, 即 r ( A) = n; 又因为
Aξi = 0,   i = 1 , 2 , … , n,
于是
A(ξ1 ,ξ2 , … ,ξn ) = (0,0, … ,0) ,
即 ABT = 0 .
因此
( ABT ) T = BAT = 0 . ( 2)

α1
α2
A = ( aij ) n× ( 2n) = ,

αn
则 AT = (α1T ,α2T ,… ,αnT ) . ( 3)
由( 2) , (3 )式可得
BαTj = 0 ,   j = 1 , … , n . ( 4)
由 r ( B) = n 及 (4 )式可见 , A 的 n 个行向量的 转置 α1T ,α2T ,… ,αTn 是 By = 0 的
基础解系 .其一般解为 y= k1 α1T + k2 α2T + … + knαTn ( 其中 k1 , k2 , … , kn 为 任意
常数) 。
27. 见第 3 章 3 .6 节中例 6 .
28. 见第 3 章 3 .5 节中例 3 .
λ 1 λ2
29. 设 A = 1 λ 1 ,
1 1 λ
已知存在三阶矩阵 B≠0,使 AB= 0,则必有 (   ) .
① λ= - 2 , | B| = 0;     ② λ= - 2 , | B| ≠0;
3 线性方程组 321

③ λ= 1 , | B| = 0;       ④ λ= | B| ≠0 .
解   答案为③ .此题与第 3 章 3 .5 节 中例 4 是类 似的 , 请参考 后自己 独
立完成 .
此题的关键为 : 由 AB = 0 可知 , B 的列 向量 β1 ,β2 ,β3 均是 Ax= 0的解 ,
且其中至少有一个是非零解 .因此必 有 | A| = 0 , 观察 即可 见 λ= 1 时 , | A | =
0, 此时 Ax = 0 的基础解系含 3 - r( A) = 3 - 1 = 2个 解向量 , 所以 r( B) ≤2 , 从
而 | B| = 0 .
30. 设
1 2 - 2
A= 4 t 3 ,
3 - 1 1
已知存在三阶非零矩阵 B, 使 AB = 0,则 t = .
解   此题与上题类似 .由 | A| = 7 t + 21 = 0 , 得 t = - 3 .
a1 b1 c1
31. 设矩阵 a2 b2 c2 是满秩的 , 则直线
a3 b3 c3
x - a3 y - b3 z - c3 x - a1 y - b1 z - c1
L1 : = =   与   L2 : = =
a1 - a2 b1 - b2 c1 - c2 a2 - a3 b2 - b3 c2 - c3
( A) 相交于一点 ;       (B) 重合 ;
( C) 平行但不重合 ; ( D) 异面 .
a1 b1 c1 a1 - a2 b1 - b2 c1 - c2
解  a2 b2 c2 → a2 - a3 b2 - b3 c2 - c3 ,
a3 b3 c3 a3 b3 c3
(由于矩阵满秩) 其中任两行不成比 例 , 即 L1 与 L2 的 方向向 量不平 行 .排 除
(B) ( C) .分析 L1 与 L2 是否共面 , 设 A( a1 , b1 , c1 ) , C( a3 , b3 , c3 ) , 由混合积
a1 - a2 b1 - b2 c1 - c2

s1 ・ ( s2 × A C) = a2 - a3 b2 - b3 c2 - c3 =0
a3 - a1 b3 - b1 c3 - c1
(因为各行加到第 1 行后 , 第 1 行全为 0) 可知 A C, L1 , L2 三线共面 , 而 L1 与
L2 又不平行 , 所以 L1 与 L2 必相交 , 选 ( A ) .
322 历年硕士研究生入学考试中线性代数试题的题解

32. 设 A∈ Mn ( R ) (即 n 阶实矩阵 ) , 则线性方程组 (Ⅰ ) , ( Ⅱ) :


( Ⅰ ) Ax = 0,     ( Ⅱ ) AT Ax= 0
的解集必为 (   ) .
① 同解( 相等 ) ;
② (Ⅰ )的解必是 ( Ⅱ) 的解 , 但 (Ⅱ ) 的解不一定是 ( Ⅰ) 的解 ;
③ (Ⅱ )的解必是 ( Ⅰ) 的解 , 但 (Ⅰ ) 的解不一定是 ( Ⅱ) 的解 ;
④ 二者的解没有关系 .
解   答案为① .因为 : 秩 ( AT A) = 秩 ( A) ,所 以二者的基 础解系所含 向量
个数相同 ; 再由 Ax= 0 可 推出 AT ( Ax) = 0,即 ( Ⅰ ) 的 解必是 ( Ⅱ ) 的解 , 从 而
(Ⅰ )的基础解系也是 (Ⅱ ) 的基础解系 .因此 , (Ⅰ ) , (Ⅱ ) 是同解方程组 .
33. 见主教材第 3 章第 3 .5 节中的例 5 .
34. 设 αi = ( ai1 , ai2 , … , ain ) T ( i = 1 , 2 , … , r; r < n) 是 n 维实向 量 , 且 α1 ,
α2 , … ,αr 线性无关 , 已知 β= ( b1 , b2 , … , bn ) T 是线性方程组
a11 x1 + a12 x2 + … + a1n x n = 0 ,
a21 x1 + a22 x2 + … + a2n x n = 0 ,
  ………………………………
ar1 x1 + ar2 x2 + … + ar n x n = 0 .
的非零解向量 , 试判断向量组 α1 ,α2 ,… ,αr ,β的线性相关性 .
解   由于 β是线性方程组的解 ,所以 β与 αi 的内积
(αi ,β) = ai1 b1 + ai2 b2 + … + ain bn = 0 ,   i = 1 , 2 , … , r .
下面判断 α1 ,α2 ,… ,αr ,β的线性相关性 .设
k1 α1 + k2 α2 + … + krαr + kβ= 0, ( 1)

( k1 α1 + k2 α2 + … + krαr + kβ,β) = (0,β) = 0 ,

  k1 (α1 ,β) + k2 (α2 ,β) + … + kr (αr ,β) + k(β,β)
= 0 + 0 + … + 0 + k(β,β) = 0 .
由于 (β,β) > 0 ,所以 k = 0 .将其代入( 1) 式 , 再由 α1 ,α2 , … ,αr 线 性无关 , 又得
k1 = k2 = … = kr = 0 .所以 , 向量组 α1 ,α2 ,… ,αr ,β线性无关 .
35. 见第 3 章 3 .5 节中例 2 .
36. 见第 3 章 3 .5 节中例 7 .
4 向量空间与线性变换 323

37. 见第 3 章 3 .5 节中例 5 .
38. 见第 3 章 3 .5 节中例 6 .
39. 见第 3 章 3 .6 节中例 3 .
40. 见第 3 章 3 .6 节中例 5 .
41. 设向量组 α1 ,α2 ,α3 线 性无关 , 向量 β1 可 由 α1 ,α2 ,α3 线性 表示 , 而
向量 β2 不能由 α1 ,α2 ,α3 线性表示 , 则对于任意常数 k , 必有
( A) α1 ,α2 ,α3 , kβ1 + β2 线性无关 ;
( B) α1 ,α2 ,α3 , kβ1 + β2 线性相关 ;
( C) α1 ,α2 ,α3 ,β1 + kβ2 线性无关 ;
( D ) α1 ,α2 ,α3 ,β1 + kβ2 线性相关 .
解   答案为 ( A ) .下面证明 : 设
c1 α1 + c2 α2 + c3 α3 + c( kβ1 + β2 ) = 0 . ( 1)
由于 β1 = k1 α1 + k2 α2 + k3 α3 , 代入 ( 1 ) 式 ,得
( c1 + ckk1 )α1 + ( c2 + ck k2 )α2 + ( c3 + ckk3 )α3 + cβ2 = 0 . ( 2)
由于 β2 不能由 α1 ,α2 ,α3 线性表示 , 所 以 ( 2 ) 式 中 β2 的 系数 c = 0 , 将 其代 入
(2) 式 , 再由 α1 ,α2 ,α3 线性无关又得 c1 = c2 = c3 = 0 .所以 ,α1 ,α2 ,α3 , kβ1 + β2
线性无关 .
注意 : ( C) , ( D) 都不正确 .因为 : 当 k = 0时 , ( C) 中向量线 性相关 ; 当 k≠
0 时 , ( D )中向量线性无关 .

4 向量空间与线性变换

1. 已知R3 的基底为 α1 = ( 1 , 1 , 0 ) , α2 = ( 1 , 0 , 1) , α3 = ( 0 ,1 , 1 ) , 求 μ=
(2 , 0 , 0 )在基底下的坐标 .
解   设   μ= x1 α1 + x2 α2 + x3 α3 ,得
x1 1 1 0 x1 2
(α1 ,α2 ,α3 ) x2 = 1 0 1 x2 = 0 .
x3 0 1 1 x3 0
解此方程组 , 得 μ的坐标 ( x1 , x2 , x3 ) T = (1 , 1 , - 1) T .
2. 已知R3 的两组基 :
324 历年硕士研究生入学考试中线性代数试题的题解

α1 = (1 , 0 , 1) T ,α2 = ( 1 , 0 , - 1) T ,α3 = ( 1 , 1 , 1 ) T ;
β1 = (1 , 2 , 1 ) T ,β2 = (2 , 3 , 4) T ,β3 = (3 , 4 , 3) T .
求{α1 ,α2 ,α3 } 到{β1 ,β2 ,β3 } 的过渡矩阵 A .
解   设 (β1 ,β2 ,β3 ) = ( α1 ,α2 ,α3 ) A,得
1 2 3 1 1 1
2 3 4 = 0 0 1 A,
1 4 3 1 -1 1
所以
- 1
1 1 1 1 2 3 1 -2 1 1 2 3
1
A= 0 0 1 2 3 4 = 1 0 -1 2 3 4
2
1 -1 1 1 4 3 0 2 0 1 4 3
-2 0 -2 -1 0 -1
1
= 0 -2 0 = 0 -1 0 .
2
4 6 8 2 3 4
3. 设 B∈R5× 4 , r ( B) = 2 , 已知 齐次线 性方程 组 Bx = 0 的 3 个解 向量 为
α1 = (1 , 1 , 2 , 3) T ,α2 = ( - 1 , 1 , 4 , - 1 ) T ,α3 = (5 , - 1 , - 8 , 9 ) T .试求 Bx = 0 的
解空间的一个标准正交基 .
解   Bx = 0 的 解 空 间 的 维 数 ( 即 基 础 解 系 所 含 向 量 个 数 ) 为
4 - r( A) = 4 - 2 = 2 .
已知的 3 个解 向 量必 有 一 个 可 由 另 外 两个 线 性 表 示 , 易 见α3 = 2α1 -
3α2 .所以解空间的基为 {α1 ,α2 } .用施密特正交化方法 , 求解 空间的一个 标正
交基 .先正交化 .
令           β1 = α1 = ( 1 ,1, 2,3 ) T ,
(α2 ,β1 ) 5
β2 = α2 - β1 = ( - 1, 1 ,4 , - 1) T - (1 ,1 ,2 , 3)T
(β1 ,β1 ) 15
T
4 2 10
= - , , , - 2 .
3 3 3
再单位化 , 有
1 1
ε1 = β= ( 1 ,1 ,2, 3 ) T ,
‖β1 ‖ 1 15
1 1
ε2 = β= ( - 2 , 1 , 5 , - 3 )T .
‖β2 ‖ 2 39
5 特征值与特征向量 矩阵的对角化 325

{ε1 ,ε2 } 就是 Bx = 0 的解空间的一个标准正交基 .

5 特征值与特征向量 矩阵的对角化

1. 见第 5 章 5 .2 节中例 6 ( C) .
2. 见第 5 章 5 .2 节中例 4 (2 ) .
3. 见第 5 章 5 .2 节中例 9 和 5 .4 节中例 6 (1 ) .
4. ( 补充题 5-43)   已知 2 , 4 , 6 , … , 2 n 是 n 阶矩阵 A 的 n 个特 征值 , 则
行列式 | A - 3 I | = (   ) .
( A) 2・ n ! - 3n ;     (B) ( 2 n - 3) ! ! = 1・3・5…( 2 n - 3) ;
( C) - (2 n - 3) ! !; ( D) 5・7・9…( 2 n + 3) .
解   n 阶矩阵 A 有 n 个不同特征值 , 所以 A 可对角化 ,即存 在可逆阵 P,
使
P - 1 AP = diag (2 , 4 , 6 , … , 2n) ,
即 A = Pdiag(2 , 4 , 6 , … , 2 n) P - 1 .
于是
| A - 3 I| = | P( diag( 2 , 4 , 6 , … , 2 n) - 3 I) P - 1 |
= | P| | P - 1 | | diag( - 1 , 1 , 3 , … , 2 n - 3) |
= - (1・3・5… (2 n - 3 ) ) = - ( 2 n - 3 ) ! !
故选( C) .
2 - 1 2
5. ( 补充 题 5-39 )   已 知   A = 5 a 3 的一 个 特征 向 量ξ=
-1 b - 2
( 1 ,1, - 1 ) T .
( 1) 确定 a , b 及ξ对应的特征值 ;
( 2 ) A 能否相似于对角矩阵 ? 说明理由 .
解   ( 1 ) 由 Aξ= λξ,即 (λI - A)ξ= 0,或
λ- 2 1 -2  1 0
-5 λ- a -3  1 = 0 ,
1 - b λ+ 2 -1 0
326 历年硕士研究生入学考试中线性代数试题的题解


λ- 2 + 1 + 2 = 0 ,        λ= - 1;
- 5 + λ- a + 3 = 0 , a = - 5 + λ+ 3 = - 3;
1 - b - (λ+ 2) = 0 , b = 1 - λ- 2 = 0 .
所以 , a = - 3 , b = 0 ,ξ对应的特征值为λ= - 1 .
( 2) 由
λ- 2 1 - 2
|λI - A| = - 5 λ+ 3 - 3 = (λ+ 1) 3 = 0 ,
1 0 λ+ 2
得 λ= - 1 是 3 重特征值 , 而秩 ( - I - A) = 2, 对应于 λ= - 1 的线性无关 的特
征向量只有一个 , 所以 A 不可对角化 ,即不能与对角阵相似 .
6. ( 补充题 5-44)   已知 n 阶矩阵 A 的行列式 | A| ≠0 ,λ1 为 A 的一个特
征值 ,则 ( A* ) 2 + E( E 为单位阵 ) 必有特征值 (   ) .
2
| A|
( A) (λ1 | A| ) 2 + 1;     B + 1;
λ1
2
| A|
( C) (1 +λ1 | A| ) 2 ; ( D) 1+ .
λ1
1
解   由 A- 1 = A* , 得 A* = | A| A - 1 .
| A|
1 | A|
当λ1 为 A 的特征值 (λ1 ≠0 )时 , 为 A - 1 的特征值 , 为 A* = | A| A - 1
λ1 λ1
的特征值 , 所以
2
| A|
( A* ) 2 + E 必有特征值 + 1 .     选 ( B) .
λ1
7. ( 补充题 5-40 )   设
a - 1 c
A= 5 b 3 .
1 - c 0 - a

已知 | A| = 1 ,且 A 有一个特征值 λ0 , 其特征向量 x = ( - 1 , - 1 , 1 ) T , 试求 a,
b, c 及λ0 .
解   | A| = b( c2 - a2 ) - 5 a - bc + 3 c - 3 = 1 . ( 1)
由上题知 : A 的特征值λ与 A * 的特征值 λ0 的关系为
5 特征值与特征向量 矩阵的对角化 327

| A|
=λ0 ,
λ
所以 λ=λ0- 1 .
由(λ0- 1 I - A) x= 0,即
λ0- 1 - a 1 - c -1 0
- 5 λ0- 1 - b - 3 -1 = 0 ,
c- 1 0 λ0- 1 + a  1 0

a - λ0- 1 - 1 - c= 0 , ( 2)
b - λ0- 1 + 2 = 0 , ( 3)
a + λ0- 1 + 1 - c= 0 . ( 4)
方程( 4) - 方程( 2) 得
2λ0- 1 + 2 = 0 , 故 λ0 = - 1; ( 5)
将( 5) 式代入( 3) 式得 b= - 3 ,
(5) 式代入( 2) 式得   a = c,   再代入 (1 )式得
c= 4 .
结论 : a = c = 4 , b = - 3 ,λ0 = - 1 .
8. 见第 5 章 5 .3 节中例 3 .
9. ( 补充题 5-45)   若 A, B 均为 n 阶矩阵 , 且 A~ B,则 (   ) .
( A) λI - A = λI - B;   ( B) A 与 B 有相同的特征值与特征向量 ;
( C) AB~ B2 ; ( D) 对于任意常数 t, 均有 tI - A~ tI - B .
解   答案为 ( D) .因为 : A~ B 即存在可逆阵 P,使
P- 1 AP= B .
从而有
P- 1 ( tI - A) P = P- 1 ( tI ) P - P - 1 AP
= tI - B,
故 tI - A~ tI - B .
读者应该想清楚 , ( A) , ( B) , ( C)为什么都不正确 .
10. 见第 5 章 5 .3 节中例 2 .
328 历年硕士研究生入学考试中线性代数试题的题解

11. 见第 5 章 5 .2 节中例 8 .
12. 已知三阶矩阵 A 的特征 值λ1 = 1 ,λ2 = 2 ,λ3 = 3 , 其对应 的特 征向 量
为 ξ1 = ( 1 ,1 ,1 ) T ,ξ2 = ( 1, 2,4 ) T ,ξ3 = ( 1 ,3 ,9 ) T ;又 β= ( 1 ,1, 3 ) T .
① 将 β用ξ1 ,ξ2 ,ξ3 线性表示 ;
② 求 Anβ( n∈ N 自然数集 ) .
解   ① 设 β= x1 ξ1 + x2 ξ2 + x3ξ3 ,即
1 1 1 x1 1
1 2 3 x2 = 1 .
1 4 9 x3 3
解该方程组得 x1 = 2 , x2 = - 2 , x3 = 1 , 故
β= 2ξ1 - 2ξ2 + ξ3 .
② 由 Aξi = λiξi   ( i = 1 , 2 , 3) , 得
λ1
A(ξ1 ,ξ2 ,ξ3 ) = (ξ1 ,ξ2 ,ξ3 ) λ2 .
λ3
1 1 1
记   P= (ξ1 ,ξ2 ,ξ3 ) = 1 2 4 ,则
1 3 9
A = Pdiag(λ1 ,λ2 ,λ3 ) P - 1 ,
An = Pdiag (λ1n ,λ2n ,λ3n ) P- 1 ,
-1
1 1 1 1 0 0 1 1 1
Anβ= 1 2 4 0 2n 0 1 2 4 β.
n
1 3 9 0 0 3 1 3 9
由①中结论 ( x1 , x2 , x3 ) T = P - 1β= (2 , - 2 , 1 ) T , 所以
1 2n 3n  2 2 - 2 n+ 1 + 3n
Anβ= 1 2 n+ 1 4・3 n - 2 = 2 - 2 n+ 2 + 4・3 n .
1 3・2 n 3 n+ 2  1 2 - 3・2 n+ 1 + 3 n+ 2
3 2 -2
13. 设 A = - k - 1 k .
4 2 -3
5 特征值与特征向量 矩阵的对角化 329

问 : k 为何值时 , 存在可逆矩阵 P,使 P- 1 AP= Λ( 对角矩阵 ) , 并求 P 和 Λ .


解   由 |λI - A| = 0, 即
λ- 3 -2 2 λ- 1 -2 2 λ- 1 -2 2
  k λ+ 1 -k = 0 λ+ 1 - k = 0 λ+ 1 -k
-4 -2 λ+ 3 λ- 1 -2 λ+ 3 0 0 λ+ 1
= (λ- 1)(λ+ 1)2 = 0,
得特征值 λ1 = 1 ,λ2 = - 1 (二重 ) .
当 λ2 = - 1 , 对 应线 性无 关 的特 征 向量 有两 个 时 , A 可对 角 化 .此 时 秩
(λ2 I - A) = 1 ,即 λ2 I - A 的任意两行 ( 列 ) 成比例 .而
-4 -2 2
(λ2 I - A) = k 0 - k ,
-4 -2 2
由此可见 , 当 k = 0 时 , 秩 (λ2 I - A) = 1 .
由 (λ2 I - A) x= 0, 即
- 4 - 2 2 x1 0
0 0 0 x2 = 0 .
- 4 - 2 2 x3 0
从而得对应于 λ2 = - 1 的两个线性无关的特征向量
ξ2 = (1 , - 2 , 0) T ,   ξ3 = (1 , 0 , 2) T .
由(λ1 I - A) x= 0, 即
- 2 - 2 2 x1 0
0 2 0 x2 = 0 .
- 4 - 2 4 x3 0
从而得对应于 λ1 = 1 的特征向量 ξ1 = ( 1 ,0 ,1 ) T .
1 1 1
取 P = (ξ1 ,ξ2 ,ξ3 ) = 0 -2 0 ,
1 0 2
λ1 1
Λ= λ2 = - 1 ,
λ3 - 1
则有 P - 1 AP = Λ .
330 历年硕士研究生入学考试中线性代数试题的题解

1 0 1
14. 设 A = 0 2 0 , 计算 An - 2 An - 1 ( 自然数 n≥2) .
1 0 1
λ- 1 0 -1
解   由 |λI - A| = 0 λ- 2 0 =λ(λ- 2) 2 = 0 ,
-1 0 λ- 1
得特征值 λ1 = 0 ,λ2 = 2 (二重 ) .
对应于 λ1 = 0 的特征向量 ξ1 = ( - 1 , 0 , 1) T ;
对应于 λ2 = 2 的两个线性无关的特征向量可由
1 0 - 1 x1 0
(λ2 I - A) x= 0 0 0 x2 = 0
-1 0 1 x3 0
(注意 : 自由未知量为 x2 , x3 , 分别取 x2 = 1 , x3 = 0 和 x2 = 0 , x3 = 1 )得到
ξ2 = ( 0 ,1 ,0 ) T ,   ξ3 = ( 1, 0,1 ) T .
- 1 0 1 0
取 P= (ξ1 ,ξ2 ,ξ3 ) = 0 1 0 ,   Λ= 2 ,则有
1 0 1 2
A = PΛP -1
,   A = PΛ P n n - 1
,   2A n-1
= 2 PΛn - 1 P - 1 .
于是
An - 2 An - 1 = P( Λn - 2Λn - 1 ) P - 1 = 0   ( 因为 Λn - 2Λn - 1 = 0) .
x1 1 xn + 1 xn x n+ 1
15. 已知 = , =A   ( n= 1 ,2 ,… ) , 求 .
y1 1 yn+ 1 yn yn + 1
1 1 1 2
① A= ,     ② A= .
0 1 3 2
此题类似于第 5 章 5 .4 节中的例 7 .请读者自己独立完成 .
16. 设 A 是 n 阶实矩阵 , AA T = I, | A| < 0 ,求 | A + I| .
解   由已知条件知 A 是正交矩阵 .
| AAT | = | A| | AT | = | A| 2 = 1,故 | A| = ±1 .现已知 | A| < 0,所以 | A| = - 1 .
| A + I| = | A + A A T | = | A( I + AT ) |
= | A| | ( I + AT ) | = - | I + A| ,
所以 | A+ I| = 0 .
5 特征值与特征向量 矩阵的对角化 331

17. 设 A 为 三 阶 实 对 称 矩 阵 , 且 满 足 条 件 A2 + 2 A = 0, 已 知 A 的 秩
r ( A) = 2 .
( 1 ) 求 A 的全部特征值 .
( 2 ) 当 k 为何值时 , 矩阵 A + kI 为正定矩阵 ,其中 I 为三阶单位矩阵 .
解   ( 1 ) 设 Ax= λx ,则 A2 x = λ2 x( x≠0 ) .由 A2 + 2 A = 0, 即 A2 = - 2 A,

A2 x= - 2 Ax   从而   λ2 x= - 2λx .
由于 x≠0,所以 λ2 = - 2λ, 即 λ= - 2 或 0 .
又因为实对称矩阵 A~Λ= diag(λ1 ,λ2 ,λ3 ) , r ( A) = r ( Λ) = 2 .所以 A 的特
征值中只有一个为 0 ,两个非零 ,因此 A 的特征值为 0 , - 2 ( 二重 ) .
( 2 ) 由于 A~ diag ( 0, - 2 , - 2 ) ,所以
A + kI ~diag( 0 , - 2 , - 2 ) + kI
= diag( k, k - 2 , k - 2) .
根据相似的实对称矩阵有相同的正定性 , 即得 k > 2 时 , A 为正定矩阵 .
18. 见第 5 章 5 .3 节中例 4 .
19. 见第 5 章 5 .4 节中例 3 .
20. 设 A 是 n 阶实对称矩阵 , P 是 n 阶可逆矩阵 .已知 n 维列向量α是 A
的属于特征值 λ的特 征 向 量 , 则 矩阵 ( P - 1 AP) T 属 于特 征 值 λ的 特征 向 量
是(   ) .
( A) P - 1 α;     ( B) PT α;     ( C) Pα;     ( D) ( P- 1 ) T α.
解   已知 Aα= λα .设 ( P- 1 AP) T 属于特征值 λ的特 征向量 为β( 下面 求
β) ,即
( P - 1 AP) Tβ= λβ,
于是     PT AT ( P- 1 ) Tβ= λβ  ( AT = A, ( P - 1 ) T = ( PT ) - 1 ) ,
PT A( PT ) -1
β= λβ,
两边左乘 ( PT ) -1

A( PT ) -1
β= λ( PT ) - 1β. ( 1)
由 ( 1 ) 式可得 , ( PT ) - 1β= α,从而
β= PT α.  故选 ( B) .
21. 设实对称矩阵
332 历年硕士研究生入学考试中线性代数试题的题解

a 1 1
A= 1 a - 1 .
1 - 1 a
求可逆矩阵 P, 使 P- 1 AP 为对角阵, 并计算行列式 | A - I| 的值 .
解   由 |λI - A| = 0, 即
λ- a - 1 - 1 λ- a - 1 - 1 - 1
  - 1 λ- a 1 = 0 λ- a 1
- 1 1 λ- a λ- a - 1 1 λ- a
λ- a - 1 -1 -1
= 0 λ- a 1 = (λ- a - 1)2 (λ- a + 2) = 0 ,
0 2 λ- a + 1
得特征值 : λ1 = a + 1 (二重 ) ,λ2 = a - 2 .求特征向量的计算略去 .
对应于 λ1 的特征向量 :ξ1 = ( 1 ,1 ,0 ) T ,ξ2 = ( 1, 0,1 ) T .
对应于 λ2 的特征向量 :ξ3 = ( - 1 , 1 , 1) T .
1 1 - 1
取 P = (ξ1 ,ξ2 ,ξ3 ) = 1 0 1 ,
0 1 1
则有 P - 1 AP = dia g( a + 1 , a + 1 , a - 2) ,
| A - I| = | P - 1 ( A - I) P| = | P - 1 AP - P- 1 IP|
= | P - 1 AP - I| = | diag( a, a, a - 3) |
= a2 ( a - 3 ) .
22. 矩阵
0 - 2 - 2
A= 2 2 - 2 的非零特征值是 .
-2 - 2 2
解  由
λ 2 2 λ 2 2
|λI - A| = - 2 λ- 2 2 = - 2 λ- 2 2
2 2 λ- 2 0 λ λ
λ 0 2
= - 2 λ- 4 2 =λ2 (λ- 4 ) = 0 ,
0 0 λ
6 二次型 333

得 A 的非零特征值λ= 4 .
23. (习题 5 -24)   设 A, B 为同阶方阵 .
( 1 ) 如果 A, B 相似 ,试证 A, B 的特征多项式相等 .
( 2 ) 举一个二阶方阵的例子说明 ( 1 ) 的逆命题不成立 .
( 3 ) 当 A, B 均为实对称矩阵时 ,试证 ( 1 ) 的逆命题成立 .
证   ( 1 ) A, B 相似 ,即存在可逆矩 阵 P, 使 P - 1 AP = B, 于是 B 的特征 多
项式
|λI - B| = |λI - P- 1 AP| = | P - 1 (λI - A) P|
= | P- 1 | | P| |λI - A| = |λI - A| .
1 1 1 0
( 2) B= , A= I = ,此时
0 1 0 1
|λI - A| = |λI - B| = (λ- 1 )2 .
但是 A 与 B 不相似 ,因为 ,对于任何可逆矩阵 P, 有
P- 1 AP = P - 1 IP = I≠ B .
( 3 ) 当 A, B 为实对称矩阵时 ,若
|λI - A| = |λI - B| = (λ- λ1 ) (λ- λ2 ) … (λ- λn ) ,
由于 A, B 都可对角化, 即 A, B 都相似于对角阵 Λ= diag (λ1 ,λ2 , … ,λn ) , 于是
存在可逆阵 P1 , P2 , 使
P1- 1 AP1 = Λ= P2- 1 BP2 .
如此 ,则有
P2 P1- 1 AP1 P2- 1 = B .
取 P = P1 P2- 1 , P- 1 = P2 P1- 1 , 就有
P- 1 AP= B,
故 A 与 B 必相似 .

6 二次型

1. 求一个正交变换 ,化二次型 f ( x1 , x2 , x3 ) = x21 + 4 x22 + 4 x23 - 4 x1 x2 -


4 x1 x3 - 8 x2 x3 为标准形 .
334 历年硕士研究生入学考试中线性代数试题的题解

此题与第 6 章 6 .3 节中例 4 类似 .留给读者练习 .


2. 见第 6 章 6 .5 节中例 2 .
3. 见第 6 章 6 .3 节中例 6 .
4. ( 补充题 6-46)   已知 f ( x1 , x2 , x3 ) = 2 x21 + 3 x22 + 3 x23 + 2 a x2 x3 通 过
正交变换 x = Qy 可 化为 标 准 形 f = y21 + 2 y22 + 5 y23 , 试 求 参 数 a 及 正 交 矩
阵Q.
解   已知的二次型对应的实对称矩阵为
2 0 0
A= 0 3 a .
0 a 3
二次型通过正交变换化成的标准形中的系数 1 , 2 , 5 是 A 的特征值 .因此 ,由
λ- 2 0 0
|λI - A| = 0 λ- 3 - a = (λ- 2) (λ- 3 + a) (λ- 3 - a) = 0 ,
0 - a λ- 3
得特征值 : λ1 = 3 - a, λ2 = 2 ,λ3 = 3 + a .所以 , 当 a = 2 时 , A 的 3 个 特征值 为
1,2 ,5 .
下面求正交变换 x= Qy 中的正交矩阵 Q( 求特征向量的计算过程略去 ) .
对应于 λ1 = 1 的特征向量为 ξ1 = ( 0 ,1 , - 1 ) T ;
对应于 λ2 = 2 的特征向量为 ξ2 = (1 , 0 , 0) T ;
对应于 λ3 = 5 的特征向量为 ξ3 = (0 , 1 , 1) T .
3 个特征向量已两两正交, 只需将它们单位化 , 得
T T
1 -1 1 1
η1 = 0, , ;   η2 = ( 1 ,0 ,0 ) ,   η3 = 0 ,
T
, .
2 2 2 2
所求的正交矩阵
0 1 0
1 1
0
Q= 2 2 .
-1 1
0
2 2
此时 ,令 x= Qy,则有 QT AQ= diag( 1 , 2 , 5 ) 从而
f = xT Ax = yT QT AQy= y21 + 2 y22 + 5 y23 .
6 二次型 335

5. 见第 6 章 6 .3 节中例 5 .
6. ( 补充题 6-48)   设

1 0 1
A= 0 2 0 ,   B = ( kE + A) 2 ,
1 0 1

k 为实数 , E 为单位矩阵 ,求对 角矩 阵 Λ, 使 AD Λ; 并问 : k 为 何值 时 , B 为 正


定矩阵 .
解   由 |λE - A| = 0, 即

λ- 1 0 -1
|λE - A| = 0 λ- 2 0 = (λ- 2 ) [ (λ- 1) 2 - 1]
-1 0 λ- 1

= λ(λ- 2) 2 = 0 ,
得特征值 λ1 = 0 ,λ2 = 2 (二重 ) .于是
AD Λ= diag ( 0, 2,2 ) .
相应地, B = ( kE + A) 2 的特征值为 μ1 = k2 ,μ2 = ( k + 2) 2 ( 二重) .因此 , 当 k 为
非零实数时 , B 的 3 个特征值均大于零 , 故 B 为正定矩阵 .
7. 见第 6 章 6 .5 节中例 3 .
8. 见第 6 章 6 .5 节中例 4 .
9. 已知实二 次 型 f ( x1 , x2 , x3 ) = a( x21 + x22 + x23 ) + 4 x1 x2 + 4 x1 x3 +
4 x2 x3 经正交变换 x = Py 可化为标准形 f = 6 y21 , 则 a = .
解   二次型对应的实对称矩阵为

a 2 2
A= 2 a 2 .
2 2 a

根据题意 , A 的特征值为λ1 = 6 ,λ2 = 0( 二重) .由

λ- a - 2 - 2 λ- a - 4 -2 -2
|λI - A| = - 2 λ- a - 2 = λ- a - 4 λ- a -2
- 2 - 2 λ- a λ- a - 4 -2 λ- a
336 历年硕士研究生入学考试中线性代数试题的题解

1 - 2 -2
= (λ- a - 4) 0 λ- a + 2 0
0 0 λ- a + 2
= (λ- a - 4) (λ- a + 2) 2 = 0 ,
得 A 的特征值 : λ1 = a + 4 ,λ2 = a - 2 (二重 ) , 由
λ2 = a - 2 = 0 ,
得 a = 2 , 此时 λ1 = 6 ,λ2 = 0( 二重) .于是 , 所求的 a = 2 .

You might also like